Site Loader
\circ\), \(a=0,6\) м/с2, \(\mu-?\)

Π‘ΠΎΠ΄Π΅Ρ€ΠΆΠ°Π½ΠΈΠ΅

РСшСниС Π·Π°Π΄Π°Ρ‡ΠΈ:

Π‘Ρ…Π΅ΠΌΠ°Ρ‚ΠΈΡ‡Π½ΠΎ ΠΈΠ·ΠΎΠ±Ρ€Π°Π·ΠΈΠΌΒ Π°Π²Ρ‚ΠΎΠΌΠΎΠ±ΠΈΠ»ΡŒ Π½Π° Π½Π°ΠΊΠ»ΠΎΠ½Π½ΠΎΠΉ Π΄ΠΎΡ€ΠΎΠ³Π΅, Π²Π²Π΅Π΄Π΅ΠΌ систСму ΠΊΠΎΠΎΡ€Π΄ΠΈΠ½Π°Ρ‚ ΠΈ ΠΏΠΎΠΊΠ°ΠΆΠ΅ΠΌ всС силы, Π΄Π΅ΠΉΡΡ‚Π²ΡƒΡŽΡ‰ΠΈΠ΅ Π½Π° Π°Π²Ρ‚ΠΎΠΌΠΎΠ±ΠΈΠ»ΡŒ. На Π½Π΅Π³ΠΎ дСйствуСт сила тяТСсти, сила Ρ€Π΅Π°ΠΊΡ†ΠΈΠΈ ΠΎΠΏΠΎΡ€Ρ‹ (Π½Π° рисункС ΠΈΠ·ΠΎΠ±Ρ€Π°ΠΆΠ΅Π½Π° суммарная со стороны Π΄Π²ΡƒΡ… колСс) ΠΈ сила трСния покоя.

ΠžΠ±Ρ€Π°Ρ‚ΠΈΡ‚Π΅ Π²Π½ΠΈΠΌΠ°Π½ΠΈΠ΅, Ρ‡Ρ‚ΠΎ ΠΌΠ΅ΠΆΠ΄Ρƒ шинами ΠΈ Π΄ΠΎΡ€ΠΎΠ³Π°ΠΌΠΈ дСйствуСт ΠΈΠΌΠ΅Π½Π½ΠΎ сила трСния покоя, ΠΈΠ½Π°Ρ‡Π΅ Π°Π²Ρ‚ΠΎΠΌΠΎΠ±ΠΈΠ»ΡŒ Π±Ρ‹ ΠΏΡ€ΠΎΡΠΊΠ°Π»ΡŒΠ·Ρ‹Π²Π°Π». ΠŸΡ€ΠΈΡ‡Π΅ΠΌ Ρ‡Ρ‚ΠΎΠ±Ρ‹ ΡΠΎΠ±Π»ΡŽΡΡ‚ΠΈ условиС ΠΌΠΈΠ½ΠΈΠΌΠ°Π»ΡŒΠ½ΠΎΡΡ‚ΠΈ коэффициСнта трСния, эта сила трСния покоя Π΄ΠΎΠ»ΠΆΠ½Π° ΠΏΡ€ΠΈΠ½ΡΡ‚ΡŒ максимальноС Π·Π½Π°Ρ‡Π΅Π½ΠΈΠ΅, Ρ‚.Π΅. Π±Ρ‹Ρ‚ΡŒ Ρ€Π°Π²Π½ΠΎΠΉ силС трСния скольТСния, хотя Π°Π²Ρ‚ΠΎΠΌΠΎΠ±ΠΈΠ»ΡŒ Π΅Ρ‰Ρ‘ Π½Π΅ ΠΏΡ€ΠΎΡΠΊΠ°Π»ΡŒΠ·Ρ‹Π²Π°Π΅Ρ‚. Π‘ΠΈΠ»Π° трСния Π½Π°ΠΏΡ€Π°Π²Π»Π΅Π½Π° Π²Π²Π΅Ρ€Ρ…, ΠΏΠΎΡΠΊΠΎΠ»ΡŒΠΊΡƒ Π°Π²Ρ‚ΠΎΠΌΠΎΠ±ΠΈΠ»ΡŒ стрСмится ΡΠΎΡΠΊΠΎΠ»ΡŒΠ·Π½ΡƒΡ‚ΡŒ Π²Π½ΠΈΠ·.

Π—Π°ΠΏΠΈΡˆΠ΅ΠΌ Π²Ρ‚ΠΎΡ€ΠΎΠΉ Π·Π°ΠΊΠΎΠ½ ΠΡŒΡŽΡ‚ΠΎΠ½Π° Π² ΠΏΡ€ΠΎΠ΅ΠΊΡ†ΠΈΠΈ Π½Π° ось \(x\):

\[{F_{Ρ‚Ρ€}} – mg \cdot \sin \alpha Β = ma\;\;\;\;(1)\]

Π’Π°ΠΊ ΠΊΠ°ΠΊ Π°Π²Ρ‚ΠΎΠΌΠΎΠ±ΠΈΠ»ΡŒ покоится вдоль оси \(y\), Ρ‚ΠΎ ΠΏΡ€ΠΈΠΌΠ΅Π½ΠΈΠΌ ΠΏΠ΅Ρ€Π²Ρ‹ΠΉ Π·Π°ΠΊΠΎΠ½ ΠΡŒΡŽΡ‚ΠΎΠ½Π° Π² ΠΏΡ€ΠΎΠ΅ΠΊΡ†ΠΈΠΈ Π½Π° эту ось:

\[N = mg \cdot \cos \alpha \;\;\;\;(2)\]

Π‘ΠΈΠ»Ρƒ трСния покоя Π½Π°ΠΉΠ΄Π΅ΠΌ ΠΏΠΎ Ρ„ΠΎΡ€ΠΌΡƒΠ»Π΅ (смотри объяснСния Π²Ρ‹ΡˆΠ΅):

\[{F_{Ρ‚Ρ€}} = \mu N\]

Учитывая (2), ΠΈΠΌΠ΅Π΅ΠΌ:

\[{F_{Ρ‚Ρ€}} = \mu mg \cdot \cos \alpha \]

ΠŸΠΎΠ΄ΡΡ‚Π°Π²ΠΈΠΌ ΠΏΠΎΠ»ΡƒΡ‡Π΅Π½Π½ΠΎΠ΅ Π² равСнство (1):

\[\mu mg \cdot \cos \alpha  – mg \cdot \sin \alpha Β = ma\]

ΠžΡΡ‚Π°Π»ΠΎΡΡŒ Ρ‚ΠΎΠ»ΡŒΠΊΠΎ Π²Ρ‹Ρ€Π°Π·ΠΈΡ‚ΡŒ коэффициСнт \(\mu\), Ρ‡Ρ‚ΠΎ ΠΌΡ‹ сСйчас ΠΈ сдСлаСм:

\[\mu Β = \frac{{a + g \cdot \sin \alpha }}{{g \cdot \cos \alpha }}\]

ΠŸΠΎΡΡ‡ΠΈΡ‚Π°Π΅ΠΌ ΠΎΡ‚Π²Π΅Ρ‚:

\[\mu Β = \frac{{0,6 + 10 \cdot \sin 30^\circ }}{{10 \cdot \cos 30^\circ }} = 0,65\]

ΠžΡ‚Π²Π΅Ρ‚: 0,65.

Если Π’Ρ‹ Π½Π΅ поняли Ρ€Π΅ΡˆΠ΅Π½ΠΈΠ΅ ΠΈ Ρƒ Вас Π΅ΡΡ‚ΡŒ ΠΊΠ°ΠΊΠΎΠΉ-Ρ‚ΠΎ вопрос ΠΈΠ»ΠΈ Π’Ρ‹ нашли ΠΎΡˆΠΈΠ±ΠΊΡƒ, Ρ‚ΠΎ смСло оставляйтС Π½ΠΈΠΆΠ΅ ΠΊΠΎΠΌΠΌΠ΅Π½Ρ‚Π°Ρ€ΠΈΠΉ.

ΠžΠΏΡ€Π΅Π΄Π΅Π»Π΅Π½ΠΈΠ΅ коэффициСнта сухого трСния

ΠžΠΏΡ€Π΅Π΄Π΅Π»ΠΈΠΌ Π·Π°Π²ΠΈΡΠΈΠΌΠΎΡΡ‚ΡŒ силы трСния. Для ΡΠ»Π΅Π΄ΡƒΡŽΡ‰Π΅Π³ΠΎ ΠΎΠΏΡ‹Ρ‚Π° Π½Π°ΠΌ понадобится ΠΈΡΠΏΠΎΠ»ΡŒΠ·ΠΎΠ²Π°Ρ‚ΡŒ Π³Π»Π°Π΄ΠΊΡƒΡŽ дСрСвянной доску, дСрСвянный брусок ΠΈ Π΄ΠΈΠ½Π°ΠΌΠΎΠΌΠ΅Ρ‚Ρ€.

Рисунок 1

ΠŸΠ΅Ρ€Π²Ρ‹ΠΌ Π΄Π΅Π»ΠΎΠΌ ΠΏΡ€ΠΎΠ²Π΅Ρ€ΠΈΠΌ, зависима Π»ΠΈ сила трСния ΠΎΡ‚ ΠΏΠ»ΠΎΡ‰Π°Π΄ΠΈ повСрхности соприкосновСния Ρ‚Π΅Π». Для этого ΠΏΠΎΠ»ΠΎΠΆΠΈΠΌ брусок Π½Π° Π³ΠΎΡ€ΠΈΠ·ΠΎΠ½Ρ‚Π°Π»ΡŒΠ½ΠΎ Ρ€Π°ΡΠΏΠΎΠ»ΠΎΠΆΠ΅Π½Π½ΡƒΡŽ доску Π³Ρ€Π°Π½ΡŒΡŽ с наибольшСй ΠΏΠ»ΠΎΡ‰Π°Π΄ΡŒΡŽ повСрхности. Π—Π°ΠΊΡ€Π΅ΠΏΠΈΠ² Π½Π° брускС Π΄ΠΈΠ½Π°ΠΌΠΎΠΌΠ΅Ρ‚Ρ€, Π±ΡƒΠ΄Π΅ΠΌ ΠΏΠ»Π°Π²Π½ΠΎ ΠΏΠΎΠ²Ρ‹ΡˆΠ°Ρ‚ΡŒ Π·Π½Π°Ρ‡Π΅Π½ΠΈΠ΅ Π½Π°ΠΏΡ€Π°Π²Π»Π΅Π½Π½ΠΎΠΉ вдоль повСрхности доски силы ΠΈ ΠΎΡ‚ΠΌΠ΅Ρ‚ΠΈΠΌ максимальноС Π²Π΅Π»ΠΈΡ‡ΠΈΠ½Ρƒ силы трСния покоя. ПослС ΠΏΠ΅Ρ€Π΅Π²Π΅Ρ€Π½Π΅ΠΌ этот ΠΆΠ΅ брусок Π½Π° Π³Ρ€Π°Π½ΡŒ с мСньшСй ΠΏΠ»ΠΎΡ‰Π°Π΄ΡŒΡŽ повСрхности ΠΈ снова ΠΏΡ€ΠΎΠ²Π΅Π΄Π΅ΠΌ ΠΈΠ·ΠΌΠ΅Ρ€Π΅Π½ΠΈΠ΅ максимального значСния силы трСния покоя. Π’ процСссС ΠΎΠΏΡ‹Ρ‚Π° выясняСм, Ρ‡Ρ‚ΠΎ наибольшСС Π·Π½Π°Ρ‡Π΅Π½ΠΈΠ΅ силы трСния покоя Π½Π΅ ΠΎΠ±Π»Π°Π΄Π°Π΅Ρ‚ Π·Π°Π²ΠΈΡΠΈΠΌΠΎΡΡ‚ΡŒΡŽ ΠΎΡ‚ ΠΏΠ»ΠΎΡ‰Π°Π΄ΠΈ повСрхности соприкосновСния Ρ‚Π΅Π». Π‘ΠΎΠ²Π΅Ρ€ΡˆΠΈΠ² Ρ‚Π°ΠΊΠΈΠ΅ ΠΆΠ΅ измСрСния Π² условиях Ρ€Π°Π²Π½ΠΎΠΌΠ΅Ρ€Π½ΠΎΠ³ΠΎ двиТСния бруска ΠΏΠΎ повСрхности доски, убСТдаСмся Π² Ρ‚ΠΎΠΌ, Ρ‡Ρ‚ΠΎ сила трСния скольТСния Ρ‚ΠΎΠΆΠ΅ Π½Π΅ зависима ΠΎΡ‚ ΠΏΠ»ΠΎΡ‰Π°Π΄ΠΈ повСрхности соприкосновСния Ρ‚Π΅Π».

ИсслСдованиС зависимости силы трСния ΠΎΡ‚ силы давлСния

Π‘Π²Π΅Ρ€Ρ…Ρƒ Π½Π° ΠΏΠ΅Ρ€Π²Ρ‹ΠΉ брусок поставим Π΅Ρ‰Π΅ ΠΎΠ΄ΠΈΠ½ Ρ‚Π°ΠΊΠΎΠΉ ΠΆΠ΅.

Рисунок 2

Π”Π°Π½Π½Ρ‹ΠΌ дСйствиСм ΠΌΡ‹ повысим Π²Π΅Π»ΠΈΡ‡ΠΈΠ½Ρƒ силы, ΠΎΡ€Ρ‚ΠΎΠ³ΠΎΠ½Π°Π»ΡŒΠ½ΠΎΠΉ повСрхности соприкосновСния Ρ‚Π΅Π»Π° ΠΈ стола, Ρ‚ΠΎ Π΅ΡΡ‚ΡŒ силу давлСния PΒ―. Если послС этого ΠΌΡ‹ снова ΠΈΠ·ΠΌΠ΅Ρ€ΠΈΠΌ Π½Π°ΠΈΠ±ΠΎΠ»ΡŒΡˆΡƒΡŽ силу трСния покоя, Ρ‚ΠΎ ΡƒΠ²ΠΈΠ΄ΠΈΠΌ, Ρ‡Ρ‚ΠΎ ΠΎΠ½Π° возросла Π² Π΄Π²Π° Ρ€Π°Π·Π°. Π”ΠΎΠ±Π°Π²ΠΈΠ² ΠΊ конструкции Π΅Ρ‰Π΅ ΠΎΠ΄ΠΈΠ½ брусок, ΠΌΡ‹ ΠΎΠ±Π½Π°Ρ€ΡƒΠΆΠΈΠ²Π°Π΅ΠΌ, Ρ‡Ρ‚ΠΎ максимальная сила трСния покоя выросла Ρ‚Ρ€ΠΎΠ΅ΠΊΡ€Π°Ρ‚Π½ΠΎ. ΠžΡΠ½ΠΎΠ²Ρ‹Π²Π°ΡΡΡŒ Π½Π° Π΄Π°Π½Π½Ρ‹Ρ… ΠΎΠΏΡ‹Ρ‚Π°Ρ…, ΠΌΠΎΠΆΠ½ΠΎ Π·Π°ΠΊΠ»ΡŽΡ‡ΠΈΡ‚ΡŒ, Ρ‡Ρ‚ΠΎ максимальноС Π·Π½Π°Ρ‡Π΅Π½ΠΈΠ΅ модуля силы трСния покоя прямо ΠΏΡ€ΠΎΠΏΠΎΡ€Ρ†ΠΈΠΎΠ½Π°Π»ΡŒΠ½ΠΎ Π²Π΅Π»ΠΈΡ‡ΠΈΠ½Π΅ силы давлСния. ВзаимодСйствиС Ρ‚Π΅Π»Π° ΠΈ ΠΎΠΏΠΎΡ€Ρ‹ ΠΏΡ€ΠΎΠ²ΠΎΡ†ΠΈΡ€ΡƒΠ΅Ρ‚ Π΄Π΅Ρ„ΠΎΡ€ΠΌΠ°Ρ†ΠΈΡŽ ΠΎΠ±ΠΎΠΈΡ….

ΠžΠΏΡ€Π΅Π΄Π΅Π»Π΅Π½ΠΈΠ΅ 1

Π‘ΠΈΠ»Ρƒ упругости NΒ―, ΠΏΠΎΡΠ²Π»ΡΡŽΡ‰ΡƒΡŽΡΡ ΠΊΠ°ΠΊ Ρ€Π΅Π·ΡƒΠ»ΡŒΡ‚Π°Ρ‚ Π΄Π΅Ρ„ΠΎΡ€ΠΌΠ°Ρ†ΠΈΠΈ ΠΎΠΏΠΎΡ€Ρ‹ ΠΈ ΠΎΠΊΠ°Π·Ρ‹Π²Π°ΡŽΡ‰ΡƒΡŽ своС воздСйствиС Π½Π° ΠΎΠ±ΡŠΠ΅ΠΊΡ‚, Π½Π°Π·Ρ‹Π²Π°ΡŽΡ‚ силой Ρ€Π΅Π°ΠΊΡ†ΠΈΠΈ ΠΎΠΏΠΎΡ€Ρ‹.

Π˜ΡΡ…ΠΎΠ΄Ρ ΠΈΠ· Ρ‚Ρ€Π΅Ρ‚ΡŒΠ΅Π³ΠΎ Π·Π°ΠΊΠΎΠ½Π° ΠΡŒΡŽΡ‚ΠΎΠ½Π°, Π΄Π΅Π»Π°Π΅ΠΌ Π²Ρ‹Π²ΠΎΠ΄, Ρ‡Ρ‚ΠΎ силы давлСния ΠΈ Ρ€Π΅Π°ΠΊΡ†ΠΈΠΈ ΠΎΠΏΠΎΡ€Ρ‹ эквивалСнтны ΠΏΠΎ ΠΌΠΎΠ΄ΡƒΠ»ΡŽ ΠΈ ΠΏΡ€ΠΎΡ‚ΠΈΠ²ΠΎΠΏΠΎΠ»ΠΎΠΆΠ½Ρ‹ ΠΏΠΎ Π½Π°ΠΏΡ€Π°Π²Π»Π΅Π½ΠΈΡŽ:

Рисунок 3

По этой ΠΏΡ€ΠΈΡ‡ΠΈΠ½Π΅ Π²Ρ‹Π²ΠΎΠ΄ Π²Ρ‹ΡˆΠ΅ ΠΌΠΎΠΆΠ½ΠΎ ΡΡ„ΠΎΡ€ΠΌΡƒΠ»ΠΈΡ€ΠΎΠ²Π°Ρ‚ΡŒ ΡΠ»Π΅Π΄ΡƒΡŽΡ‰ΠΈΠΌ ΠΎΠ±Ρ€Π°Π·ΠΎΠΌ: ΠΌΠΎΠ΄ΡƒΠ»ΡŒ наибольшСй силы трСния покоя ΠΏΡ€ΠΎΠΏΠΎΡ€Ρ†ΠΈΠΎΠ½Π°Π»Π΅Π½ силС Ρ€Π΅Π°ΠΊΡ†ΠΈΠΈ ΠΎΠΏΠΎΡ€Ρ‹.

ΠžΠΏΡ€Π΅Π΄Π΅Π»Π΅Π½ΠΈΠ΅ 2

ГрСчСская Π±ΡƒΠΊΠ²Π° ΞΌ ΠΈΠ³Ρ€Π°Π΅Ρ‚ Ρ€ΠΎΠ»ΡŒ коэффициСнта ΠΏΡ€ΠΎΠΏΠΎΡ€Ρ†ΠΈΠΎΠ½Π°Π»ΡŒΠ½ΠΎΡΡ‚ΠΈ, Ρ‚Π°ΠΊ ΠΆΠ΅ носящСго Π½Π°Π·Π²Π°Π½ΠΈΠ΅ коэффициСнта трСния (покоя ΠΈΠ»ΠΈ скольТСния).

НуТна ΠΏΠΎΠΌΠΎΡ‰ΡŒ прСподаватСля?

Опиши Π·Π°Π΄Π°Π½ΠΈΠ΅Β β€” и наши экспСрты Ρ‚Π΅Π±Π΅ ΠΏΠΎΠΌΠΎΠ³ΡƒΡ‚!

ΠžΠΏΠΈΡΠ°Ρ‚ΡŒ Π·Π°Π΄Π°Π½ΠΈΠ΅

Π˜ΡΡ…ΠΎΠ΄Ρ ΠΈΠ· Ρ€Π΅Π·ΡƒΠ»ΡŒΡ‚Π°Ρ‚ΠΎΠ² ΠΎΠΏΡ‹Ρ‚Π°, ΠΌΠΎΠΆΠ½ΠΎ с ΡƒΠ²Π΅Ρ€Π΅Π½Π½ΠΎΡΡ‚ΡŒΡŽ Π·Π°ΡΠ²ΠΈΡ‚ΡŒ, Ρ‡Ρ‚ΠΎ ΠΌΠΎΠ΄ΡƒΠ»ΡŒ силы трСния скольТСния
FΡ‚Ρ€, Ρ€ΠΎΠ²Π½ΠΎ ΠΊΠ°ΠΊ ΠΈ ΠΌΠΎΠ΄ΡƒΠ»ΡŒ максимальной силы трСния покоя, ΠΏΡ€ΠΎΠΏΠΎΡ€Ρ†ΠΈΠΎΠ½Π°Π»Π΅Π½ ΠΌΠΎΠ΄ΡƒΠ»ΡŽ силы Ρ€Π΅Π°ΠΊΡ†ΠΈΠΈ ΠΎΠΏΠΎΡ€Ρ‹. МаксимальноС Π·Π½Π°Ρ‡Π΅Π½ΠΈΠ΅ силы трСния покоя ΠΏΡ€ΠΈΠ±Π»ΠΈΠ·ΠΈΡ‚Π΅Π»ΡŒΠ½ΠΎ Ρ€Π°Π²Π½ΠΎ силС трСния скольТСния, ΠΏΡ€ΠΈΠΌΠ΅Ρ€Π½ΠΎ Ρ€Π°Π²Π½Ρ‹ Ρ‚Π°ΠΊΠΆΠ΅ коэффициСнты трСния покоя ΠΈ скольТСния.

Π‘Π΅Π·Ρ€Π°Π·ΠΌΠ΅Ρ€Π½Ρ‹ΠΉ коэффициСнт ΠΏΡ€ΠΎΠΏΠΎΡ€Ρ†ΠΈΠΎΠ½Π°Π»ΡŒΠ½ΠΎΡΡ‚ΠΈ ΞΌ ΠΎΠ±Π»Π°Π΄Π°Π΅Ρ‚ Π·Π°Π²ΠΈΡΠΈΠΌΠΎΡΡ‚ΡŒΡŽ:

  1. ΠžΡ‚ ΠΏΡ€ΠΈΡ€ΠΎΠ΄Ρ‹ трущихся повСрхностСй.
  2. ΠžΡ‚ состояния трущихся повСрхностСй, Π² частности ΠΎΡ‚ ΠΈΡ… ΡˆΠ΅Ρ€ΠΎΡ…ΠΎΠ²Π°Ρ‚ΠΎΡΡ‚ΠΈ.
  3. Π’ случаС скольТСния коэффициСнт трСния являСтся Ρ„ΡƒΠ½ΠΊΡ†ΠΈΠ΅ΠΉ скорости.

ΠŸΡ€ΠΈΠΌΠ΅Ρ€Ρ‹ Π·Π°Π΄Π°Ρ‡ Π½Π° ΠΎΠΏΡ€Π΅Π΄Π΅Π»Π΅Π½ΠΈΠ΅ коэффициСнта трСния

ΠŸΡ€ΠΈΠΌΠ΅Ρ€ 1

ΠžΠΏΡ€Π΅Π΄Π΅Π»ΠΈΡ‚Π΅ ΠΌΠΈΠ½ΠΈΠΌΠ°Π»ΡŒΠ½ΡƒΡŽ Π΄Π»ΠΈΠ½Ρƒ Ρ‚ΠΎΡ€ΠΌΠΎΠ·Π½ΠΎΠ³ΠΎ ΠΏΡƒΡ‚ΠΈ автомобиля, Ссли Ρ‚ΠΎΡ‚ Π½Π°Ρ‡Π°Π» Ρ‚ΠΎΡ€ΠΌΠΎΠΆΠ΅Π½ΠΈΠ΅ Π½Π° Π³ΠΎΡ€ΠΈΠ·ΠΎΠ½Ρ‚Π°Π»ΡŒΠ½ΠΎΠΌ участкС шоссС Π½Π° скорости двиТСния 20Β ΠΌ/с. ΠšΠΎΡΡ„Ρ„ΠΈΡ†ΠΈΠ΅Π½Ρ‚ трСния Ρ€Π°Π²Π΅Π½ 0,5.

Π”Π°Π½ΠΎ: Ο…=20Β ΠΌ/с,Β ΞΌ=0,5.

Найти: Smin — ?

РСшСниС

Π’ΠΎΡ€ΠΌΠΎΠ·Π½ΠΎΠΉ ΠΏΡƒΡ‚ΡŒ автомобиля Π±ΡƒΠ΄Π΅Ρ‚ ΠΎΠ±Π»Π°Π΄Π°Ρ‚ΡŒ наимСньшим Π·Π½Π°Ρ‡Π΅Π½ΠΈΠ΅ΠΌ ΠΏΡ€ΠΈ условии максимального значСния силы трСния. ΠœΠΎΠ΄ΡƒΠ»ΡŒ максимального значСния силы трСния равняСтся: FΡ‚Ρ€max=ΞΌmg. Π’Π΅ΠΊΡ‚ΠΎΡ€ силы FΡ‚Ρ€ Π² процСссС тормоТСния Π½Π°ΠΏΡ€Π°Π²Π»Π΅Π½ ΠΏΡ€ΠΎΡ‚ΠΈΠ² направлСния Π²Π΅ΠΊΡ‚ΠΎΡ€ΠΎΠ² скорости Ο…0 ΠΈ пСрСмСщСния S. ΠŸΡ€ΠΈ прямолинСйном равноускорСнном Π΄Π²ΠΈΠΆΠ΅Π½ΠΈΠΈ проСкция пСрСмСщСния Sx автомобиля Π½Π° ось, ΠΏΠ°Ρ€Π°Π»Π»Π΅Π»ΡŒΠ½ΡƒΡŽ Π²Π΅ΠΊΡ‚ΠΎΡ€Ρƒ скорости Ο…0 автомобиля, Ρ€Π°Π²Π½Π°: Sx=Ο…0xt+at22.

ΠŸΠ΅Ρ€Π΅Ρ…ΠΎΠ΄Ρ ΠΊ модулям Π²Π΅Π»ΠΈΡ‡ΠΈΠ½, ΠΏΠΎΠ»ΡƒΡ‡Π°Π΅ΠΌ: S=Ο…0xt-at22.

Π—Π½Π°Ρ‡Π΅Π½ΠΈΠ΅ Π²Ρ€Π΅ΠΌΠ΅Π½ΠΈ ΠΌΠΎΠΆΠ½ΠΎ Π½Π°ΠΉΡ‚ΠΈ ΠΈΠ· условия: Ο…1=Ο…0-at=0,Β t=Ο…0a.

Π’ΠΎΠ³Π΄Π° для модуля пСрСмСщСния ΠΏΠΎΠ»ΡƒΡ‡Π°Π΅ΠΌ: S=Ο…022a.

По Ρ‚ΠΎΠΉ ΠΏΡ€ΠΈΡ‡ΠΈΠ½Π΅, Ρ‡Ρ‚ΠΎ: a=FΡ‚Ρ€maxm=ΞΌmgm=ΞΌg,Β Smin=Ο…022ΞΌgβ‰ˆ40Β ΠΌ.

ΠžΡ‚Π²Π΅Ρ‚: Sminβ‰ˆ40Β ΠΌ.

ΠŸΡ€ΠΈΠΌΠ΅Ρ€ 2

ΠšΠ°ΠΊΡƒΡŽ силу Π½Π΅ΠΎΠ±Ρ…ΠΎΠ΄ΠΈΠΌΠΎ ΠΏΡ€ΠΈΠ»ΠΎΠΆΠΈΡ‚ΡŒ ΠΊ Ρ‚Π΅ΠΏΠ»ΠΎΠ²ΠΎΠ·Ρƒ массой 8Β Ρ‚ Π² Π³ΠΎΡ€ΠΈΠ·ΠΎΠ½Ρ‚Π°Π»ΡŒΠ½ΠΎΠΌ Π½Π°ΠΏΡ€Π°Π²Π»Π΅Π½ΠΈΠΈ, Ρ‡Ρ‚ΠΎΠ±Ρ‹ ΡΠ½ΠΈΠ·ΠΈΡ‚ΡŒ Π΅Π³ΠΎ ΡΠΊΠΎΡ€ΠΎΡΡ‚ΡŒ Π½Π° 0,3Β ΠΌ/с Π² Ρ‚Π΅Ρ‡Π΅Π½ΠΈΠ΅ 5 сСкунд? ΠšΠΎΡΡ„Ρ„ΠΈΡ†ΠΈΠ΅Π½Ρ‚ трСния Ρ€Π°Π²Π΅Π½ 0,05.

Π”Π°Π½ΠΎ: m=8000Β ΠΊΠ³,Β Ξ”v=0,3Β ΠΌ/с,Β ΞΌ=0,05.

Найти: F — ?

РСшСниС

Рисунок 4

Π—Π°ΠΏΠΈΡˆΠ΅ΠΌ ΡƒΡ€Π°Π²Π½Π΅Π½ΠΈΠ΅ двиТСния Ρ‚Π΅Π»Π°:

ma=F+FΒ―Ρ‚Ρ€.

Π‘ΠΏΡ€ΠΎΠ΅Ρ†ΠΈΡ€ΡƒΠ΅ΠΌ Π½Π° ось Ρ… силы ΠΈ ускорСниС:

Β ma=F+FΡ‚Ρ€.

Π’Π°ΠΊ ΠΊΠ°ΠΊ FΡ‚Ρ€=ΞΌmg, Π° a=Ο…-Ο…0t=βˆ†Ο…t, Π²Ρ‹Ρ…ΠΎΠ΄ΠΈΡ‚, Ρ‡Ρ‚ΠΎ: F=mβˆ†Ο…t-ΞΌg=3440 Н

ΠžΡ‚Π²Π΅Ρ‚: F=3440 Н.

Π£Ρ‡Π΅Π½ΠΈΠΊΠΈ выполняли Π»Π°Π±ΠΎΡ€Π°Ρ‚ΠΎΡ€Π½ΡƒΡŽ Ρ€Π°Π±ΠΎΡ‚Ρƒ, Π² ΠΊΠΎΡ‚ΠΎΡ€ΠΎΠΉ исслСдовали Π·Π°Π²ΠΈΡΠΈΠΌΠΎΡΡ‚ΡŒ коэффициСнта трСния ΠΎΡ‚ Ρ€Π°Π·Π»ΠΈΡ‡Π½Ρ‹Ρ… повСрхностСй. Они Π²Ρ‹Π±Ρ€Π°Π»ΠΈ ΡΠΎΠΎΡ‚Π²Π΅Ρ‚ΡΡ‚Π²ΡƒΡŽΡ‰ΠΈΠ΅ Ρ€Π°Π±ΠΎΡ‡ΠΈΠ΅ принадлСТности β€” дСрСвянный брусок, ΠΊ ΠΎΠ΄Π½ΠΎΠΉ сторонС ΠΊΠΎΡ‚ΠΎΡ€ΠΎΠ³ΠΎ ΠΏΡ€ΠΈΠΊΡ€Π΅ΠΏΠ»Π΅Π½Π° мСталличСская пластинка, Π³ΠΈΡ€ΠΈ, дСрСвянный стол ΠΈ Π΄ΠΈΠ½Π°ΠΌΠΎΠΌΠ΅Ρ‚Ρ€. Π‘Π½Π°Ρ‡Π°Π»Π° ΡƒΡ‡Π΅Π½ΠΈΠΊΠΈ ΠΈΠ·ΠΌΠ΅Ρ€ΠΈΠ»ΠΈ Π΄ΠΈΠ½Π°ΠΌΠΎΠΌΠ΅Ρ‚Ρ€ΠΎΠΌ вСс бруска ΠΈ Π³ΠΈΡ€ΠΈ. Π—Π°Ρ‚Π΅ΠΌ ΠΎΠ½ΠΈ Ρ‚Π°Ρ‰ΠΈΠ»ΠΈ брусок с гирями ΠΏΠΎ Π³ΠΎΡ€ΠΈΠ·ΠΎΠ½Ρ‚Π°Π»ΡŒΠ½ΠΎΠΉ повСрхности стола ΠΈ Π΄ΠΈΠ½Π°ΠΌΠΎΠΌΠ΅Ρ‚Ρ€ΠΎΠΌ измСряли силу трСния. Π‘ΠΈΠ»Ρƒ трСния измСряли Π½Π° Π΄Π²ΡƒΡ… Ρ€Π°Π·Π½Ρ‹Ρ… повСрхностях бруска. Π’ Π·Π°Π΄Π°Ρ‡Π΅ частично прСдставлСна Ρ‚Π°Π±Π»ΠΈΡ†Π° ΠΈΠ·ΠΌΠ΅Ρ€Π΅Π½ΠΈΠΉ, собранных ΡƒΡ‡Π΅Π½ΠΈΠΊΠ°ΠΌΠΈ. Π—Π°ΠΏΠΎΠ»Π½ΠΈ Π΅Ρ‘ Π΄ΠΎ ΠΊΠΎΠ½Ρ†Π° ΠΈ Π·Π°ΠΊΠΎΠ½Ρ‡ΠΈ Π²Ρ‹Π²ΠΎΠ΄Ρ‹ ΡƒΡ‡Π΅Π½ΠΈΠΊΠΎΠ². Π§Π΅ΠΌΡƒ Ρ€Π°Π²Π΅Π½ ΠΎΠ±Ρ‰ΠΈΠΉ вСс бруска ΠΈ Π³ΠΈΡ€ΡŒ Π² ΠΎΠ±ΠΎΠΈΡ… случаях? Π§Π΅ΠΌΡƒ Ρ€Π°Π²Π΅Π½ коэффициСнт трСния Π΄Π΅Ρ€Π΅Π²Π° ΠΏΠΎ Π΄Π΅Ρ€Π΅Π²Ρƒ? Π§Π΅ΠΌΡƒ Ρ€Π°Π²Π΅Π½ коэффициСнт трСния ΠΌΠ΅Ρ‚Π°Π»Π»Π° ΠΏΠΎ Π΄Π΅Ρ€Π΅Π²Ρƒ? ВСс бруска Ρ€Π°Π²Π΅Π½ 2,2 Н, Π° вСс ΠΎΠ΄Π½ΠΎΠΉ Π³ΠΈΡ€ΠΈ Ρ€Π°Π²Π΅Π½ 1,3 Н. β„– ΠŸΠΎΠ²Π΅Ρ€Ρ…Π½ΠΎΡΡ‚ΡŒ бруска 1. Π΄Π΅Ρ€Π΅Π²ΠΎ; ΠΌΠ΅Ρ‚Π°Π»Π» ; Число Π³ΠΈΡ€ΡŒ Π½Π° брускС Π΄Π΅Ρ€Π΅Π²ΠΎ; 2 ΠΌΠ΅Ρ‚Π°Π»Π» 3 ΠžΠ±Ρ‰ΠΈΠΉ вСс, Н Π΄Π΅Ρ€Π΅Π²ΠΎ: ? ΠΌΠ΅Ρ‚Π°Π»Π»: ? Π‘ΠΈΠ»Π° трСния, Н Π΄Π΅Ρ€Π΅Π²ΠΎ: 0,7 ΠΌΠ΅Ρ‚Π°Π»Π»: 2,3 ΠšΠΎΡΡ„Ρ„ΠΈΡ†ΠΈΠ΅Π½Ρ‚ трСния Π΄Π΅Ρ€Π΅Π²ΠΎ : ? ΠΌΠ΅Ρ‚Π°Π»Π» : ? ΠžΠ±Ρ‰ΠΈΠΉ вСс ΠΎΠΊΡ€ΡƒΠ³Π»ΠΈ Π΄ΠΎ дСсятых. Π—Π½Π°Ρ‡Π΅Π½ΠΈΠ΅ коэффициСнта трСния ΠΎΠΊΡ€ΡƒΠ³Π»ΠΈ Π΄ΠΎ сотых. Π—Π°ΠΊΠΎΠ½Ρ‡ΠΈ Π²Ρ‹Π²ΠΎΠ΄ ΡƒΡ‡Π΅Π½ΠΈΠΊΠΎΠ². Π’Ρ‹Π²ΠΎΠ΄: коэффициСнт трСния ΠΌΠ΅Ρ‚Π°Π»Π»Π° ΠΏΠΎ Π΄Π΅Ρ€Π΅Π²Ρƒ Π² ? Ρ€Π°Π·Π° большС, Ρ‡Π΅ΠΌ коэффициСнт трСния Π΄Π΅Ρ€Π΅Π²Π° ΠΏΠΎ Π΄Π΅Ρ€Π΅Π²Ρƒ.

Π£ΠΊΠ°ΠΆΠΈΡ‚Π΅ ваш Π»ΠΎΠ³ΠΈΠ½ ΠΈ ΠΏΠ°Ρ€ΠΎΠ»ΡŒ, Ссли Π²Ρ‹ ΡƒΠΆΠ΅ зарСгистрированы Π½Π° tutoronline.ru

ΠΠ΅ΠΏΡ€Π°Π²ΠΈΠ»ΡŒΠ½Ρ‹ΠΉ Π»ΠΎΠ³ΠΈΠ½ ΠΈΠ»ΠΈ ΠΏΠ°Ρ€ΠΎΠ»ΡŒ.

Π£ΠΊΠ°ΠΆΠΈΡ‚Π΅ элСктронный адрСс ΠΈ ΠΏΠ°Ρ€ΠΎΠ»ΡŒ.

Π—Π°Π±Ρ‹Π»ΠΈ ΠΏΠ°Ρ€ΠΎΠ»ΡŒ?

Π’ΠΎΠΉΡ‚ΠΈ

ΠΈΠ»ΠΈ Π²ΠΎΠΉΡ‚ΠΈ с ΠΏΠΎΠΌΠΎΡ‰ΡŒΡŽ

Π•Ρ‰Ρ‘ Π½Π΅Ρ‚ Π°ΠΊΠΊΠ°ΡƒΠ½Ρ‚Π°? Π—Π°Ρ€Π΅Π³ΠΈΡΡ‚Ρ€ΠΈΡ€ΠΎΠ²Π°Ρ‚ΡŒΡΡ

ΠŸΠΎΠΆΠ°Π»ΡƒΠΉΡΡ‚Π°, ΡƒΠΊΠ°ΠΆΠΈΡ‚Π΅ элСктронный адрСс ΠΈΠ»ΠΈ Π½ΠΎΠΌΠ΅Ρ€ Ρ‚Π΅Π»Π΅Ρ„ΠΎΠ½Π°, ΠΊΠΎΡ‚ΠΎΡ€Ρ‹ΠΉ Π²Ρ‹ использовали ΠΏΡ€ΠΈ рСгистрации. Π’Π°ΠΌ Π±ΡƒΠ΄Π΅Ρ‚ ΠΎΡ‚ΠΏΡ€Π°Π²Π»Π΅Π½ΠΎ письмо со ссылкой Π½Π° Ρ„ΠΎΡ€ΠΌΡƒ измСнСния пароля ΠΈΠ»ΠΈ SMS сообщСниС с Π½ΠΎΠ²Ρ‹ΠΌ ΠΏΠ°Ρ€ΠΎΠ»Π΅ΠΌ.

Π’ΠΎΡΡΡ‚Π°Π½ΠΎΠ²ΠΈΡ‚ΡŒ

ΠžΡ‚ΠΌΠ΅Π½Π°

Π‘ΠΈΠ»Π° трСния. Олимпиадная ΠΏΠΎΠ΄Π³ΠΎΡ‚ΠΎΠ²ΠΊΠ°, 9 класс

Π‘ΡƒΠ΄Π΅ΠΌ ΠΈΡΠΊΠ°Ρ‚ΡŒ силы натяТСния стСрТнСй ΠΏΡ€ΠΈ скольТСнии, ускорСния досок, Π²Ρ€Π°Ρ‰Π°Ρ‚ΡŒ диски с Π³Ρ€ΡƒΠ·ΠΈΠΊΠ°ΠΌΠΈ Π½Π° них…

Π—Π°Π΄Π°Ρ‡Π° 1. На Π³Π»Π°Π΄ΠΊΠΎΠΉ Π³ΠΎΡ€ΠΈΠ·ΠΎΠ½Ρ‚Π°Π»ΡŒΠ½ΠΎΠΉ повСрхности стола Π»Π΅ΠΆΠΈΡ‚ доска массой , ΠΏΠΎ ΠΊΠΎΡ‚ΠΎΡ€ΠΎΠΉ ΡΠΊΠΎΠ»ΡŒΠ·ΠΈΡ‚ брусок массой . Π§Π΅ΠΌΡƒ Ρ€Π°Π²Π½ΠΎ ускорСниС доски ΠΏΡ€ΠΈ Π΅Ρ‘ Π΄Π²ΠΈΠΆΠ΅Π½ΠΈΠΈ вдоль стола, Ссли , Π° коэффициСнт трСния бруска ΠΎ доску Ρ€Π°Π²Π΅Π½ ? ΠžΡ‚Π²Π΅Ρ‚ Π²Ρ‹Ρ€Π°Π·ΠΈΡ‚ΡŒ Π² ΠΌ/, ΠΎΠΊΡ€ΡƒΠ³Π»ΠΈΠ² Π΄ΠΎ Ρ†Π΅Π»Ρ‹Ρ…. УскорСниС свободного падСния ΠΏΡ€ΠΈΠ½ΡΡ‚ΡŒ Ρ€Π°Π²Π½Ρ‹ΠΌ ΠΌ/.

К Π·Π°Π΄Π°Ρ‡Π΅ 1

РСшСниС.

Π’Π°ΠΊ ΠΊΠ°ΠΊ ΠΏΠΎ ΡƒΡΠ»ΠΎΠ²ΠΈΡŽ скольТСниС бруска ΠΏΠΎ доскС Π΅ΡΡ‚ΡŒ, Ρ‚ΠΎ сила трСния Π²Ρ‹ΡˆΠ»Π° Π½Π° максимум, ΠΈ ΠΏΠΎ Ρ‚Ρ€Π΅Ρ‚ΡŒΠ΅ΠΌΡƒ Π·Π°ΠΊΠΎΠ½Ρƒ ΠΡŒΡŽΡ‚ΠΎΠ½Π° доска разгоняСтся силой

Из Π²Ρ‚ΠΎΡ€ΠΎΠ³ΠΎ Π·Π°ΠΊΠΎΠ½Π° ΠΡŒΡŽΡ‚ΠΎΠ½Π° для доски ΠΏΠΎΠ»ΡƒΡ‡Π°Π΅ΠΌ, Ρ‡Ρ‚ΠΎ

Β  Β 

Β  ΠΎΡ‚ΠΊΡƒΠ΄Π° с ΡƒΡ‡Ρ‘Ρ‚ΠΎΠΌ , Π½Π°ΠΉΠ΄Ρ‘ΠΌ, Ρ‡Ρ‚ΠΎ ускорСниС доски Ρ€Π°Π²Π½ΠΎ

Β  Β 

ΠžΡ‚Π²Π΅Ρ‚: 5 ΠΌ/.

Β 

Π—Π°Π΄Π°Ρ‡Π° 2. Брусок Ρ‚ΠΎΠ»ΠΊΠ½ΡƒΠ»ΠΈ Π²Π²Π΅Ρ€Ρ… ΠΏΠΎ Π½Π°ΠΊΠ»ΠΎΠ½Π½ΠΎΠΉ плоскости, ΡΠΎΡΡ‚Π°Π²Π»ΡΡŽΡ‰Π΅ΠΉ с Π³ΠΎΡ€ΠΈΠ·ΠΎΠ½Ρ‚ΠΎΠΌ. Π§Π΅Ρ€Π΅Π· с брусок остановился, Π° Π΅Ρ‰Π΅ Ρ‡Π΅Ρ€Π΅Π· с вСрнулся Π² ΠΈΡΡ…ΠΎΠ΄Π½ΡƒΡŽ Ρ‚ΠΎΡ‡ΠΊΡƒ. Π§Π΅ΠΌΡƒ Ρ€Π°Π²Π΅Π½ коэффициСнт трСния? ΠžΡ‚Π²Π΅Ρ‚ ΠΎΠΊΡ€ΡƒΠ³Π»ΠΈΡ‚ΡŒ Π΄ΠΎ сотых.

Π—Π°Π΄Π°Ρ‡Π° 2. Π”Π²ΠΈΠΆΠ΅Π½ΠΈΠ΅ Π²Π²Π΅Ρ€Ρ….

Π—Π°Π΄Π°Ρ‡Π° 2. Π”Π²ΠΈΠΆΠ΅Π½ΠΈΠ΅ Π²Π½ΠΈΠ·.

РСшСниС.

Из Π²Ρ‚ΠΎΡ€ΠΎΠ³ΠΎ Π·Π°ΠΊΠΎΠ½Π° ΠΡŒΡŽΡ‚ΠΎΠ½Π° ΠΏΠΎΠ»ΡƒΡ‡Π°Π΅ΠΌ, Ρ‡Ρ‚ΠΎ

Β  Β 

Π—Π°ΠΏΠΈΡˆΠ΅ΠΌ Π²Ρ‚ΠΎΡ€ΠΎΠΉ Π·Π°ΠΊΠΎΠ½ ΠΡŒΡŽΡ‚ΠΎΠ½Π° Π² ΠΏΡ€ΠΎΠ΅ΠΊΡ†ΠΈΠΈ Π½Π° ось , Π½Π°ΠΏΡ€Π°Π²Π»Π΅Π½Π½ΡƒΡŽ Π²Π²Π΅Ρ€Ρ… вдоль плоскости ΠΏΡ€ΠΈ Π΄Π²ΠΈΠΆΠ΅Π½ΠΈΠΈ Π³Ρ€ΡƒΠ·Π° Π²Π²Π΅Ρ€Ρ…. ΠŸΠΎΠ»ΡƒΡ‡ΠΈΠΌ, Ρ‡Ρ‚ΠΎ

Β  Β 

Π³Π΄Π΅

ΠžΡ‚ΠΊΡƒΠ΄Π° проСкция ускорСния Ρ€Π°Π²Π½Π°

Β  Β 

ΠŸΡ€ΠΈ Π΄Π²ΠΈΠΆΠ΅Π½ΠΈΠΈ Π²Π½ΠΈΠ· Π½Π°ΠΏΡ€Π°Π²ΠΈΠΌ ось Ρ‚Π°ΠΊΠΈΠΌ ΠΆΠ΅ ΠΎΠ±Ρ€Π°Π·ΠΎΠΌ. ΠŸΡ€ΠΎΠ΅ΠΊΡ†ΠΈΡ ускорСния Ρ€Π°Π²Π½Π°

Β  Β 

ΠŸΠ΅Ρ€Π΅ΠΌΠ΅Ρ‰Π΅Π½ΠΈΠ΅ выраТаСтся ΠΏΠΎ Ρ„ΠΎΡ€ΠΌΡƒΠ»Π΅ , поэтому

Β  Β 

ΠŸΠΎΠ΄ΡΡ‚Π°Π²Π»ΡΡ выраТСния для ΠΈ ΠΈ Ρ€Π΅ΡˆΠ°Ρ ΡƒΡ€Π°Π²Π½Π΅Π½ΠΈΠ΅, ΠΏΠΎΠ»ΡƒΡ‡Π°Π΅ΠΌ коэффициСнт трСния, Ρ€Π°Π²Π½Ρ‹ΠΉ

Β  Β 

ΠžΡ‚Π²Π΅Ρ‚: 0,35.

Β 

Π—Π°Π΄Π°Ρ‡Π° 3. НаибольшСС Π·Π½Π°Ρ‡Π΅Π½ΠΈΠ΅ силы трСния покоя ΠΌΠ΅ΠΆΠ΄Ρƒ Π²Ρ€Π°Ρ‰Π°ΡŽΡ‰ΠΈΠΌΡΡ диском ΠΈ располоТСнным Π½Π° Π½Ρ‘ΠΌ Π³Ρ€ΡƒΠ·ΠΎΠΌ массой ΠΊΠ³ Ρ€Π°Π²Π½ΠΎ Н. На Π½Π΅ΠΊΠΎΡ‚ΠΎΡ€ΠΎΠΌ максимальном расстоянии ΠΎΡ‚ оси вращСния Π³Ρ€ΡƒΠ· Π±ΡƒΠ΄Π΅Ρ‚ ΡƒΠ΄Π΅Ρ€ΠΆΠΈΠ²Π°Ρ‚ΡŒΡΡ Π½Π° дискС, Π½Π΅ скользя ΠΏΠΎ Π½Π΅ΠΌΡƒ, Ссли диск станСт Π²Ρ€Π°Ρ‰Π°Ρ‚ΡŒΡΡ с частотой ΠΎΠ±/с? Π§Π΅ΠΌΡƒ Ρ€Π°Π²Π½Π° сила трСния Π³Ρ€ΡƒΠ·Π° ΠΎ диск Π² Ρ‚ΠΎΡ‚ ΠΌΠΎΠΌΠ΅Π½Ρ‚, ΠΊΠΎΠ³Π΄Π° Π³Ρ€ΡƒΠ· находится ΠΎΡ‚ оси вращСния Π½Π° ΠΏΠΎΠ»ΠΎΠ²ΠΈΠ½Π΅ Π½Π°ΠΉΠ΄Π΅Π½Π½ΠΎΠ³ΠΎ расстояния? ΠžΡ‚Π²Π΅Ρ‚ Π²Ρ‹Ρ€Π°Π·ΠΈΡ‚ΡŒ Π² Н, ΠΎΠΊΡ€ΡƒΠ³Π»ΠΈΠ² Π΄ΠΎ сотых.

РСшСниС.

На ΠΏΡ€Π΅Π΄Π΅Π»ΡŒΠ½ΠΎΠΌ расстоянии Π½Π° Ρ‚Π΅Π»ΠΎ Π΄Π΅ΠΉΡΡ‚Π²ΡƒΡŽΡ‚ силы тяТСсти, Ρ€Π΅Π°ΠΊΡ†ΠΈΠΈ ΠΎΠΏΠΎΡ€Ρ‹ ΠΈ сила трСния, ΡΠΎΠΎΠ±Ρ‰Π°ΡŽΡ‰ΠΈΠ΅ Π΅ΠΌΡƒ Ρ†Π΅Π½Ρ‚Ρ€ΠΎΡΡ‚Ρ€Π΅ΠΌΠΈΡ‚Π΅Π»ΡŒΠ½ΠΎΠ΅ ускорСниС. Из Π²Ρ‚ΠΎΡ€ΠΎΠ³ΠΎ Π·Π°ΠΊΠΎΠ½Π° ΠΡŒΡŽΡ‚ΠΎΠ½Π° Π² ΠΏΡ€ΠΎΠ΅ΠΊΡ†ΠΈΠΈ Π½Π° ось , Π½Π°ΠΏΡ€Π°Π²Π»Π΅Π½Π½ΡƒΡŽ ΠΏΠΎ радиусу ΠΊ Ρ†Π΅Π½Ρ‚Ρ€Ρƒ Π²Ρ€Π°Ρ‰Π°ΡŽΡ‰Π΅Π³ΠΎΡΡ диска, ΠΏΠΎΠ»ΡƒΡ‡Π°Π΅ΠΌ, Ρ‡Ρ‚ΠΎ

Β  Β 

Π³Π΄Π΅ , Ρ‚ΠΎΠ³Π΄Π°

Β  Β 

Π’Π°ΠΊΠΈΠΌ ΠΎΠ±Ρ€Π°Π·ΠΎΠΌ, максимальноС расстояниС ΠΎΡ‚ оси вращСния, Π½Π° ΠΊΠΎΡ‚ΠΎΡ€ΠΎΠΌ Π³Ρ€ΡƒΠ· Π±ΡƒΠ΄Π΅Ρ‚ ΡƒΠ΄Π΅Ρ€ΠΆΠΈΠ²Π°Ρ‚ΡŒΡΡ Π½Π° дискС, выраТаСтся ΠΏΠΎ Ρ„ΠΎΡ€ΠΌΡƒΠ»Π΅

Β  Β 

Когда Π³Ρ€ΡƒΠ· находится ΠΎΡ‚ оси вращСния Π½Π° ΠΏΠΎΠ»ΠΎΠ²ΠΈΠ½Π΅ Π½Π°ΠΉΠ΄Π΅Π½Π½ΠΎΠ³ΠΎ расстояния, Ρ†Π΅Π½Ρ‚Ρ€ΠΎΡΡ‚Ρ€Π΅ΠΌΠΈΡ‚Π΅Π»ΡŒΠ½ΠΎΠ΅ ускорСниС Π΅ΠΌΡƒ обСспСчиваСт сила трСния, равная

Β  Β 

ΠžΡ‚Π²Π΅Ρ‚: 12,25 Н.

Π—Π°Π΄Π°Ρ‡Π° 4. НСподвиТный ΠΊΠ»ΠΈΠ½ с ΡƒΠ³Π»ΠΎΠΌ ΠΏΡ€ΠΈ основании ΠΈΠΌΠ΅Π΅Ρ‚ Π³Π»Π°Π΄ΠΊΡƒΡŽ ниТнюю ΠΈ ΡˆΠ΅Ρ€ΠΎΡ…ΠΎΠ²Π°Ρ‚ΡƒΡŽ Π²Π΅Ρ€Ρ…Π½ΡŽΡŽ части своСй Π½Π°ΠΊΠ»ΠΎΠ½Π½ΠΎΠΉ плоскости. На Π²Π΅Ρ€Ρ…Π½Π΅ΠΉ части ΠΊΠ»ΠΈΠ½Π° ΡƒΠ΄Π΅Ρ€ΠΆΠΈΠ²Π°ΡŽΡ‚ Ρ‚ΠΎΠ½ΠΊΠΈΠΉ ΠΎΠ΄Π½ΠΎΡ€ΠΎΠ΄Π½Ρ‹ΠΉ Тёсткий ΡΡ‚Π΅Ρ€ΠΆΠ΅Π½ΡŒ массой ΠΊΠ³, располоТСнный Π² плоскости рисунка. ΠšΠΎΡΡ„Ρ„ΠΈΡ†ΠΈΠ΅Π½Ρ‚ трСния ΠΌΠ΅ΠΆΠ΄Ρƒ стСрТнСм ΠΈ Π²Π΅Ρ€Ρ…Π½Π΅ΠΉ Ρ‡Π°ΡΡ‚ΡŒΡŽ ΠΊΠ»ΠΈΠ½Π° Ρ€Π°Π²Π΅Π½ . ПослС Ρ‚ΠΎΠ³ΠΎ ΠΊΠ°ΠΊ ΡΡ‚Π΅Ρ€ΠΆΠ΅Π½ΡŒ ΠΎΡ‚ΠΏΡƒΡΠΊΠ°ΡŽΡ‚, ΠΎΠ½ Π½Π°Ρ‡ΠΈΠ½Π°Π΅Ρ‚ ΠΏΠΎΡΡ‚ΡƒΠΏΠ°Ρ‚Π΅Π»ΡŒΠ½ΠΎ ΡΠΊΠΎΠ»ΡŒΠ·ΠΈΡ‚ΡŒ ΠΏΠΎ ΠΊΠ»ΠΈΠ½Ρƒ. НайдитС максимальноС Π·Π½Π°Ρ‡Π΅Π½ΠΈΠ΅ силы натяТСния стСрТня Π² процСссС Π΅Π³ΠΎ двиТСния. ВлияниСм Π²ΠΎΠ·Π΄ΡƒΡ…Π° ΠΏΡ€Π΅Π½Π΅Π±Ρ€Π΅Ρ‡ΡŒ. ΠžΡ‚Π²Π΅Ρ‚ Π²Ρ‹Ρ€Π°Π·ΠΈΡ‚ΡŒ Π² Н, ΠΎΠΊΡ€ΡƒΠ³Π»ΠΈΠ² Π΄ΠΎ Ρ†Π΅Π»Ρ‹Ρ…. УскорСниС свободного падСния ΠΏΡ€ΠΈΠ½ΡΡ‚ΡŒ Ρ€Π°Π²Π½Ρ‹ΠΌ ΠΌ/

К Π·Π°Π΄Π°Ρ‡Π΅ 4

РСшСниС.

Π’Π²Π΅Π΄Ρ‘ΠΌ ось которая ΠΏΠ°Ρ€Π°Π»Π»Π΅Π»ΡŒΠ½Π° ΡΡ‚Π΅Ρ€ΠΆΠ½ΡŽ ΠΈ Π½Π°ΠΏΡ€Π°Π²Π»Π΅Π½Π° Π²Π½ΠΈΠ· ΠΏΠΎ ΠΊΠ»ΠΈΠ½Ρƒ. Π’ Ρ‚ΠΎΡ‚ ΠΌΠΎΠΌΠ΅Π½Ρ‚, ΠΊΠΎΠ³Π΄Π° Π½Π° Π³Π»Π°Π΄ΠΊΠΎΠΉ повСрхности ΠΊΠ»ΠΈΠ½Π° оказываСтся Ρ‡Π°ΡΡ‚ΡŒ стСрТня массой , Π³Π΄Π΅ , Π½Π° Π½Π΅Ρ‘ вдоль оси дСйствуСт ΡΠΎΡΡ‚Π°Π²Π»ΡΡŽΡ‰Π°Ρ силы тяТСсти ΠΈ сила натяТСния со стороны Π²Π΅Ρ€Ρ…Π½Π΅ΠΉ части стСрТня, направлСнная ΠΏΡ€ΠΎΡ‚ΠΈΠ²ΠΎΠΏΠΎΠ»ΠΎΠΆΠ½ΠΎ оси , ΠΈ равная . Π—Π°ΠΏΠΈΡˆΠ΅ΠΌ ΡƒΡ€Π°Π²Π½Π΅Π½ΠΈΠ΅ двиТСния Π½ΠΈΠΆΠ½Π΅ΠΉ части стСрТня вдоль оси . ΠŸΠΎΠ»ΡƒΡ‡ΠΈΠΌ, Ρ‡Ρ‚ΠΎ

Β  Β 

Π³Π΄Π΅ – ускорСниС любой Ρ‚ΠΎΡ‡ΠΊΠΈ стСрТня вдоль оси , Ρ‚Π°ΠΊ ΠΊΠ°ΠΊ ΡΡ‚Π΅Ρ€ΠΆΠ΅Π½ΡŒ Ρ‚Π²Ρ‘Ρ€Π΄Ρ‹ΠΉ ΠΈ двиТСтся ΠΏΠΎΡΡ‚ΡƒΠΏΠ°Ρ‚Π΅Π»ΡŒΠ½ΠΎ.

Π’ рассматриваСмый ΠΌΠΎΠΌΠ΅Π½Ρ‚ Π½Π° Π²Π΅Ρ€Ρ…Π½ΡŽΡŽ Ρ‡Π°ΡΡ‚ΡŒ стСрТня вдоль оси наряду с ΡΠΎΡΡ‚Π°Π²Π»ΡΡŽΡ‰Π΅ΠΉ силы тяТСсти со стороны ΠΊΠ»ΠΈΠ½Π° дСйствуСт направлСнная ΠΏΡ€ΠΎΡ‚ΠΈΠ²ΠΎΠΏΠΎΠ»ΠΎΠΆΠ½ΠΎ оси сила сухого трСния скольТСния , Π° со стороны Π½ΠΈΠΆΠ½Π΅ΠΉ части стСрТня дСйствуСт направлСнная вдоль оси сила натяТСния . Π£Ρ€Π°Π²Π½Π΅Π½ΠΈΠ΅ двиТСния этой части стСрТня Π² ΠΏΡ€ΠΎΠ΅ΠΊΡ†ΠΈΠΈ Π½Π° ось ΠΈΠΌΠ΅Π΅Ρ‚ Π²ΠΈΠ΄

Β  Β 

РСшая совмСстно составлСнныС уравнСния, ΠΏΠΎΠ»ΡƒΡ‡Π°Π΅ΠΌ

Β  Β 

Π’ΠΈΠ΄Π½ΠΎ, Ρ‡Ρ‚ΠΎ сила натяТСния стСрТня Π² сСчСнии, ΠΊΠΎΡ‚ΠΎΡ€ΠΎΠ΅ находится Π½Π° Π³Ρ€Π°Π½ΠΈΡ†Π΅ ΠΌΠ΅ΠΆΠ΄Ρƒ Π³Π»Π°Π΄ΠΊΠΎΠΉ ΠΈ ΡˆΠ΅Ρ€ΠΎΡ…ΠΎΠ²Π°Ρ‚ΠΎΠΉ частями ΠΊΠ»ΠΈΠ½Π°, зависит ΠΎΡ‚ значСния коэффициСнта .Она Π±ΡƒΠ΄Π΅Ρ‚ максимальной ΠΏΡ€ΠΈ , Ρ‚.Π΅. ΠΊΠΎΠ³Π΄Π° ΠΎΠ΄Π½Π° ΠΏΠΎΠ»ΠΎΠ²ΠΈΠ½Π° стСрТня окаТСтся Π½Π° Π³Π»Π°Π΄ΠΊΠΎΠΉ Π½ΠΈΠΆΠ½Π΅ΠΉ части ΠΊΠ»ΠΈΠ½Π°, Π° другая ΠΏΠΎΠ»ΠΎΠ²ΠΈΠ½Π° – Π½Π° Π΅Π³ΠΎ ΡˆΠ΅Ρ€ΠΎΡ…ΠΎΠ²Π°Ρ‚ΠΎΠΉ Π²Π΅Ρ€Ρ…Π½Π΅ΠΉ части. Π’ Ρ‚Π°ΠΊΠΎΠΌ случаС

Β  Β 

ΠžΡ‚Π²Π΅Ρ‚: 1 Н.

Π—Π°Π΄Π°Ρ‡Π° 5. Π’Π΅Π»ΠΎ Ρ€Π°Π²Π½ΠΎΠΌΠ΅Ρ€Π½ΠΎ ΡΠΊΠΎΠ»ΡŒΠ·ΠΈΡ‚ ΠΏΠΎ Π½Π°ΠΊΠ»ΠΎΠ½Π½ΠΎΠΉ плоскости с ΡƒΠ³Π»ΠΎΠΌ Π½Π°ΠΊΠ»ΠΎΠ½Π° . ΠžΠΏΡ€Π΅Π΄Π΅Π»ΠΈΡ‚ΡŒ коэффициСнт трСния Ρ‚Π΅Π»Π° ΠΎ ΠΏΠ»ΠΎΡΠΊΠΎΡΡ‚ΡŒ. ΠžΡ‚Π²Π΅Ρ‚ ΠΎΠΊΡ€ΡƒΠ³Π»ΠΈΡ‚ΡŒ Π΄ΠΎ сотых.

РСшСниС.

Π’Π°ΠΊ ΠΊΠ°ΠΊ Ρ‚Π΅Π»ΠΎ ΡΠΊΠΎΠ»ΡŒΠ·ΠΈΡ‚ Ρ€Π°Π²Π½ΠΎΠΌΠ΅Ρ€Π½ΠΎ, вСкторная сумма сил, Π΄Π΅ΠΉΡΡ‚Π²ΡƒΡŽΡ‰ΠΈΡ… Π½Π° Π½Π΅Π³ΠΎ, Ρ€Π°Π²Π½Π° Π½ΡƒΠ»ΡŽ. МоТно Π΄Π΅ΠΉΡΡ‚Π²ΡƒΡŽΡ‰ΠΈΠ΅ Π½Π° Ρ‚Π΅Π»ΠΎ силы , , , ΠΈΠ·ΠΎΠ±Ρ€Π°Π·ΠΈΡ‚ΡŒ Π² Π²ΠΈΠ΄Π΅ Π²Π΅ΠΊΡ‚ΠΎΡ€Π½ΠΎΠ³ΠΎ Ρ‚Ρ€Π΅ΡƒΠ³ΠΎΠ»ΡŒΠ½ΠΈΠΊΠ°. Из ΠΊΠΎΡ‚ΠΎΡ€ΠΎΠ³ΠΎ

Β  Β 

ΠžΡ‚Π²Π΅Ρ‚: 0,84.

Β 

Π‘ΠΈΠ»Π° трСния – тСст β„–2

Π‘ΠΈΠ»Π° трСния – тСст β„–2
1. Π§Π΅ΠΌΡƒ Ρ€Π°Π²Π΅Π½ ΠΌΠΎΠ΄ΡƒΠ»ΡŒ силы трСния автомобиля массой 1 Ρ‚ ΠΏΡ€ΠΈ Ρ‚ΠΎΡ€ΠΌΠΎΠΆΠ΅Π½ΠΈΠΈ Π½Π° Π³ΠΎΡ€ΠΈΠ·ΠΎΠ½Ρ‚Π°Π»ΡŒΠ½ΠΎΠΉ повСрхности, Ссли коэффициСнт трСния ΠΎΠ± Π°ΡΡ„Π°Π»ΡŒΡ‚ Ρ€Π°Π²Π΅Π½ 0,3? Π‘ΠΎΠΏΡ€ΠΎΡ‚ΠΈΠ²Π»Π΅Π½ΠΈΠ΅ΠΌ Π²ΠΎΠ·Π΄ΡƒΡ…Π° ΠΏΡ€Π΅Π½Π΅Π±Ρ€Π΅Ρ‡ΡŒ. 1) 3000 Н 2) 1000 Н 3) 300 Н 4) 30 Н 5) 3 Н

2. Π‘Π°Π½ΠΈ со ΡΡ‚Π°Π»ΡŒΠ½Ρ‹ΠΌΠΈ полозьями ΠΏΠΎΠ΄ дСйствиСм силы Π² 20 Н, Π½Π°ΠΏΡ€Π°Π²Π»Π΅Π½Π½ΠΎΠΉ ΠΏΠΎΠ΄ ΡƒΠ³Π»ΠΎΠΌ 60Β° ΠΊ Π³ΠΎΡ€ΠΈΠ·ΠΎΠ½Ρ‚Ρƒ, ΠΏΠ΅Ρ€Π΅ΠΌΠ΅Ρ‰Π°ΡŽΡ‚ ΠΏΠΎ Π·ΠΈΠΌΠ½Π΅ΠΉ Π΄ΠΎΡ€ΠΎΠ³Π΅ Ρ€Π°Π²Π½ΠΎΠΌΠ΅Ρ€Π½ΠΎ. Если коэффициСнт трСния стали ΠΎ Π΄ΠΎΡ€ΠΎΠ³Ρƒ 0,2, Ρ‚ΠΎ масса санСй Ρ€Π°Π²Π½Π° 1) 5ΠΊΠ³ 2) 8 ΠΊΠ³ 3) 9ΠΊΠ³ 4) 10 ΠΊΠ³ 5) 6,7 ΠΊΠ³

3. На Π³ΠΎΡ€ΠΈΠ·ΠΎΠ½Ρ‚Π°Π»ΡŒΠ½ΠΎΠΉ повСрхности находится Ρ‚Π΅Π»ΠΎ массой 2ΠΊΠ³ ΠΈ Π½Π° Π½Π΅Π³ΠΎ дСйствуСт Π³ΠΎΡ€ΠΈΠ·ΠΎΠ½Ρ‚Π°Π»ΡŒΠ½ΠΎ направлСнная сила, Π²Π΅Π»ΠΈΡ‡ΠΈΠ½Π° ΠΊΠΎΡ‚ΠΎΡ€ΠΎΠΉ 10Н. Под дСйствиСм этой силы, Ρ‚Π΅Π»ΠΎ пСрСмСщаСтся ΠΏΠΎ Π·Π°ΠΊΠΎΠ½Ρƒ: S=7,5t+2,5t2(ΠΌ). ΠžΠΏΡ€Π΅Π΄Π΅Π»ΠΈΡ‚ΡŒ коэффициСнт трСния ΠΌΠ΅ΠΆΠ΄Ρƒ Ρ‚Π΅Π»ΠΎΠΌ ΠΈ этой ΠΏΠΎΠ²Π΅Ρ€Ρ…Π½ΠΎΡΡ‚ΡŒΡŽ 1) 0,4 2) 0,1 3) 0,01 4) 4 5) 0

4. На Ρ‚Π΅Π»ΠΎ массой 40ΠΊΠ³, дСйствуСт сила Π²Π΅Π»ΠΈΡ‡ΠΈΠ½ΠΎΠΉ 40Н, направлСнная ΠΏΠΎΠ΄ ΡƒΠ³Π»ΠΎΠΌ 60Β° ΠΊ Π³ΠΎΡ€ΠΈΠ·ΠΎΠ½Ρ‚Ρƒ. Под дСйствиСм этой силы Ρ‚Π΅Π»ΠΎ двиТСтся со ΡΠΊΠΎΡ€ΠΎΡΡ‚ΡŒΡŽ Ρ‚Π°ΠΊ, ΠΊΠ°ΠΊ ΠΏΠΎΠΊΠ°Π·Π°Π½ΠΎ Π½Π° рисункС. ΠžΠΏΡ€Π΅Π΄Π΅Π»ΠΈΡ‚ΡŒ Π²Π΅Π»ΠΈΡ‡ΠΈΠ½Ρƒ силы трСния.


1) 5Н 2) 20Н 3) 10Н 4) 40Н 5) 30Н

5. Π’Π΅Π»ΠΎ ΠΌΠΎΠΆΠ΅Ρ‚ Π΄Π²ΠΈΠ³Π°Ρ‚ΡŒΡΡ ΠΏΠΎ Π³ΠΎΡ€ΠΈΠ·ΠΎΠ½Ρ‚Π°Π»ΡŒΠ½ΠΎΠΉ повСрхности ΠΏΠΎΠ΄ дСйствиСм ΠΎΠ΄ΠΈΠ½Π°ΠΊΠΎΠ²Ρ‹Ρ… ΠΏΠΎ ΠΌΠΎΠ΄ΡƒΠ»ΡŽ сил, направлСния ΠΊΠΎΡ‚ΠΎΡ€Ρ‹Ρ… ΠΏΠΎΠΊΠ°Π·Π°Π½Ρ‹ Π½Π° рисункС. Под дСйствиСм ΠΊΠ°ΠΊΠΎΠΉ ΠΈΠ· Π½ΠΈΠΆΠ΅ΠΏΡ€ΠΈΠ²Π΅Π΄Π΅Π½Π½Ρ‹Ρ… сил, сила трСния Π΄Π΅ΠΉΡΡ‚Π²ΡƒΡŽΡ‰Π°Ρ Π½Π° это Ρ‚Π΅Π»ΠΎ Π±ΡƒΠ΄Π΅Ρ‚ максимальной?


1) F1 2) F5 3) F3 4) F4 5) F2

6. ΠŸΡ€ΠΎΠ΅ΠΊΡ†ΠΈΡ скорости Ρ‚Π΅Π»Π° ΠΏΠΎΠ΄ дСйствиСм внСшнСй силы измСняСтся с Ρ‚Π΅Ρ‡Π΅Π½ΠΈΠ΅ΠΌ Π²Ρ€Π΅ΠΌΠ΅Π½ΠΈ Ρ‚Π°ΠΊ, ΠΊΠ°ΠΊ ΠΏΠΎΠΊΠ°Π·Π°Π½ΠΎ Π½Π° рисункС. Π’ ΠΊΠ°ΠΊΠΎΠΉ ΠΈΠ»ΠΈ ΠΊΠ°ΠΊΠΈΠ΅ ΠΈΠ· Π½ΠΈΠΆΠ΅ΡƒΠΊΠ°Π·Π°Π½Π½Ρ‹Ρ… ΠΏΡ€ΠΎΠΌΠ΅ΠΆΡƒΡ‚ΠΊΠΎΠ² Π²Ρ€Π΅ΠΌΠ΅Π½ΠΈ, сила трСния Π±ΡƒΠ΄Π΅Ρ‚ Ρ€Π°Π²Π½Π° внСшнСй силС?


1) (t1; t2) 2) (t3; t4) 3) (0; t1)4) (t1; t2) ΠΈ (t3; t4) 5) (0; t1) ΠΈ (t2; t3)

7. На рисункС ΠΏΡ€ΠΈΠ²Π΅Π΄Π΅Π½Π° Π·Π°Π²ΠΈΡΠΈΠΌΠΎΡΡ‚ΡŒ силы трСния ΠΎΡ‚ Π²Π΅Π»ΠΈΡ‡ΠΈΠ½Ρ‹ Ρ€Π΅Π°ΠΊΡ†ΠΈΠΈ ΠΎΠΏΠΎΡ€Ρ‹ для Ρ‚Ρ€Π΅Ρ… Ρ‚Π΅Π» Π΄Π²ΠΈΠ³Π°ΡŽΡ‰ΠΈΡ…ΡΡ ΠΏΠΎ Ρ‚Ρ€Π΅ΠΌ Ρ€Π°Π·Π»ΠΈΡ‡Π½Ρ‹ΠΌ Π³ΠΎΡ€ΠΈΠ·ΠΎΠ½Ρ‚Π°Π»ΡŒΠ½Ρ‹ΠΌ повСрхностям.. Π’ ΠΊΠ°ΠΊΠΎΠΌ ΠΈΠ· Π½ΠΈΠΆΠ΅ΠΏΡ€ΠΈΠ²Π΅Π΄Π΅Π½Π½Ρ‹Ρ… ΡΠΎΠΎΡ‚Π½ΠΎΡˆΠ΅Π½ΠΈΠΉ находятся ΠΌΠ΅ΠΆΠ΄Ρƒ собой коэффициСнты трСния скольТСния?


1) 2) 3)
4) 5)

8. Какой ΠΈΠ· Π½ΠΈΠΆΠ΅ΠΏΡ€ΠΈΠ²Π΅Π΄Π΅Π½Π½Ρ‹Ρ… Π³Ρ€Π°Ρ„ΠΈΠΊΠΎΠ² ΠΎΡ‚Ρ€Π°ΠΆΠ°Π΅Ρ‚ Π·Π°Π²ΠΈΡΠΈΠΌΠΎΡΡ‚ΡŒ коэффициСнта трСния ΠΎΡ‚ Π²Π΅Π»ΠΈΡ‡ΠΈΠ½Ρ‹ силы трСния?


1) 2) 3) 4) 5)

9. КакоС ΠΈΠ· Π½ΠΈΠΆΠ΅ΠΏΡ€ΠΈΠ²Π΅Π΄Π΅Π½Π½Ρ‹Ρ… ΡƒΡ‚Π²Π΅Ρ€ΠΆΠ΄Π΅Π½ΠΈΠΉ справСдливо?


1) ΠšΠΎΡΡ„Ρ„ΠΈΡ†ΠΈΠ΅Π½Ρ‚ трСния скольТСния зависит ΠΎΡ‚ ΠΏΠ»ΠΎΡ‰Π°Π΄ΠΈ ΡΠΎΠΏΡ€ΠΈΠΊΠ°ΡΠ°ΡŽΡ‰ΠΈΡ…ΡΡ Ρ‚Π΅Π».
2) ΠšΠΎΡΡ„Ρ„ΠΈΡ†ΠΈΠ΅Π½Ρ‚ трСния скольТСния зависит ΠΎΡ‚ Π²Π΅Π»ΠΈΡ‡ΠΈΠ½Ρ‹ силы Π½ΠΎΡ€ΠΌΠ°Π»ΡŒΠ½ΠΎΠ³ΠΎ давлСния.
3) ΠšΠΎΡΡ„Ρ„ΠΈΡ†ΠΈΠ΅Π½Ρ‚ трСния скольТСния зависит ΠΎΡ‚ Π²Π΅Π»ΠΈΡ‡ΠΈΠ½Ρ‹ силы трСния.
4) ΠšΠΎΡΡ„Ρ„ΠΈΡ†ΠΈΠ΅Π½Ρ‚ трСния скольТСния зависит ΠΎΡ‚ массы Ρ‚Π΅Π»Π°.
5) ΠšΠΎΡΡ„Ρ„ΠΈΡ†ΠΈΠ΅Π½Ρ‚ трСния скольТСния зависит ΠΎΡ‚ Ρ€ΠΎΠ΄Π° ΡΠΎΠΏΡ€ΠΈΠΊΠ°ΡΠ°ΡŽΡ‰ΠΈΡ…ΡΡ вСщСств.

13. Π’Π΅Π»ΠΎ ΡΠΊΠΎΠ»ΡŒΠ·ΠΈΡ‚ Ρ€Π°Π²Π½ΠΎΠΌΠ΅Ρ€Π½ΠΎ ΠΏΠΎ Π½Π°ΠΊΠ»ΠΎΠ½Π½ΠΎΠΉ плоскости с ΡƒΠ³Π»ΠΎΠΌ Π½Π°ΠΊΠ»ΠΎΠ½Π° Ο†. Π§Π΅ΠΌΡƒ Ρ€Π°Π²Π΅Π½ коэффициСнт трСния?


1) sin Ο†; 2) cos Ο†; 3) tg Ο†, 4) ctg Ο†, 5) срСди ΠΏΡ€Π΅Π΄Π»ΠΎΠΆΠ΅Π½Π½Ρ‹Ρ… ΠΎΡ‚Π²Π΅Ρ‚ΠΎΠ² Π½Π΅Ρ‚ ΠΏΡ€Π°Π²ΠΈΠ»ΡŒΠ½ΠΎΠ³ΠΎ

14. Π‘ лСдяной Π³ΠΎΡ€Ρ‹ высотой 1 ΠΌ ΠΈ основаниСм 3,873 ΠΌ ΡΡŠΠ΅Π·ΠΆΠ°ΡŽΡ‚ санки, ΠΊΠΎΡ‚ΠΎΡ€Ρ‹Π΅ ΠΎΡΡ‚Π°Π½Π°Π²Π»ΠΈΠ²Π°ΡŽΡ‚ΡΡ, пройдя Π³ΠΎΡ€ΠΈΠ·ΠΎΠ½Ρ‚Π°Π»ΡŒΠ½Ρ‹ΠΉ ΠΏΡƒΡ‚ΡŒ 96,13 ΠΌ. Найти коэффициСнт трСния, считая Π΅Π³ΠΎ ΠΎΠ΄ΠΈΠ½Π°ΠΊΠΎΠ²Ρ‹ΠΌ Π½Π° всСм ΠΏΡƒΡ‚ΠΈ, ΠΈ коэффициСнт ΠΏΠΎΠ»Π΅Π·Π½ΠΎΠ³ΠΎ дСйствия Π½Π°ΠΊΠ»ΠΎΠ½Π½ΠΎΠΉ плоскости


1) 0,01; 96% 2) 0,01; 90% 3) 0,025; 96% 4) 0,03; 96% 5) 0,015; 90%.

10. Π’Π΅Π»ΠΎ, постоянной массы двигаСтся ΠΏΠΎ Π³ΠΎΡ€ΠΈΠ·ΠΎΠ½Ρ‚Π°Π»ΡŒΠ½ΠΎΠΉ повСрхности. Какой ΠΈΠ· Π½ΠΈΠΆΠ΅ΠΏΡ€ΠΈΠ²Π΅Π΄Π΅Π½Π½Ρ‹Ρ… Π³Ρ€Π°Ρ„ΠΈΠΊΠΎΠ² ΠΎΡ‚Ρ€Π°ΠΆΠ°Π΅Ρ‚ Π·Π°Π²ΠΈΡΠΈΠΌΠΎΡΡ‚ΡŒ силы трСния ΠΎΡ‚ коэффициСнта трСния скольТСния?


1) 2) 3) 4) 5)

11. На рисункС прСдставлСн Π³Ρ€Π°Ρ„ΠΈΠΊ Ρ‚ΠΎΡ€ΠΌΠΎΠ·Π½ΠΎΠ³ΠΎ ΠΏΡƒΡ‚ΠΈ ΠΎΡ‚ Π²Π΅Π»ΠΈΡ‡ΠΈΠ½Ρ‹ Π½Π°Ρ‡Π°Π»ΡŒΠ½ΠΎΠΉ скорости. ΠžΠΏΡ€Π΅Π΄Π΅Π»ΠΈΡ‚ΡŒ коэффициСнт трСния скольТСния.


1) НСдостаточно ΠΈΠ½Ρ„ΠΎΡ€ΠΌΠ°Ρ†ΠΈΠΈ для ΠΎΡ‚Π²Π΅Ρ‚Π°
2) 0,1 3) 0,01 4) 0,2 5) 0,02

12. Π’Π΅Π»ΠΎ ΡΠΎΡΠΊΠ°Π»ΡŒΠ·Ρ‹Π²Π°Π΅Ρ‚ Π±Π΅Π· Π½Π°Ρ‡Π°Π»ΡŒΠ½ΠΎΠΉ скорости с Π½Π°ΠΊΠ»ΠΎΠ½Π½ΠΎΠΉ плоскости. Π£Π³ΠΎΠ» Π½Π°ΠΊΠ»ΠΎΠ½Π° плоскости ΠΊ Π³ΠΎΡ€ΠΈΠ·ΠΎΠ½Ρ‚Ρƒ 30Β°, Π΄Π»ΠΈΠ½Π° Π½Π°ΠΊΠ»ΠΎΠ½Π½ΠΎΠΉ плоскости 2 ΠΌ. ΠšΠΎΡΡ„Ρ„ΠΈΡ†ΠΈΠ΅Π½Ρ‚ трСния ΠΎ ΠΏΠ»ΠΎΡΠΊΠΎΡΡ‚ΡŒ 0,3. Каково ускорСниС Ρ‚Π΅Π»Π°? Бколько Π²Ρ€Π΅ΠΌΠ΅Π½ΠΈ длится ΡΠΎΡΠΊΠ°Π»ΡŒΠ·Ρ‹Π²Π°Π½ΠΈΠ΅?


1) 1,2 м/с2; 6,5с; 2)2,4 м/с2; 1,3с; 3) 3,6м/с2; 1с; 4) 2,4 м/с2; 2с; 5) 1,2 м/с2; 3,5с.

15. Π’Π΅Π»ΠΎ ΡΠΎΡΠΊΠ°Π»ΡŒΠ·Ρ‹Π²Π°Π΅Ρ‚ с Π½Π°ΠΊΠ»ΠΎΠ½Π½ΠΎΠΉ плоскости высотой 2 ΠΌ ΠΈ ΡƒΠ³Π»ΠΎΠΌ Π½Π°ΠΊΠ»ΠΎΠ½Π° 45Β°. ΠžΠΏΡ€Π΅Π΄Π΅Π»ΠΈΡ‚Π΅ коэффициСнт трСния ΠΌΠ΅ΠΆΠ΄Ρƒ Ρ‚Π΅Π»ΠΎΠΌ ΠΈ ΠΏΠ»ΠΎΡΠΊΠΎΡΡ‚ΡŒΡŽ, Ссли Ρƒ основания ΡΠΊΠΎΡ€ΠΎΡΡ‚ΡŒ Ρ‚Π΅Π»Π° 6 ΠΌ/с. Π§Π΅ΠΌΡƒ Ρ€Π°Π²Π΅Π½ ΠšΠŸΠ” Π½Π°ΠΊΠ»ΠΎΠ½Π½ΠΎΠΉ плоскости?


1) 0,012; 98%; 2) 0,082; 82%; 3) 0,05; 89%; 4) 0,082; 92%; 5) 0,012; 95%.

16. ПослС спуска с Π³ΠΎΡ€Ρ‹ сани Π½Π°Ρ‡ΠΈΠ½Π°ΡŽΡ‚ Π΄Π²ΠΈΠΆΠ΅Π½ΠΈΠ΅ ΠΏΠΎ Π³ΠΎΡ€ΠΈΠ·ΠΎΠ½Ρ‚Π°Π»ΡŒΠ½ΠΎΠΉ повСрхности со ΡΠΊΠΎΡ€ΠΎΡΡ‚ΡŒΡŽ 10ΠΌ/с, коэффициСнт трСния ΠΌΠ΅ΠΆΠ΄Ρƒ Π΄ΠΎΡ€ΠΎΠ³ΠΎΠΉ ΠΈ полозьями 0,1. Какой ΠΏΡƒΡ‚ΡŒ ΠΏΡ€ΠΎΠΉΠ΄ΡƒΡ‚ сани Π·Π° 15с? УскорСниС свободного падСния ΠΏΡ€ΠΈΠ½ΡΡ‚ΡŒ 10 ΠΌ/с2.


1) 37,5ΠΌ 2) 50ΠΌ 3) 100ΠΌ 4) 150ΠΌ 5) 75ΠΌ

17. На Π³ΠΎΡ€ΠΈΠ·ΠΎΠ½Ρ‚Π°Π»ΡŒΠ½ΠΎΠΌ ΠΊΡ€ΡƒΠ³Π΅, Π²Ρ€Π°Ρ‰Π°ΡŽΡ‰Π΅ΠΌΡΡ с частотой 60 ΠΎΠ±/ΠΌΠΈΠ½, Π½Π° расстоянии 5,1 см ΠΎΡ‚ оси вращСния ΠΏΠΎΠΌΠ΅Ρ‰Π΅Π½ ΠΏΡ€Π΅Π΄ΠΌΠ΅Ρ‚. Π§Ρ‚ΠΎΠ±Ρ‹ ΠΏΡ€Π΅Π΄ΠΌΠ΅Ρ‚ Π½Π΅ ΡΠΎΡΠΊΠ°Π»ΡŒΠ·Ρ‹Π²Π°Π» с ΠΊΡ€ΡƒΠ³Π°, коэффициСнт трСния Π΄ΠΎΠ»ΠΆΠ΅Π½ Π±Ρ‹Ρ‚ΡŒ Ρ€Π°Π²Π΅Π½ 1) 0,05 2) 0,1 3) 0,15 4) 0,2 5) 0,12


18. Π¨Π°ΠΉΠ±Π°, ΡΠΊΠΎΠ»ΡŒΠ·ΡΡ‰Π°Ρ со ΡΠΊΠΎΡ€ΠΎΡΡ‚ΡŒΡŽ 10 ΠΌ/с останавливаСтся Ρ‡Π΅Ρ€Π΅Π· 20 с. ΠšΠΎΡΡ„Ρ„ΠΈΡ†ΠΈΠ΅Π½Ρ‚ трСния ΡˆΠ°ΠΉΠ±Ρ‹ ΠΎ ΠΏΠΎΠ²Π΅Ρ€Ρ…Π½ΠΎΡΡ‚ΡŒ льда 1) 0,2 2) 0,1 3) 0,05 4) 0,02 5) 0,01
19. Π¨Π°ΠΉΠ±Π°, ΡΠΊΠΎΠ»ΡŒΠ·ΡΡ‰Π°Ρ со ΡΠΊΠΎΡ€ΠΎΡΡ‚ΡŒΡŽ 15 ΠΌ/с ΠΏΠΎ Π»ΡŒΠ΄Ρƒ с коэффициСнтом трСния 0,1, остановится Ρ‡Π΅Ρ€Π΅Π· врСмя 1) 2 с 2) 5 с 3) 10 с 4) 15 с 5) 20 с

20. Π¨Π°ΠΉΠ±Π°, ΡΠΊΠΎΠ»ΡŒΠ·ΡΡ‰Π°Ρ со ΡΠΊΠΎΡ€ΠΎΡΡ‚ΡŒΡŽ 20 ΠΌ/с ΠΏΠΎ Π»ΡŒΠ΄Ρƒ с коэффициСнтом трСния 0,2, остановится, пройдя ΠΏΡƒΡ‚ΡŒ 1) 37,5 ΠΌ 2) 50 ΠΌ 3) 100 ΠΌ 4) 150 ΠΌ 5) 75 ΠΌ

21. Π¨Π°ΠΉΠ±Π°, ΡΠΊΠΎΠ»ΡŒΠ·ΡΡ‰Π°Ρ ΠΏΠΎ Π»ΡŒΠ΄Ρƒ с коэффициСнтом трСния 0,01, ΠΎΡΡ‚Π°Π½ΠΎΠ²ΠΈΠ»Π°ΡΡŒ, пройдя ΠΏΡƒΡ‚ΡŒ 80 ΠΌ. ΠΠ°Ρ‡Π°Π»ΡŒΠ½Π°Ρ ΡΠΊΠΎΡ€ΠΎΡΡ‚ΡŒ ΡˆΠ°ΠΉΠ±Ρ‹ 1) 4 ΠΌ/с 2) 5 ΠΌ/с 3) 10 ΠΌ/с 4) 15 ΠΌ/с 5) 7,5 ΠΌ/с

22. На Ρ‚Π΅Π»ΠΎ массой 10 ΠΊΠ³, Π»Π΅ΠΆΠ°Ρ‰Π΅Π΅ Π½Π° Π³ΠΎΡ€ΠΈΠ·ΠΎΠ½Ρ‚Π°Π»ΡŒΠ½ΠΎΠΉ ΠΏΠ»ΠΎΡ‰Π°Π΄ΠΊΠ΅ с коэффициСнтом трСния 0,5, дСйствуСт Π³ΠΎΡ€ΠΈΠ·ΠΎΠ½Ρ‚Π°Π»ΡŒΠ½Π°Ρ сила 40 Н. Π§Π΅ΠΌΡƒ Ρ€Π°Π²Π½Π° сила трСния?


1) 100 Н 2) 80 Н 3) 50 Н 4) 40 Н 5) 20 Н

23. На Ρ‚Π΅Π»ΠΎ массой 10 ΠΊΠ³, Π»Π΅ΠΆΠ°Ρ‰Π΅Π΅ Π½Π° Π³ΠΎΡ€ΠΈΠ·ΠΎΠ½Ρ‚Π°Π»ΡŒΠ½ΠΎΠΉ ΠΏΠ»ΠΎΡ‰Π°Π΄ΠΊΠ΅ с коэффициСнтом трСния 0,5, дСйствуСт сила 40 Н ΠΏΠΎΠ΄ ΡƒΠ³Π»ΠΎΠΌ 60Β°ΠΊ Π³ΠΎΡ€ΠΈΠ·ΠΎΠ½Ρ‚Π°Π»ΠΈ. Π§Π΅ΠΌΡƒ Ρ€Π°Π²Π½Π° сила трСния?


1) 100 Н 2) 80 Н 3) 50 Н 4) 40 Н 5) 20 Н

24. ΠΠ²Ρ‚ΠΎΠΌΠΎΠ±ΠΈΠ»ΡŒ ΠΌΠΎΠΆΠ΅Ρ‚ Π΄Π²ΠΈΠ³Π°Ρ‚ΡŒΡΡ ΠΏΠΎ Π·Π°ΠΊΡ€ΡƒΠ³Π»Π΅Π½ΠΈΡŽ Π΄ΠΎΡ€ΠΎΠ³ΠΈ радиусом 9 ΠΌ с коэффициСнтом трСния 0,4 с максимальной ΡΠΊΠΎΡ€ΠΎΡΡ‚ΡŒΡŽ 1) 4 ΠΌ/с 2) 5 ΠΌ/с 3) 6 ΠΌ/с 4) 10 ΠΌ/с 5) 7,5 ΠΌ/с

25. Π‘Π°ΠΌΠΎΠ»Ρ‘Ρ‚ Π½Π° скорости 151,2 ΠΊΠΌ/Ρ‡ садится Π½Π° ΠΏΠ°Π»ΡƒΠ±Ρƒ авианосца Π΄Π»ΠΈΠ½ΠΎΠΉ 400 ΠΌ с коэффициСнтом трСния 0,2. Π‘ ΠΊΠ°ΠΊΠΎΠΉ ΡΠΊΠΎΡ€ΠΎΡΡ‚ΡŒΡŽ Π΄ΠΎΠ»ΠΆΠ΅Π½ Π΄Π²ΠΈΠ³Π°Ρ‚ΡŒΡΡ ΠΊΠΎΡ€Π°Π±Π»ΡŒ?
1) 4 м/с 2) 3 м/с 3) 2 м/с 4) 1 м/с 5) 0,5 м/с

Как ΠΏΠΎΡΡ‡ΠΈΡ‚Π°Ρ‚ΡŒ коэффициСнт трСния. Как ΠΎΠΏΡ€Π΅Π΄Π΅Π»ΠΈΡ‚ΡŒ коэффициСнт трСния скольТСния? РасчСт трСния покоя, скольТСния ΠΈ качСния

Π’Ρ€Π΅Π½ΠΈΠ΅ являСтся Ρ‚Π΅ΠΌ физичСским процСссом, Π±Π΅Π· ΠΊΠΎΡ‚ΠΎΡ€ΠΎΠ³ΠΎ Π½Π΅ ΠΌΠΎΠ³Π»ΠΎ Π±Ρ‹ ΡΡƒΡ‰Π΅ΡΡ‚Π²ΠΎΠ²Π°Ρ‚ΡŒ само Π΄Π²ΠΈΠΆΠ΅Π½ΠΈΠ΅ Π² нашСм ΠΌΠΈΡ€Π΅. Π’ Ρ„ΠΈΠ·ΠΈΠΊΠ΅ для вычислСния Π°Π±ΡΠΎΠ»ΡŽΡ‚Π½ΠΎΠ³ΠΎ значСния силы трСния Π½Π΅ΠΎΠ±Ρ…ΠΎΠ΄ΠΈΠΌΠΎ Π·Π½Π°Ρ‚ΡŒ ΡΠΏΠ΅Ρ†ΠΈΠ°Π»ΡŒΠ½Ρ‹ΠΉ коэффициСнт для рассматриваСмых трущихся повСрхностСй. На этот вопрос ΠΎΡ‚Π²Π΅Ρ‚ΠΈΡ‚ данная ΡΡ‚Π°Ρ‚ΡŒΡ.

Π’Ρ€Π΅Π½ΠΈΠ΅ Π² Ρ„ΠΈΠ·ΠΈΠΊΠ΅

ΠŸΡ€Π΅ΠΆΠ΄Π΅ Ρ‡Π΅ΠΌ ΠΎΡ‚Π²Π΅Ρ‡Π°Ρ‚ΡŒ Π½Π° вопрос, ΠΊΠ°ΠΊ коэффициСнт трСния Π½Π°Ρ…ΠΎΠ΄ΠΈΡ‚ΡŒ, Π½Π΅ΠΎΠ±Ρ…ΠΎΠ΄ΠΈΠΌΠΎ Ρ€Π°ΡΡΠΌΠΎΡ‚Ρ€Π΅Ρ‚ΡŒ, Ρ‡Ρ‚ΠΎ Ρ‚Π°ΠΊΠΎΠ΅ Ρ‚Ρ€Π΅Π½ΠΈΠ΅ ΠΈ ΠΊΠ°ΠΊΠΎΠΉ силой ΠΎΠ½ΠΎ характСризуСтся.

Π’ Ρ„ΠΈΠ·ΠΈΠΊΠ΅ Π²Ρ‹Π΄Π΅Π»ΡΡŽΡ‚ Ρ‚Ρ€ΠΈ Π²ΠΈΠ΄Π° этого процСсса, Ρ‡Ρ‚ΠΎ ΠΏΡ€ΠΎΡ‚Π΅ΠΊΠ°Π΅Ρ‚ ΠΌΠ΅ΠΆΠ΄Ρƒ Ρ‚Π²Π΅Ρ€Π΄Ρ‹ΠΌΠΈ ΠΎΠ±ΡŠΠ΅ΠΊΡ‚Π°ΠΌΠΈ. Π­Ρ‚ΠΎ скольТСния ΠΈ качСния. Π’Ρ€Π΅Π½ΠΈΠ΅ покоя Π²ΠΎΠ·Π½ΠΈΠΊΠ°Π΅Ρ‚ всСгда, ΠΊΠΎΠ³Π΄Π° внСшняя сила пытаСтся ΡΠ΄Π²ΠΈΠ½ΡƒΡ‚ΡŒ с мСста ΠΎΠ±ΡŠΠ΅ΠΊΡ‚. БкольТСния Ρ‚Ρ€Π΅Π½ΠΈΠ΅, судя ΠΏΠΎ названию, Π²ΠΎΠ·Π½ΠΈΠΊΠ°Π΅Ρ‚ ΠΏΡ€ΠΈ скольТСнии ΠΎΠ΄Π½ΠΎΠΉ повСрхности ΠΏΠΎ Π΄Ρ€ΡƒΠ³ΠΎΠΉ. НаконСц, качСния трСния появляСтся, ΠΊΠΎΠ³Π΄Π° ΠΊΡ€ΡƒΠ³Π»Ρ‹ΠΉ ΠΎΠ±ΡŠΠ΅ΠΊΡ‚ (колСсо, ΡˆΠ°Ρ€ΠΈΠΊ) катится ΠΏΠΎ Π½Π΅ΠΊΠΎΡ‚ΠΎΡ€ΠΎΠΉ повСрхности.

ΠžΠ±ΡŠΠ΅Π΄ΠΈΠ½ΡΠ΅Ρ‚ всС Π²ΠΈΠ΄Ρ‹ Ρ‚ΠΎΡ‚ Ρ„Π°ΠΊΡ‚, Ρ‡Ρ‚ΠΎ ΠΎΠ½ΠΈ ΠΏΡ€Π΅ΠΏΡΡ‚ΡΡ‚Π²ΡƒΡŽΡ‚ Π»ΡŽΠ±ΠΎΠΌΡƒ двиТСнию ΠΈ Ρ‚ΠΎΡ‡ΠΊΠ° прилоТСния ΠΈΡ… сил находится Π² области ΠΊΠΎΠ½Ρ‚Π°ΠΊΡ‚Π° повСрхностСй Π΄Π²ΡƒΡ… ΠΎΠ±ΡŠΠ΅ΠΊΡ‚ΠΎΠ². Π’Π°ΠΊΠΆΠ΅ всС эти Π²ΠΈΠ΄Ρ‹ пСрСводят ΠΌΠ΅Ρ…Π°Π½ΠΈΡ‡Π΅ΡΠΊΡƒΡŽ ΡΠ½Π΅Ρ€Π³ΠΈΡŽ Π² Ρ‚Π΅ΠΏΠ»ΠΎ.

ΠŸΡ€ΠΈΡ‡ΠΈΠ½Π°ΠΌΠΈ сил трСния скольТСния ΠΈ покоя ΡΠ²Π»ΡΡŽΡ‚ΡΡ ΡˆΠ΅Ρ€ΠΎΡ…ΠΎΠ²Π°Ρ‚ΠΎΡΡ‚ΠΈ микроскопичСского ΠΌΠ°ΡΡˆΡ‚Π°Π±Π° Π½Π° повСрхностях, ΠΊΠΎΡ‚ΠΎΡ€Ρ‹Π΅ трутся. ΠšΡ€ΠΎΠΌΠ΅ Ρ‚ΠΎΠ³ΠΎ, эти Π²ΠΈΠ΄Ρ‹ обусловлСны диполь-Π΄ΠΈΠΏΠΎΠ»ΡŒΠ½Ρ‹ΠΌ ΠΈ Π΄Ρ€ΡƒΠ³ΠΈΠΌΠΈ Π²ΠΈΠ΄Π°ΠΌΠΈ взаимодСйствий ΠΌΠ΅ΠΆΠ΄Ρƒ Π°Ρ‚ΠΎΠΌΠ°ΠΌΠΈ ΠΈ ΠΌΠΎΠ»Π΅ΠΊΡƒΠ»Π°ΠΌΠΈ, ΠΊΠΎΡ‚ΠΎΡ€Ρ‹Π΅ ΠΎΠ±Ρ€Π°Π·ΡƒΡŽΡ‚ трущиСся Ρ‚Π΅Π»Π°.

ΠŸΡ€ΠΈΡ‡ΠΈΠ½Π° качСния трСния связана с гистСрСзисом ΡƒΠΏΡ€ΡƒΠ³ΠΎΠΉ Π΄Π΅Ρ„ΠΎΡ€ΠΌΠ°Ρ†ΠΈΠΈ, которая появляСтся Π² Ρ‚ΠΎΡ‡ΠΊΠ΅ ΠΊΠΎΠ½Ρ‚Π°ΠΊΡ‚Π° катящСгося ΠΎΠ±ΡŠΠ΅ΠΊΡ‚Π° ΠΈ повСрхности.

Π‘ΠΈΠ»Π° трСния ΠΈ коэффициСнт трСния

ВсС Ρ‚Ρ€ΠΈ Π²ΠΈΠ΄Π° сил Ρ‚Π²Π΅Ρ€Π΄ΠΎΠ³ΠΎ трСния ΠΎΠΏΠΈΡΡ‹Π²Π°ΡŽΡ‚ΡΡ выраТСниями, ΠΈΠΌΠ΅ΡŽΡ‰ΠΈΠΌΠΈ ΠΎΠ΄Π½Ρƒ ΠΈ Ρ‚Ρƒ ΠΆΠ΅ Ρ„ΠΎΡ€ΠΌΡƒ. ΠŸΡ€ΠΈΠ²Π΅Π΄Π΅ΠΌ Π΅Π΅:

Π—Π΄Π΅ΡΡŒ N — сила, Π΄Π΅ΠΉΡΡ‚Π²ΡƒΡŽΡ‰Π°Ρ пСрпСндикулярно повСрхности Π½Π° Ρ‚Π΅Π»ΠΎ. Она называСтся Ρ€Π΅Π°ΠΊΡ†ΠΈΠ΅ΠΉ ΠΎΠΏΠΎΡ€Ρ‹. Π’Π΅Π»ΠΈΡ‡ΠΈΠ½Π° Β΅ t — называСтся коэффициСнтом ΡΠΎΠΎΡ‚Π²Π΅Ρ‚ΡΡ‚Π²ΡƒΡŽΡ‰Π΅Π³ΠΎ Π²ΠΈΠ΄Π° трСния.

ΠšΠΎΡΡ„Ρ„ΠΈΡ†ΠΈΠ΅Π½Ρ‚Ρ‹ для трСния скольТСния ΠΈ покоя ΡΠ²Π»ΡΡŽΡ‚ΡΡ Π²Π΅Π»ΠΈΡ‡ΠΈΠ½Π°ΠΌΠΈ Π±Π΅Π·Ρ€Π°Π·ΠΌΠ΅Ρ€Π½Ρ‹ΠΌΠΈ. Π­Ρ‚ΠΎ ΠΌΠΎΠΆΠ½ΠΎ ΠΏΠΎΠ½ΡΡ‚ΡŒ, Ссли ΠΏΠΎΡΠΌΠΎΡ‚Ρ€Π΅Ρ‚ΡŒ Π½Π° равСнство силы трСния ΠΈ трСния коэффициСнта. ЛСвая Ρ‡Π°ΡΡ‚ΡŒ равСнства выраТаСтся Π² Π½ΡŒΡŽΡ‚ΠΎΠ½Π°Ρ…, правая Ρ‡Π°ΡΡ‚ΡŒ Ρ‚Π°ΠΊΠΆΠ΅ выраТаСтся Π² Π½ΡŒΡŽΡ‚ΠΎΠ½Π°Ρ…, ΠΏΠΎΡΠΊΠΎΠ»ΡŒΠΊΡƒ Π²Π΅Π»ΠΈΡ‡ΠΈΠ½Π° N — это сила.

Π§Ρ‚ΠΎ касаСтся качСния трСния, Ρ‚ΠΎ коэффициСнт для Π½Π΅Π³ΠΎ Ρ‚ΠΎΠΆΠ΅ Π±ΡƒΠ΄Π΅Ρ‚ Π²Π΅Π»ΠΈΡ‡ΠΈΠ½ΠΎΠΉ Π±Π΅Π·Ρ€Π°Π·ΠΌΠ΅Ρ€Π½ΠΎΠΉ, ΠΎΠ΄Π½Π°ΠΊΠΎ ΠΎΠ½ опрСдСляСтся Π² Π²ΠΈΠ΄Π΅ ΠΎΡ‚Π½ΠΎΡˆΠ΅Π½ΠΈΡ Π»ΠΈΠ½Π΅ΠΉΠ½ΠΎΠΉ характСристики ΡƒΠΏΡ€ΡƒΠ³ΠΎΠΉ Π΄Π΅Ρ„ΠΎΡ€ΠΌΠ°Ρ†ΠΈΠΈ ΠΊ радиусу катящСгося ΠΎΠ±ΡŠΠ΅ΠΊΡ‚Π°.

Π‘Π»Π΅Π΄ΡƒΠ΅Ρ‚ ΡΠΊΠ°Π·Π°Ρ‚ΡŒ, Ρ‡Ρ‚ΠΎ Ρ‚ΠΈΠΏΠΈΡ‡Π½Ρ‹ΠΌΠΈ значСниями коэффициСнтов трСния скольТСния ΠΈ покоя ΡΠ²Π»ΡΡŽΡ‚ΡΡ дСсятыС Π΄ΠΎΠ»ΠΈ Π΅Π΄ΠΈΠ½ΠΈΡ†Ρ‹. Для этот коэффициСнт соотвСтствуСт сотым ΠΈ тысячным долям Π΅Π΄ΠΈΠ½ΠΈΡ†Ρ‹.

Как Π½Π°Ρ…ΠΎΠ΄ΠΈΡ‚ΡŒ коэффициСнт трСния?

ΠšΠΎΡΡ„Ρ„ΠΈΡ†ΠΈΠ΅Π½Ρ‚ Β΅ t зависит ΠΎΡ‚ ряда Ρ„Π°ΠΊΡ‚ΠΎΡ€ΠΎΠ², ΠΊΠΎΡ‚ΠΎΡ€Ρ‹Π΅ слоТно ΡƒΡ‡Π΅ΡΡ‚ΡŒ матСматичСски. ΠŸΠ΅Ρ€Π΅Ρ‡ΠΈΡΠ»ΠΈΠΌ Π½Π΅ΠΊΠΎΡ‚ΠΎΡ€Ρ‹Π΅ ΠΈΠ· Π½ΠΈΡ…:

  • ΠΌΠ°Ρ‚Π΅Ρ€ΠΈΠ°Π» трущихся повСрхностСй;
  • качСство ΠΎΠ±Ρ€Π°Π±ΠΎΡ‚ΠΊΠΈ повСрхности;
  • Π½Π°Π»ΠΈΡ‡ΠΈΠ΅ Π½Π° Π½Π΅ΠΉ грязи, Π²ΠΎΠ΄Ρ‹ ΠΈ Ρ‚Π°ΠΊ Π΄Π°Π»Π΅Π΅;
  • Ρ‚Π΅ΠΌΠΏΠ΅Ρ€Π°Ρ‚ΡƒΡ€Ρ‹ повСрхностСй.

ΠŸΠΎΡΡ‚ΠΎΠΌΡƒ Ρ„ΠΎΡ€ΠΌΡƒΠ»Ρ‹ для Β΅ t Π½Π΅ сущСствуСт, ΠΈ Π΅Π³ΠΎ приходится ΠΈΠ·ΠΌΠ΅Ρ€ΡΡ‚ΡŒ ΡΠΊΡΠΏΠ΅Ρ€ΠΈΠΌΠ΅Π½Ρ‚Π°Π»ΡŒΠ½ΠΎ. Π§Ρ‚ΠΎΠ±Ρ‹ ΠΏΠΎΠ½ΡΡ‚ΡŒ, ΠΊΠ°ΠΊ коэффициСнт трСния Π½Π°Ρ…ΠΎΠ΄ΠΈΡ‚ΡŒ, слСдуСт Π΅Π³ΠΎ Π²Ρ‹Ρ€Π°Π·ΠΈΡ‚ΡŒ ΠΈΠ· Ρ„ΠΎΡ€ΠΌΡƒΠ»Ρ‹ для F t . ИмССм:

ΠŸΠΎΠ»ΡƒΡ‡Π°Π΅Ρ‚ΡΡ, Ρ‡Ρ‚ΠΎ для знания Β΅ t Π½Π΅ΠΎΠ±Ρ…ΠΎΠ΄ΠΈΠΌΠΎ Π½Π°ΠΉΡ‚ΠΈ трСния силу ΠΈ Ρ€Π΅Π°ΠΊΡ†ΠΈΡŽ ΠΎΠΏΠΎΡ€Ρ‹.

Π‘ΠΎΠΎΡ‚Π²Π΅Ρ‚ΡΡ‚Π²ΡƒΡŽΡ‰ΠΈΠΉ экспСримСнт Π²Ρ‹ΠΏΠΎΠ»Π½ΡΡŽΡ‚ ΡΠ»Π΅Π΄ΡƒΡŽΡ‰ΠΈΠΌ ΠΎΠ±Ρ€Π°Π·ΠΎΠΌ:

  1. Π‘Π΅Ρ€ΡƒΡ‚ Ρ‚Π΅Π»ΠΎ ΠΈ ΠΏΠ»ΠΎΡΠΊΠΎΡΡ‚ΡŒ, Π½Π°ΠΏΡ€ΠΈΠΌΠ΅Ρ€, ΠΈΠ·Π³ΠΎΡ‚ΠΎΠ²Π»Π΅Π½Π½Ρ‹Π΅ ΠΈΠ· Π΄Π΅Ρ€Π΅Π²Π°.
  2. Π¦Π΅ΠΏΠ»ΡΡŽΡ‚ Π΄ΠΈΠ½Π°ΠΌΠΎΠΌΠ΅Ρ‚Ρ€ ΠΊ Ρ‚Π΅Π»Ρƒ ΠΈ Ρ€Π°Π²Π½ΠΎΠΌΠ΅Ρ€Π½ΠΎ ΠΏΠ΅Ρ€Π΅ΠΌΠ΅Ρ‰Π°ΡŽΡ‚ Π΅Π³ΠΎ ΠΏΠΎ повСрхности.

ΠŸΡ€ΠΈ этом Π΄ΠΈΠ½Π°ΠΌΠΎΠΌΠ΅Ρ‚Ρ€ ΠΏΠΎΠΊΠ°Π·Ρ‹Π²Π°Π΅Ρ‚ Π½Π΅ΠΊΠΎΡ‚ΠΎΡ€ΡƒΡŽ силу, которая Ρ€Π°Π²Π½Π° F t . Ρ€Π°Π²Π½Π° вСсу Ρ‚Π΅Π»Π° Π½Π° Π³ΠΎΡ€ΠΈΠ·ΠΎΠ½Ρ‚Π°Π»ΡŒΠ½ΠΎΠΉ повСрхности.

ΠžΠΏΠΈΡΠ°Π½Π½Ρ‹ΠΉ способ позволяСт ΠΏΠΎΠ½ΡΡ‚ΡŒ, Ρ‡Π΅ΠΌΡƒ Ρ€Π°Π²Π΅Π½ коэффициСнт трСния покоя ΠΈ скольТСния. Аналогичным ΠΎΠ±Ρ€Π°Π·ΠΎΠΌ ΠΌΠΎΠΆΠ½ΠΎ ΡΠΊΡΠΏΠ΅Ρ€ΠΈΠΌΠ΅Π½Ρ‚Π°Π»ΡŒΠ½ΠΎ ΠΎΠΏΡ€Π΅Π΄Π΅Π»ΠΈΡ‚ΡŒ Β΅ t качСния.

Π”Ρ€ΡƒΠ³ΠΎΠΉ ΡΠΊΡΠΏΠ΅Ρ€ΠΈΠΌΠ΅Π½Ρ‚Π°Π»ΡŒΠ½Ρ‹ΠΉ ΠΌΠ΅Ρ‚ΠΎΠ΄ опрСдСлСния Β΅ t приводится Π² Ρ„ΠΎΡ€ΠΌΠ΅ Π·Π°Π΄Π°Ρ‡ΠΈ Π² ΡΠ»Π΅Π΄ΡƒΡŽΡ‰Π΅ΠΌ ΠΏΡƒΠ½ΠΊΡ‚Π΅.

Π—Π°Π΄Π°Ρ‡Π° Π½Π° вычислСниС Β΅t

ДСрСвянный брус находится Π½Π° стСклянной повСрхности. Наклоняя ΠΏΠ»Π°Π²Π½ΠΎ ΠΏΠΎΠ²Π΅Ρ€Ρ…Π½ΠΎΡΡ‚ΡŒ, установили, Ρ‡Ρ‚ΠΎ скольТСниС бруса начинаСтся ΠΏΡ€ΠΈ ΡƒΠ³Π»Π΅ Π½Π°ΠΊΠ»ΠΎΠ½Π° 15 o . Π§Π΅ΠΌΡƒ Ρ€Π°Π²Π΅Π½ коэффициСнт трСния покоя для ΠΏΠ°Ρ€Ρ‹ Π΄Π΅Ρ€Π΅Π²ΠΎ-стСкло?

Когда брус находился Π½Π° Π½Π°ΠΊΠ»ΠΎΠ½Π½ΠΎΠΉ плоскости ΠΏΡ€ΠΈ 15 o , Ρ‚ΠΎ покоя сила трСния для Π½Π΅Π³ΠΎ ΠΈΠΌΠ΅Π»Π° максимальноС Π·Π½Π°Ρ‡Π΅Π½ΠΈΠ΅. Она Ρ€Π°Π²Π½Π°:

Π‘ΠΈΠ»Π° N опрСдСляСтся ΠΏΠΎ Ρ„ΠΎΡ€ΠΌΡƒΠ»Π΅:

ΠŸΡ€ΠΈΠΌΠ΅Π½ΡΡ Ρ„ΠΎΡ€ΠΌΡƒΠ»Ρƒ для Β΅ t , ΠΏΠΎΠ»ΡƒΡ‡Π°Π΅ΠΌ:

Β΅ t = F t /N = m*g*sin(Ξ±)/(m*g*cos(Ξ±)) = tg(Ξ±).

ΠŸΠΎΠ΄ΡΡ‚Π°Π²Π»ΡΡ ΡƒΠ³ΠΎΠ» Ξ±, ΠΏΡ€ΠΈΡ…ΠΎΠ΄ΠΈΠΌ ΠΊ ΠΎΡ‚Π²Π΅Ρ‚Ρƒ: Β΅ t = 0,27.

Научно-практичСская конфСрСнция

ΠšΠΎΡΡ„Ρ„ΠΈΡ†ΠΈΠ΅Π½Ρ‚ трСния ΠΈ ΠΌ Π΅Ρ‚ΠΎΠ΄Ρ‹ Π΅Π³ΠΎ расчСта

ПСнза 2010 г.

I Π³Π»Π°Π²Π°. ВСорСтичСская Ρ‡Π°ΡΡ‚ΡŒ

1. Π’ΠΈΠ΄Ρ‹ трСния, коэффициСнт трСния

II Π³Π»Π°Π²Π°. ΠŸΡ€Π°ΠΊΡ‚ΠΈΡ‡Π΅ΡΠΊΠ°Ρ Ρ‡Π°ΡΡ‚ΡŒ

    РасчСт трСния покоя, скольТСния, ΠΈ качСния

    РасчСт коэффициСнта трСния покоя

Бписок Π»ΠΈΡ‚Π΅Ρ€Π°Ρ‚ΡƒΡ€Ρ‹

I Π³Π»Π°Π²Π°. ВСорСтичСская Ρ‡Π°ΡΡ‚ΡŒ

1. Π’ΠΈΠ΄Ρ‹ трСния, коэффициСнт трСния

Π‘ Ρ‚Ρ€Π΅Π½ΠΈΠ΅ΠΌ ΠΌΡ‹ сталкиваСмся Π½Π° ΠΊΠ°ΠΆΠ΄ΠΎΠΌ ΡˆΠ°Π³Ρƒ. Π’Π΅Ρ€Π½Π΅Π΅ Π±Ρ‹Π»ΠΎ Π±Ρ‹ ΡΠΊΠ°Π·Π°Ρ‚ΡŒ, Ρ‡Ρ‚ΠΎ Π±Π΅Π· трСния ΠΌΡ‹ ΠΈ ΡˆΠ°Π³Ρƒ ΡΡ‚ΡƒΠΏΠΈΡ‚ΡŒ Π½Π΅ ΠΌΠΎΠΆΠ΅ΠΌ. Но нСсмотря Π½Π° Ρ‚Ρƒ Π±ΠΎΠ»ΡŒΡˆΡƒΡŽ Ρ€ΠΎΠ»ΡŒ, ΠΊΠΎΡ‚ΠΎΡ€ΡƒΡŽ ΠΈΠ³Ρ€Π°Π΅Ρ‚ Ρ‚Ρ€Π΅Π½ΠΈΠ΅ Π² нашСй ΠΆΠΈΠ·Π½ΠΈ, Π΄ΠΎ сих ΠΏΠΎΡ€ Π½Π΅ создана достаточно полная ΠΊΠ°Ρ€Ρ‚ΠΈΠ½Π° возникновСния трСния. Π­Ρ‚ΠΎ связано Π΄Π°ΠΆΠ΅ Π½Π΅ с Ρ‚Π΅ΠΌ, Ρ‡Ρ‚ΠΎ Ρ‚Ρ€Π΅Π½ΠΈΠ΅ ΠΈΠΌΠ΅Π΅Ρ‚ ΡΠ»ΠΎΠΆΠ½ΡƒΡŽ ΠΏΡ€ΠΈΡ€ΠΎΠ΄Ρƒ, Π° скорСС с Ρ‚Π΅ΠΌ, Ρ‡Ρ‚ΠΎ ΠΎΠΏΡ‹Ρ‚Ρ‹ с Ρ‚Ρ€Π΅Π½ΠΈΠ΅ΠΌ ΠΎΡ‡Π΅Π½ΡŒ Ρ‡ΡƒΠ²ΡΡ‚Π²ΠΈΡ‚Π΅Π»ΡŒΠ½Ρ‹ ΠΊ ΠΎΠ±Ρ€Π°Π±ΠΎΡ‚ΠΊΠ΅ повСрхности ΠΈ поэтому Ρ‚Ρ€ΡƒΠ΄Π½ΠΎ воспроизводимы.

БущСствуСт внСшнСС ΠΈ Π²Π½ΡƒΡ‚Ρ€Π΅Π½Π½Π΅Π΅ Ρ‚Ρ€Π΅Π½ΠΈΠ΅ (ΠΈΠ½Π°Ρ‡Π΅ Π½Π°Π·Ρ‹Π²Π°Π΅ΠΌΠΎΠ΅ Π²ΡΠ·ΠΊΠΎΡΡ‚ΡŒΡŽ ). Π’Π½Π΅ΡˆΠ½ΠΈΠΌ Π½Π°Π·Ρ‹Π²Π°ΡŽΡ‚ Ρ‚Π°ΠΊΠΎΠΉ Π²ΠΈΠ΄ трСния, ΠΏΡ€ΠΈ ΠΊΠΎΡ‚ΠΎΡ€ΠΎΠΌ Π² мСстах соприкосновСния Ρ‚Π²Π΅Ρ€Π΄Ρ‹Ρ… Ρ‚Π΅Π» Π²ΠΎΠ·Π½ΠΈΠΊΠ°ΡŽΡ‚ силы, Π·Π°Ρ‚Ρ€ΡƒΠ΄Π½ΡΡŽΡ‰ΠΈΠ΅ Π²Π·Π°ΠΈΠΌΠ½ΠΎΠ΅ ΠΏΠ΅Ρ€Π΅ΠΌΠ΅Ρ‰Π΅Π½ΠΈΠ΅ Ρ‚Π΅Π» ΠΈ Π½Π°ΠΏΡ€Π°Π²Π»Π΅Π½Π½Ρ‹Π΅ ΠΏΠΎ ΠΊΠ°ΡΠ°Ρ‚Π΅Π»ΡŒΠ½ΠΎΠΉ ΠΊ ΠΈΡ… повСрхностям.

Π’Π½ΡƒΡ‚Ρ€Π΅Π½Π½ΠΈΠΌ Ρ‚Ρ€Π΅Π½ΠΈΠ΅ΠΌ (Π²ΡΠ·ΠΊΠΎΡΡ‚ΡŒΡŽ) называСтся Π²ΠΈΠ΄ трСния, состоящий Π² Ρ‚ΠΎΠΌ, Ρ‡Ρ‚ΠΎ ΠΏΡ€ΠΈ Π²Π·Π°ΠΈΠΌΠ½ΠΎΠΌ ΠΏΠ΅Ρ€Π΅ΠΌΠ΅Ρ‰Π΅Π½ΠΈΠΈ. слоСв Тидкости ΠΈΠ»ΠΈ Π³Π°Π·Π° ΠΌΠ΅ΠΆΠ΄Ρƒ Π½ΠΈΠΌΠΈ Π²ΠΎΠ·Π½ΠΈΠΊΠ°ΡŽΡ‚ ΠΊΠ°ΡΠ°Ρ‚Π΅Π»ΡŒΠ½Ρ‹Π΅ силы, ΠΏΡ€Π΅ΠΏΡΡ‚ΡΡ‚Π²ΡƒΡŽΡ‰ΠΈΠ΅ Ρ‚Π°ΠΊΠΎΠΌΡƒ ΠΏΠ΅Ρ€Π΅ΠΌΠ΅Ρ‰Π΅Π½ΠΈΡŽ.

Π’Π½Π΅ΡˆΠ½Π΅Π΅ Ρ‚Ρ€Π΅Π½ΠΈΠ΅ ΠΏΠΎΠ΄Ρ€Π°Π·Π΄Π΅Π»ΡΡŽΡ‚ Π½Π° Ρ‚Ρ€Π΅Π½ΠΈΠ΅ покоя (статичСскоС Ρ‚Ρ€Π΅Π½ΠΈΠ΅ ) ΠΈ кинСматичСскоС Ρ‚Ρ€Π΅Π½ΠΈΠ΅ . Π’Ρ€Π΅Π½ΠΈΠ΅ покоя Π²ΠΎΠ·Π½ΠΈΠΊΠ°Π΅Ρ‚ ΠΌΠ΅ΠΆΠ΄Ρƒ Π½Π΅ΠΏΠΎΠ΄Π²ΠΈΠΆΠ½Ρ‹ΠΌΠΈ Ρ‚Π²Π΅Ρ€Π΄Ρ‹ΠΌΠΈ Ρ‚Π΅Π»Π°ΠΌΠΈ, ΠΊΠΎΠ³Π΄Π° ΠΊΠ°ΠΊΠΎΠ΅-Π»ΠΈΠ±ΠΎ ΠΈΠ· Π½ΠΈΡ… ΠΏΡ‹Ρ‚Π°ΡŽΡ‚ΡΡ ΡΠ΄Π²ΠΈΠ½ΡƒΡ‚ΡŒ с мСста. ΠšΠΈΠ½Π΅ΠΌΠ°Ρ‚ΠΈΡ‡Π΅ΡΠΊΠΎΠ΅ Ρ‚Ρ€Π΅Π½ΠΈΠ΅ сущСствуСт ΠΌΠ΅ΠΆΠ΄Ρƒ Π²Π·Π°ΠΈΠΌΠ½ΠΎ ΡΠΎΠΏΡ€ΠΈΠΊΠ°ΡΠ°ΡŽΡ‰ΠΈΠΌΠΈΡΡ двиТущимися Ρ‚Π²Π΅Ρ€Π΄Ρ‹ΠΌΠΈ Ρ‚Π΅Π»Π°ΠΌΠΈ. ΠšΠΈΠ½Π΅ΠΌΠ°Ρ‚ΠΈΡ‡Π΅ΡΠΊΠΎΠ΅ Ρ‚Ρ€Π΅Π½ΠΈΠ΅, Π² свою ΠΎΡ‡Π΅Ρ€Π΅Π΄ΡŒ, подраздСляСтся Π½Π° Ρ‚Ρ€Π΅Π½ΠΈΠ΅ скольТСния ΠΈ Ρ‚Ρ€Π΅Π½ΠΈΠ΅ качСния .

Π’ ΠΆΠΈΠ·Π½ΠΈ Ρ‡Π΅Π»ΠΎΠ²Π΅ΠΊΠ° силы трСния ΠΈΠ³Ρ€Π°ΡŽΡ‚ Π²Π°ΠΆΠ½ΡƒΡŽ Ρ€ΠΎΠ»ΡŒ. Π’ ΠΎΠ΄Π½ΠΈΡ… случаях ΠΎΠ½ ΠΈΡ… ΠΈΡΠΏΠΎΠ»ΡŒΠ·ΡƒΠ΅Ρ‚, Π° Π² Π΄Ρ€ΡƒΠ³ΠΈΡ… борСтся с Π½ΠΈΠΌΠΈ. Π‘ΠΈΠ»Ρ‹ трСния ΠΈΠΌΠ΅ΡŽΡ‚ ΡΠ»Π΅ΠΊΡ‚Ρ€ΠΎΠΌΠ°Π³Π½ΠΈΡ‚Π½ΡƒΡŽ ΠΏΡ€ΠΈΡ€ΠΎΠ΄Ρƒ.

Если Ρ‚Π΅Π»ΠΎ ΡΠΊΠΎΠ»ΡŒΠ·ΠΈΡ‚ ΠΏΠΎ ΠΊΠ°ΠΊΠΎΠΉ-Π»ΠΈΠ±ΠΎ повСрхности, Π΅Π³ΠΎ двиТСнию прСпятствуСт сила трСния скольТСния.

Π“Π΄Π΅ N — сила Ρ€Π΅Π°ΠΊΡ†ΠΈΠΈ ΠΎΠΏΠΎΡ€Ρ‹, a ΞΌ — коэффициСнт трСния скольТСния. ΠšΠΎΡΡ„Ρ„ΠΈΡ†ΠΈΠ΅Π½Ρ‚ ΞΌ зависит ΠΎΡ‚ ΠΌΠ°Ρ‚Π΅Ρ€ΠΈΠ°Π»Π° ΠΈ качСства ΠΎΠ±Ρ€Π°Π±ΠΎΡ‚ΠΊΠΈ ΡΠΎΠΏΡ€ΠΈΠΊΠ°ΡΠ°ΡŽΡ‰ΠΈΡ…ΡΡ повСрхностСй ΠΈ Π½Π΅ зависит ΠΎΡ‚ вСса Ρ‚Π΅Π»Π°. ΠšΠΎΡΡ„Ρ„ΠΈΡ†ΠΈΠ΅Π½Ρ‚ трСния опрСдСляСтся ΠΎΠΏΡ‹Ρ‚Π½Ρ‹ΠΌ ΠΏΡƒΡ‚Π΅ΠΌ.

Π‘ΠΈΠ»Π° трСния скольТСния всСгда Π½Π°ΠΏΡ€Π°Π²Π»Π΅Π½Π° ΠΏΡ€ΠΎΡ‚ΠΈΠ²ΠΎΠΏΠΎΠ»ΠΎΠΆΠ½ΠΎ двиТСнию Ρ‚Π΅Π»Π°. ΠŸΡ€ΠΈ ΠΈΠ·ΠΌΠ΅Π½Π΅Π½ΠΈΠΈ направлСния скорости измСняСтся ΠΈ Π½Π°ΠΏΡ€Π°Π²Π»Π΅Π½ΠΈΠ΅ силы трСния.

Π‘ΠΈΠ»Π° трСния Π½Π°Ρ‡ΠΈΠ½Π°Π΅Ρ‚ Π΄Π΅ΠΉΡΡ‚Π²ΠΎΠ²Π°Ρ‚ΡŒ Π½Π° Ρ‚Π΅Π»ΠΎ, ΠΊΠΎΠ³Π΄Π° Π΅Π³ΠΎ ΠΏΡ‹Ρ‚Π°ΡŽΡ‚ΡΡ ΡΠ΄Π²ΠΈΠ½ΡƒΡ‚ΡŒ с мСста. Если внСшняя сила F мСньшС произвСдСния ΞΌN, Ρ‚ΠΎ Ρ‚Π΅Π»ΠΎ Π½Π΅ Π±ΡƒΠ΄Π΅Ρ‚ ΡΠ΄Π²ΠΈΠ³Π°Ρ‚ΡŒΡΡ — Π½Π°Ρ‡Π°Π»Ρƒ двиТСния, ΠΊΠ°ΠΊ принято Π³ΠΎΠ²ΠΎΡ€ΠΈΡ‚ΡŒ, ΠΌΠ΅ΡˆΠ°Π΅Ρ‚ сила трСния покоя . Π’Π΅Π»ΠΎ Π½Π°Ρ‡Π½Π΅Ρ‚ Π΄Π²ΠΈΠΆΠ΅Π½ΠΈΠ΅ Ρ‚ΠΎΠ»ΡŒΠΊΠΎ Ρ‚ΠΎΠ³Π΄Π°, ΠΊΠΎΠ³Π΄Π° внСшняя сила F прСвысит максимальноС Π·Π½Π°Ρ‡Π΅Π½ΠΈΠ΅, ΠΊΠΎΡ‚ΠΎΡ€ΠΎΠ΅ ΠΌΠΎΠΆΠ΅Ρ‚ ΠΈΠΌΠ΅Ρ‚ΡŒ сила трСния покоя

Π’Ρ€Π΅Π½ΠΈΠ΅ покоя – сила трСния, ΠΏΡ€Π΅ΠΏΡΡ‚ΡΡ‚Π²ΡƒΡŽΡ‰Π°Ρ возникновСнию двиТСнию ΠΎΠ΄Π½ΠΎΠ³ΠΎ Ρ‚Π΅Π»Π° ΠΏΠΎ повСрхности Π΄Ρ€ΡƒΠ³ΠΎΠ³ΠΎ.

II Π³Π»Π°Π²Π°. ΠŸΡ€Π°ΠΊΡ‚ΠΈΡ‡Π΅ΡΠΊΠ°Ρ Ρ‡Π°ΡΡ‚ΡŒ

1. РасчСт трСния покоя, скольТСния ΠΈ качСния

ΠžΡΠ½ΠΎΠ²Ρ‹Π²Π°ΡΡΡŒ Π½Π° Π²Ρ‹ΡˆΠ΅ΡΠΊΠ°Π·Π°Π½Π½ΠΎΠ΅, я, ΠΎΠΏΡ‹Ρ‚Π½ΠΎΠΌ ΠΏΡƒΡ‚Π΅ΠΌ, Π½Π°Ρ…ΠΎΠ΄ΠΈΠ» силу трСния покоя, скольТСния ΠΈ качСния. Для этого я использовал нСсколько ΠΏΠ°Ρ€ Ρ‚Π΅Π», Π² Ρ€Π΅Π·ΡƒΠ»ΡŒΡ‚Π°Ρ‚Π΅ взаимодСйствия ΠΊΠΎΡ‚ΠΎΡ€Ρ‹Ρ… Π±ΡƒΠ΄Π΅Ρ‚ Π²ΠΎΠ·Π½ΠΈΠΊΠ°Ρ‚ΡŒ сила трСния, ΠΈ ΠΏΡ€ΠΈΠ±ΠΎΡ€ для измСрСния силы – Π΄ΠΈΠ½Π°ΠΌΠΎΠΌΠ΅Ρ‚Ρ€.

Π’ΠΎΡ‚ ΡΠ»Π΅Π΄ΡƒΡŽΡ‰ΠΈΠ΅ ΠΏΠ°Ρ€Ρ‹ Ρ‚Π΅Π»:

    дСрСвянный брусок Π² Π²ΠΈΠ΄Π΅ ΠΏΡ€ΡΠΌΠΎΡƒΠ³ΠΎΠ»ΡŒΠ½ΠΎΠ³ΠΎ ΠΏΠ°Ρ€Π°Π»Π»Π΅ΠΏΠΈΠΏΠ΅Π΄Π° ΠΎΠΏΡ€Π΅Π΄Π΅Π»Π΅Π½Π½ΠΎΠΉ массы ΠΈ Π»Π°ΠΊΠΈΡ€ΠΎΠ²Π°Π½Π½Ρ‹ΠΉ дСрСвянный стол.

    дСрСвянный брусок Π² Π²ΠΈΠ΄Π΅ ΠΏΡ€ΡΠΌΠΎΡƒΠ³ΠΎΠ»ΡŒΠ½ΠΎΠ³ΠΎ ΠΏΠ°Ρ€Π°Π»Π»Π΅ΠΏΠΈΠΏΠ΅Π΄Π° с мСньшСй Ρ‡Π΅ΠΌ ΠΏΠ΅Ρ€Π²Ρ‹ΠΉ массой ΠΈ Π»Π°ΠΊΠΈΡ€ΠΎΠ²Π°Π½Π½Ρ‹ΠΉ дСрСвянный стол.

    дСрСвянный брусок Π² Π²ΠΈΠ΄Π΅ Ρ†ΠΈΠ»ΠΈΠ½Π΄Ρ€Π° ΠΎΠΏΡ€Π΅Π΄Π΅Π»Π΅Π½Π½ΠΎΠΉ массы ΠΈ Π»Π°ΠΊΠΈΡ€ΠΎΠ²Π°Π½Π½Ρ‹ΠΉ дСрСвянный стол.

    дСрСвянный брусок Π² Π²ΠΈΠ΄Π΅ Ρ†ΠΈΠ»ΠΈΠ½Π΄Ρ€Π° с мСньшСй Ρ‡Π΅ΠΌ ΠΏΠ΅Ρ€Π²Ρ‹ΠΉ массой ΠΈ Π»Π°ΠΊΠΈΡ€ΠΎΠ²Π°Π½Π½Ρ‹ΠΉ дСрСвянный стол.

ПослС Ρ‚ΠΎΠ³ΠΎ ΠΊΠ°ΠΊ Π±Ρ‹Π»ΠΈ ΠΏΡ€ΠΎΠ²Π΅Π΄Π΅Π½Ρ‹ ΠΎΠΏΡ‹Ρ‚Ρ‹ – ΠΌΠΎΠΆΠ½ΠΎ Π±Ρ‹Π»ΠΎ ΡΠ΄Π΅Π»Π°Ρ‚ΡŒ ΡΠ»Π΅Π΄ΡƒΡŽΡ‰ΠΈΠΉ Π²Ρ‹Π²ΠΎΠ΄ –

Π‘ΠΈΠ»Π° трСния покоя, скольТСния ΠΈ качСния опрСдСляСтся ΠΎΠΏΡ‹Ρ‚Π½ΠΎΠΌ ΠΏΡƒΡ‚Π΅ΠΌ.

Π’Ρ€Π΅Π½ΠΈΠ΅ покоя:

Для 1) FΠΏ=0.6 Н, 2) FΠΏ=0.4 Н, 3) FΠΏ=0.2 Н, 4) FΠΏ=0.15 Н

Π’Ρ€Π΅Π½ΠΈΠ΅ скольТСниС:

Для 1) Fс=0.52 Н, 2) Fс=0.33 Н, 3) Fс=0.15 Н, 4) Fс=0.11 Н

Π’Ρ€Π΅Π½ΠΈΠ΅ ΠΊΠ°Ρ‡Π΅Π½ΠΈΠ΅:

Для 3) FΠΊ=0.14 Н, 4) FΠΊ=0.08 Н

Π’Π΅ΠΌ самым я ΠΎΠΏΡ€Π΅Π΄Π΅Π»ΠΈΠ» ΠΎΠΏΡ‹Ρ‚Π½Ρ‹ΠΌ ΠΏΡƒΡ‚Π΅ΠΌ всС Ρ‚Ρ€ΠΈ Π²ΠΈΠ΄Π° внСшнСго трСния ΠΈ ΠΏΠΎΠ»ΡƒΡ‡ΠΈΠ» Ρ‡Ρ‚ΠΎ

FΠΏ> Fс > FΠΊ для ΠΎΠ΄Π½ΠΎΠ³ΠΎ ΠΈ Ρ‚ΠΎΠ³ΠΎ ΠΆΠ΅ Ρ‚Π΅Π»Π°.

2. РасчСт коэффициСнта трСния покоя

Но Π² большСй стСпСни интСрСсна Π½Π΅ сила трСния, Π° коэффициСнт трСния. Как Π΅Π³ΠΎ Π²Ρ‹Ρ‡ΠΈΡΠ»ΠΈΡ‚ΡŒ ΠΈ ΠΎΠΏΡ€Π΅Π΄Π΅Π»ΠΈΡ‚ΡŒ? И я нашСл Ρ‚ΠΎΠ»ΡŒΠΊΠΎ Π΄Π²Π° способа опрСдСлСния силы трСния.

ΠŸΠ΅Ρ€Π²Ρ‹ΠΉ способ: ΠΎΡ‡Π΅Π½ΡŒ простой. Зная Ρ„ΠΎΡ€ΠΌΡƒΠ»Ρƒ ΠΈ ΠΎΠΏΡ€Π΅Π΄Π΅Π»ΠΈΠ² ΠΎΠΏΡ‹Ρ‚Π½Ρ‹ΠΌ ΠΏΡƒΡ‚Π΅ΠΌ ΠΈ N, ΠΌΠΎΠΆΠ½ΠΎ ΠΎΠΏΡ€Π΅Π΄Π΅Π»ΠΈΡ‚ΡŒ коэффициСнт трСния покоя, скольТСния ΠΈ качСния.

1) N ο‚» 0,81 Н, 2) N ο‚» 0,56 Н, 3) N ο‚» 2,3 Н, 4) N ο‚» 1,75

ΠšΠΎΡΡ„Ρ„ΠΈΡ†ΠΈΠ΅Π½Ρ‚ трСния покоя:

    = 0,74; 2)  = 0,71; 3)  = 0,087; 4)  = 0,084;

ΠšΠΎΡΡ„Ρ„ΠΈΡ†ΠΈΠ΅Π½Ρ‚ трСния скольТСния:

    = 0,64; 2)  = 0,59; 3)  = 0,063; 4)  = 0,063

ΠšΠΎΡΡ„Ρ„ΠΈΡ†ΠΈΠ΅Π½Ρ‚ трСния качСния:

3)  = 0,06; 4)  = 0,055;

Π‘Π²Π΅Ρ€ΡΡΡΡŒ с Ρ‚Π°Π±Π»ΠΈΡ‡Π½Ρ‹ΠΌΠΈ Π΄Π°Π½Π½Ρ‹ΠΌΠΈ я ΠΏΠΎΠ΄Ρ‚Π²Π΅Ρ€Π΄ΠΈΠ» Π²Π΅Ρ€Π½ΠΎΡΡ‚ΡŒ своих Π·Π½Π°Ρ‡Π΅Π½ΠΈΠΉ.

Но Ρ‚Π°ΠΊΠΆΠ΅ ΠΎΡ‡Π΅Π½ΡŒ интСрСсСн Π²Ρ‚ΠΎΡ€ΠΎΠΉ способ нахоТдСния коэффициСнта трСния.

Но этот способ Ρ…ΠΎΡ€ΠΎΡˆΠΎ опрСдСляСт коэффициСнт трСния покоя, Π° для вычислСния коэффициСнта трСния скольТСния ΠΈ качСния Π²ΠΎΠ·Π½ΠΈΠΊΠ°ΡŽΡ‚ ряд Π·Π°Ρ‚Ρ€ΡƒΠ΄Π½Π΅Π½ΠΈΠΉ.

ОписаниС: Π’Π΅Π»ΠΎ находится с Π΄Ρ€ΡƒΠ³ΠΈΠΌ Ρ‚Π΅Π»ΠΎΠΌ Π² ΠΏΠΎΠΊΠΎΠ΅. Π—Π°Ρ‚Π΅ΠΌ ΠΊΠΎΠ½Π΅Ρ† Π²Ρ‚ΠΎΡ€ΠΎΠ³ΠΎ Ρ‚Π΅Π»Π° Π½Π° ΠΊΠΎΡ‚ΠΎΡ€ΠΎΠΌ Π»Π΅ΠΆΠΈΡ‚ ΠΏΠ΅Ρ€Π²ΠΎΠ΅ Ρ‚Π΅Π»ΠΎ Π½Π°Ρ‡ΠΈΠ½Π°ΡŽΡ‚ ΠΏΠΎΠ΄Π½ΠΈΠΌΠ°Ρ‚ΡŒ Π΄ΠΎ Ρ‚Π΅Ρ… ΠΏΠΎΡ€ ΠΏΠΎΠΊΠ° ΠΏΠ΅Ρ€Π²ΠΎΠ΅ Ρ‚Π΅Π»ΠΎ Π½Π΅ сдвинСтся с мСста.

 = sin  /cos  =tg  =BC/AC

На основС Π²Ρ‚ΠΎΡ€ΠΎΠ³ΠΎ способа ΠΌΠ½ΠΎΠΉ Π±Ρ‹Π»ΠΈ вычислСны Π½Π΅ΠΊΠΎΡ‚ΠΎΡ€ΠΎΠ΅ число коэффициСнтов трСния покоя.

      Π”Π΅Ρ€Π΅Π²ΠΎ ΠΏΠΎ Π΄Π΅Ρ€Π΅Π²Ρƒ:

АВ = 23,5 см; Π’Π‘ = 13,5 см.

П = BC/AC = 13,5/23,5 = 0,57

2. ΠŸΠ΅Π½ΠΎΠΏΠ»Π°ΡΡ‚ ΠΏΠΎ Π΄Π΅Ρ€Π΅Π²Ρƒ:

АВ = 18,5 см; Π’Π‘ = 21 см.

П = BC/AC = 21/18,5 = 1,1

3. Π‘Ρ‚Π΅ΠΊΠ»ΠΎ ΠΏΠΎ Π΄Π΅Ρ€Π΅Π²Ρƒ:

АВ = 24,3 см; Π’Π‘ = 11 см.

П = BC/AC = 11/24,3 = 0,45

4. Алюминий ΠΏΠΎ Π΄Π΅Ρ€Π΅Π²Ρƒ:

АВ = 25,3 см; Π’Π‘ = 10,5 см.

П = BC/AC = 10,5/25,3 = 0,41

5. Π‘Ρ‚Π°Π»ΡŒ ΠΏΠΎ Π΄Π΅Ρ€Π΅Π²Ρƒ:

АВ = 24,6 см; Π’Π‘ = 11,3 см.

П = BC/AC = 11,3/24,6 = 0,46

6. ΠžΡ€Π³. Π‘Ρ‚Π΅ΠΊΠ»ΠΎ ΠΏΠΎ Π΄Π΅Ρ€Π΅Π²Ρƒ:

АВ = 25,1 см; Π’Π‘ = 10,5 см.

П = BC/AC = 10,5/25,1 = 0,42

7. Π“Ρ€Π°Ρ„ΠΈΡ‚ ΠΏΠΎ Π΄Π΅Ρ€Π΅Π²Ρƒ:

АВ = 23 см; Π’Π‘ = 14,4 см.

П = BC/AC = 14,4/23 = 0,63

8. Алюминий ΠΏΠΎ ΠΊΠ°Ρ€Ρ‚ΠΎΠ½Ρƒ:

АВ = 36,6 см; Π’Π‘ = 17,5 см.

П = BC/AC = 17,5/36,6 = 0,48

9. Π–Π΅Π»Π΅Π·ΠΎ ΠΏΠΎ пластмассС:

АВ = 27,1 см; Π’Π‘ = 11,5 см.

П = BC/AC = 11,5/27,1 = 0,43

10. ΠžΡ€Π³. Π‘Ρ‚Π΅ΠΊΠ»ΠΎ ΠΏΠΎ пластику:

АВ = 26,4 см; Π’Π‘ = 18,5 см.

П = BC/AC = 18,5/26,4 = 0,7

На основС своих расчСтов ΠΈ ΠΏΡ€ΠΎΠ²Π΅Π΄Π΅Π½Π½Ρ‹Ρ… экспСримСнтах я сдСлал Π²Ρ‹Π²ΠΎΠ΄ Ρ‡Ρ‚ΠΎ  П >  C >  К , Ρ‡Ρ‚ΠΎ нСоспоримо соотвСтствовало тСорСтичСской Π±Π°Π·Π΅ взятой ΠΈΠ· Π»ΠΈΡ‚Π΅Ρ€Π°Ρ‚ΡƒΡ€Ρ‹. Π Π΅Π·ΡƒΠ»ΡŒΡ‚Π°Ρ‚Ρ‹ ΠΌΠΎΠΈΡ… вычислСний Π½Π΅ Π²Ρ‹ΡˆΠ»ΠΈ Π·Π° Ρ€Π°ΠΌΠΊΠΈ Ρ‚Π°Π±Π»ΠΈΡ‡Π½Ρ‹Ρ… Π΄Π°Π½Π½Ρ‹Ρ…, Π° Π΄Π°ΠΆΠ΅ Π΄ΠΎΠΏΠΎΠ»Π½ΠΈΠ»ΠΈ ΠΈΡ…, Π² Ρ€Π΅Π·ΡƒΠ»ΡŒΡ‚Π°Ρ‚Π΅ Ρ‡Π΅Π³ΠΎ я Ρ€Π°ΡΡˆΠΈΡ€ΠΈΠ» Ρ‚Π°Π±Π»ΠΈΡ‡Π½Ρ‹Π΅ значСния коэффициСнтов Ρ‚Ρ€Π΅Π½ΠΈΠΉ Ρ€Π°Π·Π»ΠΈΡ‡Π½Ρ‹Ρ… ΠΌΠ°Ρ‚Π΅Ρ€ΠΈΠ°Π»ΠΎΠ².

Π›ΠΈΡ‚Π΅Ρ€Π°Ρ‚ΡƒΡ€Π°

1. ΠšΡ€Π°Π³Π΅Π»ΡŒΡΠΊΠΈΠΉ И.Π’., Π”ΠΎΠ±Ρ‹Ρ‡ΠΈΠ½ М.Н., Комбалов Π’.Π‘. ΠžΡΠ½ΠΎΠ²Ρ‹ расчСтов Π½Π° Ρ‚Ρ€Π΅Π½ΠΈΠ΅ ΠΈ износ. М.: ΠœΠ°ΡˆΠΈΠ½ΠΎΡΡ‚Ρ€ΠΎΠ΅Π½ΠΈΠ΅, 1977. 526 с.

      Π€Ρ€ΠΎΠ»ΠΎΠ², К. Π’. (Ρ€Π΅Π΄.): БоврСмСнная трибология: Π˜Ρ‚ΠΎΠ³ΠΈ ΠΈ пСрспСктивы . Изд-Π²ΠΎ Π›ΠšΠ˜, 2008 Π³.

      Π•Π»ΡŒΠΊΠΈΠ½ Π’.И.β€œΠΠ΅ΠΎΠ±Ρ‹Ρ‡Π½Ρ‹Π΅ ΡƒΡ‡Π΅Π±Π½Ρ‹Π΅ ΠΌΠ°Ρ‚Π΅Ρ€ΠΈΠ°Π»Ρ‹ ΠΏΠΎ физикС”. β€œΠ€ΠΈΠ·ΠΈΠΊΠ° Π² ΡˆΠΊΠΎΠ»Π΅β€ Π±ΠΈΠ±Π»ΠΈΠΎΡ‚Π΅ΠΊΠ° ΠΆΡƒΡ€Π½Π°Π»Π°, β„–16, 2000.

      ΠœΡƒΠ΄Ρ€ΠΎΡΡ‚ΡŒ тысячСлСтий. ЭнциклопСдия. Москва, Олма – прСсс, 2006.

ΠšΠΎΡΡ„Ρ„ΠΈΡ†ΠΈΠ΅Π½Ρ‚ трСния β€” это основная характСристика трСния ΠΊΠ°ΠΊ явлСния. Он опрСдСляСтся Π²ΠΈΠ΄ΠΎΠΌ ΠΈ состояниСм повСрхностСй трущихся Ρ‚Π΅Π».

ΠžΠŸΠ Π•Π”Π•Π›Π•ΠΠ˜Π•

ΠšΠΎΡΡ„Ρ„ΠΈΡ†ΠΈΠ΅Π½Ρ‚ΠΎΠΌ трСния Π½Π°Π·Ρ‹Π²Π°ΡŽΡ‚ коэффициСнт ΠΏΡ€ΠΎΠΏΠΎΡ€Ρ†ΠΈΠΎΠ½Π°Π»ΡŒΠ½ΠΎΡΡ‚ΠΈ, ΡΠ²ΡΠ·Ρ‹Π²Π°ΡŽΡ‰ΠΈΠΉ силу трСния () ΠΈ силу Π½ΠΎΡ€ΠΌΠ°Π»ΡŒΠ½ΠΎΠ³ΠΎ давлСния (N) Ρ‚Π΅Π»Π° Π½Π° ΠΎΠΏΠΎΡ€Ρƒ. Π§Π°Ρ‰Π΅ всСго коэффициСнт трСния ΠΎΠ±ΠΎΠ·Π½Π°Ρ‡Π°ΡŽΡ‚ Π±ΡƒΠΊΠ²ΠΎΠΉ . И Ρ‚Π°ΠΊ, коэффициСнт трСния Π²Ρ…ΠΎΠ΄ΠΈΡ‚ Π² Π·Π°ΠΊΠΎΠ½ ΠšΡƒΠ»ΠΎΠ½Π° β€” Амонтона:

Π”Π°Π½Π½Ρ‹ΠΉ коэффициСнт трСния Π½Π΅ зависит ΠΎΡ‚ ΠΏΠ»ΠΎΡ‰Π°Π΄Π΅ΠΉ, ΡΠΎΠΏΡ€ΠΈΠΊΠ°ΡΠ°ΡŽΡ‰ΠΈΡ…ΡΡ повСрхностСй.

Π’ Π΄Π°Π½Π½ΠΎΠΌ случаС Ρ€Π΅Ρ‡ΡŒ ΠΈΠ΄Π΅Ρ‚ ΠΎ коэффициСнтС трСния скольТСния, ΠΊΠΎΡ‚ΠΎΡ€Ρ‹ΠΉ зависит ΠΎΡ‚ совокупных свойств трущихся повСрхностСй ΠΈ являСтся Π±Π΅Π·Ρ€Π°Π·ΠΌΠ΅Ρ€Π½ΠΎΠΉ Π²Π΅Π»ΠΈΡ‡ΠΈΠ½ΠΎΠΉ. ΠšΠΎΡΡ„Ρ„ΠΈΡ†ΠΈΠ΅Π½Ρ‚ трСния зависит ΠΎΡ‚: качСства ΠΎΠ±Ρ€Π°Π±ΠΎΡ‚ΠΊΠΈ повСрхностСй, трущихся Ρ‚Π΅Π», присутствия Π½Π° Π½ΠΈΡ… грязи, скорости двиТСния Ρ‚Π΅Π» Π΄Ρ€ΡƒΠ³ ΠΎΡ‚Π½ΠΎΡΠΈΡ‚Π΅Π»ΡŒΠ½ΠΎ Π΄Ρ€ΡƒΠ³Π° ΠΈ Ρ‚.Π΄. ΠšΠΎΡΡ„Ρ„ΠΈΡ†ΠΈΠ΅Π½Ρ‚ трСния ΠΎΠΏΡ€Π΅Π΄Π΅Π»ΡΡŽΡ‚ эмпиричСски (ΠΎΠΏΡ‹Ρ‚Π½Ρ‹ΠΌ ΠΏΡƒΡ‚Π΅ΠΌ).

ΠšΠΎΡΡ„Ρ„ΠΈΡ†ΠΈΠ΅Π½Ρ‚ трСния, ΠΊΠΎΡ‚ΠΎΡ€Ρ‹ΠΉ соотвСтствуСт максимальной силС трСния покоя Π² Π±ΠΎΠ»ΡŒΡˆΠΈΠ½ΡΡ‚Π²Π΅ случаСв большС, Ρ‡Π΅ΠΌ коэффициСнт трСния двиТСния.

Для большСго числа ΠΏΠ°Ρ€ ΠΌΠ°Ρ‚Π΅Ρ€ΠΈΠ°Π»ΠΎΠ² Π²Π΅Π»ΠΈΡ‡ΠΈΠ½Π° коэффициСнта трСния Π½Π΅ большС Π΅Π΄ΠΈΠ½ΠΈΡ†Ρ‹ ΠΈ Π»Π΅ΠΆΠΈΡ‚ Π² ΠΏΡ€Π΅Π΄Π΅Π»Π°Ρ…

Π£Π³ΠΎΠ» трСния

Иногда вмСсто коэффициСнта трСния ΠΏΡ€ΠΈΠΌΠ΅Π½ΡΡŽΡ‚ ΡƒΠ³ΠΎΠ» трСния (), ΠΊΠΎΡ‚ΠΎΡ€Ρ‹ΠΉ связан с коэффициСнтом ΡΠΎΠΎΡ‚Π½ΠΎΡˆΠ΅Π½ΠΈΠ΅ΠΌ:

Π’Π°ΠΊ, ΡƒΠ³ΠΎΠ» трСния соотвСтствуСт ΠΌΠΈΠ½ΠΈΠΌΠ°Π»ΡŒΠ½ΠΎΠΌΡƒ ΡƒΠ³Π»Ρƒ Π½Π°ΠΊΠ»ΠΎΠ½Π° плоскости ΠΏΠΎ ΠΎΡ‚Π½ΠΎΡˆΠ΅Π½ΠΈΡŽ ΠΊ Π³ΠΎΡ€ΠΈΠ·ΠΎΠ½Ρ‚Ρƒ, ΠΏΡ€ΠΈ ΠΊΠΎΡ‚ΠΎΡ€ΠΎΠΌ Ρ‚Π΅Π»ΠΎ, Π»Π΅ΠΆΠ°Ρ‰Π΅Π΅ Π½Π° этой плоскости, Π½Π°Ρ‡Π½Π΅Ρ‚ ΡΠΊΠΎΠ»ΡŒΠ·ΠΈΡ‚ΡŒ Π²Π½ΠΈΠ· ΠΏΠΎΠ΄ воздСйствиСм силы тяТСсти. ΠŸΡ€ΠΈ этом выполняСтся равСнство:

Π˜ΡΡ‚ΠΈΠ½Π½Ρ‹ΠΉ коэффициСнт трСния

Π—Π°ΠΊΠΎΠ½ трСния, ΠΊΠΎΡ‚ΠΎΡ€Ρ‹ΠΉ ΡƒΡ‡ΠΈΡ‚Ρ‹Π²Π°Π΅Ρ‚ влияниС сил притяТСния ΠΌΠ΅ΠΆΠ΄Ρƒ ΠΌΠΎΠ»Π΅ΠΊΡƒΠ»Π°ΠΌΠΈ, трущихся повСрхностСй Π·Π°ΠΏΠΈΡΡ‹Π²Π°ΡŽ ΡΠ»Π΅Π΄ΡƒΡŽΡ‰ΠΈΠΌ ΠΎΠ±Ρ€Π°Π·ΠΎΠΌ:

Π³Π΄Π΅ β€” Π½Π°Π·Ρ‹Π²Π°ΡŽΡ‚ истинным коэффициСнтом трСния, β€” Π΄ΠΎΠ±Π°Π²ΠΎΡ‡Π½ΠΎΠ΅ Π΄Π°Π²Π»Π΅Π½ΠΈΠ΅, ΠΊΠΎΡ‚ΠΎΡ€ΠΎΠ΅ вызываСтся силами мСТмолСкулярного притяТСния, S β€” общая ΠΏΠ»ΠΎΡ‰Π°Π΄ΡŒ нСпосрСдствСнного ΠΊΠΎΠ½Ρ‚Π°ΠΊΡ‚Π° трущихся Ρ‚Π΅Π».

ΠšΠΎΡΡ„Ρ„ΠΈΡ†ΠΈΠ΅Π½Ρ‚ трСния качСния

ΠšΠΎΡΡ„Ρ„ΠΈΡ†ΠΈΠ΅Π½Ρ‚ трСния качСния (k) ΠΌΠΎΠΆΠ½ΠΎ ΠΎΠΏΡ€Π΅Π΄Π΅Π»ΠΈΡ‚ΡŒ ΠΊΠ°ΠΊ ΠΎΡ‚Π½ΠΎΡˆΠ΅Π½ΠΈΠ΅ ΠΌΠΎΠΌΠ΅Π½Ρ‚Π° силы трСния качСния () ΠΊ силС с ΠΊΠΎΡ‚ΠΎΡ€ΠΎΠΉ Ρ‚Π΅Π»ΠΎ приТимаСтся ΠΊ ΠΎΠΏΠΎΡ€Π΅ (N):

ΠžΡ‚ΠΌΠ΅Ρ‚ΠΈΠΌ, Ρ‡Ρ‚ΠΎ коэффициСнт трСния качСния ΠΎΠ±ΠΎΠ·Π½Π°Ρ‡Π°ΡŽΡ‚ Ρ‡Π°Ρ‰Π΅ Π±ΡƒΠΊΠ²ΠΎΠΉ . Π­Ρ‚ΠΎΡ‚ коэффициСнт, Π² ΠΎΡ‚Π»ΠΈΡ‡ΠΈΠ΅ ΠΎΡ‚ Π²Ρ‹ΡˆΠ΅ пСрСчислСнных коэффициСнтов трСния, ΠΈΠΌΠ΅Π΅Ρ‚ Ρ€Π°Π·ΠΌΠ΅Ρ€Π½ΠΎΡΡ‚ΡŒ Π΄Π»ΠΈΠ½Ρ‹. Π’ΠΎ Π΅ΡΡ‚ΡŒ Π² систСмС БИ ΠΎΠ½ измСряСтся Π² ΠΌΠ΅Ρ‚Ρ€Π°Ρ….

ΠšΠΎΡΡ„Ρ„ΠΈΡ†ΠΈΠ΅Π½Ρ‚ трСния качСния ΠΌΠ½ΠΎΠ³ΠΎ мСньшС, Ρ‡Π΅ΠΌ коэффициСнт трСния скольТСния.

ΠŸΡ€ΠΈΠΌΠ΅Ρ€Ρ‹ Ρ€Π΅ΡˆΠ΅Π½ΠΈΡ Π·Π°Π΄Π°Ρ‡

ΠŸΠ Π˜ΠœΠ•Π  1

Π—Π°Π΄Π°Π½ΠΈΠ΅Π’Π΅Ρ€Π΅Π²ΠΊΠ° Π»Π΅ΠΆΠΈΡ‚ частично Π½Π° столС, Ρ‡Π°ΡΡ‚ΡŒ Π΅Π΅ ΡΠ²Π΅ΡˆΠΈΠ²Π°Π΅Ρ‚ΡΡ со стола. Если Ρ‚Ρ€Π΅Ρ‚ΡŒ Π΄Π»ΠΈΠ½Ρ‹ Π²Π΅Ρ€Π΅Π²ΠΊΠΈ свСсится со стола, Ρ‚ΠΎ ΠΎΠ½Π° Π½Π°Ρ‡ΠΈΠ½Π°Π΅Ρ‚ ΡΠΊΠΎΠ»ΡŒΠ·ΠΈΡ‚ΡŒ. Каков коэффициСнт трСния Π²Π΅Ρ€Π΅Π²ΠΊΠΈ ΠΎ стол?
Π Π΅ΡˆΠ΅Π½ΠΈΠ΅Π’Π΅Ρ€Π΅Π²ΠΊΠ° ΡΠΊΠΎΠ»ΡŒΠ·ΠΈΡ‚ со стола ΠΏΠΎΠ΄ дСйствиСм силы тяТСсти. ΠžΠ±ΠΎΠ·Π½Π°Ρ‡ΠΈΠΌ силу тяТСсти, которая дСйствуСт Π½Π° Π΅Π΄ΠΈΠ½ΠΈΡ†Ρƒ Π΄Π»ΠΈΠ½Ρ‹ Π²Π΅Ρ€Π΅Π²ΠΊΠΈ ΠΊΠ°ΠΊ . Π’ Ρ‚Π°ΠΊΠΎΠΌ случаС Π² ΠΌΠΎΠΌΠ΅Π½Ρ‚ Π½Π°Ρ‡Π°Π»Π° скольТСния сила тяТСсти, которая дСйствуСт Π½Π° ΡΠ²Π΅ΡˆΠΈΠ²Π°ΡŽΡ‰ΡƒΡŽΡΡ Ρ‡Π°ΡΡ‚ΡŒ Π²Π΅Ρ€Π΅Π²ΠΊΠΈ, Ρ€Π°Π²Π½Π°:

Π”ΠΎ Π½Π°Ρ‡Π°Π»Π° скольТСния эта сила ΡƒΡ€Π°Π²Π½ΠΎΠ²Π΅ΡˆΠΈΠ²Π°Π΅Ρ‚ΡΡ силой трСния, которая дСйствуСт Π½Π° Ρ‡Π°ΡΡ‚ΡŒ Π²Π΅Ρ€Π΅Π²ΠΊΠΈ, которая Π»Π΅ΠΆΠΈΡ‚ Π½Π° столС:

Π’Π°ΠΊ ΠΊΠ°ΠΊ силы ΡƒΡ€Π°Π²Π½ΠΎΠ²Π΅ΡˆΠΈΠ²Π°ΡŽΡ‚ΡΡ, Ρ‚ΠΎ ΠΌΠΎΠΆΠ½ΠΎ Π·Π°ΠΏΠΈΡΠ°Ρ‚ΡŒ ():

ΠžΡ‚Π²Π΅Ρ‚

ΠŸΠ Π˜ΠœΠ•Π  2

Π—Π°Π΄Π°Π½ΠΈΠ΅ΠšΠ°ΠΊΠΎΠ² коэффициСнт трСния Ρ‚Π΅Π»Π° ΠΎ ΠΏΠ»ΠΎΡΠΊΠΎΡΡ‚ΡŒ (), Ссли Π·Π°Π²ΠΈΡΠΈΠΌΠΎΡΡ‚ΡŒ ΠΏΡƒΡ‚ΠΈ, ΠΊΠΎΡ‚ΠΎΡ€ΠΎΠ΅ ΠΎΠ½ΠΎ ΠΏΡ€ΠΎΡ…ΠΎΠ΄ΠΈΡ‚ Π·Π°Π΄Π°Π½ΠΎ ΡƒΡ€Π°Π²Π½Π΅Π½ΠΈΠ΅ΠΌ: Π³Π΄Π΅ ΠŸΠ»ΠΎΡΠΊΠΎΡΡ‚ΡŒ составляСт ΡƒΠ³ΠΎΠ» с Π³ΠΎΡ€ΠΈΠ·ΠΎΠ½Ρ‚ΠΎΠΌ.
Π Π΅ΡˆΠ΅Π½ΠΈΠ΅Π—Π°ΠΏΠΈΡˆΠ΅ΠΌ Π²Ρ‚ΠΎΡ€ΠΎΠΉ Π·Π°ΠΊΠΎΠ½ ΠΡŒΡŽΡ‚ΠΎΠ½Π° для сил, ΠΏΡ€ΠΈΠ»ΠΎΠΆΠ΅Π½Π½Ρ‹Ρ… ΠΊ двиТущСмуся Ρ‚Π΅Π»Ρƒ:

Если брусок тянут с ΠΏΠΎΠΌΠΎΡ‰ΡŒΡŽ Π΄ΠΈΠ½Π°ΠΌΠΎΠΌΠ΅Ρ‚Ρ€Π° с постоянной ΡΠΊΠΎΡ€ΠΎΡΡ‚ΡŒΡŽ, Ρ‚ΠΎ Π΄ΠΈΠ½Π°ΠΌΠΎΠΌΠ΅Ρ‚Ρ€ ΠΏΠΎΠΊΠ°Π·Ρ‹Π²Π°Π΅Ρ‚ ΠΌΠΎΠ΄ΡƒΠ»ΡŒ силы трСния скольТСния (F Ρ‚Ρ€). Π—Π΄Π΅ΡΡŒ сила упругости ΠΏΡ€ΡƒΠΆΠΈΠ½Ρ‹ Π΄ΠΈΠ½Π°ΠΌΠΎΠΌΠ΅Ρ‚Ρ€Π° ΡƒΡ€Π°Π²Π½ΠΎΠ²Π΅ΡˆΠΈΠ²Π°Π΅Ρ‚ силу трСния скольТСния.

Π‘ Π΄Ρ€ΡƒΠ³ΠΎΠΉ стороны, сила трСния скольТСния зависит ΠΎΡ‚ силы Π½ΠΎΡ€ΠΌΠ°Π»ΡŒΠ½ΠΎΠΉ Ρ€Π΅Π°ΠΊΡ†ΠΈΠΈ ΠΎΠΏΠΎΡ€Ρ‹ (N), которая Π²ΠΎΠ·Π½ΠΈΠΊΠ°Π΅Ρ‚ Π² слСдствиС дСйствия вСса Ρ‚Π΅Π»Π°. Π§Π΅ΠΌ вСс большС, Ρ‚Π΅ΠΌ большС сила Π½ΠΎΡ€ΠΌΠ°Π»ΡŒΠ½ΠΎΠΉ Ρ€Π΅Π°ΠΊΡ†ΠΈΠΈ. И Ρ‡Π΅ΠΌ большС сила Π½ΠΎΡ€ΠΌΠ°Π»ΡŒΠ½ΠΎΠΉ Ρ€Π΅Π°ΠΊΡ†ΠΈΠΈ, Ρ‚Π΅ΠΌ большС сила трСния . ΠœΠ΅ΠΆΠ΄Ρƒ этими силами сущСствуСт прямая ΠΏΡ€ΠΎΠΏΠΎΡ€Ρ†ΠΈΠΎΠ½Π°Π»ΡŒΠ½Π°Ρ Π·Π°Π²ΠΈΡΠΈΠΌΠΎΡΡ‚ΡŒ, ΠΊΠΎΡ‚ΠΎΡ€ΡƒΡŽ ΠΌΠΎΠΆΠ½ΠΎ Π²Ρ‹Ρ€Π°Π·ΠΈΡ‚ΡŒ Ρ„ΠΎΡ€ΠΌΡƒΠ»ΠΎΠΉ:

Π—Π΄Π΅ΡΡŒ ΞΌ – это коэффициСнт трСния . Он ΠΏΠΎΠΊΠ°Π·Ρ‹Π²Π°Π΅Ρ‚, ΠΊΠ°ΠΊ ΠΈΠΌΠ΅Π½Π½ΠΎ сила трСния скольТСния зависит ΠΎΡ‚ силы Π½ΠΎΡ€ΠΌΠ°Π»ΡŒΠ½ΠΎΠΉ Ρ€Π΅Π°ΠΊΡ†ΠΈΠΈ (ΠΈΠ»ΠΈ, ΠΌΠΎΠΆΠ½ΠΎ ΡΠΊΠ°Π·Π°Ρ‚ΡŒ, ΠΎΡ‚ вСса Ρ‚Π΅Π»Π°), ΠΊΠ°ΠΊΡƒΡŽ долю ΠΎΡ‚ Π½Π΅Π΅ составляСт. ΠšΠΎΡΡ„Ρ„ΠΈΡ†ΠΈΠ΅Π½Ρ‚ трСния — бСзразмСрная Π²Π΅Π»ΠΈΡ‡ΠΈΠ½Π°. Для Ρ€Π°Π·Π½Ρ‹Ρ… ΠΏΠ°Ρ€ повСрхностСй ΞΌ ΠΈΠΌΠ΅Π΅Ρ‚ Ρ€Π°Π·Π½ΠΎΠ΅ Π·Π½Π°Ρ‡Π΅Π½ΠΈΠ΅.

Π’Π°ΠΊ, Π½Π°ΠΏΡ€ΠΈΠΌΠ΅Ρ€, дСрСвянныС ΠΏΡ€Π΅Π΄ΠΌΠ΅Ρ‚Ρ‹ трутся Π΄Ρ€ΡƒΠ³ ΠΎ Π΄Ρ€ΡƒΠ³Π° с коэффициСнтом ΠΎΡ‚ 0,2 Π΄ΠΎ 0,5 (Π² зависимости ΠΎΡ‚ Π²ΠΈΠ΄Π° дСрСвянных повСрхностСй). Π­Ρ‚ΠΎ Π·Π½Π°Ρ‡ΠΈΡ‚, Ρ‡Ρ‚ΠΎ Ссли сила Π½ΠΎΡ€ΠΌΠ°Π»ΡŒΠ½ΠΎΠΉ Ρ€Π΅Π°ΠΊΡ†ΠΈΠΈ ΠΎΠΏΠΎΡ€Ρ‹ 1 Н, Ρ‚ΠΎ ΠΏΡ€ΠΈ Π΄Π²ΠΈΠΆΠ΅Π½ΠΈΠΈ сила трСния скольТСния ΠΌΠΎΠΆΠ΅Ρ‚ ΡΠΎΡΡ‚Π°Π²ΠΈΡ‚ΡŒ Π·Π½Π°Ρ‡Π΅Π½ΠΈΠ΅, Π»Π΅ΠΆΠ°Ρ‰Π΅Π΅ Π² ΠΏΡ€ΠΎΠΌΠ΅ΠΆΡƒΡ‚ΠΊΠ΅ ΠΎΡ‚ 0,2 Н Π΄ΠΎ 0,5 Н.

Из Ρ„ΠΎΡ€ΠΌΡƒΠ»Ρ‹ F Ρ‚Ρ€ = ΞΌN слСдуСт, Ρ‡Ρ‚ΠΎ зная силы трСния ΠΈ Π½ΠΎΡ€ΠΌΠ°Π»ΡŒΠ½ΠΎΠΉ Ρ€Π΅Π°ΠΊΡ†ΠΈΠΈ, ΠΌΠΎΠΆΠ½ΠΎ ΠΎΠΏΡ€Π΅Π΄Π΅Π»ΠΈΡ‚ΡŒ коэффициСнт трСния для Π»ΡŽΠ±Ρ‹Ρ… повСрхностСй:

Π‘ΠΈΠ»Π° Π½ΠΎΡ€ΠΌΠ°Π»ΡŒΠ½ΠΎΠΉ Ρ€Π΅Π°ΠΊΡ†ΠΈΠΈ ΠΎΠΏΠΎΡ€Ρ‹ зависит ΠΎΡ‚ вСса Ρ‚Π΅Π»Π°. Она Ρ€Π°Π²Π½Π° Π΅ΠΌΡƒ ΠΏΠΎ ΠΌΠΎΠ΄ΡƒΠ»ΡŽ, Π½ΠΎ ΠΏΡ€ΠΎΡ‚ΠΈΠ²ΠΎΠΏΠΎΠ»ΠΎΠΆΠ½Π° ΠΏΠΎ Π½Π°ΠΏΡ€Π°Π²Π»Π΅Π½ΠΈΡŽ. ВСс Ρ‚Π΅Π»Π° (P) ΠΌΠΎΠΆΠ½ΠΎ Π²Ρ‹Ρ‡ΠΈΡΠ»ΠΈΡ‚ΡŒ, зная массу Ρ‚Π΅Π»Π°. Π’Π°ΠΊΠΈΠΌ ΠΎΠ±Ρ€Π°Π·ΠΎΠΌ, Ссли Π½Π΅ ΡƒΡ‡ΠΈΡ‚Ρ‹Π²Π°Ρ‚ΡŒ Π²Π΅ΠΊΡ‚ΠΎΡ€Π½ΠΎΡΡ‚ΡŒ Π²Π΅Π»ΠΈΡ‡ΠΈΠ½, ΠΌΠΎΠΆΠ½ΠΎ Π·Π°ΠΏΠΈΡΠ°Ρ‚ΡŒ, Ρ‡Ρ‚ΠΎ N = P = mg. Π’ΠΎΠ³Π΄Π° коэффициСнт трСния находится ΠΏΠΎ Ρ„ΠΎΡ€ΠΌΡƒΠ»Π΅:

ΞΌ = F Ρ‚Ρ€ / (mg)

НапримСр, Ссли извСстно, Ρ‡Ρ‚ΠΎ сила трСния Ρ‚Π΅Π»Π° массой 5 ΠΊΠ³, двиТущСгося ΠΏΠΎ повСрхности, Ρ€Π°Π²Π½Π° 12 Н, Ρ‚ΠΎ ΠΌΠΎΠΆΠ½ΠΎ Π½Π°ΠΉΡ‚ΠΈ коэффициСнт трСния: ΞΌ = 12 Н / (5 ΠΊΠ³ βˆ™ 9,8 Н/ΠΊΠ³) = 12 Н / 49 Н β‰ˆ 0,245.

БкольТСния: FΡ‚Ρ€ = ΠΌN, Π³Π΄Π΅ ΠΌ – коэффициСнт трСния скольТСния, N – сила Ρ€Π΅Π°ΠΊΡ†ΠΈΠΈ ΠΎΠΏΠΎΡ€Ρ‹, Н. Для Ρ‚Π΅Π»Π°, ΡΠΊΠΎΠ»ΡŒΠ·ΡΡ‰Π΅Π³ΠΎ ΠΏΠΎ Π³ΠΎΡ€ΠΈΠ·ΠΎΠ½Ρ‚Π°Π»ΡŒΠ½ΠΎΠΉ плоскости, N = G = mg, Π³Π΄Π΅ G — вСс Ρ‚Π΅Π»Π°, Н; m – масса Ρ‚Π΅Π»Π°, ΠΊΠ³; g – ускорСниС свободного падСния, ΠΌ/с2. ЗначСния Π±Π΅Π·Ρ€Π°Π·ΠΌΠ΅Ρ€Π½ΠΎΠ³ΠΎ коэффициСнта ΠΌ для Π΄Π°Π½Π½ΠΎΠΉ ΠΏΠ°Ρ€Ρ‹ ΠΌΠ°Ρ‚Π΅Ρ€ΠΈΠ°Π»ΠΎΠ² Π΄Π°Π½Ρ‹ Π² справочной . Зная массу Ρ‚Π΅Π»Π° ΠΈ ΠΏΠ°Ρ€Ρƒ ΠΌΠ°Ρ‚Π΅Ρ€ΠΈΠ°Π»ΠΎΠ². ΡΠΊΠΎΠ»ΡŒΠ·ΡΡ‰ΠΈΡ… Π΄Ρ€ΡƒΠ³ ΠΎΡ‚Π½ΠΎΡΠΈΡ‚Π΅Π»ΡŒΠ½ΠΎ Π΄Ρ€ΡƒΠ³Π°, Π½Π°ΠΉΠ΄ΠΈΡ‚Π΅ силу трСния.

Π‘Π»ΡƒΡ‡Π°ΠΉ 2. РассмотритС Ρ‚Π΅Π»ΠΎ, ΡΠΊΠΎΠ»ΡŒΠ·ΡΡ‰Π΅Π΅ ΠΏΠΎ Π³ΠΎΡ€ΠΈΠ·ΠΎΠ½Ρ‚Π°Π»ΡŒΠ½ΠΎΠΉ повСрхности ΠΈ Π΄Π²ΠΈΠ³Π°ΡŽΡ‰Π΅Π΅ΡΡ равноускорСнно. На Π½Π΅Π³ΠΎ Π΄Π΅ΠΉΡΡ‚Π²ΡƒΡŽΡ‚ Ρ‡Π΅Ρ‚Ρ‹Ρ€Π΅ силы: сила, приводящСС Ρ‚Π΅Π»ΠΎ Π² Π΄Π²ΠΈΠΆΠ΅Π½ΠΈΠ΅, сила тяТСсти, сила Ρ€Π΅Π°ΠΊΡ†ΠΈΠΈ ΠΎΠΏΠΎΡ€Ρ‹, сила трСния скольТСния. Π’Π°ΠΊ ΠΊΠ°ΠΊ ΠΏΠΎΠ²Π΅Ρ€Ρ…Π½ΠΎΡΡ‚ΡŒ Π³ΠΎΡ€ΠΈΠ·ΠΎΠ½Ρ‚Π°Π»ΡŒΠ½Π°Ρ, сила Ρ€Π΅Π°ΠΊΡ†ΠΈΠΈ ΠΎΠΏΠΎΡ€Ρ‹ ΠΈ сила тяТСсти Π½Π°ΠΏΡ€Π°Π²Π»Π΅Π½Ρ‹ вдоль ΠΎΠ΄Π½ΠΎΠΉ прямой ΠΈ ΡƒΡ€Π°Π²Π½ΠΎΠ²Π΅ΡˆΠΈΠ²Π°ΡŽΡ‚ Π΄Ρ€ΡƒΠ³ Π΄Ρ€ΡƒΠ³Π°. ΠŸΠ΅Ρ€Π΅ΠΌΠ΅Ρ‰Π΅Π½ΠΈΠ΅ описываСт ΡƒΡ€Π°Π²Π½Π΅Π½ΠΈΠ΅: FΠ΄Π² — FΡ‚Ρ€ = ma; Π³Π΄Π΅ FΠ΄Π² – ΠΌΠΎΠ΄ΡƒΠ»ΡŒ силы, приводящСй Ρ‚Π΅Π»ΠΎ Π² Π΄Π²ΠΈΠΆΠ΅Π½ΠΈΠ΅, Н; FΡ‚Ρ€ – ΠΌΠΎΠ΄ΡƒΠ»ΡŒ силы трСния, Н; m – масса Ρ‚Π΅Π»Π°, ΠΊΠ³; a – ускорСниС, ΠΌ/с2. Зная значСния массы, ускорСния Ρ‚Π΅Π»Π° ΠΈ силы, Π²ΠΎΠ·Π΄Π΅ΠΉΡΡ‚Π²ΡƒΡŽΡ‰Π΅ΠΉ Π½Π° Π½Π΅Π³ΠΎ, Π½Π°ΠΉΠ΄ΠΈΡ‚Π΅ силу трСния. Если эти значСния Π½Π΅ Π·Π°Π΄Π°Π½Ρ‹ прямо, посмотритС, Π΅ΡΡ‚ΡŒ Π»ΠΈ Π² условии Π΄Π°Π½Π½Ρ‹Π΅, ΠΈΠ· ΠΊΠΎΡ‚ΠΎΡ€Ρ‹Ρ… ΠΌΠΎΠΆΠ½ΠΎ Π½Π°ΠΉΡ‚ΠΈ эти Π²Π΅Π»ΠΈΡ‡ΠΈΠ½Ρ‹.

ΠŸΡ€ΠΈΠΌΠ΅Ρ€ Π·Π°Π΄Π°Ρ‡ΠΈ 1: Π½Π° брусок массой 5 ΠΊΠ³, Π»Π΅ΠΆΠ°Ρ‰ΠΈΠΉ Π½Π° повСрхности, Π²ΠΎΠ·Π΄Π΅ΠΉΡΡ‚Π²ΡƒΡŽΡ‚ силой 10 Н.2 = 0,8 ΠΌ/с2. Π’Π΅ΠΏΠ΅Ρ€ΡŒ Π½Π°ΠΉΠ΄ΠΈΡ‚Π΅ силу трСния: FΡ‚Ρ€ = ma = 0,8*1 = 0,8 Н.

Π‘Π»ΡƒΡ‡Π°ΠΉ 4. На Ρ‚Π΅Π»ΠΎ, ΡΠ°ΠΌΠΎΠΏΡ€ΠΎΠΈΠ·Π²ΠΎΠ»ΡŒΠ½ΠΎ ΡΠΊΠΎΠ»ΡŒΠ·ΡΡ‰Π΅Π΅ ΠΏΠΎ Π½Π°ΠΊΠ»ΠΎΠ½Π½ΠΎΠΉ плоскости, Π΄Π΅ΠΉΡΡ‚Π²ΡƒΡŽΡ‚ Ρ‚Ρ€ΠΈ силы: сила тяТСсти (G), сила Ρ€Π΅Π°ΠΊΡ†ΠΈΠΈ ΠΎΠΏΠΎΡ€Ρ‹ (N) ΠΈ сила трСния (FΡ‚Ρ€). Π‘ΠΈΠ»Π° тяТСсти ΠΌΠΎΠΆΠ΅Ρ‚ Π±Ρ‹Ρ‚ΡŒ записана Π² Ρ‚Π°ΠΊΠΎΠΌ Π²ΠΈΠ΄Π΅: G = mg, Н, Π³Π΄Π΅ m – масса Ρ‚Π΅Π»Π°, ΠΊΠ³; g – ускорСниС свободного падСния, ΠΌ/с2. ΠŸΠΎΡΠΊΠΎΠ»ΡŒΠΊΡƒ эти силы Π½Π°ΠΏΡ€Π°Π²Π»Π΅Π½Ρ‹ Π½Π΅ вдоль ΠΎΠ΄Π½ΠΎΠΉ прямой, Π·Π°ΠΏΠΈΡˆΠΈΡ‚Π΅ ΡƒΡ€Π°Π²Π½Π΅Π½ΠΈΠ΅ двиТСния Π² Π²Π΅ΠΊΡ‚ΠΎΡ€Π½ΠΎΠΌ Π²ΠΈΠ΄Π΅.

Π‘Π»ΠΎΠΆΠΈΠ² ΠΏΠΎ ΠΏΡ€Π°Π²ΠΈΠ»Ρƒ ΠΏΠ°Ρ€Π°Π»Π»Π΅Π»ΠΎΠ³Ρ€Π°ΠΌΠΌΠ° силы N ΠΈ mg, Π²Ρ‹ ΠΏΠΎΠ»ΡƒΡ‡ΠΈΡ‚Π΅ Ρ€Π΅Π·ΡƒΠ»ΡŒΡ‚ΠΈΡ€ΡƒΡŽΡ‰ΡƒΡŽ силу F’. Из рисунка ΠΌΠΎΠΆΠ½ΠΎ ΡΠ΄Π΅Π»Π°Ρ‚ΡŒ Π²Ρ‹Π²ΠΎΠ΄Ρ‹: N = mg*cosΞ±; F’ = mg*sinΞ±. Π“Π΄Π΅ Ξ± – ΡƒΠ³ΠΎΠ» Π½Π°ΠΊΠ»ΠΎΠ½Π° плоскости. Π‘ΠΈΠ»Ρƒ трСния ΠΌΠΎΠΆΠ½ΠΎ Π·Π°ΠΏΠΈΡΠ°Ρ‚ΡŒ Ρ„ΠΎΡ€ΠΌΡƒΠ»ΠΎΠΉ: FΡ‚Ρ€ = ΠΌ*N = ΠΌ*mg*cosΞ±. Π£Ρ€Π°Π²Π½Π΅Π½ΠΈΠ΅ для двиТСния ΠΏΡ€ΠΈΠ½ΠΈΠΌΠ°Π΅Ρ‚ Π²ΠΈΠ΄: F’-FΡ‚Ρ€ = ma. Или: FΡ‚Ρ€ = mg*sinΞ±-ma.

Π‘Π»ΡƒΡ‡Π°ΠΉ 6. Π’Π΅Π»ΠΎ двигаСтся ΠΏΠΎ Π½Π°ΠΊΠ»ΠΎΠ½Π½ΠΎΠΉ повСрхности Ρ€Π°Π²Π½ΠΎΠΌΠ΅Ρ€Π½ΠΎ. Π—Π½Π°Ρ‡ΠΈΡ‚, ΠΏΠΎ Π²Ρ‚ΠΎΡ€ΠΎΠΌΡƒ Π·Π°ΠΊΠΎΠ½Ρƒ ΠΡŒΡŽΡ‚ΠΎΠ½Π° систСма находится Π² равновСсии. Если скольТСниС ΡΠ°ΠΌΠΎΠΏΡ€ΠΎΠΈΠ·Π²ΠΎΠ»ΡŒΠ½ΠΎΠ΅, Π΄Π²ΠΈΠΆΠ΅Π½ΠΈΠ΅ Ρ‚Π΅Π»Π° подчиняСтся ΡƒΡ€Π°Π²Π½Π΅Π½ΠΈΡŽ: mg*sinΞ± = FΡ‚Ρ€.

Если ΠΆΠ΅ ΠΊ Ρ‚Π΅Π»Ρƒ ΠΏΡ€ΠΈΠ»ΠΎΠΆΠ΅Π½Π° Π΄ΠΎΠΏΠΎΠ»Π½ΠΈΡ‚Π΅Π»ΡŒΠ½Π°Ρ сила (F), ΠΏΡ€Π΅ΠΏΡΡ‚ΡΡ‚Π²ΡƒΡŽΡ‰Π°Ρ равноускорСнному ΠΏΠ΅Ρ€Π΅ΠΌΠ΅Ρ‰Π΅Π½ΠΈΡŽ, Π²Ρ‹Ρ€Π°ΠΆΠ΅Π½ΠΈΠ΅ для двиТСния ΠΈΠΌΠ΅Π΅Ρ‚ Π²ΠΈΠ΄: mg*sinα–FΡ‚Ρ€-F = 0. ΠžΡ‚ΡΡŽΠ΄Π° Π½Π°ΠΉΠ΄ΠΈΡ‚Π΅ силу трСния: FΡ‚Ρ€ = mg*sinΞ±-F.

Найти коэффициСнт трСния Ссли. Как ΠΎΠΏΡ€Π΅Π΄Π΅Π»ΠΈΡ‚ΡŒ коэффициСнт трСния скольТСния? Π—Π°Π΄Π°Ρ‡Π° Π½Π° вычислСниС Β΅t

Лабораторная Ρ€Π°Π±ΠΎΡ‚Π° β„– 3 Β«Π˜Π·ΠΌΠ΅Ρ€Π΅Π½ΠΈΠ΅ коэффициСнта трСния скольТСния»

ЦСль Ρ€Π°Π±ΠΎΡ‚Ρ‹: Π½Π°ΠΉΡ‚ΠΈ коэффициСнт трСния дрСвСсного бруска, ΡΠΊΠΎΠ»ΡŒΠ·ΡΡ‰Π΅Π³ΠΎ ΠΏΠΎ дрСвСсной Π»ΠΈΠ½Π΅ΠΉΠΊΠ΅, ΠΈΡΠΏΠΎΠ»ΡŒΠ·ΡƒΡ Ρ„ΠΎΡ€ΠΌΡƒΠ»Ρƒ F Ρ‚Ρ€ = = ΞΌΠ . ΠŸΡ€ΠΈ ΠΏΠΎΠΌΠΎΡ‰ΠΈ Π΄ΠΈΠ½Π°ΠΌΠΎΠΌΠ΅Ρ‚Ρ€Π° ΠΎΠΏΡ€Π΅Π΄Π΅Π»ΡΡŽΡ‚ силу, с ΠΊΠΎΡ‚ΠΎΡ€ΠΎΠΉ Π½Π΅ΠΎΠ±Ρ…ΠΎΠ΄ΠΈΠΌΠΎ Ρ‚ΡΠ½ΡƒΡ‚ΡŒ брусок с Π³Ρ€ΡƒΠ·Π°ΠΌΠΈ ΠΏΠΎ Π³ΠΎΡ€ΠΈΠ·ΠΎΠ½Ρ‚Π°Π»ΡŒΠ½ΠΎΠΉ повСрхности Ρ‚Π°ΠΊ, Ρ‡Ρ‚ΠΎΠ± ΠΎΠ½ двигался ΡƒΠΌΠ΅Ρ€Π΅Π½Π½ΠΎ. Π­Ρ‚Π° сила Ρ€Π°Π²Π½Π° ΠΏΠΎ ΠΌΠΎΠ΄ΡƒΠ»ΡŽ силС трСния F Ρ‚p , Π΄Π΅ΠΉΡΡ‚Π²ΡƒΡŽΡ‰Π΅ΠΉ Π½Π° брусок. ΠŸΡ€ΠΈ ΠΏΠΎΠΌΠΎΡ‰ΠΈ Ρ‚Π°ΠΊΠΎΠ³ΠΎ ΠΆΠ΅ Π΄ΠΈΠ½Π°ΠΌΠΎΠΌΠ΅Ρ‚Ρ€Π° ΠΌΠΎΠΆΠ½ΠΎ ΠΎΡ‚Ρ‹ΡΠΊΠ°Ρ‚ΡŒ вСс бруска с Π³Ρ€ΡƒΠ·ΠΎΠΌ. Π­Ρ‚ΠΎΡ‚ вСс ΠΏΠΎ ΠΌΠΎΠ΄ΡƒΠ»ΡŽ Ρ€Π°Π²Π΅Π½ силС ΠΎΠ±Ρ‹Ρ‡Π½ΠΎΠ³ΠΎ давлСния N бруска Π½Π° ΠΏΠΎΠ²Π΅Ρ€Ρ…Π½ΠΎΡΡ‚ΡŒ, ΠΏΠΎ ΠΊΠΎΡ‚ΠΎΡ€ΠΎΠΉ ΠΎΠ½ ΡΠΊΠΎΠ»ΡŒΠ·ΠΈΡ‚. ΠžΠΏΡ€Π΅Π΄Π΅Π»ΠΈΠ² Ρ‚Π°ΠΊΠΈΠΌ ΠΌΠ°ΠΊΠ°Ρ€ΠΎΠΌ значСния силы трСния ΠΏΡ€ΠΈ Ρ€Π°Π·Π½Ρ‹Ρ… значСниях силы ΠΎΠ±Ρ‹Ρ‡Π½ΠΎΠ³ΠΎ давлСния, Π½ΡƒΠΆΠ½ΠΎ Π²Ρ‹ΡΡ‚Ρ€ΠΎΠΈΡ‚ΡŒ Π³Ρ€Π°Ρ„ΠΈΠΊ зависимости F Ρ‚Ρ€ ΠΎΡ‚ Π  ΠΈ Π½Π°ΠΉΡ‚ΠΈ срСднСС Π·Π½Π°Ρ‡Π΅Π½ΠΈΠ΅ коэффициСнта трСния (см. Ρ€Π°Π±ΠΎΡ‚Ρƒ β„– 2).

ΠšΠΎΡΡ„Ρ„ΠΈΡ†ΠΈΠ΅Π½Ρ‚ трСния — Π€ΠΈΠ·ΠΈΠΊΠ° Π² ΠΎΠΏΡ‹Ρ‚Π°Ρ… ΠΈ экспСримСнтах

Π“Π»Π°Π²Π½Ρ‹ΠΌ ΠΈΠ·ΠΌΠ΅Ρ€ΠΈΡ‚Π΅Π»ΡŒΠ½Ρ‹ΠΌ устройством Π² этой Ρ€Π°Π±ΠΎΡ‚Π΅ являСтся Π΄ΠΈΠ½Π°ΠΌΠΎΠΌΠ΅Ρ‚Ρ€. Π”ΠΈΠ½Π°ΠΌΠΎΠΌΠ΅Ρ‚Ρ€ ΠΈΠΌΠ΅Π΅Ρ‚ ΠΏΠΎΠ³Ρ€Π΅ΡˆΠ½ΠΎΡΡ‚ΡŒ Ξ” Π΄ =0,05 Н. Она ΠΈ Ρ€Π°Π²Π½Π° ΠΏΠΎΠ³Ρ€Π΅ΡˆΠ½ΠΎΡΡ‚ΠΈ измСрСния, Ссли ΡƒΠΊΠ°Π·Π°Ρ‚Π΅Π»ΡŒ совпадаСт со ΡˆΡ‚Ρ€ΠΈΡ…ΠΎΠΌ ΡˆΠΊΠ°Π»Ρ‹. Если ΠΆΠ΅ ΡƒΠΊΠ°Π·Π°Ρ‚Π΅Π»ΡŒ Π² процСссС измСрСния Π½Π΅ совпадаСт со ΡˆΡ‚Ρ€ΠΈΡ…ΠΎΠΌ ΡˆΠΊΠ°Π»Ρ‹ (Π»ΠΈΠ±ΠΎ колСблСтся), Ρ‚ΠΎ ΠΏΠΎΠ³Ρ€Π΅ΡˆΠ½ΠΎΡΡ‚ΡŒ измСрСния силы Ρ€Π°Π²Π½Π° Ξ”F = = 0,1 Н.

БрСдства измСрСния: Π΄ΠΈΠ½Π°ΠΌΠΎΠΌΠ΅Ρ‚Ρ€.

ΠœΠ°Ρ‚Π΅Ρ€ΠΈΠ°Π»Ρ‹: 1) дрСвСсный брусок; 2) дрСвСсная Π»ΠΈΠ½Π΅ΠΉΠΊΠ°; 3) Π½Π°Π±ΠΎΡ€ Π³Ρ€ΡƒΠ·ΠΎΠ².

ΠŸΠΎΡ€ΡΠ΄ΠΎΠΊ выполнСния Ρ€Π°Π±ΠΎΡ‚Ρ‹.

1. ΠŸΠΎΠ»ΠΎΠΆΠΈΡ‚Π΅ брусок Π½Π° Π³ΠΎΡ€ΠΈΠ·ΠΎΠ½Ρ‚Π°Π»ΡŒΠ½ΠΎ Ρ€Π°ΡΠΏΠΎΠ»ΠΎΠΆΠ΅Π½Π½ΡƒΡŽ Π΄Ρ€Π΅Π²Π΅ΡΠ½ΡƒΡŽ Π»ΠΈΠ½Π΅ΠΉΠΊΡƒ. На брусок ΠΏΠΎΡΡ‚Π°Π²ΡŒΡ‚Π΅ Π³Ρ€ΡƒΠ·.

2. ΠŸΡ€ΠΈΠΊΡ€Π΅ΠΏΠΈΠ² ΠΊ бруску Π΄ΠΈΠ½Π°ΠΌΠΎΠΌΠ΅Ρ‚Ρ€, ΠΊΠ°ΠΊ ΠΌΠΎΠΆΠ½ΠΎ Π±ΠΎΠ»Π΅Π΅ ΡƒΠΌΠ΅Ρ€Π΅Π½Π½ΠΎ тянитС Π΅Π³ΠΎ вдоль Π»ΠΈΠ½Π΅ΠΉΠΊΠΈ. Π—Π°ΠΌΠ΅Ρ€ΡŒΡ‚Π΅ ΠΏΡ€ΠΈ всСм этом ΠΏΠΎΠΊΠ°Π·Π°Π½ΠΈΠ΅ Π΄ΠΈΠ½Π°ΠΌΠΎΠΌΠ΅Ρ‚Ρ€Π°.

3. Π’Π·Π²Π΅ΡΡŒΡ‚Π΅ брусок ΠΈ Π³Ρ€ΡƒΠ·.

4. К ΠΏΠ΅Ρ€Π²ΠΎΠΌΡƒ Π³Ρ€ΡƒΠ·Ρƒ Π΄ΠΎΠ±Π°Π²ΡŒΡ‚Π΅ 2-ΠΎΠΉ, 3-ΠΈΠΉ Π³Ρ€ΡƒΠ·Ρ‹, всякий Ρ€Π°Π· взвСшивая брусок ΠΈ Π³Ρ€ΡƒΠ·Ρ‹ ΠΈ измСряя силу трСния.

По Ρ€Π΅Π·ΡƒΠ»ΡŒΡ‚Π°Ρ‚Π°ΠΌ ΠΈΠ·ΠΌΠ΅Ρ€Π΅Π½ΠΈΠΉ Π·Π°ΠΏΠΎΠ»Π½ΠΈΡ‚Π΅ Ρ‚Π°Π±Π»ΠΈΡ†Ρƒ:

5. По Ρ€Π΅Π·ΡƒΠ»ΡŒΡ‚Π°Ρ‚Π°ΠΌ ΠΈΠ·ΠΌΠ΅Ρ€Π΅Π½ΠΈΠΉ постройтС Π³Ρ€Π°Ρ„ΠΈΠΊ зависимости силы трСния ΠΎΡ‚ силы давлСния ΠΈ, ΠΏΠΎΠ»ΡŒΠ·ΡƒΡΡΡŒ ΠΈΠΌ, ΠΎΠ±ΡƒΡΠ»ΠΎΠ²ΡŒΡ‚Π΅ срСднСС Π·Π½Π°Ρ‡Π΅Π½ΠΈΠ΅ коэффициСнта трСния ΞΌ ср (см. Ρ€Π°Π±ΠΎΡ‚Ρƒ β„– 2).

6. ВысчитайтС Π½Π°ΠΈΠ²Ρ‹ΡΡˆΡƒΡŽ ΠΎΡ‚Π½ΠΎΡΠΈΡ‚Π΅Π»ΡŒΠ½ΡƒΡŽ ΠΏΠΎΠ³Ρ€Π΅ΡˆΠ½ΠΎΡΡ‚ΡŒ измСрСния коэффициСнта трСния. ΠŸΠΎΡ‚ΠΎΠΌΡƒ Ρ‡Ρ‚ΠΎ.

(см. Ρ„ΠΎΡ€ΠΌΡƒΠ»Ρƒ (1) Ρ€Π°Π±ΠΎΡ‚Ρ‹ β„– 2).

Из Ρ„ΠΎΡ€ΠΌΡƒΠ»Ρ‹ (1) слСдуСт, Ρ‡Ρ‚ΠΎ с большСй ΠΏΠΎΠ³Ρ€Π΅ΡˆΠ½ΠΎΡΡ‚ΡŒΡŽ ΠΈΠ·ΠΌΠ΅Ρ€Π΅Π½ коэффициСнт трСния Π² ΠΎΠΏΡ‹Ρ‚Π΅ с ΠΎΠ΄Π½ΠΈΠΌ Π³Ρ€ΡƒΠ·ΠΎΠΌ (ΠΏΠΎΡ‚ΠΎΠΌΡƒ Ρ‡Ρ‚ΠΎ Π² Π΄Π°Π½Π½ΠΎΠΌ случаС Π·Π½Π°ΠΌΠ΅Π½Π°Ρ‚Π΅Π»ΠΈ ΠΈΠΌΠ΅ΡŽΡ‚ мСньшСС Π·Π½Π°Ρ‡Π΅Π½ΠΈΠ΅) .

7. НайдитС Π°Π±ΡΠΎΠ»ΡŽΡ‚Π½ΡƒΡŽ ΠΏΠΎΠ³Ρ€Π΅ΡˆΠ½ΠΎΡΡ‚ΡŒ.

ΠΈ Π·Π°ΠΏΠΈΡˆΠΈΡ‚Π΅ ΠΎΡ‚Π²Π΅Ρ‚ Π² Π²ΠΈΠ΄Π΅:

ВрСбуСтся Π½Π°ΠΉΡ‚ΠΈ коэффициСнт трСния скольТСния дрСвСсного бруска, ΡΠΊΠΎΠ»ΡŒΠ·ΡΡ‰Π΅Π³ΠΎ ΠΏΠΎ дрСвСсной Π»ΠΈΠ½Π΅ΠΉΠΊΠ΅.

Π‘ΠΈΠ»Π° трСния скольТСния.

Π³Π΄Π΅ N — рСакция ΠΎΠΏΠΎΡ€Ρ‹; ΞΌ — ΠΊΠΎ.

эффициСнт трСния скольТСния, ΠΎΡ‚ΠΊΡƒΠ΄Π° ΞΌ=F Ρ‚Ρ€ /N;

Π‘ΠΈΠ»Π° трСния ΠΏΠΎ ΠΌΠΎΠ΄ΡƒΠ»ΡŽ Ρ€Π°Π²Π½Π° силС, Π½Π°ΠΏΡ€Π°Π²Π»Π΅Π½Π½ΠΎΠΉ ΠΏΠ°Ρ€Π°Π»Π»Π΅Π»ΡŒΠ½ΠΎ повСрхности скольТСния, которая трСбуСтся для Ρ€Π°Π²Π½ΠΎΠΌΠ΅Ρ€Π½ΠΎΠ³ΠΎ пСрСмСщСния бруска с Π³Ρ€ΡƒΠ·ΠΎΠΌ. РСакция ΠΎΠΏΠΎΡ€Ρ‹ ΠΏΠΎ ΠΌΠΎΠ΄ΡƒΠ»ΡŽ Ρ€Π°Π²Π½Π° вСсу бруска с Π³Ρ€ΡƒΠ·ΠΎΠΌ. Π˜Π·ΠΌΠ΅Ρ€Π΅Π½ΠΈΡ ΠΎΠ±ΠΎΠΈΡ… сил проводятся с ΠΏΠΎΠΌΠΎΡ‰ΡŒΡŽ школьного Π΄ΠΈΠ½Π°ΠΌΠΎΠΌΠ΅Ρ‚Ρ€Π°. ΠŸΡ€ΠΈ ΠΏΠ΅Ρ€Π΅ΠΌΠ΅Ρ‰Π΅Π½ΠΈΠΈ бруска ΠΏΠΎ Π»ΠΈΠ½Π΅ΠΉΠΊΠ΅ ΠΏΡ€ΠΈΠ½Ρ†ΠΈΠΏΠΈΠ°Π»ΡŒΠ½ΠΎ Π΄ΠΎΡΡ‚ΠΈΠ³Π½ΡƒΡ‚ΡŒ Ρ€Π°Π²Π½ΠΎΠΌΠ΅Ρ€Π½ΠΎΠ³ΠΎ Π΅Π³ΠΎ двиТСния, Ρ‡Ρ‚ΠΎΠ± показания Π΄ΠΈΠ½Π°ΠΌΠΎΠΌΠ΅Ρ‚Ρ€Π° ΠΎΡΡ‚Π°Π²Π°Π»ΠΈΡΡŒ Π½Π΅ΠΈΠ·ΠΌΠ΅Π½Π½Ρ‹ΠΌΠΈ ΠΈ ΠΈΡ… ΠΌΠΎΠΆΠ½ΠΎ Π±Ρ‹Π»ΠΎ ΠΏΠΎΡ‚ΠΎΡ‡Π½Π΅Π΅ Π½Π°ΠΉΡ‚ΠΈ.

ВСс бруска с Π³Ρ€ΡƒΠ·ΠΎΠΌ Π , Н.

РассчитаСм ΠΎΡ‚Π½ΠΎΡΠΈΡ‚Π΅Π»ΡŒΠ½ΡƒΡŽ ΠΏΠΎΠ³Ρ€Π΅ΡˆΠ½ΠΎΡΡ‚ΡŒ:

Π’ΠΈΠ΄Π½ΠΎ, Ρ‡Ρ‚ΠΎ большая ΠΎΡ‚Π½ΠΎΡΠΈΡ‚Π΅Π»ΡŒΠ½Π°Ρ ΠΏΠΎΠ³Ρ€Π΅ΡˆΠ½ΠΎΡΡ‚ΡŒ Π±ΡƒΠ΄Π΅Ρ‚ Π² ΠΎΠΏΡ‹Ρ‚Π΅ с ΠΌΠΈΠ½ΠΈΠΌΠ°Π»ΡŒΠ½Ρ‹ΠΌ Π³Ρ€ΡƒΠ·ΠΎΠΌ, Ρ‚.ΠΊ. Π·Π½Π°ΠΌΠ΅Π½Π°Ρ‚Π΅Π»ΡŒ мСньшС.

РассчитаСм Π°Π±ΡΠΎΠ»ΡŽΡ‚Π½ΡƒΡŽ ΠΏΠΎΠ³Ρ€Π΅ΡˆΠ½ΠΎΡΡ‚ΡŒ.

ΠŸΡ€ΠΈΠΎΠ±Ρ€Π΅Ρ‚Π΅Π½Π½Ρ‹ΠΉ Π² ΠΈΡ‚ΠΎΠ³Π΅ ΠΎΠΏΡ‹Ρ‚ΠΎΠ² коэффициСнт трСния скольТСния ΠΌΠΎΠΆΠ½ΠΎ Π·Π°ΠΏΠΈΡΠ°Ρ‚ΡŒ ΠΊΠ°ΠΊ: ΞΌ = 0,35 Β± 0,05.

Π’Ρ‹Π΄Π΅Π»ΠΈΡ‚Π΅ Π΅Ρ‘ ΠΌΡ‹ΡˆΠΊΠΎΠΉ ΠΈ Π½Π°ΠΆΠΌΠΈΡ‚Π΅ CTRL ENTER.

ΠžΠ³Ρ€ΠΎΠΌΠ½ΠΎΠ΅ спасибо всСм, ΠΊΡ‚ΠΎ ΠΏΠΎΠΌΠΎΠ³Π°Π΅Ρ‚ Π΄Π΅Π»Π°Ρ‚ΡŒ Π²Π΅Π±-сайт Π»ΡƒΡ‡ΡˆΠ΅! =)

ВСзисы

Как ΠΎΡ‚Ρ‹ΡΠΊΠ°Ρ‚ΡŒ силу трСния скольТСния f трСния Ρ„ΠΎΡ€ΠΌΡƒΠ»Π°. Π€ΠΎΡ€ΠΌΡƒΠ»Π° силы трСния. Она сущСствуСт всСгда, ΠΏΠΎΡ‚ΠΎΠΌΡƒ Ρ‡Ρ‚ΠΎ ΠΏΠΎΠ»Π½ΠΎΡΡ‚ΡŒΡŽ Π³Π»Π°Π΄ΠΊΠΈΡ… Ρ‚Π΅Π» Π½Π΅ Π±Ρ‹Π²Π°Π΅Ρ‚. ΠžΡ‚Ρ‹ΡΠΊΠ°Ρ‚ΡŒ силу трСния. Как Π½Π°ΠΉΡ‚ΠΈ коэффициСнт трСния ΠšΠΎΡΡ„Ρ„ΠΈΡ†ΠΈΠ΅Π½Ρ‚ трСния. Находим силу трСния. Π€ΠΎΡ€ΠΌΡƒΠ»Π° силы трСния. Π”Π΅Ρ‚Π°Π»ΠΈ Π°Π²Ρ‚ΠΎΠΌΠΎΠ±ΠΈΠ»Π΅ΠΉ Π±Π΅Π· смазки ΠŸΠ΅Ρ€Π΅Π΄ Ρ‚Π΅ΠΌ ΠΊΠ°ΠΊ Π½Π°ΠΉΡ‚ΠΈ силу трСния , коэффициСнта трСния. Π‘ΠΈΠ»Π° трСния. Π‘ΠΈΠ»Ρ‹ трСния, ΠΊΠ°ΠΊ ΠΈ Π² ΠΏΠΎΡ‡Ρ‚ΠΈ всСх случаях ΠΏΡ€ΠΈΠ±Π»ΠΈΠΆΠ΅Π½Π½ΠΎ силу трСния скольТСния ΠΌΠΎΠΆΠ½ΠΎ. ΠšΠžΠ­Π€Π€Π˜Π¦Π˜Π•ΠΠ’ Π’Π Π•ΠΠ˜Π― — это Π§Ρ‚ΠΎ Ρ‚Π°ΠΊΠΎΠ΅ ΠšΠžΠ­Π€Π€Π˜Π¦Π˜Π•ΠΠ’ Π’Π Π•ΠΠ˜Π―? Если ΠΎΠ±ΠΎΠ·Π½Π°Ρ‡ΠΈΡ‚ΡŒ вСс ΠΏΡ€Π΅Π΄ΠΌΠ΅Ρ‚Π° ΠΊΠ°ΠΊ N, Π° коэффициСнт Π’Π Π•ΠΠ˜Π― m, покоя опрСдСляСт силу. ΠšΠΎΡΡ„Ρ„ΠΈΡ†ΠΈΠ΅Π½Ρ‚ трСния Π­Ρ‚Ρƒ силу Π½ΡƒΠΆΠ½ΠΎ ΠΏΡ€Π΅ΠΎΠ΄ΠΎΠ»Π΅Ρ‚ΡŒ Ρ€Π°Π·Π»ΠΈΡ‡Π½ΠΎΠΉ Ρ‚ΠΎΠ»Ρ‰ΠΈΠ½Ρ‹ — ΠΊΠ°ΠΊ. Лабораторная Ρ€Π°Π±ΠΎΡ‚Π° β„– 3 Β«Π˜Π·ΠΌΠ΅Ρ€Π΅Π½ΠΈΠ΅ коэффициСнта трСния. Π“Π”Π— ΠΊ Лабораторная Ρ€Π°Π±ΠΎΡ‚Π° β„– 3 Β«Π˜Π·ΠΌΠ΅Ρ€Π΅Π½ΠΈΠ΅ коэффициСнта трСния ΠΊΠ°ΠΊ ΠΌΠΎΠΆΠ½ΠΎ силу трСния. ΠžΡ‚Π²Π΅Ρ‚Ρ‹ | Π›Π°Π±. ΠžΠΏΡ€Π΅Π΄Π΅Π»Π΅Π½ΠΈΠ΅ коэффициСнта трСния Как ΠΏΡ€ΠΈ ΠΏΠΎΠΌΠΎΡ‰ΠΈ Π»ΠΈΠ½Π΅ΠΉΠΊΠΈ, силу тяТСсти Π² направлСниях. НС Π±ΡƒΠ΄ΡŒ трСния — Π²Ρ€ΠΎΠ΄Π΅ Π±Ρ‹ ΠΌΡ‹ Π‘ ΡƒΡ‡Π΅Ρ‚ΠΎΠΌ коэффициСнта трСния ВычисляСм Π½ΠΎΡ€ΠΌΠ°Π»ΡŒΠ½ΡƒΡŽ силу f.

(ЗанятиС каникулярной ΡˆΠΊΠΎΠ»Ρ‹ для учащихся 8–9 ΠΊΠ».)

  • Активизация ΠΌΡ‹ΡΠ»ΠΈΡ‚Π΅Π»ΡŒΠ½ΠΎΠΉ Π΄Π΅ΡΡ‚Π΅Π»ΡŒΠ½ΠΎΡΡ‚ΠΈ учащихся.
  • Π€ΠΎΡ€ΠΌΠΈΡ€ΠΎΠ²Π°Π½ΠΈΠ΅ ΠΎΠ±ΠΎΠ±Ρ‰Π΅Π½Π½ΠΎΠ³ΠΎ умСния ΠΏΡ€ΠΎΠ²ΠΎΠ΄ΠΈΡ‚ΡŒ физичСскиС измСрСния.
  • Π€ΠΎΡ€ΠΌΠΈΡ€ΠΎΠ²Π°Π½ΠΈΠ΅ ΠΎΠ±ΠΎΠ±Ρ‰Π΅Π½Π½ΠΎΠ³ΠΎ умСния ΠΏΡ€ΠΎΠ²ΠΎΠ΄ΠΈΡ‚ΡŒ ΡΠΊΡΠΏΠ΅Ρ€ΠΈΠΌΠ΅Π½Ρ‚Π°Π»ΡŒΠ½ΡƒΡŽ ΠΏΡ€ΠΎΠ²Π΅Ρ€ΠΊΡƒ физичСских закономСрностСй.
  • Π€ΠΎΡ€ΠΌΠΈΡ€ΠΎΠ²Π°Π½ΠΈΠ΅ умСния ΡΠΈΡΡ‚Π΅ΠΌΠ°Ρ‚ΠΈΠ·ΠΈΡ€ΠΎΠ²Π°Ρ‚ΡŒ ΠΏΠΎΠ»ΡƒΡ‡Π΅Π½Π½Ρ‹Π΅ Ρ€Π΅Π·ΡƒΠ»ΡŒΡ‚Π°Ρ‚Ρ‹ Π² Π²ΠΈΠ΄Π΅ Ρ‚Π°Π±Π»ΠΈΡ†Ρ‹, ΡƒΠΌΠ΅Π½ΠΈΠ΅ Π΄Π΅Π»Π°Ρ‚ΡŒ Π²Ρ‹Π²ΠΎΠ΄ Π½Π° основС экспСримСнта.

ΠžΡ€Π³Π°Π½ΠΈΠ·Π°Ρ†ΠΈΡ провСдСния ΠΏΡ€Π°ΠΊΡ‚ΠΈΠΊΡƒΠΌΠ°: ВсС учащиСся ΠΏΡ€ΠΈΠ½ΠΈΠΌΠ°ΡŽΡ‰ΠΈΠ΅ участиС Π² Ρ€Π°Π±ΠΎΡ‚Π΅ ΠΏΡ€Π°ΠΊΡ‚ΠΈΠΊΡƒΠΌΠ° дСлятся Π½Π° Π³Ρ€ΡƒΠΏΠΏΡ‹. КаТдая Π³Ρ€ΡƒΠΏΠΏΠ° учащихся ΠΏΠΎΠ»ΡƒΡ‡Π°Π΅Ρ‚ Π·Π°Π΄Π°Π½ΠΈΠ΅ с ΠΊΡ€Π°Ρ‚ΠΊΠΈΠΌ описаниСм Ρ€Π°Π±ΠΎΡ‚Ρ‹.

По ΠΎΠΊΠΎΠ½Ρ‡Π°Π½ΠΈΠΈ выполнСния Ρ€Π°Π±ΠΎΡ‚Ρ‹ учащимся Π½Π΅ΠΎΠ±Ρ…ΠΎΠ΄ΠΈΠΌΠΎ ΡΠΎΡΡ‚Π°Π²ΠΈΡ‚ΡŒ ΠΎΡ‚Ρ‡Π΅Ρ‚. ΠžΡ‚Ρ‡Π΅Ρ‚ состоит ΠΈΠ· Ρ‚Π°Π±Π»ΠΈΡ†Ρ‹, вычислСния искомой Π²Π΅Π»ΠΈΡ‡ΠΈΠ½Ρ‹ ΠΈ Π΅Π΅ ΠΏΠΎΠ³Ρ€Π΅ΡˆΠ½ΠΎΡΡ‚ΠΈ, Π²Ρ‹Π²ΠΎΠ΄Π° ΠΏΠΎ Ρ€Π°Π±ΠΎΡ‚Π΅.

Π₯ΠΎΠ΄ Ρ€Π°Π±ΠΎΡ‚Ρ‹

I. Π’ΡΡ‚ΡƒΠΏΠΈΡ‚Π΅Π»ΡŒΠ½ΠΎΠ΅ слово учитСля:

Если ΠΏΠΎΠ»ΠΎΠΆΠΈΡ‚ΡŒ Π½Π° Π³ΠΎΡ€ΠΈΠ·ΠΎΠ½Ρ‚Π°Π»ΡŒΠ½ΡƒΡŽ ΠΏΠΎΠ²Π΅Ρ€Ρ…Π½ΠΎΡΡ‚ΡŒ брусок ΠΈ ΠΏΠΎΠ΄Π΅ΠΉΡΡ‚Π²ΠΎΠ²Π°Ρ‚ΡŒ Π½Π° Π½Π΅Π³ΠΎ с достаточной силой Π² Π³ΠΎΡ€ΠΈΠ·ΠΎΠ½Ρ‚Π°Π»ΡŒΠ½ΠΎΠΌ Π½Π°ΠΏΡ€Π°Π²Π»Π΅Π½ΠΈΠΈ, Ρ‚ΠΎ брусок станСт Π΄Π²ΠΈΠ³Π°Ρ‚ΡŒΡΡ. НСтрудно ΡƒΠ±Π΅Π΄ΠΈΡ‚ΡŒΡΡ, Ρ‡Ρ‚ΠΎ Π² этом случаС Π½Π° брусок Π΄Π΅ΠΉΡΡ‚Π²ΡƒΡŽΡ‚ Ρ‡Π΅Ρ‚Ρ‹Ρ€Π΅ силы: Π² Π²Π΅Ρ€Ρ‚ΠΈΠΊΠ°Π»ΡŒΠ½ΠΎΠΌ Π½Π°ΠΏΡ€Π°Π²Π»Π΅Π½ΠΈΠΈ – сила тяТСсти P ΠΈ сила Ρ€Π΅Π°ΠΊΡ†ΠΈΠΈ ΠΎΠΏΠΎΡ€Ρ‹ Q, Ρ€Π°Π²Π½Ρ‹Π΅ ΠΏΠΎ ΠΌΠΎΠ΄ΡƒΠ»ΡŽ ΠΏΡ€ΠΎΡ‚ΠΈΠ²ΠΎΠΏΠΎΠ»ΠΎΠΆΠ½Ρ‹Π΅ ΠΏΠΎ Π½Π°ΠΏΡ€Π°Π²Π»Π΅Π½ΠΈΡŽ; Π² Π³ΠΎΡ€ΠΈΠ·ΠΎΠ½Ρ‚Π°Π»ΡŒΠ½ΠΎΠΌ Π½Π°ΠΏΡ€Π°Π²Π»Π΅Π½ΠΈΠΈ – сила тяги F ΠΈ противополоТная ΠΏΠΎ Π½Π°ΠΏΡ€Π°Π²Π»Π΅Π½ΠΈΡŽ сила трСния F mp .

Π§Ρ‚ΠΎΠ±Ρ‹ брусок двигался Ρ€Π°Π²Π½ΠΎΠΌΠ΅Ρ€Π½ΠΎ ΠΈ прямолинСйно, Π½ΡƒΠΆΠ½ΠΎ, Ρ‡Ρ‚ΠΎΠ±Ρ‹ ΠΌΠΎΠ΄ΡƒΠ»ΡŒ силы тяги Π±Ρ‹Π» Ρ€Π°Π²Π΅Π½ ΠΌΠΎΠ΄ΡƒΠ»ΡŽ силы трСния.

На этом основан ΠΌΠ΅Ρ‚ΠΎΠ΄ измСрСния силы трСния. Π‘Π»Π΅Π΄ΡƒΠ΅Ρ‚ ΠΏΡ€ΠΈΠ»ΠΎΠΆΠΈΡ‚ΡŒ ΠΊ бруску силу тяги, которая Π±ΡƒΠ΄Π΅Ρ‚ ΠΏΠΎΠ΄Π΄Π΅Ρ€ΠΆΠΈΠ²Π°Ρ‚ΡŒ Ρ€Π°Π²Π½ΠΎΠΌΠ΅Ρ€Π½ΠΎΠ΅ прямолинСйноС Π΄Π²ΠΈΠΆΠ΅Π½ΠΈΠ΅ этого Ρ‚Π΅Π»Π°. По этой силС тяги ΠΎΠΏΡ€Π΅Π΄Π΅Π»ΡΡŽΡ‚ ΠΌΠΎΠ΄ΡƒΠ»ΡŒ силы трСния.

II. ΠŸΡ€Π°ΠΊΡ‚ΠΈΠΊΡƒΠΌ.

Π—Π°Π΄Π°Π½ΠΈΠ΅ Π³Ρ€ΡƒΠΏΠΏΠ΅ I.

ΠžΠΏΡ€Π΅Π΄Π΅Π»ΠΈΡ‚Π΅ коэффициСнт трСния скольТСния ΠΏΡ€ΠΈ Π΄Π²ΠΈΠΆΠ΅Π½ΠΈΠΈ бруска ΠΏΠΎ Π³ΠΎΡ€ΠΈΠ·ΠΎΠ½Ρ‚Π°Π»ΡŒΠ½ΠΎΠΉ повСрхности стола.

ΠžΠ±ΠΎΡ€ΡƒΠ΄ΠΎΠ²Π°Π½ΠΈΠ΅: Ρ‚Ρ€ΠΈΠ±ΠΎΠΌΠ΅Ρ‚Ρ€, дСрСвянная Π»ΠΈΠ½Π΅ΠΉΠΊΠ°, дСрСвянный бруска с трСмя отвСрстиями; Π΄ΠΈΠ½Π°ΠΌΠΎΠΌΠ΅Ρ‚Ρ€; Π½Π°Π±ΠΎΡ€ Π³Ρ€ΡƒΠ·ΠΎΠ² ΠΏΠΎ ΠΌΠ΅Ρ…Π°Π½ΠΈΠΊΠ΅.

ΠŸΠΎΡ€ΡΠ΄ΠΎΠΊ выполнСния Ρ€Π°Π±ΠΎΡ‚Ρ‹.

  1. ВычислитС Ρ†Π΅Π½Ρƒ дСлСния ΡˆΠΊΠ°Π»Ρ‹ Π΄ΠΈΠ½Π°ΠΌΠΎΠΌΠ΅Ρ‚Ρ€Π°.
  2. Π˜Π·ΠΌΠ΅Ρ€ΡŒΡ‚Π΅ вСс бруска ΠΏΡ€ΠΈ ΠΏΠΎΠΌΠΎΡ‰ΠΈ Π΄ΠΈΠ½Π°ΠΌΠΎΠΌΠ΅Ρ‚Ρ€Π°. Π Π΅Π·ΡƒΠ»ΡŒΡ‚Π°Ρ‚ измСрСния вСса Π·Π°ΠΏΠΈΡˆΠΈΡ‚Π΅ Π² Ρ‚Π°Π±Π»ΠΈΡ†Ρƒ.
  3. Π˜Π·ΠΌΠ΅Ρ€ΡŒΡ‚Π΅ силу трСния скольТСния бруска с Π³Ρ€ΡƒΠ·Π°ΠΌΠΈ ΠΏΠΎ столу. Для этого ΠΏΠ΅Ρ€Π΅ΠΌΠ΅Ρ‰Π°ΠΉΡ‚Π΅ брусок с Π³Ρ€ΡƒΠ·Π°ΠΌΠΈ Ρ€Π°Π²Π½ΠΎΠΌΠ΅Ρ€Π½ΠΎ ΠΏΠΎ столу ΠΏΡ€ΠΈ ΠΏΠΎΠΌΠΎΡ‰ΠΈ Π΄ΠΈΠ½Π°ΠΌΠΎΠΌΠ΅Ρ‚Ρ€Π°.
  4. Π Π΅Π·ΡƒΠ»ΡŒΡ‚Π°Ρ‚ измСрСния Π·Π°ΠΏΠΈΡˆΠΈΡ‚Π΅ Π² Ρ‚Π°Π±Π»ΠΈΡ†Ρƒ.
  5. НагруТая брусок ΠΎΠ΄Π½ΠΈΠΌ, двумя ΠΈ трСмя Π³Ρ€ΡƒΠ·Π°ΠΌΠΈ, ΠΈΠ·ΠΌΠ΅Ρ€ΡŒΡ‚Π΅ Π² ΠΊΠ°ΠΆΠ΄ΠΎΠΌ случаС силу трСния. Π”Π°Π½Π½Ρ‹Π΅ занСситС Π² Ρ‚Π°Π±Π»ΠΈΡ†Ρƒ.
  6. ВычислитС коэффициСнт трСния скольТСния
  7. ΠžΠΏΡ€Π΅Π΄Π΅Π»ΠΈΡ‚Π΅ ΠΈΠ½ΡΡ‚Ρ€ΡƒΠΌΠ΅Π½Ρ‚Π°Π»ΡŒΠ½ΡƒΡŽ ΠΏΠΎΠ³Ρ€Π΅ΡˆΠ½ΠΎΡΡ‚ΡŒ коэффициСнта трСния.
  8. Π‘Π΄Π΅Π»Π°ΠΉΡ‚Π΅ Π²Ρ‹Π²ΠΎΠ΄.

Π›Π΅Π³ΠΊΠΎ ΡƒΠ±Π΅Π΄ΠΈΡ‚ΡŒΡΡ, Ρ‡Ρ‚ΠΎ Π² случаС двиТСния Ρ‚Π΅Π»Π° ΠΏΠΎ Π³ΠΎΡ€ΠΈΠ·ΠΎΠ½Ρ‚Π°Π»ΡŒΠ½ΠΎΠΉ повСрхности сила Π½ΠΎΡ€ΠΌΠ°Π»ΡŒΠ½ΠΎΠ³ΠΎ давлСния Ρ€Π°Π²Π½Π° силС тяТСсти, Π΄Π΅ΠΉΡΡ‚Π²ΡƒΡŽΡ‰Π΅ΠΉ Π½Π° это Ρ‚Π΅Π»ΠΎ: N = P . Π­Ρ‚ΠΎ позволяСт Π²Ρ‹Ρ‡ΠΈΡΠ»ΠΈΡ‚ΡŒ коэффициСнт трСния:

Π¦Π΅Π½Π° дСлСния ΡˆΠΊΠ°Π»Ρ‹ Π΄ΠΈΠ½Π°ΠΌΠΎΠΌΠ΅Ρ‚Ρ€Π°, Ρ†.Π΄ = 0,1 Н.

1. ΠžΠΏΡ€Π΅Π΄Π΅Π»ΠΈΠ»ΠΈ вСс бруска ΠΈ Π³Ρ€ΡƒΠ·Π° с ΠΏΠΎΠΌΠΎΡ‰ΡŒΡŽ Π΄ΠΈΠ½Π°ΠΌΠΎΠΌΠ΅Ρ‚Ρ€Π°, записали Π² Ρ‚Π°Π±Π»ΠΈΡ†Ρƒ.

2. Двигая брусок Ρ€Π°Π²Π½ΠΎΠΌΠ΅Ρ€Π½ΠΎ ΠΏΠΎ дСрСвянной Π»ΠΈΠ½Π΅ΠΉΠΊΠ΅, ΠΎΠΏΡ€Π΅Π΄Π΅Π»ΠΈΠ»ΠΈ силу тяги, которая Ρ€Π°Π²Π½Π° силС трСния. Записали Π΅Π΅ Π·Π½Π°Ρ‡Π΅Π½ΠΈΠ΅ Π² Ρ‚Π°Π±Π»ΠΈΡ†Ρƒ.

3. ΠžΠΏΡ€Π΅Π΄Π΅Π»ΠΈΠ»ΠΈ коэффициСнт трСния для ΠΊΠ°ΠΆΠ΄ΠΎΠ³ΠΎ измСрСния силы трСния, занСсли ΠΈΡ… Π² Ρ‚Π°Π±Π»ΠΈΡ†Ρƒ.

4. ΠžΠΏΡ€Π΅Π΄Π΅Π»ΠΈΠ»ΠΈ ΠΏΠΎΠ³Ρ€Π΅ΡˆΠ½ΠΎΡΡ‚ΡŒ измСрСния для ΠΊΠ°ΠΆΠ΄ΠΎΠ³ΠΎ значСния коэффициСнта силы трСния.

1. ΠšΠΎΡΡ„Ρ„ΠΈΡ†ΠΈΠ΅Π½Ρ‚ трСния Ρ€Π°Π²Π΅Π½ 0,2.
2. Π˜Π½ΡΡ‚Ρ€ΡƒΠΌΠ΅Π½Ρ‚Π°Π»ΡŒΠ½Π°Ρ ΠΏΠΎΠ³Ρ€Π΅ΡˆΠ½ΠΎΡΡ‚ΡŒ измСрСния Ρ€Π°Π²Π½Π° 0,06.
3. ΠšΠΎΡΡ„Ρ„ΠΈΡ†ΠΈΠ΅Π½Ρ‚ трСния скольТСния ΠΏΡ€ΠΈ Π²Π·Π°ΠΈΠΌΠ½ΠΎΠΌ Π΄Π²ΠΈΠΆΠ΅Π½ΠΈΠΈ Ρ‚Π΅Π»Π° ΠΏΠΎ повСрхности стола являСтся Π²Π΅Π»ΠΈΡ‡ΠΈΠ½ΠΎΠΉ постоянной Π½Π΅ зависящСй ΠΎΡ‚ силы Π½ΠΎΡ€ΠΌΠ°Π»ΡŒΠ½ΠΎΠ³ΠΎ давлСния.

2. Π‘Ρ€Π°Π²Π½ΠΈΡ‚Π΅ коэффициСнт трСния покоя, скольТСния ΠΈ качСния. Π‘Π΄Π΅Π»Π°ΠΉΡ‚Π΅ Π²Ρ‹Π²ΠΎΠ΄.

ΠžΠ±ΠΎΡ€ΡƒΠ΄ΠΎΠ²Π°Π½ΠΈΠ΅: Π΄ΠΈΠ½Π°ΠΌΠΎΠΌΠ΅Ρ‚Ρ€, брусок дСрСвянный, Π³Ρ€ΡƒΠ·Ρ‹ с двумя ΠΊΡ€ΡŽΡ‡ΠΊΠ°ΠΌΠΈ – 2 ΡˆΡ‚., ΠΊΠ°Ρ€Π°Π½Π΄Π°ΡˆΠΈ ΠΊΡ€ΡƒΠ³Π»Ρ‹Π΅ – 2 ΡˆΡ‚.

ΠŸΠΎΡ€ΡΠ΄ΠΎΠΊ выполнСния Ρ€Π°Π±ΠΎΡ‚Ρ‹.

2. Π˜Π·ΠΌΠ΅Ρ€ΡŒΡ‚Π΅ вСс бруска с двумя Π³Ρ€ΡƒΠ·Π°ΠΌΠΈ ΠΏΡ€ΠΈ ΠΏΠΎΠΌΠΎΡ‰ΠΈ Π΄ΠΈΠ½Π°ΠΌΠΎΠΌΠ΅Ρ‚Ρ€Π°. Π Π΅Π·ΡƒΠ»ΡŒΡ‚Π°Ρ‚ измСрСния вСса Π·Π°ΠΏΠΈΡˆΠΈΡ‚Π΅ Π² Ρ‚Π΅Ρ‚Ρ€Π°Π΄ΡŒ.

3. Π˜Π·ΠΌΠ΅Ρ€ΡŒΡ‚Π΅ ΠΌΠ°ΠΊΡΠΈΠΌΠ°Π»ΡŒΠ½ΡƒΡŽ силу трСния покоя бруска ΠΏΠΎ столу. Для этого ΠΏΠΎΠ»ΠΎΠΆΠΈΡ‚Π΅ брусок Π½Π° стол, Π° Π½Π° брусок Π΄Π²Π° Π³Ρ€ΡƒΠ·Π°; ΠΊ бруску ΠΏΡ€ΠΈΡ†Π΅ΠΏΠΈΡ‚Π΅ Π΄ΠΈΠ½Π°ΠΌΠΎΠΌΠ΅Ρ‚Ρ€ ΠΈ ΠΏΡ€ΠΈΠ²Π΅Π΄ΠΈΡ‚Π΅ брусок с Π³Ρ€ΡƒΠ·Π°ΠΌΠΈ Π² Π΄Π²ΠΈΠΆΠ΅Π½ΠΈΠ΅. Π—Π°ΠΏΠΈΡˆΠΈΡ‚Π΅ показания Π΄ΠΈΠ½Π°ΠΌΠΎΠΌΠ΅Ρ‚Ρ€Π°, ΡΠΎΠΎΡ‚Π²Π΅Ρ‚ΡΡ‚Π²ΡƒΡŽΡ‰Π΅Π΅ Π½Π°Ρ‡Π°Π»Ρƒ двиТСния бруска.

4. Π˜Π·ΠΌΠ΅Ρ€ΡŒΡ‚Π΅ силу трСния скольТСния бруска с Π³Ρ€ΡƒΠ·Π°ΠΌΠΈ ΠΏΠΎ столу. Для этого ΠΏΠ΅Ρ€Π΅ΠΌΠ΅Ρ‰Π°ΠΉΡ‚Π΅ брусок с Π³Ρ€ΡƒΠ·Π°ΠΌΠΈ Ρ€Π°Π²Π½ΠΎΠΌΠ΅Ρ€Π½ΠΎ ΠΏΠΎ столу ΠΏΡ€ΠΈ ΠΏΠΎΠΌΠΎΡ‰ΠΈ Π΄ΠΈΠ½Π°ΠΌΠΎΠΌΠ΅Ρ‚Ρ€Π°. Π Π΅Π·ΡƒΠ»ΡŒΡ‚Π°Ρ‚ измСрСния силы Π·Π°ΠΏΠΈΡˆΠΈΡ‚Π΅ Π² Ρ‚Π΅Ρ‚Ρ€Π°Π΄ΡŒ.

5. Π˜Π·ΠΌΠ΅Ρ€ΡŒΡ‚Π΅ силу трСния качСния бруска ΠΏΠΎ столу. Для этого ΠΏΠΎΠ»ΠΎΠΆΠΈΡ‚Π΅ брусок с двумя Π³Ρ€ΡƒΠ·Π°ΠΌΠΈ Π½Π° Π΄Π²Π° ΠΊΡ€ΡƒΠ³Π»Ρ‹Ρ… ΠΊΠ°Ρ€Π°Π½Π΄Π°ΡˆΠ° ΠΈ ΠΏΠ΅Ρ€Π΅ΠΌΠ΅Ρ‰Π°ΠΉΡ‚Π΅ Ρ€Π°Π²Π½ΠΎΠΌΠ΅Ρ€Π½ΠΎ брусок ΠΏΠΎ столу ΠΏΡ€ΠΈ ΠΏΠΎΠΌΠΎΡ‰ΠΈ Π΄ΠΈΠ½Π°ΠΌΠΎΠΌΠ΅Ρ‚Ρ€Π°. Π Π΅Π·ΡƒΠ»ΡŒΡ‚Π°Ρ‚ измСрСния силы Π·Π°ΠΏΠΈΡˆΠΈΡ‚Π΅ Π² Ρ‚Π΅Ρ‚Ρ€Π°Π΄ΡŒ.

6. Π‘Π΄Π΅Π»Π°ΠΉΡ‚Π΅ Π²Ρ‹Π²ΠΎΠ΄ ΠΎ Ρ‚ΠΎΠΌ, какая сила большС:
Π°) вСс Ρ‚Π΅Π»Π° ΠΈΠ»ΠΈ максимальная сила трСния покоя?
Π±) максимальная сила трСния покоя ΠΈΠ»ΠΈ сила трСния скольТСния?
Π²) сила трСния скольТСния ΠΈΠ»ΠΈ сила трСния качСния?

7. Π‘Ρ€Π°Π²Π½ΠΈΡ‚Π΅ коэффициСнт трСния покоя, трСния скольТСния ΠΈ трСния качСния.

Π°) ВСс Ρ‚Π΅Π»Π° большС Ρ‡Π΅ΠΌ максимальная сила трСния покоя.

Π±) Максимальная сила трСния покоя большС Ρ‡Π΅ΠΌ сила трСния скольТСния.

Π²) Π‘ΠΈΠ»Π° трСния скольТСния большС Ρ‡Π΅ΠΌ сила трСния качСния.

Π³) ΠŸΡ€ΠΈ Π½Π΅ΠΈΠ·ΠΌΠ΅Π½Π½ΠΎΠΌ вСсС Ρ‚Π΅Π»Π°, наимСньшСС Π·Π½Π°Ρ‡Π΅Π½ΠΈΠ΅ коэффициСнт трСния ΠΈΠΌΠ΅Π΅Ρ‚ ΠΏΡ€ΠΈ ΠΊΠ°Ρ‡Π΅Π½ΠΈΠΈ Ρ‚Π΅Π»Π°, Π° наибольшСС Π² случаС покоя.

3. ΠžΠΏΡ€Π΅Π΄Π΅Π»ΠΈΡ‚Π΅ коэффициСнт трСния скольТСния ΠΏΡ€ΠΈ Π΄Π²ΠΈΠΆΠ΅Π½ΠΈΠΈ бруска вдоль повСрхности Ρ€Π΅Π·ΠΈΠ½Ρ‹, Π½Π΅ΡˆΠ»ΠΈΡ„ΠΎΠ²Π°Π½Π½ΠΎΠΉ дСрСвянной Ρ€Π΅ΠΉΠΊΠΈ, Π½Π°ΠΆΠ΄Π°Ρ‡Π½ΠΎΠΉ Π±ΡƒΠΌΠ°Π³ΠΈ.

ΠžΠ±ΠΎΡ€ΡƒΠ΄ΠΎΠ²Π°Π½ΠΈΠ΅: Π΄ΠΈΠ½Π°ΠΌΠΎΠΌΠ΅Ρ‚Ρ€, брусок дСрСвянный, Π³Ρ€ΡƒΠ·Ρ‹ с двумя ΠΊΡ€ΡŽΡ‡ΠΊΠ°ΠΌΠΈ – 2 ΡˆΡ‚., ΠΎΡ‚Ρ€Π΅Π· Π»ΠΈΠ½ΠΎΠ»Π΅ΡƒΠΌΠ°, дСрСвянная Π½Π΅ΡˆΠ»ΠΈΡ„ΠΎΠ²Π°Π½Π½Π°Ρ Ρ€Π΅ΠΉΠΊΠ°, наТдачная Π±ΡƒΠΌΠ°Π³Π°.

ΠŸΠΎΡ€ΡΠ΄ΠΎΠΊ выполнСния Ρ€Π°Π±ΠΎΡ‚Ρ‹.

1. ВычислитС Ρ†Π΅Π½Ρƒ дСлСния ΡˆΠΊΠ°Π»Ρ‹ Π΄ΠΈΠ½Π°ΠΌΠΎΠΌΠ΅Ρ‚Ρ€Π°.
2. Π˜Π·ΠΌΠ΅Ρ€ΡŒΡ‚Π΅ вСс бруска ΠΏΡ€ΠΈ ΠΏΠΎΠΌΠΎΡ‰ΠΈ Π΄ΠΈΠ½Π°ΠΌΠΎΠΌΠ΅Ρ‚Ρ€Π°. Π Π΅Π·ΡƒΠ»ΡŒΡ‚Π°Ρ‚ измСрСния вСса Π·Π°ΠΏΠΈΡˆΠΈΡ‚Π΅ Π² Ρ‚Π°Π±Π»ΠΈΡ†Ρƒ.
3. Π˜Π·ΠΌΠ΅Ρ€ΡŒΡ‚Π΅ силу трСния скольТСния бруска с Π³Ρ€ΡƒΠ·Π°ΠΌΠΈ ΠΏΠΎ повСрхности Ρ€Π΅Π·ΠΈΠ½Ρ‹, дСрСвянной Π½Π΅ΡˆΠ»ΠΈΡ„ΠΎΠ²Π°Π½Π½ΠΎΠΉ Π»ΠΈΠ½Π΅ΠΉΠΊΠΈ ΠΈ ΠΏΠΎ повСрхности Π½Π°ΠΆΠ΄Π°Ρ‡Π½ΠΎΠΉ Π±ΡƒΠΌΠ°Π³ΠΈ. Для этого ΠΏΠ΅Ρ€Π΅ΠΌΠ΅Ρ‰Π°ΠΉΡ‚Π΅ брусок с Π³Ρ€ΡƒΠ·Π°ΠΌΠΈ Ρ€Π°Π²Π½ΠΎΠΌΠ΅Ρ€Π½ΠΎ ΠΏΠΎ столу ΠΏΡ€ΠΈ ΠΏΠΎΠΌΠΎΡ‰ΠΈ Π΄ΠΈΠ½Π°ΠΌΠΎΠΌΠ΅Ρ‚Ρ€Π°. Π Π΅Π·ΡƒΠ»ΡŒΡ‚Π°Ρ‚ измСрСния Π·Π°ΠΏΠΈΡˆΠΈΡ‚Π΅ Π² Ρ‚Π°Π±Π»ΠΈΡ†Ρƒ.
4. ВычислитС коэффициСнт трСния скольТСния.
5. Π‘Π΄Π΅Π»Π°ΠΉΡ‚Π΅ Π²Ρ‹Π²ΠΎΠ΄.

Π¦Π΅Π½Π° дСлСния ΡˆΠΊΠ°Π»Ρ‹ Π΄ΠΈΠ½Π°ΠΌΠΎΠΌΠ΅Ρ‚Ρ€Π°, Ρ†.Π΄ = 0,1 Н.

1. Π‘ΠΈΠ»Π° трСния:

Π°) зависит ΠΎΡ‚ Ρ€ΠΎΠ΄Π° трущихся повСрхностСй.
Π±) зависит ΠΎΡ‚ ΡˆΠ΅Ρ€ΠΎΡ…ΠΎΠ²Π°Ρ‚ΠΎΡΡ‚ΠΈ трущихся повСрхностСй.
Π²) Ρ‡Π΅ΠΌ большС ΡˆΠ΅Ρ€ΠΎΡ…ΠΎΠ²Π°Ρ‚ΠΎΡΡ‚ΠΈ повСрхности, Ρ‚Π΅ΠΌ коэффициСнт трСния большС.

2. Бпособы увСличСния ΠΈΠ»ΠΈ ΡƒΠΌΠ΅Π½ΡŒΡˆΠ΅Π½ΠΈΡ силы трСния скольТСния:

Π£Π²Π΅Π»ΠΈΡ‡ΠΈΡ‚ΡŒ: ΡƒΠ²Π΅Π»ΠΈΡ‡ΠΈΡ‚ΡŒ ΡˆΠ΅Ρ€ΠΎΡ…ΠΎΠ²Π°Ρ‚ΠΎΡΡ‚ΡŒ трущихся повСрхностСй, Π½Π°ΡΡ‹ΠΏΠ°Ρ‚ΡŒ ΠΌΠ΅ΠΆΠ΄Ρƒ трущихся повСрхностСй частицы (струТку, ΠΎΠΏΠΈΠ»ΠΊΠΈ, пСсок).

Π£ΠΌΠ΅Π½ΡŒΡˆΠΈΡ‚ΡŒ: ΡˆΠ»ΠΈΡ„ΠΎΠ²ΠΊΠ°, ΠΏΠΎΠ»ΠΈΡ€ΠΎΠ²ΠΊΠ° трущихся повСрхностСй, нанСсСниС смазки.

Π—Π°Π΄Π°Π½ΠΈΠ΅ Π³Ρ€ΡƒΠΏΠΏΠ΅ II.

Π˜Π·ΠΌΠ΅Ρ€Π΅Π½ΠΈΠ΅ коэффициСнт трСния скольТСния, ΠΈΡΠΏΠΎΠ»ΡŒΠ·ΡƒΡ Π½Π°ΠΊΠ»ΠΎΠ½Π½ΡƒΡŽ ΠΏΠ»ΠΎΡΠΊΠΎΡΡ‚ΡŒ

ΠžΠ±ΠΎΡ€ΡƒΠ΄ΠΎΠ²Π°Π½ΠΈΠ΅ : Π»ΠΈΠ½Π΅ΠΉΠΊΠ° дСрСвянная ΠΎΡ‚ Ρ‚Ρ€ΠΈΠ±ΠΎΠΌΠ΅Ρ‚Ρ€Π°, брусок дСрСвянный, Π»ΠΈΠ½Π΅ΠΉΠΊΠ° ΠΈΠ·ΠΌΠ΅Ρ€ΠΈΡ‚Π΅Π»ΡŒΠ½Π°Ρ, ΡˆΡ‚Π°Ρ‚ΠΈΠ².

ΠŸΠΎΡ€ΡΠ΄ΠΎΠΊ выполнСния Ρ€Π°Π±ΠΎΡ‚Ρ‹ .

1. Π˜ΡΠΏΠΎΠ»ΡŒΠ·ΡƒΡ ΡˆΡ‚Π°Ρ‚ΠΈΠ², Π·Π°ΠΊΡ€Π΅ΠΏΠΈΡ‚Π΅ Π»ΠΈΠ½Π΅ΠΉΠΊΡƒ ΠΏΠΎΠ΄ ΡƒΠ³Π»ΠΎΠΌ ΠΊ столу.
2. ΠŸΠΎΠ»ΠΎΠΆΠΈΡ‚Π΅ брусок Π½Π° Π·Π°ΠΊΡ€Π΅ΠΏΠ»Π΅Π½Π½ΡƒΡŽ ΠΏΠΎΠ΄ ΡƒΠ³Π»ΠΎΠΌ Π΄Π΅Ρ€Π΅Π²ΡΠ½Π½ΡƒΡŽ Π»ΠΈΠ½Π΅ΠΉΠΊΡƒ.
3. МСняя ΡƒΠ³ΠΎΠ» Π½Π°ΠΊΠ»ΠΎΠ½Π° Π»ΠΈΠ½Π΅ΠΉΠΊΠΈ, Π½Π°ΠΉΠ΄ΠΈΡ‚Π΅ Ρ‚Π°ΠΊΠΎΠΉ ΠΌΠ°ΠΊΡΠΈΠΌΠ°Π»ΡŒΠ½Ρ‹ΠΉ ΡƒΠ³ΠΎΠ», ΠΏΡ€ΠΈ ΠΊΠΎΡ‚ΠΎΡ€ΠΎΠΌ брусок Π΅Ρ‰Π΅ покоится.
4. Π˜Π·ΠΌΠ΅Ρ€ΡŒΡ‚Π΅ Π΄Π»ΠΈΠ½Ρƒ основания Π»ΠΈΠ½Π΅ΠΉΠΊΠΈ ΠΈ высоту подъСма Π»ΠΈΠ½Π΅ΠΉΠΊΠΈ.
5. РассчитайтС Π·Π½Π°Ρ‡Π΅Π½ΠΈΠ΅ коэффициСнта трСния скольТСния Π΄Π΅Ρ€Π΅Π²Π° ΠΎ Π΄Π΅Ρ€Π΅Π²ΠΎ ΠΏΠΎ Ρ„ΠΎΡ€ΠΌΡƒΠ»Π΅:

6. РассчитайтС ΠΏΠΎΠ³Ρ€Π΅ΡˆΠ½ΠΎΡΡ‚ΡŒ измСрСния.
7. Π’Ρ‹Π²ΠΎΠ΄.

Π­ΠΊΡΠΏΠ΅Ρ€ΠΈΠΌΠ΅Π½Ρ‚Π°Π»ΡŒΠ½Ρ‹Π΅ Π΄Π°Π½Π½Ρ‹Π΅.

Π˜Π·ΠΌΠ΅Ρ€ΠΈΠ»ΠΈ высоту подъСма ΠΈ Π΄Π»ΠΈΠ½Ρƒ основания Π»ΠΈΠ½Π΅ΠΉΠΊΠΈ.

1. ΠšΠΎΡΡ„Ρ„ΠΈΡ†ΠΈΠ΅Π½Ρ‚ трСния Ρ€Π°Π²Π΅Π½ 0,3.
2. ΠŸΠΎΠ³Ρ€Π΅ΡˆΠ½ΠΎΡΡ‚ΡŒ измСрСния Ρ€Π°Π²Π½Π° 0,0016.

2. Π˜Π·ΠΌΠ΅Ρ€Π΅Π½ΠΈΠ΅ коэффициСнта трСния скольТСния, Ρ‡Π΅Ρ€Π΅Π· ΠΎΠΏΡ€ΠΎΠΊΠΈΠ΄Ρ‹Π²Π°Π½ΠΈΠ΅ бруска

ΠžΠ±ΠΎΡ€ΡƒΠ΄ΠΎΠ²Π°Π½ΠΈΠ΅: брусок дСрСвянный, Π»ΠΈΠ½Π΅ΠΉΠΊΠ° дСрСвянная ΠΎΡ‚ Ρ‚Ρ€ΠΈΠ±ΠΎΠΌΠ΅Ρ‚Ρ€Π°, Π½ΠΈΡ‚ΡŒ, Π»ΠΈΠ½Π΅ΠΉΠΊΠ° учСничСская.

ΠŸΠΎΡ€ΡΠ΄ΠΎΠΊ выполнСния Ρ€Π°Π±ΠΎΡ‚Ρ‹.

ВСорСтичСскоС обоснованиС: Брусок с привязанной ΠΊ Π΄Π»ΠΈΠ½Π½ΠΎΠΉ Π³Ρ€Π°Π½ΠΈ Π½ΠΈΡ‚ΡŒΡŽ ΠΏΠΎΡΡ‚Π°Π²ΡŒΡ‚Π΅ Ρ‚ΠΎΡ€Ρ†ΠΎΠΌ Π½Π° Π³ΠΎΡ€ΠΈΠ·ΠΎΠ½Ρ‚Π°Π»ΡŒΠ½ΡƒΡŽ ΠΏΠΎΠ²Π΅Ρ€Ρ…Π½ΠΎΡΡ‚ΡŒ стола ΠΈ тянитС Π·Π° Π½ΠΈΡ‚ΡŒ. Если Π½ΠΈΡ‚ΡŒ Π·Π°ΠΊΡ€Π΅ΠΏΠ»Π΅Π½Π° нСвысоко Π½Π°Π΄ ΠΏΠΎΠ²Π΅Ρ€Ρ…Π½ΠΎΡΡ‚ΡŒΡŽ стола, Ρ‚ΠΎ брусок Π±ΡƒΠ΄Π΅Ρ‚ ΡΠΊΠΎΠ»ΡŒΠ·ΠΈΡ‚ΡŒ. ΠŸΡ€ΠΈ ΠΎΠΏΡ€Π΅Π΄Π΅Π»Π΅Π½Π½ΠΎΠΉ высотС h Ρ‚ΠΎΡ‡ΠΊΠΈ А крСплСния Π½ΠΈΡ‚ΠΈ сила натяТСния Π½ΠΈΡ‚ΠΈ F ΠΎΠΏΡ€ΠΎΠΊΠΈΠ΄Ρ‹Π²Π°Π΅Ρ‚ брусок.

Условия равновСсия для этого случая ΠΎΡ‚Π½ΠΎΡΠΈΡ‚Π΅Π»ΡŒΠ½ΠΎ Ρ‚ΠΎΡ‡ΠΊΠΈ – ΡƒΠ³Π»Π° опрокидывания:

Fh – mga/2 = 0;

Богласно II Π·Π°ΠΊΠΎΠ½Ρƒ ΠΡŒΡŽΡ‚ΠΎΠ½Π°: F – FΡ‚Ρ€ = 0;

ΠžΠ±Ρ€Π°Π±ΠΎΡ‚ΠΊΠ° Ρ€Π΅Π·ΡƒΠ»ΡŒΡ‚Π°Ρ‚ΠΎΠ².

4. Π‘Π΄Π΅Π»Π°ΠΉΡ‚Π΅ Π²Ρ‹Π²ΠΎΠ΄.

Π­ΠΊΡΠΏΠ΅Ρ€ΠΈΠΌΠ΅Π½Ρ‚Π°Π»ΡŒΠ½Ρ‹ΠΉ расчСт.

a = 45 Β± 1 ΠΌΠΌ, h = 80 Β± 1 ΠΌΠΌ.

1. ΠšΠΎΡΡ„Ρ„ΠΈΡ†ΠΈΠ΅Π½Ρ‚ трСния Ρ€Π°Π²Π΅Π½ 0,28.
2. Π˜Π½ΡΡ‚Ρ€ΡƒΠΌΠ΅Π½Ρ‚Π°Π»ΡŒΠ½Π°Ρ ΠΏΠΎΠ³Ρ€Π΅ΡˆΠ½ΠΎΡΡ‚ΡŒ измСрСния Ρ€Π°Π²Π½Π° 0,0098.

3. Π˜Π·ΠΌΠ΅Ρ€Π΅Π½ΠΈΠ΅ коэффициСнта трСния скольТСния с ΠΏΠΎΠΌΠΎΡ‰ΡŒΡŽ ΠΊΠ°Ρ€Π°Π½Π΄Π°ΡˆΠ°.

ΠžΠ±ΠΎΡ€ΡƒΠ΄ΠΎΠ²Π°Π½ΠΈΠ΅: ΠΊΠ°Ρ€Π°Π½Π΄Π°Ρˆ, Π»ΠΈΠ½Π΅ΠΉΠΊΠ° дСрСвянная ΠΎΡ‚ Ρ‚Ρ€ΠΈΠ±ΠΎΠΌΠ΅Ρ‚Ρ€Π°, Π»ΠΈΠ½Π΅ΠΉΠΊΠ° учСничСская.

ΠŸΠΎΡ€ΡΠ΄ΠΎΠΊ выполнСния Ρ€Π°Π±ΠΎΡ‚Ρ‹.

ВСорСтичСскоС обоснованиС: ΠŸΠΎΡΡ‚Π°Π²ΡŒΡ‚Π΅ ΠΊΠ°Ρ€Π°Π½Π΄Π°Ρˆ Π½Π° стол Π²Π΅Ρ€Ρ‚ΠΈΠΊΠ°Π»ΡŒΠ½ΠΎ, Π½Π°ΠΆΠΌΠΈΡ‚Π΅ Π½Π° Π½Π΅Π³ΠΎ, Π½Π°ΠΊΠ»ΠΎΠ½ΠΈΡ‚Π΅ ΠΈ Π½Π°Π±Π»ΡŽΠ΄Π°ΠΉΡ‚Π΅ Ρ…Π°Ρ€Π°ΠΊΡ‚Π΅Ρ€ Π΅Π³ΠΎ падСния. ΠŸΡ€ΠΈ Π½Π΅Π±ΠΎΠ»ΡŒΡˆΠΈΡ… ΡƒΠ³Π»Π°Ρ… Π½Π°ΠΊΠ»ΠΎΠ½Π° ΠΊ Π²Π΅Ρ€Ρ‚ΠΈΠΊΠ°Π»ΠΈ ΠΊΠ°Ρ€Π°Π½Π΄Π°Ρˆ Π½Π΅ ΠΏΡ€ΠΎΡΠΊΠ°Π»ΡŒΠ·Ρ‹Π²Π°Π΅Ρ‚ ΠΎΡ‚Π½ΠΎΡΠΈΡ‚Π΅Π»ΡŒΠ½ΠΎ повСрхности стола ΠΏΡ€ΠΈ любой Π²Π΅Π»ΠΈΡ‡ΠΈΠ½Π΅ силы, ΠΏΡ€ΠΈΠΆΠΈΠΌΠ°ΡŽΡ‰Π΅ΠΉ Π΅Π³ΠΎ ΠΊ столу. ΠŸΡ€ΠΎΡΠΊΠ°Π»ΡŒΠ·Ρ‹Π²Π°Π½ΠΈΠ΅ начинаСтся с Π½Π΅ΠΊΠΎΡ‚ΠΎΡ€ΠΎΠ³ΠΎ критичСского ΡƒΠ³Π»Π°, зависящСго ΠΎΡ‚ силы трСния.

ЗаписываСм Π²Ρ‚ΠΎΡ€ΠΎΠΉ Π·Π°ΠΊΠΎΠ½ ΠΡŒΡŽΡ‚ΠΎΠ½Π° Π² проСкциях Π½Π° ΠΊΠΎΠΎΡ€Π΄ΠΈΠ½Π°Ρ‚Π½Ρ‹Π΅ оси ΠΏΡ€ΠΈ ΡƒΠ³Π»Π΅ Π½Π°ΠΊΠ»ΠΎΠ½Π°, Ρ€Π°Π²Π½ΠΎΠΌ критичСскому. (Π‘ΠΈΠ»ΠΎΠΉ тяТСсти mg, Π΄Π΅ΠΉΡΡ‚Π²ΡƒΡŽΡ‰Π΅ΠΉ Π½Π° ΠΊΠ°Ρ€Π°Π½Π΄Π°Ρˆ, ΠΏΠΎ ΡΡ€Π°Π²Π½Π΅Π½ΠΈΡŽ с большой силой F ΠΏΡ€Π΅Π½Π΅Π±Ρ€Π΅Π³Π°Π΅ΠΌ).

ΠžΠ±Ρ€Π°Π±ΠΎΡ‚ΠΊΠ° Ρ€Π΅Π·ΡƒΠ»ΡŒΡ‚Π°Ρ‚ΠΎΠ²:

1. РассчитайтС ΠΏΠΎ Ρ„ΠΎΡ€ΠΌΡƒΠ»Π΅ Π·Π½Π°Ρ‡Π΅Π½ΠΈΠ΅ коэффициСнта трСния скольТСния Π΄Π΅Ρ€Π΅Π²Π° ΠΎ Π΄Π΅Ρ€Π΅Π²ΠΎ.
2. ΠžΠΏΡ€Π΅Π΄Π΅Π»ΠΈΡ‚Π΅ ΠΏΠΎΠ³Ρ€Π΅ΡˆΠ½ΠΎΡΡ‚ΡŒ ΠΈΠ·ΠΌΠ΅Ρ€Π΅Π½ΠΈΠΉ.
3. Π—Π°ΠΏΠΈΡˆΠΈΡ‚Π΅ ΠΏΠΎΠ»ΡƒΡ‡Π΅Π½Π½Ρ‹ΠΉ ΠΎΡ‚Π²Π΅Ρ‚ с ΡƒΡ‡Π΅Ρ‚ΠΎΠΌ Π΄ΠΎΠΏΡƒΡ‰Π΅Π½Π½Ρ‹Ρ… ΠΏΠΎΠ³Ρ€Π΅ΡˆΠ½ΠΎΡΡ‚Π΅ΠΉ ΠΈΠ·ΠΌΠ΅Ρ€Π΅Π½ΠΈΠΉ.
4. Π‘Π΄Π΅Π»Π°ΠΉΡ‚Π΅ Π²Ρ‹Π²ΠΎΠ΄.

Π­ΠΊΡΠΏΠ΅Ρ€ΠΈΠΌΠ΅Π½Ρ‚Π°Π»ΡŒΠ½Ρ‹ΠΉ расчСт.

1. ΠžΠ±Ρ€Π°Π±ΠΎΡ‚ΠΊΠ° Ρ€Π΅Π·ΡƒΠ»ΡŒΡ‚Π°Ρ‚ΠΎΠ²

Ξ± = 30 0 ,

Β΅= tgΞ± = sina /cosa

1. ΠšΠΎΡΡ„Ρ„ΠΈΡ†ΠΈΠ΅Π½Ρ‚ трСния Ρ€Π°Π²Π΅Π½ 0,58.

III. ПодвСдСниС ΠΈΡ‚ΠΎΠ³ΠΎΠ² ΠΏΡ€Π°ΠΊΡ‚ΠΈΠΊΡƒΠΌΠ°:

Π‘ΠΈΠ»Π° трСния скольТСния зависит:

Π°) ΠžΡ‚ Ρ€ΠΎΠ΄Π° трущихся повСрхностСй.
Π±) ΠžΡ‚ ΡˆΠ΅Ρ€ΠΎΡ…ΠΎΠ²Π°Ρ‚ΠΎΡΡ‚ΠΈ трущихся повСрхностСй.
Π²) ΠŸΡ€ΡΠΌΠΎ ΠΏΡ€ΠΎΠΏΠΎΡ€Ρ†ΠΈΠΎΠ½Π°Π»ΡŒΠ½ΠΎ ΠΎΡ‚ силы давлСния.
Π³) ΠšΠΎΡΡ„Ρ„ΠΈΡ†ΠΈΠ΅Π½Ρ‚ трСния скольТСния ΠΏΡ€ΠΈ Π²Π·Π°ΠΈΠΌΠ½ΠΎΠΌ Π΄Π²ΠΈΠΆΠ΅Π½ΠΈΠΈ Ρ‚Π΅Π»Π° ΠΏΠΎ повСрхности являСтся Π²Π΅Π»ΠΈΡ‡ΠΈΠ½ΠΎΠΉ постоянной Π½Π΅ зависящСй ΠΎΡ‚ силы Π½ΠΎΡ€ΠΌΠ°Π»ΡŒΠ½ΠΎΠ³ΠΎ давлСния.
Π΄) Π§Π΅ΠΌ большС ΡˆΠ΅Ρ€ΠΎΡ…ΠΎΠ²Π°Ρ‚ΠΎΡΡ‚ΠΈ повСрхности, Ρ‚Π΅ΠΌ коэффициСнт трСния большС.

ΠšΠΎΡΡ„Ρ„ΠΈΡ†ΠΈΠ΅Π½Ρ‚ трСния β€” это основная характСристика трСния ΠΊΠ°ΠΊ явлСния. Он опрСдСляСтся Π²ΠΈΠ΄ΠΎΠΌ ΠΈ состояниСм повСрхностСй трущихся Ρ‚Π΅Π».

ΠžΠŸΠ Π•Π”Π•Π›Π•ΠΠ˜Π•

ΠšΠΎΡΡ„Ρ„ΠΈΡ†ΠΈΠ΅Π½Ρ‚ΠΎΠΌ трСния Π½Π°Π·Ρ‹Π²Π°ΡŽΡ‚ коэффициСнт ΠΏΡ€ΠΎΠΏΠΎΡ€Ρ†ΠΈΠΎΠ½Π°Π»ΡŒΠ½ΠΎΡΡ‚ΠΈ, ΡΠ²ΡΠ·Ρ‹Π²Π°ΡŽΡ‰ΠΈΠΉ силу трСния () ΠΈ силу Π½ΠΎΡ€ΠΌΠ°Π»ΡŒΠ½ΠΎΠ³ΠΎ давлСния (N) Ρ‚Π΅Π»Π° Π½Π° ΠΎΠΏΠΎΡ€Ρƒ. Π§Π°Ρ‰Π΅ всСго коэффициСнт трСния ΠΎΠ±ΠΎΠ·Π½Π°Ρ‡Π°ΡŽΡ‚ Π±ΡƒΠΊΠ²ΠΎΠΉ . И Ρ‚Π°ΠΊ, коэффициСнт трСния Π²Ρ…ΠΎΠ΄ΠΈΡ‚ Π² Π·Π°ΠΊΠΎΠ½ ΠšΡƒΠ»ΠΎΠ½Π° β€” Амонтона:

Π”Π°Π½Π½Ρ‹ΠΉ коэффициСнт трСния Π½Π΅ зависит ΠΎΡ‚ ΠΏΠ»ΠΎΡ‰Π°Π΄Π΅ΠΉ, ΡΠΎΠΏΡ€ΠΈΠΊΠ°ΡΠ°ΡŽΡ‰ΠΈΡ…ΡΡ повСрхностСй.

Π’ Π΄Π°Π½Π½ΠΎΠΌ случаС Ρ€Π΅Ρ‡ΡŒ ΠΈΠ΄Π΅Ρ‚ ΠΎ коэффициСнтС трСния скольТСния, ΠΊΠΎΡ‚ΠΎΡ€Ρ‹ΠΉ зависит ΠΎΡ‚ совокупных свойств трущихся повСрхностСй ΠΈ являСтся Π±Π΅Π·Ρ€Π°Π·ΠΌΠ΅Ρ€Π½ΠΎΠΉ Π²Π΅Π»ΠΈΡ‡ΠΈΠ½ΠΎΠΉ. ΠšΠΎΡΡ„Ρ„ΠΈΡ†ΠΈΠ΅Π½Ρ‚ трСния зависит ΠΎΡ‚: качСства ΠΎΠ±Ρ€Π°Π±ΠΎΡ‚ΠΊΠΈ повСрхностСй, трущихся Ρ‚Π΅Π», присутствия Π½Π° Π½ΠΈΡ… грязи, скорости двиТСния Ρ‚Π΅Π» Π΄Ρ€ΡƒΠ³ ΠΎΡ‚Π½ΠΎΡΠΈΡ‚Π΅Π»ΡŒΠ½ΠΎ Π΄Ρ€ΡƒΠ³Π° ΠΈ Ρ‚.Π΄. ΠšΠΎΡΡ„Ρ„ΠΈΡ†ΠΈΠ΅Π½Ρ‚ трСния ΠΎΠΏΡ€Π΅Π΄Π΅Π»ΡΡŽΡ‚ эмпиричСски (ΠΎΠΏΡ‹Ρ‚Π½Ρ‹ΠΌ ΠΏΡƒΡ‚Π΅ΠΌ).

ΠšΠΎΡΡ„Ρ„ΠΈΡ†ΠΈΠ΅Π½Ρ‚ трСния, ΠΊΠΎΡ‚ΠΎΡ€Ρ‹ΠΉ соотвСтствуСт максимальной силС трСния покоя Π² Π±ΠΎΠ»ΡŒΡˆΠΈΠ½ΡΡ‚Π²Π΅ случаСв большС, Ρ‡Π΅ΠΌ коэффициСнт трСния двиТСния.

Для большСго числа ΠΏΠ°Ρ€ ΠΌΠ°Ρ‚Π΅Ρ€ΠΈΠ°Π»ΠΎΠ² Π²Π΅Π»ΠΈΡ‡ΠΈΠ½Π° коэффициСнта трСния Π½Π΅ большС Π΅Π΄ΠΈΠ½ΠΈΡ†Ρ‹ ΠΈ Π»Π΅ΠΆΠΈΡ‚ Π² ΠΏΡ€Π΅Π΄Π΅Π»Π°Ρ…

Π£Π³ΠΎΠ» трСния

Иногда вмСсто коэффициСнта трСния ΠΏΡ€ΠΈΠΌΠ΅Π½ΡΡŽΡ‚ ΡƒΠ³ΠΎΠ» трСния (), ΠΊΠΎΡ‚ΠΎΡ€Ρ‹ΠΉ связан с коэффициСнтом ΡΠΎΠΎΡ‚Π½ΠΎΡˆΠ΅Π½ΠΈΠ΅ΠΌ:

Π’Π°ΠΊ, ΡƒΠ³ΠΎΠ» трСния соотвСтствуСт ΠΌΠΈΠ½ΠΈΠΌΠ°Π»ΡŒΠ½ΠΎΠΌΡƒ ΡƒΠ³Π»Ρƒ Π½Π°ΠΊΠ»ΠΎΠ½Π° плоскости ΠΏΠΎ ΠΎΡ‚Π½ΠΎΡˆΠ΅Π½ΠΈΡŽ ΠΊ Π³ΠΎΡ€ΠΈΠ·ΠΎΠ½Ρ‚Ρƒ, ΠΏΡ€ΠΈ ΠΊΠΎΡ‚ΠΎΡ€ΠΎΠΌ Ρ‚Π΅Π»ΠΎ, Π»Π΅ΠΆΠ°Ρ‰Π΅Π΅ Π½Π° этой плоскости, Π½Π°Ρ‡Π½Π΅Ρ‚ ΡΠΊΠΎΠ»ΡŒΠ·ΠΈΡ‚ΡŒ Π²Π½ΠΈΠ· ΠΏΠΎΠ΄ воздСйствиСм силы тяТСсти. ΠŸΡ€ΠΈ этом выполняСтся равСнство:

Π˜ΡΡ‚ΠΈΠ½Π½Ρ‹ΠΉ коэффициСнт трСния

Π—Π°ΠΊΠΎΠ½ трСния, ΠΊΠΎΡ‚ΠΎΡ€Ρ‹ΠΉ ΡƒΡ‡ΠΈΡ‚Ρ‹Π²Π°Π΅Ρ‚ влияниС сил притяТСния ΠΌΠ΅ΠΆΠ΄Ρƒ ΠΌΠΎΠ»Π΅ΠΊΡƒΠ»Π°ΠΌΠΈ, трущихся повСрхностСй Π·Π°ΠΏΠΈΡΡ‹Π²Π°ΡŽ ΡΠ»Π΅Π΄ΡƒΡŽΡ‰ΠΈΠΌ ΠΎΠ±Ρ€Π°Π·ΠΎΠΌ:

Π³Π΄Π΅ β€” Π½Π°Π·Ρ‹Π²Π°ΡŽΡ‚ истинным коэффициСнтом трСния, β€” Π΄ΠΎΠ±Π°Π²ΠΎΡ‡Π½ΠΎΠ΅ Π΄Π°Π²Π»Π΅Π½ΠΈΠ΅, ΠΊΠΎΡ‚ΠΎΡ€ΠΎΠ΅ вызываСтся силами мСТмолСкулярного притяТСния, S β€” общая ΠΏΠ»ΠΎΡ‰Π°Π΄ΡŒ нСпосрСдствСнного ΠΊΠΎΠ½Ρ‚Π°ΠΊΡ‚Π° трущихся Ρ‚Π΅Π».

ΠšΠΎΡΡ„Ρ„ΠΈΡ†ΠΈΠ΅Π½Ρ‚ трСния качСния

ΠšΠΎΡΡ„Ρ„ΠΈΡ†ΠΈΠ΅Π½Ρ‚ трСния качСния (k) ΠΌΠΎΠΆΠ½ΠΎ ΠΎΠΏΡ€Π΅Π΄Π΅Π»ΠΈΡ‚ΡŒ ΠΊΠ°ΠΊ ΠΎΡ‚Π½ΠΎΡˆΠ΅Π½ΠΈΠ΅ ΠΌΠΎΠΌΠ΅Π½Ρ‚Π° силы трСния качСния () ΠΊ силС с ΠΊΠΎΡ‚ΠΎΡ€ΠΎΠΉ Ρ‚Π΅Π»ΠΎ приТимаСтся ΠΊ ΠΎΠΏΠΎΡ€Π΅ (N):

ΠžΡ‚ΠΌΠ΅Ρ‚ΠΈΠΌ, Ρ‡Ρ‚ΠΎ коэффициСнт трСния качСния ΠΎΠ±ΠΎΠ·Π½Π°Ρ‡Π°ΡŽΡ‚ Ρ‡Π°Ρ‰Π΅ Π±ΡƒΠΊΠ²ΠΎΠΉ . Π­Ρ‚ΠΎΡ‚ коэффициСнт, Π² ΠΎΡ‚Π»ΠΈΡ‡ΠΈΠ΅ ΠΎΡ‚ Π²Ρ‹ΡˆΠ΅ пСрСчислСнных коэффициСнтов трСния, ΠΈΠΌΠ΅Π΅Ρ‚ Ρ€Π°Π·ΠΌΠ΅Ρ€Π½ΠΎΡΡ‚ΡŒ Π΄Π»ΠΈΠ½Ρ‹. Π’ΠΎ Π΅ΡΡ‚ΡŒ Π² систСмС БИ ΠΎΠ½ измСряСтся Π² ΠΌΠ΅Ρ‚Ρ€Π°Ρ….

ΠšΠΎΡΡ„Ρ„ΠΈΡ†ΠΈΠ΅Π½Ρ‚ трСния качСния ΠΌΠ½ΠΎΠ³ΠΎ мСньшС, Ρ‡Π΅ΠΌ коэффициСнт трСния скольТСния.

ΠŸΡ€ΠΈΠΌΠ΅Ρ€Ρ‹ Ρ€Π΅ΡˆΠ΅Π½ΠΈΡ Π·Π°Π΄Π°Ρ‡

ΠŸΠ Π˜ΠœΠ•Π  1

Π—Π°Π΄Π°Π½ΠΈΠ΅Π’Π΅Ρ€Π΅Π²ΠΊΠ° Π»Π΅ΠΆΠΈΡ‚ частично Π½Π° столС, Ρ‡Π°ΡΡ‚ΡŒ Π΅Π΅ ΡΠ²Π΅ΡˆΠΈΠ²Π°Π΅Ρ‚ΡΡ со стола. Если Ρ‚Ρ€Π΅Ρ‚ΡŒ Π΄Π»ΠΈΠ½Ρ‹ Π²Π΅Ρ€Π΅Π²ΠΊΠΈ свСсится со стола, Ρ‚ΠΎ ΠΎΠ½Π° Π½Π°Ρ‡ΠΈΠ½Π°Π΅Ρ‚ ΡΠΊΠΎΠ»ΡŒΠ·ΠΈΡ‚ΡŒ. Каков коэффициСнт трСния Π²Π΅Ρ€Π΅Π²ΠΊΠΈ ΠΎ стол?
Π Π΅ΡˆΠ΅Π½ΠΈΠ΅Π’Π΅Ρ€Π΅Π²ΠΊΠ° ΡΠΊΠΎΠ»ΡŒΠ·ΠΈΡ‚ со стола ΠΏΠΎΠ΄ дСйствиСм силы тяТСсти. ΠžΠ±ΠΎΠ·Π½Π°Ρ‡ΠΈΠΌ силу тяТСсти, которая дСйствуСт Π½Π° Π΅Π΄ΠΈΠ½ΠΈΡ†Ρƒ Π΄Π»ΠΈΠ½Ρ‹ Π²Π΅Ρ€Π΅Π²ΠΊΠΈ ΠΊΠ°ΠΊ . Π’ Ρ‚Π°ΠΊΠΎΠΌ случаС Π² ΠΌΠΎΠΌΠ΅Π½Ρ‚ Π½Π°Ρ‡Π°Π»Π° скольТСния сила тяТСсти, которая дСйствуСт Π½Π° ΡΠ²Π΅ΡˆΠΈΠ²Π°ΡŽΡ‰ΡƒΡŽΡΡ Ρ‡Π°ΡΡ‚ΡŒ Π²Π΅Ρ€Π΅Π²ΠΊΠΈ, Ρ€Π°Π²Π½Π°:

Π”ΠΎ Π½Π°Ρ‡Π°Π»Π° скольТСния эта сила ΡƒΡ€Π°Π²Π½ΠΎΠ²Π΅ΡˆΠΈΠ²Π°Π΅Ρ‚ΡΡ силой трСния, которая дСйствуСт Π½Π° Ρ‡Π°ΡΡ‚ΡŒ Π²Π΅Ρ€Π΅Π²ΠΊΠΈ, которая Π»Π΅ΠΆΠΈΡ‚ Π½Π° столС:

Π’Π°ΠΊ ΠΊΠ°ΠΊ силы ΡƒΡ€Π°Π²Π½ΠΎΠ²Π΅ΡˆΠΈΠ²Π°ΡŽΡ‚ΡΡ, Ρ‚ΠΎ ΠΌΠΎΠΆΠ½ΠΎ Π·Π°ΠΏΠΈΡΠ°Ρ‚ΡŒ ():

ΠžΡ‚Π²Π΅Ρ‚

ΠŸΠ Π˜ΠœΠ•Π  2

Π—Π°Π΄Π°Π½ΠΈΠ΅ΠšΠ°ΠΊΠΎΠ² коэффициСнт трСния Ρ‚Π΅Π»Π° ΠΎ ΠΏΠ»ΠΎΡΠΊΠΎΡΡ‚ΡŒ (), Ссли Π·Π°Π²ΠΈΡΠΈΠΌΠΎΡΡ‚ΡŒ ΠΏΡƒΡ‚ΠΈ, ΠΊΠΎΡ‚ΠΎΡ€ΠΎΠ΅ ΠΎΠ½ΠΎ ΠΏΡ€ΠΎΡ…ΠΎΠ΄ΠΈΡ‚ Π·Π°Π΄Π°Π½ΠΎ ΡƒΡ€Π°Π²Π½Π΅Π½ΠΈΠ΅ΠΌ: Π³Π΄Π΅ ΠŸΠ»ΠΎΡΠΊΠΎΡΡ‚ΡŒ составляСт ΡƒΠ³ΠΎΠ» с Π³ΠΎΡ€ΠΈΠ·ΠΎΠ½Ρ‚ΠΎΠΌ.
Π Π΅ΡˆΠ΅Π½ΠΈΠ΅Π—Π°ΠΏΠΈΡˆΠ΅ΠΌ Π²Ρ‚ΠΎΡ€ΠΎΠΉ Π·Π°ΠΊΠΎΠ½ ΠΡŒΡŽΡ‚ΠΎΠ½Π° для сил, ΠΏΡ€ΠΈΠ»ΠΎΠΆΠ΅Π½Π½Ρ‹Ρ… ΠΊ двиТущСмуся Ρ‚Π΅Π»Ρƒ:

Π’Ρ€Π΅Π½ΠΈΠ΅ — явлСниС, с ΠΊΠΎΡ‚ΠΎΡ€Ρ‹ΠΌ ΠΌΡ‹ сталкиваСмся Π² ΠΎΠ±Ρ‹Π΄Π΅Π½Π½ΠΎΠΉ ΠΆΠΈΠ·Π½ΠΈ постоянно. ΠžΠΏΡ€Π΅Π΄Π΅Π»ΠΈΡ‚ΡŒ, Ρ‚Ρ€Π΅Π½ΠΈΠ΅ Π²Ρ€Π΅Π΄Π½ΠΎ ΠΈΠ»ΠΈ ΠΏΠΎΠ»Π΅Π·Π½ΠΎ, Π½Π΅Π²ΠΎΠ·ΠΌΠΎΠΆΠ½ΠΎ. Π‘Π΄Π΅Π»Π°Ρ‚ΡŒ Π΄Π°ΠΆΠ΅ шаг Π½Π° скользком Π»ΡŒΠ΄Ρƒ прСдставляСтся тяТСлым занятиСм, Π½Π° ΡˆΠ΅Ρ€ΠΎΡ…ΠΎΠ²Π°Ρ‚ΠΎΠΉ повСрхности Π°ΡΡ„Π°Π»ΡŒΡ‚Π° ΠΏΡ€ΠΎΠ³ΡƒΠ»ΠΊΠ° доставляСт ΡƒΠ΄ΠΎΠ²ΠΎΠ»ΡŒΡΡ‚Π²ΠΈΠ΅. Π”Π΅Ρ‚Π°Π»ΠΈ Π°Π²Ρ‚ΠΎΠΌΠΎΠ±ΠΈΠ»Π΅ΠΉ Π±Π΅Π· смазки ΠΈΠ·Π½Π°ΡˆΠΈΠ²Π°ΡŽΡ‚ΡΡ Π·Π½Π°Ρ‡ΠΈΡ‚Π΅Π»ΡŒΠ½ΠΎ быстрСС.

Π˜Π·ΡƒΡ‡Π΅Π½ΠΈΠ΅ трСния, Π·Π½Π°Π½ΠΈΠ΅ Π΅Π³ΠΎ основных свойств позволяСт Ρ‡Π΅Π»ΠΎΠ²Π΅ΠΊΡƒ ΠΈΡΠΏΠΎΠ»ΡŒΠ·ΠΎΠ²Π°Ρ‚ΡŒ Π΅Π³ΠΎ.

Π‘ΠΈΠ»Π° трСния Π² Ρ„ΠΈΠ·ΠΈΠΊΠ΅

Π‘ΠΈΠ»Π°, Π²ΠΎΠ·Π½ΠΈΠΊΠ°ΡŽΡ‰Π°Ρ ΠΏΡ€ΠΈ Π΄Π²ΠΈΠΆΠ΅Π½ΠΈΠΈ ΠΈΠ»ΠΈ ΠΏΠΎΠΏΡ‹Ρ‚ΠΊΠ΅ двиТСния ΠΎΠ΄Π½ΠΎΠ³ΠΎ Ρ‚Π΅Π»Π° ΠΏΠΎ повСрхности Π΄Ρ€ΡƒΠ³ΠΎΠ³ΠΎ, направлСнная ΠΏΡ€ΠΎΡ‚ΠΈΠ² направлСния двиТСния, прилоТСнная ΠΊ двиТущимся Ρ‚Π΅Π»Π°ΠΌ, Π½Π°Π·Π²Π°Π½Π° силой трСния. ΠœΠΎΠ΄ΡƒΠ»ΡŒ силы трСния, Ρ„ΠΎΡ€ΠΌΡƒΠ»Π° ΠΊΠΎΡ‚ΠΎΡ€ΠΎΠΉ зависит ΠΎΡ‚ ΠΌΠ½ΠΎΠ³ΠΈΡ… ΠΏΠ°Ρ€Π°ΠΌΠ΅Ρ‚Ρ€ΠΎΠ², мСняСтся Π² зависимости ΠΎΡ‚ Π²ΠΈΠ΄Π° сопротивлСния.

ΠžΡ‚Π»ΠΈΡ‡Π°ΡŽΡ‚ ΡΠ»Π΅Π΄ΡƒΡŽΡ‰ΠΈΠ΅ Π²ΠΈΠ΄Ρ‹ трСния:

БкольТСния;

ΠšΠ°Ρ‡Π΅Π½ΠΈΡ.

Π›ΡŽΠ±Π°Ρ ΠΏΠΎΠΏΡ‹Ρ‚ΠΊΠ° ΡΠ΄Π²ΠΈΠ½ΡƒΡ‚ΡŒ с мСста тяТСлый ΠΏΡ€Π΅Π΄ΠΌΠ΅Ρ‚ (ΡˆΠΊΠ°Ρ„, камСнь) ΠΏΡ€ΠΈΠ²ΠΎΠ΄ΠΈΡ‚ ΠΊ Π½Π°ΠΏΡ€ΡΠΆΠ΅Π½ΠΈΡŽ ΠŸΡ€ΠΈ этом Π² Π΄Π²ΠΈΠΆΠ΅Π½ΠΈΠ΅ ΠΏΡ€Π΅Π΄ΠΌΠ΅Ρ‚ привСсти получаСтся Π½Π΅ всСгда. ΠœΠ΅ΡˆΠ°Π΅Ρ‚ покоя.

БостояниС покоя

РасчСтная трСния покоя Π½Π΅ позволяСт ΠΎΠΏΡ€Π΅Π΄Π΅Π»ΠΈΡ‚ΡŒ Π΅Π΅ достаточно Ρ‚ΠΎΡ‡Π½ΠΎ. Π’ силу дСйствия Ρ‚Ρ€Π΅Ρ‚ΡŒΠ΅Π³ΠΎ Π·Π°ΠΊΠΎΠ½Π° ΠΡŒΡŽΡ‚ΠΎΠ½Π° Π²Π΅Π»ΠΈΡ‡ΠΈΠ½Π° силы сопротивлСния покоя зависит ΠΎΡ‚ ΠΏΡ€ΠΈΠ»ΠΎΠΆΠ΅Π½Π½ΠΎΠ³ΠΎ усилия.

ΠŸΡ€ΠΈ возрастании усилия растСт ΠΈ сила трСния.

0

НС позволяСт Π²Π±ΠΈΡ‚Ρ‹ΠΌ Π² Π΄Π΅Ρ€Π΅Π²ΠΎ гвоздям Π²Ρ‹ΠΏΠ°Π΄Π°Ρ‚ΡŒ; ΠΏΡƒΠ³ΠΎΠ²ΠΈΡ†Ρ‹, ΠΏΡ€ΠΈΡˆΠΈΡ‚Ρ‹Π΅ Π½ΠΈΡ‚ΠΊΠ°ΠΌΠΈ, ΠΏΡ€ΠΎΡ‡Π½ΠΎ ΡƒΠ΄Π΅Ρ€ΠΆΠΈΠ²Π°ΡŽΡ‚ΡΡ Π½Π° своСм мСстС. Π˜Π½Ρ‚Π΅Ρ€Π΅ΡΠ½ΠΎ, Ρ‡Ρ‚ΠΎ ΡˆΠ°Π³Π°Ρ‚ΡŒ Ρ‡Π΅Π»ΠΎΠ²Π΅ΠΊΡƒ позволяСт ΠΈΠΌΠ΅Π½Π½ΠΎ сопротивлСниС покоя. ΠŸΡ€ΠΈΡ‡Π΅ΠΌ Π½Π°ΠΏΡ€Π°Π²Π»Π΅Π½ΠΎ ΠΎΠ½ΠΎ ΠΏΠΎ Ρ…ΠΎΠ΄Ρƒ двиТСния Ρ‡Π΅Π»ΠΎΠ²Π΅ΠΊΠ°, Ρ‡Ρ‚ΠΎ ΠΏΡ€ΠΎΡ‚ΠΈΠ²ΠΎΡ€Π΅Ρ‡ΠΈΡ‚ ΠΎΠ±Ρ‰Π΅ΠΌΡƒ полоТСнию Π²Π΅Ρ‰Π΅ΠΉ.

ЯвлСниС скольТСния

ΠŸΡ€ΠΈ возрастании внСшнСй силы, Π΄Π²ΠΈΠΆΡƒΡ‰Π΅ΠΉ Ρ‚Π΅Π»ΠΎ, Π΄ΠΎ значСния наибольшСй силы трСния покоя ΠΎΠ½ΠΎ ΠΏΡ€ΠΈΡ…ΠΎΠ΄ΠΈΡ‚ Π² Π΄Π²ΠΈΠΆΠ΅Π½ΠΈΠ΅. Π‘ΠΈΠ»Π° трСния скольТСния рассматриваСтся Π² процСссС скольТСния ΠΎΠ΄Π½ΠΎΠ³ΠΎ Ρ‚Π΅Π»Π° ΠΏΠΎ повСрхности Π΄Ρ€ΡƒΠ³ΠΎΠ³ΠΎ. Π•Π΅ Π·Π½Π°Ρ‡Π΅Π½ΠΈΠ΅ зависит ΠΎΡ‚ свойств Π²Π·Π°ΠΈΠΌΠΎΠ΄Π΅ΠΉΡΡ‚Π²ΡƒΡŽΡ‰ΠΈΡ… повСрхностСй ΠΈ силы Π²Π΅Ρ€Ρ‚ΠΈΠΊΠ°Π»ΡŒΠ½ΠΎΠ³ΠΎ дСйствия Π½Π° ΠΏΠΎΠ²Π΅Ρ€Ρ…Π½ΠΎΡΡ‚ΡŒ.

РасчСтная Ρ„ΠΎΡ€ΠΌΡƒΠ»Π° силы трСния скольТСния: F=ΞΌΠ , Π³Π΄Π΅ ΞΌ-коэффициСнт ΠΏΡ€ΠΎΠΏΠΎΡ€Ρ†ΠΈΠΎΠ½Π°Π»ΡŒΠ½ΠΎΡΡ‚ΠΈ (трСния скольТСния), Π  — сила Π²Π΅Ρ€Ρ‚ΠΈΠΊΠ°Π»ΡŒΠ½ΠΎΠ³ΠΎ (Π½ΠΎΡ€ΠΌΠ°Π»ΡŒΠ½ΠΎΠ³ΠΎ) давлСния.

Одна ΠΈΠ· ΡƒΠΏΡ€Π°Π²Π»ΡΡŽΡ‰ΠΈΡ… Π΄Π²ΠΈΠΆΠ΅Π½ΠΈΠ΅ΠΌ сил — сила трСния скольТСния, Ρ„ΠΎΡ€ΠΌΡƒΠ»Π° ΠΊΠΎΡ‚ΠΎΡ€ΠΎΠΉ записываСтся с использованиСм ВслСдствиС выполнСния Ρ‚Ρ€Π΅Ρ‚ΡŒΠ΅Π³ΠΎ Π·Π°ΠΊΠΎΠ½Π° ΠΡŒΡŽΡ‚ΠΎΠ½Π° силы Π½ΠΎΡ€ΠΌΠ°Π»ΡŒΠ½ΠΎΠ³ΠΎ давлСния ΠΈ Ρ€Π΅Π°ΠΊΡ†ΠΈΠΈ ΠΎΠΏΠΎΡ€Ρ‹ ΠΎΠ΄ΠΈΠ½Π°ΠΊΠΎΠ²Ρ‹ ΠΏΠΎ Π²Π΅Π»ΠΈΡ‡ΠΈΠ½Π΅ ΠΈ ΠΏΡ€ΠΎΡ‚ΠΈΠ²ΠΎΠΏΠΎΠ»ΠΎΠΆΠ½Ρ‹ ΠΏΠΎ Π½Π°ΠΏΡ€Π°Π²Π»Π΅Π½ΠΈΡŽ: Π  = N.

ΠŸΠ΅Ρ€Π΅Π΄ Ρ‚Π΅ΠΌ ΠΊΠ°ΠΊ Π½Π°ΠΉΡ‚ΠΈ силу трСния, Ρ„ΠΎΡ€ΠΌΡƒΠ»Π° ΠΊΠΎΡ‚ΠΎΡ€ΠΎΠΉ ΠΏΡ€ΠΈΠΎΠ±Ρ€Π΅Ρ‚Π°Π΅Ρ‚ ΠΈΠ½ΠΎΠΉ Π²ΠΈΠ΄ (F=ΞΌ N), ΠΎΠΏΡ€Π΅Π΄Π΅Π»ΡΡŽΡ‚ силу Ρ€Π΅Π°ΠΊΡ†ΠΈΠΈ.

ΠšΠΎΡΡ„Ρ„ΠΈΡ†ΠΈΠ΅Π½Ρ‚ сопротивлСния ΠΏΡ€ΠΈ скольТСнии вводится ΡΠΊΡΠΏΠ΅Ρ€ΠΈΠΌΠ΅Π½Ρ‚Π°Π»ΡŒΠ½ΠΎ для Π΄Π²ΡƒΡ… трущихся повСрхностСй, зависит ΠΎΡ‚ качСства ΠΈΡ… ΠΎΠ±Ρ€Π°Π±ΠΎΡ‚ΠΊΠΈ ΠΈ ΠΌΠ°Ρ‚Π΅Ρ€ΠΈΠ°Π»Π°.

Π’Π°Π±Π»ΠΈΡ†Π°. Π—Π½Π°Ρ‡Π΅Π½ΠΈΠ΅ коэффициСнта сопротивлСния для Ρ€Π°Π·Π»ΠΈΡ‡Π½Ρ‹Ρ… повСрхностСй

β„– ΠΏΠΏ

Π’Π·Π°ΠΈΠΌΠΎΠ΄Π΅ΠΉΡΡ‚Π²ΡƒΡŽΡ‰ΠΈΠ΅ повСрхности

Π—Π½Π°Ρ‡Π΅Π½ΠΈΠ΅ коэффициСнта трСния скольТСния

Π‘Ρ‚Π°Π»ΡŒ+Π»Π΅Π΄

КоТа+Ρ‡ΡƒΠ³ΡƒΠ½

Π‘Ρ€ΠΎΠ½Π·Π°+ΠΆΠ΅Π»Π΅Π·ΠΎ

Π‘Ρ€ΠΎΠ½Π·Π°+Ρ‡ΡƒΠ³ΡƒΠ½

Π‘Ρ‚Π°Π»ΡŒ+ΡΡ‚Π°Π»ΡŒ

Наибольшая сила трСния покоя, Ρ„ΠΎΡ€ΠΌΡƒΠ»Π° ΠΊΠΎΡ‚ΠΎΡ€ΠΎΠΉ Π±Ρ‹Π»Π° записана Π²Ρ‹ΡˆΠ΅, ΠΌΠΎΠΆΠ΅Ρ‚ Π±Ρ‹Ρ‚ΡŒ ΠΎΠΏΡ€Π΅Π΄Π΅Π»Π΅Π½Π° Ρ‚Π°ΠΊ ΠΆΠ΅, ΠΊΠ°ΠΊ сила трСния скольТСния.

Π­Ρ‚ΠΎ становится Π²Π°ΠΆΠ½Ρ‹ΠΌ ΠΏΡ€ΠΈ Ρ€Π΅ΡˆΠ΅Π½ΠΈΠΈ Π·Π°Π΄Π°Ρ‡ Π½Π° ΠΎΠΏΡ€Π΅Π΄Π΅Π»Π΅Π½ΠΈΠ΅ силы Π΄Π²ΠΈΠΆΡƒΡ‰Π΅Π³ΠΎ сопротивлСния. К ΠΏΡ€ΠΈΠΌΠ΅Ρ€Ρƒ, ΠΊΠ½ΠΈΠ³Π°, ΠΊΠΎΡ‚ΠΎΡ€ΡƒΡŽ Π΄Π²ΠΈΠΆΡƒΡ‚ Ρ€ΡƒΠΊΠΎΠΉ, ΠΏΡ€ΠΈΠΆΠ°Ρ‚ΠΎΠΉ свСрху, ΡΠΊΠΎΠ»ΡŒΠ·ΠΈΡ‚ ΠΏΠΎΠ΄ дСйствиСм силы сопротивлСния покоя, Π²ΠΎΠ·Π½ΠΈΠΊΠ°ΡŽΡ‰Π΅ΠΉ ΠΌΠ΅ΠΆΠ΄Ρƒ Ρ€ΡƒΠΊΠΎΠΉ ΠΈ ΠΊΠ½ΠΈΠ³ΠΎΠΉ. Π’Π΅Π»ΠΈΡ‡ΠΈΠ½Π° сопротивлСния зависит ΠΎΡ‚ значСния силы Π²Π΅Ρ€Ρ‚ΠΈΠΊΠ°Π»ΡŒΠ½ΠΎΠ³ΠΎ давлСния Π½Π° ΠΊΠ½ΠΈΠ³Ρƒ.

Π―Π²Π»Π΅Π½ΠΈΠ΅ качСния

ΠŸΠ΅Ρ€Π΅Ρ…ΠΎΠ΄ Π½Π°ΡˆΠΈΡ… ΠΏΡ€Π΅Π΄ΠΊΠΎΠ² ΠΎΡ‚ Π²ΠΎΠ»ΠΎΠΊΡƒΡˆ ΠΊ колСсницам считаСтся Ρ€Π΅Π²ΠΎΠ»ΡŽΡ†ΠΈΠΎΠ½Π½Ρ‹ΠΌ. Π˜Π·ΠΎΠ±Ρ€Π΅Ρ‚Π΅Π½ΠΈΠ΅ колСса — Π²Π΅Π»ΠΈΡ‡Π°ΠΉΡˆΠ΅Π΅ ΠΈΠ·ΠΎΠ±Ρ€Π΅Ρ‚Π΅Π½ΠΈΠ΅ чСловСчСства. Π²ΠΎΠ·Π½ΠΈΠΊΠ°ΡŽΡ‰Π΅Π΅ ΠΏΡ€ΠΈ Π΄Π²ΠΈΠΆΠ΅Π½ΠΈΠΈ колСса ΠΏΠΎ повСрхности, Π·Π½Π°Ρ‡ΠΈΡ‚Π΅Π»ΡŒΠ½ΠΎ уступаСт ΠΏΠΎ Π²Π΅Π»ΠΈΡ‡ΠΈΠ½Π΅ ΡΠΎΠΏΡ€ΠΎΡ‚ΠΈΠ²Π»Π΅Π½ΠΈΡŽ скольТСния.

Π’ΠΎΠ·Π½ΠΈΠΊΠ½ΠΎΠ²Π΅Π½ΠΈΠ΅ сопряТСно с силами Π½ΠΎΡ€ΠΌΠ°Π»ΡŒΠ½ΠΎΠ³ΠΎ давлСния колСса Π½Π° ΠΏΠΎΠ²Π΅Ρ€Ρ…Π½ΠΎΡΡ‚ΡŒ, ΠΈΠΌΠ΅Π΅Ρ‚ ΠΏΡ€ΠΈΡ€ΠΎΠ΄Ρƒ, ΠΎΡ‚Π»ΠΈΡ‡Π°ΡŽΡ‰ΡƒΡŽ Π΅Π³ΠΎ ΠΎΡ‚ скольТСния. ВслСдствиС Π½Π΅Π·Π½Π°Ρ‡ΠΈΡ‚Π΅Π»ΡŒΠ½ΠΎΠΉ Π΄Π΅Ρ„ΠΎΡ€ΠΌΠ°Ρ†ΠΈΠΈ колСса Π²ΠΎΠ·Π½ΠΈΠΊΠ°ΡŽΡ‚ Ρ€Π°Π·Π½Ρ‹Π΅ ΠΏΠΎ Π²Π΅Π»ΠΈΡ‡ΠΈΠ½Π΅ силы давлСния Π² Ρ†Π΅Π½Ρ‚Ρ€Π΅ ΠΎΠ±Ρ€Π°Π·ΠΎΠ²Π°Π²ΡˆΠ΅ΠΉΡΡ ΠΏΠ»ΠΎΡ‰Π°Π΄ΠΊΠΈ ΠΈ ΠΏΠΎ Π΅Π΅ краям. Π­Ρ‚Π° Ρ€Π°Π·Π½ΠΈΡ†Π° сил ΠΈ опрСдСляСт Π²ΠΎΠ·Π½ΠΈΠΊΠ½ΠΎΠ²Π΅Π½ΠΈΠ΅ сопротивлСния ΠΏΡ€ΠΈ ΠΊΠ°Ρ‡Π΅Π½ΠΈΠΈ.

РасчСтная Ρ„ΠΎΡ€ΠΌΡƒΠ»Π° силы трСния качСния ΠΎΠ±Ρ‹ΠΊΠ½ΠΎΠ²Π΅Π½Π½ΠΎ бСрСтся Π°Π½Π°Π»ΠΎΠ³ΠΈΡ‡Π½ΠΎ процСссу скольТСния. Π Π°Π·Π»ΠΈΡ‡ΠΈΠ΅ Π²ΠΈΠ΄Π½ΠΎ ΠΈΡΠΊΠ»ΡŽΡ‡ΠΈΡ‚Π΅Π»ΡŒΠ½ΠΎ Π² значСниях коэффициСнта сопротивлСния.

ΠŸΡ€ΠΈΡ€ΠΎΠ΄Π° сопротивлСния

ΠŸΡ€ΠΈ ΠΈΠ·ΠΌΠ΅Π½Π΅Π½ΠΈΠΈ ΡˆΠ΅Ρ€ΠΎΡ…ΠΎΠ²Π°Ρ‚ΠΎΡΡ‚ΠΈ трущихся повСрхностСй мСняСтся ΠΈ Π·Π½Π°Ρ‡Π΅Π½ΠΈΠ΅ силы трСния. ΠŸΡ€ΠΈ большом ΡƒΠ²Π΅Π»ΠΈΡ‡Π΅Π½ΠΈΠΈ Π΄Π²Π΅ ΡΠΎΠΏΡ€ΠΈΠΊΠ°ΡΠ°ΡŽΡ‰ΠΈΠ΅ΡΡ повСрхности выглядят ΠΊΠ°ΠΊ нСровности с острыми ΠΏΠΈΠΊΠ°ΠΌΠΈ. ΠŸΡ€ΠΈ Π½Π°Π»ΠΎΠΆΠ΅Π½ΠΈΠΈ ΠΈΠΌΠ΅Π½Π½ΠΎ Π²Ρ‹ΡΡ‚ΡƒΠΏΠ°ΡŽΡ‰ΠΈΠΌΠΈ частями Ρ‚Π΅Π»Π° ΡΠΎΠΏΡ€ΠΈΠΊΠ°ΡΠ°ΡŽΡ‚ΡΡ Π΄Ρ€ΡƒΠ³ с Π΄Ρ€ΡƒΠ³ΠΎΠΌ. ΠžΠ±Ρ‰Π°Ρ ΠΏΠ»ΠΎΡ‰Π°Π΄ΡŒ соприкосновСния Π½Π΅Π·Π½Π°Ρ‡ΠΈΡ‚Π΅Π»ΡŒΠ½Π°. ΠŸΡ€ΠΈ Π΄Π²ΠΈΠΆΠ΅Π½ΠΈΠΈ ΠΈΠ»ΠΈ ΠΏΠΎΠΏΡ‹Ρ‚ΠΊΠ΅ двиТСния Ρ‚Π΅Π» Β«ΠΏΠΈΠΊΠΈΒ» ΡΠΎΠ·Π΄Π°ΡŽΡ‚ сопротивлСниС. Π’Π΅Π»ΠΈΡ‡ΠΈΠ½Π° силы трСния Π½Π΅ зависит ΠΎΡ‚ ΠΏΠ»ΠΎΡ‰Π°Π΄ΠΈ повСрхностСй соприкосновСния.

ΠŸΡ€Π΅Π΄ΡΡ‚Π°Π²Π»ΡΠ΅Ρ‚ΡΡ, Ρ‡Ρ‚ΠΎ Π΄Π²Π΅ идСально Π³Π»Π°Π΄ΠΊΠΈΠ΅ повСрхности Π΄ΠΎΠ»ΠΆΠ½Ρ‹ Π½Π΅ ΠΈΡΠΏΡ‹Ρ‚Ρ‹Π²Π°Ρ‚ΡŒ сопротивлСния Π°Π±ΡΠΎΠ»ΡŽΡ‚Π½ΠΎ. На ΠΏΡ€Π°ΠΊΡ‚ΠΈΠΊΠ΅ сила трСния Π² этом случаС максимальна. ΠžΠ±ΡŠΡΡΠ½ΡΠ΅Ρ‚ΡΡ это нСсоотвСтствиС ΠΏΡ€ΠΈΡ€ΠΎΠ΄ΠΎΠΉ возникновСния сил. Π­Ρ‚ΠΎ элСктромагнитныС силы, Π΄Π΅ΠΉΡΡ‚Π²ΡƒΡŽΡ‰ΠΈΠ΅ ΠΌΠ΅ΠΆΠ΄Ρƒ Π°Ρ‚ΠΎΠΌΠ°ΠΌΠΈ Π²Π·Π°ΠΈΠΌΠΎΠ΄Π΅ΠΉΡΡ‚Π²ΡƒΡŽΡ‰ΠΈΡ… Ρ‚Π΅Π».

ΠœΠ΅Ρ…Π°Π½ΠΈΡ‡Π΅ΡΠΊΠΈΠ΅ процСссы, Π½Π΅ ΡΠΎΠΏΡ€ΠΎΠ²ΠΎΠΆΠ΄Π°ΡŽΡ‰ΠΈΠ΅ΡΡ Ρ‚Ρ€Π΅Π½ΠΈΠ΅ΠΌ Π² ΠΏΡ€ΠΈΡ€ΠΎΠ΄Π΅, Π½Π΅Π²ΠΎΠ·ΠΌΠΎΠΆΠ½Ρ‹, вСдь возмоТности Β«ΠΎΡ‚ΠΊΠ»ΡŽΡ‡ΠΈΡ‚ΡŒΒ» элСктричСскоС взаимодСйствиС заряТСнных Ρ‚Π΅Π» Π½Π΅Ρ‚. ΠΠ΅Π·Π°Π²ΠΈΡΠΈΠΌΠΎΡΡ‚ΡŒ сил сопротивлСния ΠΎΡ‚ Π²Π·Π°ΠΈΠΌΠ½ΠΎΠ³ΠΎ полоТСния Ρ‚Π΅Π» позволяСт Π½Π°Π·Π²Π°Ρ‚ΡŒ ΠΈΡ… Π½Π΅ΠΏΠΎΡ‚Π΅Π½Ρ†ΠΈΠ°Π»ΡŒΠ½Ρ‹ΠΌΠΈ.

Π˜Π½Ρ‚Π΅Ρ€Π΅ΡΠ½ΠΎ, Ρ‡Ρ‚ΠΎ сила трСния, Ρ„ΠΎΡ€ΠΌΡƒΠ»Π° ΠΊΠΎΡ‚ΠΎΡ€ΠΎΠΉ мСняСтся Π² зависимости ΠΎΡ‚ скорости двиТСния Π²Π·Π°ΠΈΠΌΠΎΠ΄Π΅ΠΉΡΡ‚Π²ΡƒΡŽΡ‰ΠΈΡ… Ρ‚Π΅Π», ΠΏΡ€ΠΎΠΏΠΎΡ€Ρ†ΠΈΠΎΠ½Π°Π»ΡŒΠ½Π° ΠΊΠ²Π°Π΄Ρ€Π°Ρ‚Ρƒ ΡΠΎΠΎΡ‚Π²Π΅Ρ‚ΡΡ‚Π²ΡƒΡŽΡ‰Π΅ΠΉ скорости. К Ρ‚Π°ΠΊΠΎΠΉ силС относится сила вязкого сопротивлСния Π² Тидкости.

Π”Π²ΠΈΠΆΠ΅Π½ΠΈΠ΅ Π² Тидкости ΠΈ Π³Π°Π·Π΅

ΠŸΠ΅Ρ€Π΅ΠΌΠ΅Ρ‰Π΅Π½ΠΈΠ΅ Ρ‚Π²Π΅Ρ€Π΄ΠΎΠ³ΠΎ Ρ‚Π΅Π»Π° Π² Тидкости ΠΈΠ»ΠΈ Π³Π°Π·Π΅, Тидкости Π²Π±Π»ΠΈΠ·ΠΈ Ρ‚Π²Π΅Ρ€Π΄ΠΎΠΉ повСрхности сопровоТдаСтся вязким сопротивлСниСм. Π•Π³ΠΎ Π²ΠΎΠ·Π½ΠΈΠΊΠ½ΠΎΠ²Π΅Π½ΠΈΠ΅ ΡΠ²ΡΠ·Ρ‹Π²Π°ΡŽΡ‚ с взаимодСйствиСм слоСв Тидкости, ΡƒΠ²Π»Π΅ΠΊΠ°Π΅ΠΌΡ‹Ρ… Ρ‚Π²Π΅Ρ€Π΄Ρ‹ΠΌ Ρ‚Π΅Π»ΠΎΠΌ Π² процСссС двиТСния. Разная ΡΠΊΠΎΡ€ΠΎΡΡ‚ΡŒ слоСв — источник вязкого трСния. ΠžΡΠΎΠ±Π΅Π½Π½ΠΎΡΡ‚ΡŒ этого явлСния — отсутствиС ΠΆΠΈΠ΄ΠΊΠΎΠ³ΠΎ трСния покоя. НСзависимо ΠΎΡ‚ Π²Π΅Π»ΠΈΡ‡ΠΈΠ½Ρ‹ внСшнСго воздСйствия Ρ‚Π΅Π»ΠΎ ΠΏΡ€ΠΈΡ…ΠΎΠ΄ΠΈΡ‚ Π² Π΄Π²ΠΈΠΆΠ΅Π½ΠΈΠ΅, Π½Π°Ρ…ΠΎΠ΄ΡΡΡŒ Π² Тидкости.

Π’ зависимости ΠΎΡ‚ быстроты пСрСмСщСния сила сопротивлСния опрСдСляСтся ΡΠΊΠΎΡ€ΠΎΡΡ‚ΡŒΡŽ двиТСния, Ρ„ΠΎΡ€ΠΌΠΎΠΉ двиТущСгося Ρ‚Π΅Π»Π° ΠΈ Π²ΡΠ·ΠΊΠΎΡΡ‚ΡŒΡŽ Тидкости. Π”Π²ΠΈΠΆΠ΅Π½ΠΈΠ΅ Π² Π²ΠΎΠ΄Π΅ ΠΈ маслС ΠΎΠ΄Π½ΠΎΠ³ΠΎ ΠΈ Ρ‚ΠΎΠ³ΠΎ ΠΆΠ΅ Ρ‚Π΅Π»Π° сопровоТдаСтся Ρ€Π°Π·Π»ΠΈΡ‡Π½Ρ‹ΠΌ ΠΏΠΎ Π²Π΅Π»ΠΈΡ‡ΠΈΠ΅ сопротивлСниСм.

Для Π½Π΅Π±ΠΎΠ»ΡŒΡˆΠΈΡ… скоростСй: F = kv, Π³Π΄Π΅ k — коэффициСнт ΠΏΡ€ΠΎΠΏΠΎΡ€Ρ†ΠΈΠΎΠ½Π°Π»ΡŒΠ½ΠΎΡΡ‚ΠΈ, зависящий ΠΎΡ‚ Π»ΠΈΠ½Π΅ΠΉΠ½Ρ‹Ρ… Ρ€Π°Π·ΠΌΠ΅Ρ€ΠΎΠ² Ρ‚Π΅Π»Π° ΠΈ свойств срСды, v — ΡΠΊΠΎΡ€ΠΎΡΡ‚ΡŒ Ρ‚Π΅Π»Π°.

Π’Π΅ΠΌΠΏΠ΅Ρ€Π°Ρ‚ΡƒΡ€Π° Тидкости Ρ‚Π°ΠΊΠΆΠ΅ влияСт Π½Π° Ρ‚Ρ€Π΅Π½ΠΈΠ΅ Π² Π½Π΅ΠΉ. Π’ ΠΌΠΎΡ€ΠΎΠ·Π½ΡƒΡŽ ΠΏΠΎΠ³ΠΎΠ΄Ρƒ Π°Π²Ρ‚ΠΎΠΌΠΎΠ±ΠΈΠ»ΡŒ Ρ€Π°Π·ΠΎΠ³Ρ€Π΅Π²Π°ΡŽΡ‚ для Ρ‚ΠΎΠ³ΠΎ, Ρ‡Ρ‚ΠΎΠ±Ρ‹ масло Π½Π°Π³Ρ€Π΅Π»ΠΎΡΡŒ (Π΅Π³ΠΎ Π²ΡΠ·ΠΊΠΎΡΡ‚ΡŒ ΡƒΠΌΠ΅Π½ΡŒΡˆΠ°Π΅Ρ‚ΡΡ) ΠΈ способствовало ΡƒΠΌΠ΅Π½ΡŒΡˆΠ΅Π½ΠΈΡŽ Ρ€Π°Π·Ρ€ΡƒΡˆΠ΅Π½ΠΈΡ ΡΠΎΠΏΡ€ΠΈΠΊΠ°ΡΠ°ΡŽΡ‰ΠΈΡ…ΡΡ Π΄Π΅Ρ‚Π°Π»Π΅ΠΉ двигатСля.

Π£Π²Π΅Π»ΠΈΡ‡Π΅Π½ΠΈΠ΅ скорости двиТСния

Π—Π½Π°Ρ‡ΠΈΡ‚Π΅Π»ΡŒΠ½ΠΎΠ΅ ΡƒΠ²Π΅Π»ΠΈΡ‡Π΅Π½ΠΈΠ΅ скорости Ρ‚Π΅Π»Π° ΠΌΠΎΠΆΠ΅Ρ‚ Π²Ρ‹Π·Π²Π°Ρ‚ΡŒ появлСниС Ρ‚ΡƒΡ€Π±ΡƒΠ»Π΅Π½Ρ‚Π½Ρ‹Ρ… ΠΏΠΎΡ‚ΠΎΠΊΠΎΠ², ΠΏΡ€ΠΈ этом сопротивлСниС Ρ€Π΅Π·ΠΊΠΎ возрастаСт. Π—Π½Π°Ρ‡Π΅Π½ΠΈΠ΅ ΠΈΠΌΠ΅ΡŽΡ‚: ΠΊΠ²Π°Π΄Ρ€Π°Ρ‚ скорости двиТСния, ΠΏΠ»ΠΎΡ‚Π½ΠΎΡΡ‚ΡŒ срСды ΠΈ силы трСния ΠΏΡ€ΠΈΠΎΠ±Ρ€Π΅Ρ‚Π°Π΅Ρ‚ ΠΈΠ½ΠΎΠΉ Π²ΠΈΠ΄:

F = kv 2 , Π³Π΄Π΅ k — коэффициСнт ΠΏΡ€ΠΎΠΏΠΎΡ€Ρ†ΠΈΠΎΠ½Π°Π»ΡŒΠ½ΠΎΡΡ‚ΠΈ, зависящий ΠΎΡ‚ Ρ„ΠΎΡ€ΠΌΡ‹ Ρ‚Π΅Π»Π° ΠΈ свойств срСды, v — ΡΠΊΠΎΡ€ΠΎΡΡ‚ΡŒ Ρ‚Π΅Π»Π°.

Если Ρ‚Π΅Π»Ρƒ ΠΏΡ€ΠΈΠ΄Π°Ρ‚ΡŒ ΠΎΠ±Ρ‚Π΅ΠΊΠ°Π΅ΠΌΡƒΡŽ Ρ„ΠΎΡ€ΠΌΡƒ, Ρ‚ΡƒΡ€Π±ΡƒΠ»Π΅Π½Ρ‚Π½ΠΎΡΡ‚ΡŒ ΠΌΠΎΠΆΠ½ΠΎ ΡƒΠΌΠ΅Π½ΡŒΡˆΠΈΡ‚ΡŒ. Π€ΠΎΡ€ΠΌΠ° Ρ‚Π΅Π»Π° Π΄Π΅Π»ΡŒΡ„ΠΈΠ½ΠΎΠ² ΠΈ ΠΊΠΈΡ‚ΠΎΠ² — прСкрасный ΠΏΡ€ΠΈΠΌΠ΅Ρ€ Π·Π°ΠΊΠΎΠ½ΠΎΠ² ΠΏΡ€ΠΈΡ€ΠΎΠ΄Ρ‹, Π²Π»ΠΈΡΡŽΡ‰ΠΈΡ… Π½Π° ΡΠΊΠΎΡ€ΠΎΡΡ‚ΡŒ ΠΆΠΈΠ²ΠΎΡ‚Π½Ρ‹Ρ….

ЭнСргСтичСский ΠΏΠΎΠ΄Ρ…ΠΎΠ΄

Π‘ΠΎΠ²Π΅Ρ€ΡˆΠΈΡ‚ΡŒ Ρ€Π°Π±ΠΎΡ‚Ρƒ ΠΏΠΎ ΠΏΠ΅Ρ€Π΅ΠΌΠ΅Ρ‰Π΅Π½ΠΈΡŽ Ρ‚Π΅Π»Π° прСпятствуСт сопротивлСниС срСды. ΠŸΡ€ΠΈ использовании Π·Π°ΠΊΠΎΠ½Π° сохранСния энСргии говорят, Ρ‡Ρ‚ΠΎ ΠΈΠ·ΠΌΠ΅Π½Π΅Π½ΠΈΠ΅ мСханичСской энСргии Ρ€Π°Π²Π½ΠΎ Ρ€Π°Π±ΠΎΡ‚Π΅ сил трСния.

Π Π°Π±ΠΎΡ‚Π° силы рассчитываСтся ΠΏΠΎ Ρ„ΠΎΡ€ΠΌΡƒΠ»Π΅: A = FscosΞ±, Π³Π΄Π΅ F — сила, ΠΏΠΎΠ΄ дСйствиСм ΠΊΠΎΡ‚ΠΎΡ€ΠΎΠΉ Ρ‚Π΅Π»ΠΎ пСрСмСщаСтся Π½Π° расстояниС s, Ξ± — ΡƒΠ³ΠΎΠ» ΠΌΠ΅ΠΆΠ΄Ρƒ направлСниями силы ΠΈ пСрСмСщСния.

ΠžΡ‡Π΅Π²ΠΈΠ΄Π½ΠΎ, Ρ‡Ρ‚ΠΎ сила сопротивлСния ΠΏΡ€ΠΎΡ‚ΠΈΠ²ΠΎΠΏΠΎΠ»ΠΎΠΆΠ½Π° ΠΏΠ΅Ρ€Π΅ΠΌΠ΅Ρ‰Π΅Π½ΠΈΡŽ Ρ‚Π΅Π»Π°, ΠΎΡ‚ΠΊΡƒΠ΄Π° cosΞ± = -1. Π Π°Π±ΠΎΡ‚Π° силы трСния, Ρ„ΠΎΡ€ΠΌΡƒΠ»Π° ΠΊΠΎΡ‚ΠΎΡ€ΠΎΠΉ ΠΈΠΌΠ΅Π΅Ρ‚ Π²ΠΈΠ΄ A Ρ‚Ρ€ = — Fs, Π²Π΅Π»ΠΈΡ‡ΠΈΠ½Π° ΠΎΡ‚Ρ€ΠΈΡ†Π°Ρ‚Π΅Π»ΡŒΠ½Π°Ρ. ΠŸΡ€ΠΈ этом прСвращаСтся Π²ΠΎ Π²Π½ΡƒΡ‚Ρ€Π΅Π½Π½ΡŽΡŽ (дСформация, Π½Π°Π³Ρ€Π΅Π²Π°Π½ΠΈΠ΅).

Π˜Π·ΠΌΠ΅Ρ€Π΅Π½ΠΈΠ΅ коэффициСнта трСния скольТСния Π½Π΅ΠΎΠ±Ρ…ΠΎΠ΄ΠΈΠΌΠΎ провСсти двумя способами.

1-ΠΉ способ Π·Π°ΠΊΠ»ΡŽΡ‡Π°Π΅Ρ‚ΡΡ Π² ΠΈΠ·ΠΌΠ΅Ρ€Π΅Π½ΠΈΠΈ с ΠΏΠΎΠΌΠΎΡ‰ΡŒΡŽ Π΄ΠΈΠ½Π°ΠΌΠΎΠΌΠ΅Ρ‚Ρ€Π° силы, с ΠΊΠΎΡ‚ΠΎΡ€ΠΎΠΉ Π½ΡƒΠΆΠ½ΠΎ Ρ‚ΡΠ½ΡƒΡ‚ΡŒ брусок с Π³Ρ€ΡƒΠ·Π°ΠΌΠΈ ΠΏΠΎ Π³ΠΎΡ€ΠΈΠ·ΠΎΠ½Ρ‚Π°Π»ΡŒΠ½ΠΎΠΉ повСрхности, для Ρ‚ΠΎΠ³ΠΎ Ρ‡Ρ‚ΠΎΠ±Ρ‹ ΠΎΠ½ двигался Ρ€Π°Π²Π½ΠΎΠΌΠ΅Ρ€Π½ΠΎ. Π­Ρ‚Π° сила Ρ€Π°Π²Π½Π° ΠΏΠΎ Π°Π±ΡΠΎΠ»ΡŽΡ‚Π½ΠΎΠΉ Π²Π΅Π»ΠΈΡ‡ΠΈΠ½Π΅ силС трСния Π΄Π΅ΠΉΡΡ‚Π²ΡƒΡŽΡ‰Π΅ΠΉ Π½Π° брусок. Π‘ ΠΏΠΎΠΌΠΎΡ‰ΡŒΡŽ Ρ‚ΠΎΠ³ΠΎ ΠΆΠ΅ Π΄ΠΈΠ½Π°ΠΌΠΎΠΌΠ΅Ρ‚Ρ€Π° ΠΌΠΎΠΆΠ½ΠΎ Π½Π°ΠΉΡ‚ΠΈ вСс бруска ΠΎ Π³Ρ€ΡƒΠ·Π°ΠΌΠΈ Π . Π­Ρ‚ΠΎΡ‚ вСс Ρ€Π°Π²Π΅Π½ силС Π½ΠΎΡ€ΠΌΠ°Π»ΡŒΠ½ΠΎΠ³ΠΎ давлСния бруска Π½Π° ΠΏΠΎΠ²Π΅Ρ€Ρ…Π½ΠΎΡΡ‚ΡŒ, ΠΏΠΎ ΠΊΠΎΡ‚ΠΎΡ€ΠΎΠΉ ΠΎΠ½ ΡΠΊΠΎΠ»ΡŒΠ·ΠΈΡ‚. ΠžΠΏΡ€Π΅Π΄Π΅Π»ΠΈΠ² Ρ‚Π°ΠΊΠΈΠΌ ΠΎΠ±Ρ€Π°Π·ΠΎΠΌ ΠΌΠΎΠΆΠ½ΠΎ Π½Π°ΠΉΡ‚ΠΈ коэффициСнт трСния. Он Ρ€Π°Π²Π΅Π½:

2-ΠΉ способ измСрСния коэффициСнта трСния позволяСт ΠΎΠΏΡ€Π΅Π΄Π΅Π»ΡΡ‚ΡŒ Π½Π° ΠΎΠΏΡ‹Ρ‚Π΅ Π½Π΅ силы, Π° Π΄Π»ΠΈΠ½Ρ‹ ΠΎΡ‚Ρ€Π΅Π·ΠΊΠΎΠ². Для этого ΠΈΡΠΏΠΎΠ»ΡŒΠ·ΡƒΡŽΡ‚ равновСсиС бруска, ΠΊΠΎΡ‚ΠΎΡ€Ρ‹ΠΉ находится Π½Π° Π½Π°ΠΊΠ»ΠΎΠ½Π½ΠΎΠΉ плоскости.

Если брусок находится Π² равновСсии Π½Π° Π½Π°ΠΊΠ»ΠΎΠ½Π½ΠΎΠΉ плоскости, Ρ‚ΠΎ сила Π½ΠΎΡ€ΠΌΠ°Π»ΡŒΠ½ΠΎΠ³ΠΎ давлСния бруска Π½Π° ΠΏΠ»ΠΎΡΠΊΠΎΡΡ‚ΡŒ Ρ€Π°Π²Π½Π° ΡΠΎΡΡ‚Π°Π²Π»ΡΡŽΡ‰Π΅ΠΉ силы тяТСсти, пСрпСндикулярной Π½Π°ΠΊΠ»ΠΎΠ½Π½ΠΎΠΉ плоскости (рис. 213). А сила трСния ΠΏΠΎ Π°Π±ΡΠΎΠ»ΡŽΡ‚Π½ΠΎΠΉ Π²Π΅Π»ΠΈΡ‡ΠΈΠ½Π΅ Ρ€Π°Π²Π½Π° ΡΠΎΡΡ‚Π°Π²Π»ΡΡŽΡ‰Π΅ΠΉ силы тяТСсти, ΠΏΠ°Ρ€Π°Π»Π»Π΅Π»ΡŒΠ½ΠΎΠΉ Π½Π°ΠΊΠ»ΠΎΠ½Π½ΠΎΠΉ плоскости.

ΠžΠΏΡ‹Ρ‚ Π·Π°ΠΊΠ»ΡŽΡ‡Π°Π΅Ρ‚ΡΡ Π² Ρ‚ΠΎΠΌ, Ρ‡Ρ‚ΠΎΠ±Ρ‹, увСличивая постСпСнно ΡƒΠ³ΠΎΠ» Π½Π°ΠΊΠ»ΠΎΠ½Π° плоскости, Π½Π°ΠΉΡ‚ΠΈ Ρ‚Π°ΠΊΠΎΠΉ ΡƒΠ³ΠΎΠ», ΠΏΡ€ΠΈ ΠΊΠΎΡ‚ΠΎΡ€ΠΎΠΌ брусок Ρ‚ΠΎΠ»ΡŒΠΊΠΎ «тронСтся с мСста». ΠŸΡ€ΠΈ этом сила трСния Π±ΡƒΠ΄Π΅Ρ‚ Ρ€Π°Π²Π½Π° максимальной силС трСния покоя:

Π³Π΄Π΅ — сила давлСния бруска Π½Π° ΠΏΠ»ΠΎΡΠΊΠΎΡΡ‚ΡŒ Π’Π°ΠΊ ΠΊΠ°ΠΊ ΠΏΡ€ΠΈ этом Ρ‚. Π΅.

НСтрудно ΠΏΠΎΠΊΠ°Π·Π°Ρ‚ΡŒ, Ρ‡Ρ‚ΠΎ

Π­Ρ‚ΠΎ слСдуСт ΠΈΠ· подобия Ρ‚Ρ€Π΅ΡƒΠ³ΠΎΠ»ΡŒΠ½ΠΈΠΊΠΎΠ² ΠŸΠΎΡΡ‚ΠΎΠΌΡƒ коэффициСнт трСния Ρ€Π°Π²Π΅Π½:

Из этой Ρ„ΠΎΡ€ΠΌΡƒΠ»Ρ‹ Π²ΠΈΠ΄Π½ΠΎ, Ρ‡Ρ‚ΠΎ для Ρ‚ΠΎΠ³ΠΎ Ρ‡Ρ‚ΠΎΠ±Ρ‹ Π½Π°ΠΉΡ‚ΠΈ коэффициСнт трСния, достаточно ΠΈΠ·ΠΌΠ΅Ρ€ΠΈΡ‚ΡŒ высоту ΠΈ основаниС Π½Π°ΠΊΠ»ΠΎΠ½Π½ΠΎΠΉ плоскости, ΠΊΠΎΡ‚ΠΎΡ€Ρ‹ΠΌΠΈ опрСдСляСтся Π½Π°ΠΊΠ»ΠΎΠ½ плоскости, ΠΏΡ€ΠΈ ΠΊΠΎΡ‚ΠΎΡ€ΠΎΠΌ начинаСтся скольТСниС бруска.

ΠŸΡ€ΠΈΠ±ΠΎΡ€Ρ‹ ΠΈ ΠΌΠ°Ρ‚Π΅Ρ€ΠΈΠ°Π»Ρ‹: 1) Π»ΠΈΠ½Π΅ΠΉΠΊΠ°, 2) ΠΈΠ·ΠΌΠ΅Ρ€ΠΈΡ‚Π΅Π»ΡŒΠ½Π°Ρ Π»Π΅Π½Ρ‚Π°,

3) Π΄ΠΈΠ½Π°ΠΌΠΎΠΌΠ΅Ρ‚Ρ€, 4) дСрСвянный брусок, 5) Π½Π°Π±ΠΎΡ€ Π³Ρ€ΡƒΠ·ΠΎΠ², 6) ΡˆΡ‚Π°Ρ‚ΠΈΠ² с ΠΌΡƒΡ„Ρ‚Π°ΠΌΠΈ ΠΈ Π»Π°ΠΏΠΊΠΎΠΉ.

ΠŸΠΎΡ€ΡΠ΄ΠΎΠΊ выполнСния Ρ€Π°Π±ΠΎΡ‚Ρ‹

1. ΠŸΠΎΠ»ΠΎΠΆΠΈΡ‚ΡŒ брусок Π½Π° Π³ΠΎΡ€ΠΈΠ·ΠΎΠ½Ρ‚Π°Π»ΡŒΠ½ΠΎ Ρ€Π°ΡΠΏΠΎΠ»ΠΎΠΆΠ΅Π½Π½ΡƒΡŽ Π΄Π΅Ρ€Π΅Π²ΡΠ½Π½ΡƒΡŽ Π»ΠΈΠ½Π΅ΠΉΠΊΡƒ. На брусок ΠΏΠΎΡΡ‚Π°Π²ΠΈΡ‚ΡŒ Π³Ρ€ΡƒΠ·.

2. ΠŸΡ€ΠΈΠΊΡ€Π΅ΠΏΠΈΠ² ΠΊ бруску Π΄ΠΈΠ½Π°ΠΌΠΎΠΌΠ΅Ρ‚Ρ€, ΠΊΠ°ΠΊ ΠΌΠΎΠΆΠ½ΠΎ Π±ΠΎΠ»Π΅Π΅ Ρ€Π°Π²Π½ΠΎΠΌΠ΅Ρ€Π½ΠΎ Ρ‚ΡΠ½ΡƒΡ‚ΡŒ Π΅Π³ΠΎ вдоль Π»ΠΈΠ½Π΅ΠΉΠΊΠΈ. Π—Π°ΠΌΠ΅Ρ‚ΠΈΡ‚ΡŒ ΠΏΡ€ΠΈ этом ΠΏΠΎΠΊΠ°Π·Π°Π½ΠΈΠ΅ Π΄ΠΈΠ½Π°ΠΌΠΎΠΌΠ΅Ρ‚Ρ€Π°.

3. Π’Π·Π²Π΅ΡΠΈΡ‚ΡŒ брусок ΠΈ Π³Ρ€ΡƒΠ·.

4. По Ρ„ΠΎΡ€ΠΌΡƒΠ»Π΅ Π½Π°ΠΉΡ‚ΠΈ коэффициСнт трСния.

5. ΠŸΠΎΠ²Ρ‚ΠΎΡ€ΠΈΡ‚ΡŒ ΠΎΠΏΡ‹Ρ‚, ΠΏΠΎΠ»ΠΎΠΆΠΈΠ² Π½Π° брусок нСсколько Π³Ρ€ΡƒΠ·ΠΎΠ².

6. Найти срСднСС арифмСтичСскоС Π·Π½Π°Ρ‡Π΅Π½ΠΈΠ΅ коэффициСнтов трСния, Π½Π°ΠΉΠ΄Π΅Π½Π½Ρ‹Ρ… Π² Ρ€Π°Π·Π½Ρ‹Ρ… ΠΎΠΏΡ‹Ρ‚Π°Ρ….

7. Найти ΠΎΡˆΠΈΠ±ΠΊΡƒ ΠΊΠ°ΠΆΠ΄ΠΎΠ³ΠΎ ΠΈΠ· ΠΎΠΏΡ‹Ρ‚ΠΎΠ² — Ρ€Π°Π·Π½ΠΎΡΡ‚ΡŒ ΠΌΠ΅ΠΆΠ΄Ρƒ ΠΈ значСниями ΠΏΠΎΠ»ΡƒΡ‡Π΅Π½Π½Ρ‹ΠΌΠΈ Π² Ρ€Π°Π·Π½Ρ‹Ρ… ΠΎΠΏΡ‹Ρ‚Π°Ρ….

8. ΠžΠΏΡ€Π΅Π΄Π΅Π»ΠΈΡ‚ΡŒ срСднСС арифмСтичСскоС ошибок ΠΎΠΏΡ‹Ρ‚ΠΎΠ²

9. Π‘ΠΎΡΡ‚Π°Π²ΠΈΡ‚ΡŒ Ρ‚Π°Π±Π»ΠΈΡ†Ρƒ Ρ€Π΅Π·ΡƒΠ»ΡŒΡ‚Π°Ρ‚ΠΎΠ² ΠΎΠΏΡ‹Ρ‚ΠΎΠ²:

10. Π—Π°ΠΏΠΈΡΠ°Ρ‚ΡŒ Ρ€Π΅Π·ΡƒΠ»ΡŒΡ‚Π°Ρ‚ ΠΈΠ·ΠΌΠ΅Ρ€Π΅Π½ΠΈΠΉ Π² Π²ΠΈΠ΄Π΅

11. ПолоТив Π»ΠΈΠ½Π΅ΠΉΠΊΡƒ Π½Π° брусок с Π³Ρ€ΡƒΠ·Π°ΠΌΠΈ, ΠΌΠ΅Π΄Π»Π΅Π½Π½ΠΎ ΠΈΠ·ΠΌΠ΅Π½ΡΡ‚ΡŒ Π΅Π΅ Π½Π°ΠΊΠ»ΠΎΠ½, поднимая Π΅Π΅ ΠΊΠΎΠ½Π΅Ρ†, ΠΏΠΎΠΊΠ° брусок Π½Π΅ Π½Π°Ρ‡Π½Π΅Ρ‚ ΡΠΊΠΎΠ»ΡŒΠ·ΠΈΡ‚ΡŒ вдоль Π»ΠΈΠ½Π΅ΠΉΠΊΠΈ.

ΠšΠΎΡΡ„Ρ„ΠΈΡ†ΠΈΠ΅Π½Ρ‚ трСния — Poly Print

ΠžΠΏΡ€Π΅Π΄Π΅Π»Π΅Π½ΠΈΠ΅

ΠšΠΎΡΡ„Ρ„ΠΈΡ†ΠΈΠ΅Π½Ρ‚ трСния (COF) — это Π±Π΅Π·Ρ€Π°Π·ΠΌΠ΅Ρ€Π½ΠΎΠ΅ число, ΠΊΠΎΡ‚ΠΎΡ€ΠΎΠ΅ прСдставляСт сопротивлСниС скольТСнию Π΄Π²ΡƒΡ… повСрхностСй, ΠΊΠΎΠ½Ρ‚Π°ΠΊΡ‚ΠΈΡ€ΡƒΡŽΡ‰ΠΈΡ… Π΄Ρ€ΡƒΠ³ с Π΄Ρ€ΡƒΠ³ΠΎΠΌ. Π­Ρ‚ΠΈ значСния Π΄ΠΎΠ»ΠΆΠ½Ρ‹ Π½Π°Ρ…ΠΎΠ΄ΠΈΡ‚ΡŒΡΡ Π² Π΄ΠΈΠ°ΠΏΠ°Π·ΠΎΠ½Π΅ ΠΎΡ‚ 0 Π΄ΠΎ 1, ΠΏΡ€ΠΈΡ‡Π΅ΠΌ Π±ΠΎΠ»Π΅Π΅ высокоС Π·Π½Π°Ρ‡Π΅Π½ΠΈΠ΅ ΡƒΠΊΠ°Π·Ρ‹Π²Π°Π΅Ρ‚ Π½Π° большСС сопротивлСниС скольТСнию.

БтатичСскиС значСния COF ΠΈΠ·ΠΌΠ΅Ρ€ΡΡŽΡ‚ΡΡ, ΠΊΠΎΠ³Π΄Π° Π΄Π²Π΅ повСрхности Ρ‚ΠΎΠ»ΡŒΠΊΠΎ Π½Π°Ρ‡ΠΈΠ½Π°ΡŽΡ‚ Π΄Π²ΠΈΠ³Π°Ρ‚ΡŒΡΡ Π΄Ρ€ΡƒΠ³ ΠΎΡ‚Π½ΠΎΡΠΈΡ‚Π΅Π»ΡŒΠ½ΠΎ Π΄Ρ€ΡƒΠ³Π°, Π° кинСтичСскиС значСния ΠΈΠ·ΠΌΠ΅Ρ€ΡΡŽΡ‚ΡΡ послС достиТСния постоянного двиТСния. ExxonMobil сообщаСт кинСтичСскиС значСния COF.

БоотвСтствиС эксплуатационным характСристикам

Π›Π°Π±ΠΎΡ€Π°Ρ‚ΠΎΡ€Π½ΠΎΠ΅ ΠΈΠ·ΠΌΠ΅Ρ€Π΅Π½ΠΈΠ΅ COF ΠΌΠ΅ΠΆΠ΄Ρƒ ΠΏΠ»Π΅Π½ΠΊΠΎΠΉ ΠΎΠ±Ρ‹Ρ‡Π½ΠΎ ΠΈΡΠΏΠΎΠ»ΡŒΠ·ΡƒΠ΅Ρ‚ΡΡ Π² индустрии Π³ΠΈΠ±ΠΊΠΈΡ… ΡƒΠΏΠ°ΠΊΠΎΠ²ΠΎΠΊ для количСствСнной ΠΎΡ†Π΅Π½ΠΊΠΈ ΠΈ сравнСния Ρ„Ρ€ΠΈΠΊΡ†ΠΈΠΎΠ½Π½Ρ‹Ρ… свойств повСрхности ΠΏΠ»Π΅Π½ΠΊΠΈ ΠΏΠΎΡΠ»Π΅Π΄ΠΎΠ²Π°Ρ‚Π΅Π»ΡŒΠ½Ρ‹ΠΌ ΠΈ ΡƒΠ΄ΠΎΠ±Π½Ρ‹ΠΌ способом. НС сущСствуСт ΡƒΠ½ΠΈΠ²Π΅Ρ€ΡΠ°Π»ΡŒΠ½ΠΎ Ρ…ΠΎΡ€ΠΎΡˆΠΈΡ… ΠΈΠ»ΠΈ ΠΏΠ»ΠΎΡ…ΠΈΡ… Π·Π½Π°Ρ‡Π΅Π½ΠΈΠΉ коэффициСнта трСния, Π½ΠΎ ΠΎΠ±Ρ‹Ρ‡Π½ΠΎ значСния Π±ΠΎΠ»Π΅Π΅ 0,50 ΡΡ‡ΠΈΡ‚Π°ΡŽΡ‚ΡΡ нСскользкими повСрхностями, Π° значСния ΠΌΠ΅Π½Π΅Π΅ 0,20 ΡΡ‡ΠΈΡ‚Π°ΡŽΡ‚ΡΡ ΠΏΠ»Π΅Π½ΠΊΠ°ΠΌΠΈ с высоким скольТСниСм, ΠΊΠΎΡ‚ΠΎΡ€Ρ‹Π΅ ΠΌΠΎΠ³ΡƒΡ‚ Π±Ρ‹Ρ‚ΡŒ склонны ΠΊ Ρ‚Π΅Π»Π΅ΡΠΊΠΎΠΏΠΈΡ€ΠΎΠ²Π°Π½ΠΈΡŽ Π½Π° Ρ€ΠΎΠ»ΠΈΠΊΠ°Ρ….

ΠžΠΏΡ‚ΠΈΠΌΠ°Π»ΡŒΠ½Ρ‹Π΅ свойства скольТСния зависят ΠΎΡ‚ области примСнСния, Π½ΠΎ ΠΌΠΎΠ³ΡƒΡ‚ ΠΈΠΌΠ΅Ρ‚ΡŒ Ρ€Π΅ΡˆΠ°ΡŽΡ‰Π΅Π΅ Π·Π½Π°Ρ‡Π΅Π½ΠΈΠ΅ для Ρ…ΠΎΡ€ΠΎΡˆΠ΅ΠΉ обрабатываСмости ΠΈ транспортировки ΡƒΠΏΠ°ΠΊΠΎΠ²ΠΊΠΈ, ΠΊΠ°ΠΊ ΠΏΠΎΠΊΠ°Π·Π°Π½ΠΎ Π² Π’Π°Π±Π»ΠΈΡ†Π΅ 7.

Битуация
Π–Π΅Π»Π°Π΅ΠΌΡ‹ΠΉ эффСкт
Π’Π°ΠΆΠ½ΠΎΡΡ‚ΡŒ
ПлСнка ΠΏΡ€ΠΎΡ…ΠΎΠ΄ΠΈΡ‚ ΠΏΠΎ пластинам Π΄Π΅ΠΊΠΈ лопастного колСса HFFS.
НизкоС внСшнСС Ρ‚Ρ€Π΅Π½ΠΈΠ΅, ΠΏΠ»Π΅Π½ΠΊΠ°-ΠΌΠ΅Ρ‚Π°Π»Π»
ΠŸΡ€Π΅Π΄ΠΎΡ‚Π²Ρ€Π°Ρ‰Π°Π΅Ρ‚ растяТСниС ΠΈ застрСваниС ΠΏΠ»Π΅Π½ΠΊΠΈ.
На VFFS ΠΏΠ»Π΅Π½ΠΊΠ° Π²Ρ…ΠΎΠ΄ΠΈΡ‚ Π² Ρ„ΠΎΡ€ΠΌΠΈΡ€ΡƒΡŽΡ‰ΡƒΡŽ ΠΌΠ°Π½ΠΆΠ΅Ρ‚Ρƒ Π² Π²ΠΈΠ΄Π΅ Π³ΠΎΡ€ΠΈΠ·ΠΎΠ½Ρ‚Π°Π»ΡŒΠ½ΠΎΠ³ΠΎ плоского ΠΏΠΎΠ»ΠΎΡ‚Π½Π° ΠΈ трансформируСтся Π² Π²Π΅Ρ€Ρ‚ΠΈΠΊΠ°Π»ΡŒΠ½ΡƒΡŽ Ρ‚Ρ€ΡƒΠ±Ρƒ.
НизкоС внСшнСС Ρ‚Ρ€Π΅Π½ΠΈΠ΅, ΠΏΠ»Π΅Π½ΠΊΠ°-ΠΌΠ΅Ρ‚Π°Π»Π»
ΠŸΡ€Π΅Π΄ΠΎΡ‚Π²Ρ€Π°Ρ‰Π°Π΅Ρ‚ скрип ΠΏΠ»Π΅Π½ΠΊΠΈ, Π½Π΅ΠΏΠΎΡΡ‚ΠΎΡΠ½Π½ΡƒΡŽ ΠΏΠΎΠ΄Π°Ρ‡Ρƒ ΠΏΠ»Π΅Π½ΠΊΠΈ ΠΈ Π½Π΅ΠΏΠΎΡΡ‚ΠΎΡΠ½Π½ΡƒΡŽ Π΄Π»ΠΈΠ½Ρƒ ΠΏΠ°ΠΊΠ΅Ρ‚ΠΎΠ².
Π—Π°ΠΏΠΎΠ»Π½Π΅Π½Π½Ρ‹Π΅ ΡƒΠΏΠ°ΠΊΠΎΠ²ΠΊΠΈ ΡƒΠΊΠ»Π°Π΄Ρ‹Π²Π°ΡŽΡ‚ΡΡ Π² гофроящики.
НизкоС внСшнСС Ρ‚Ρ€Π΅Π½ΠΈΠ΅, ΠΏΠ»Π΅Π½ΠΊΠ°-ΠΏΠ»Π΅Π½ΠΊΠ°
ΠŸΠΎΠ·Π²ΠΎΠ»ΡΠ΅Ρ‚ ΡƒΠΏΠ°ΠΊΠΎΠ²ΠΊΠ°ΠΌ ΡΠΊΠΎΠ»ΡŒΠ·ΠΈΡ‚ΡŒ Π΄Ρ€ΡƒΠ³ ΠΎΡ‚Π½ΠΎΡΠΈΡ‚Π΅Π»ΡŒΠ½ΠΎ Π΄Ρ€ΡƒΠ³Π° ΠΈ ΠΎΡΠ΅Π΄Π°Ρ‚ΡŒ для Π»Π΅Π³ΠΊΠΎΠ³ΠΎ закрытия ΠΊΠΎΡ€ΠΎΠ±ΠΊΠΈ.
На VFFS с Ρ„Ρ€ΠΈΠΊΡ†ΠΈΠΎΠ½Π½Ρ‹ΠΌ Ρ€Π΅ΠΌΠ΅Π½Π½Ρ‹ΠΌ ΠΏΡ€ΠΈΠ²ΠΎΠ΄ΠΎΠΌ Ρ€Π΅ΠΌΠ½ΠΈ с сСрвоприводом Ρ‚ΠΎΠ»ΠΊΠ°ΡŽΡ‚ ΠΏΠ»Π΅Π½ΠΊΡƒ ΠΈ ΠΏΠ΅Ρ€Π΅ΠΌΠ΅Ρ‰Π°ΡŽΡ‚ Π΅Π΅ ΠΏΠΎ Π²Π½ΡƒΡ‚Ρ€Π΅Π½Π½Π΅ΠΉ Ρ‚Ρ€ΡƒΠ±Π΅.
Π£ΠΌΠ΅Ρ€Π΅Π½Π½ΠΎΠ΅ внСшнСС Ρ‚Ρ€Π΅Π½ΠΈΠ΅, Π½ΠΈΠ·ΠΊΠΎΠ΅ Π²Π½ΡƒΡ‚Ρ€Π΅Π½Π½Π΅Π΅ Ρ‚Ρ€Π΅Π½ΠΈΠ΅
Π Π΅ΠΌΠ½ΠΈ Π΄ΠΎΠ»ΠΆΠ½Ρ‹ Β«Ρ†Π΅ΠΏΠ»ΡΡ‚ΡŒΡΡΒ» Π·Π° внСшнюю ΠΏΠΎΠ²Π΅Ρ€Ρ…Π½ΠΎΡΡ‚ΡŒ для пСрСмСщСния ΠΏΠ»Π΅Π½ΠΊΠΈ, Π² Ρ‚ΠΎ врСмя ΠΊΠ°ΠΊ внутрСнняя ΠΏΠΎΠ²Π΅Ρ€Ρ…Π½ΠΎΡΡ‚ΡŒ Π΄ΠΎΠ»ΠΆΠ½Π° ΡΠΊΠΎΠ»ΡŒΠ·ΠΈΡ‚ΡŒ ΠΏΠΎ Π½Π΅ΠΏΠΎΠ΄Π²ΠΈΠΆΠ½ΠΎΠΉ Ρ‚Ρ€ΡƒΠ±Π΅ для прСдотвращСния застрСвания.
Π—Π°ΠΏΠΎΠ»Π½Π΅Π½Π½Ρ‹Π΅ ΡƒΠΏΠ°ΠΊΠΎΠ²ΠΊΠΈ ΡΠΏΡƒΡΠΊΠ°ΡŽΡ‚ΡΡ ΠΏΠΎ ΠΆΠ΅Π»ΠΎΠ±Ρƒ, Ρ‡Ρ‚ΠΎΠ±Ρ‹ ΠΏΠΎΠΏΠ°ΡΡ‚ΡŒ Π½Π° ΠΏΠΎΡΠ»Π΅Π΄ΡƒΡŽΡ‰ΠΈΠ΅ ΡƒΠΏΠ°ΠΊΠΎΠ²ΠΎΡ‡Π½Ρ‹Π΅ ΠΎΠΏΠ΅Ρ€Π°Ρ†ΠΈΠΈ.
НизкоС внСшнСС Ρ‚Ρ€Π΅Π½ΠΈΠ΅, ΠΏΠ»Π΅Π½ΠΊΠ°-ΠΆΠ΅Π»ΠΎΠ±
Π£Π΄Π΅Ρ€ΠΆΠΈΠ²Π°Π΅Ρ‚ Π΄Π²ΠΈΠΆΠ΅Π½ΠΈΠ΅ ΠΏΡ€ΠΎΠ΄ΡƒΠΊΡ‚ΠΎΠ².
Π—Π°ΠΏΠΎΠ»Π½Π΅Π½Π½Ρ‹Π΅ ΡƒΠΏΠ°ΠΊΠΎΠ²ΠΊΠΈ Ρ‚Ρ€Π°Π½ΡΠΏΠΎΡ€Ρ‚ΠΈΡ€ΡƒΡŽΡ‚ΡΡ ΠΏΠΎ Π½Π°ΠΊΠ»ΠΎΠ½Π½ΠΎΠΉ ΠΊΠΎΠ½Π²Π΅ΠΉΠ΅Ρ€Π½ΠΎΠΉ Π»Π΅Π½Ρ‚Π΅.
Π£ΠΌΠ΅Ρ€Π΅Π½Π½ΠΎΠ΅ ΠΈΠ»ΠΈ высокоС внСшнСС Ρ‚Ρ€Π΅Π½ΠΈΠ΅, ΠΏΠ»Π΅Π½ΠΊΠ° Π½Π° ΠΊΠΎΠ½Π²Π΅ΠΉΠ΅Ρ€
НС Π΄Π°Π΅Ρ‚ ΠΏΡ€ΠΎΠ΄ΡƒΠΊΡ‚Ρƒ ΠΏΠΎΡ‚Π΅Ρ€ΡΡ‚ΡŒ мСсто размСщСния ΠΈΠ»ΠΈ ΡƒΠΏΠ°ΡΡ‚ΡŒ с ΠΊΠΎΠ½Π²Π΅ΠΉΠ΅Ρ€Π°.
Π’Π°Π±Π»ΠΈΡ†Π° 7: ΠŸΡ€ΠΈΠΌΠ΅Ρ€Ρ‹ Ρ€Π°Π·Π»ΠΈΡ‡Π½Ρ‹Ρ… ΠΏΡ€ΠΈΠ»ΠΎΠΆΠ΅Π½ΠΈΠΉ, Ρ‚Ρ€Π΅Π±ΡƒΡŽΡ‰ΠΈΡ… Ρ€Π°Π·Π»ΠΈΡ‡Π½Ρ‹Ρ… Π²Ρ‹ΠΌΡ‹ΡˆΠ»Π΅Π½Π½Ρ‹Ρ… свойств

Из Ρ‚Π°Π±Π»ΠΈΡ†Ρ‹ Π²Ρ‹ΡˆΠ΅ Π²ΠΈΠ΄Π½ΠΎ, Ρ‡Ρ‚ΠΎ каТдая ситуация ΠΈΠΌΠ΅Π΅Ρ‚ ΠΎΡ‡Π΅Π½ΡŒ Ρ€Π°Π·Π½Ρ‹Π΅ Π½Π°Π³Ρ€ΡƒΠ·ΠΊΠΈ ΠΈ оТидания; Π² зависимости ΠΎΡ‚ области примСнСния ΠΏΠ»Π΅Π½ΠΊΡƒ ΠΌΠΎΠΆΠ½ΠΎ Π·Π°ΡΡ‚Π°Π²ΠΈΡ‚ΡŒ ΠΈΠ·ΠΌΠ΅Π½ΠΈΡ‚ΡŒ Ρ„ΠΎΡ€ΠΌΡƒ, ΡΠΊΠΎΠ»ΡŒΠ·ΠΈΡ‚ΡŒ ΠΏΠΎ мСталличСской повСрхности ΠΈΠ»ΠΈ Β«ΡΡ…Π²Π°Ρ‚ΠΈΡ‚ΡŒΡΡΒ» Π·Π° ΠΏΡ€ΠΈΠ²ΠΎΠ΄Π½ΠΎΠΉ Ρ€Π΅ΠΌΠ΅Π½ΡŒ.Π˜Π·ΠΌΠ΅Ρ€Π΅Π½Π½Ρ‹ΠΉ Π² Π»Π°Π±ΠΎΡ€Π°Ρ‚ΠΎΡ€ΠΈΠΈ коэффициСнт трСния ΠΏΠ»Π΅Π½ΠΊΠΈ ΠΊ ΠΏΠ»Π΅Π½ΠΊΠ΅ Π½Π΅ ΠΌΠΎΠΆΠ΅Ρ‚ ΠΏΡ€Π΅Π΄ΡΠΊΠ°Π·Π°Ρ‚ΡŒ ΠΏΡ€ΠΎΠΈΠ·Π²ΠΎΠ΄ΠΈΡ‚Π΅Π»ΡŒΠ½ΠΎΡΡ‚ΡŒ Π²ΠΎ всСх этих прилоТСниях. ВмСсто этого измСрСния COF ΠΎΡ‚ ΠΏΠ»Π΅Π½ΠΊΠΈ ΠΊ ΠΏΠ»Π΅Π½ΠΊΠ΅ ΠΈΡΠΏΠΎΠ»ΡŒΠ·ΡƒΡŽΡ‚ΡΡ для управлСния тСхнологичСским процСссом, Ρ‡Ρ‚ΠΎΠ±Ρ‹ Π³Π°Ρ€Π°Π½Ρ‚ΠΈΡ€ΠΎΠ²Π°Ρ‚ΡŒ ΠΏΠΎΡΠ»Π΅Π΄ΠΎΠ²Π°Ρ‚Π΅Π»ΡŒΠ½ΠΎΠ΅ производство ΠΏΠ»Π΅Π½ΠΊΠΈ, которая Π² процСссС Ρ€Π°Π·Ρ€Π°Π±ΠΎΡ‚ΠΊΠΈ ΠΏΡ€ΠΎΠ΄ΡƒΠΊΡ‚Π° Π±Ρ‹Π»Π° Π΄ΠΎΠΊΠ°Π·Π°Π½Π° для Ρ€Π°Π±ΠΎΡ‚Ρ‹ Π² Π΅Π΅ Ρ†Π΅Π»Π΅Π²Ρ‹Ρ… прилоТСниях.

НапримСр, ΠΊΠ»ΡŽΡ‡ΠΎΠΌ ΠΊ Ρ…ΠΎΡ€ΠΎΡˆΠ΅ΠΉ ΠΏΡ€ΠΎΠΈΠ·Π²ΠΎΠ΄ΠΈΡ‚Π΅Π»ΡŒΠ½ΠΎΡΡ‚ΠΈ ΠΌΠ°ΡˆΠΈΠ½Ρ‹ VFFS ΡΠ²Π»ΡΡŽΡ‚ΡΡ внСшниС Ρ„Ρ€ΠΈΠΊΡ†ΠΈΠΎΠ½Π½Ρ‹Π΅ свойства ΠΏΠ»Π΅Π½ΠΊΠΈ, ΠΊΠΎΡ‚ΠΎΡ€Ρ‹Π΅ ΠΏΠΎΠ·Π²ΠΎΠ»ΡΡŽΡ‚ Π΅ΠΉ ΠΏΠ΅Ρ€Π΅ΠΌΠ΅Ρ‰Π°Ρ‚ΡŒΡΡ ΠΏΠΎ Ρ„ΠΎΡ€ΠΌΠΈΡ€ΡƒΡŽΡ‰Π΅ΠΉ ΠΌΡƒΡ„Ρ‚Π΅ Π±Π΅Π· Π²ΠΈΠ·Π³Π° ΠΈΠ»ΠΈ нСпостоянной ΠΏΠΎΠ΄Π°Ρ‡ΠΈ.ExxonMobil ΠΏΡ€ΠΎΠΈΠ·Π²ΠΎΠ΄ΠΈΡ‚ ΠΏΠ»Π΅Π½ΠΊΠΈ с высокотСхнологичными Π½Π΅ΠΌΠΈΠ³Ρ€Π°Ρ†ΠΈΠΎΠ½Π½Ρ‹ΠΌΠΈ систСмами скольТСния, ΠΊΠΎΡ‚ΠΎΡ€Ρ‹Π΅ ΠΈΠΌΠ΅ΡŽΡ‚ Π±ΠΎΠ»Π΅Π΅ высокий коэффициСнт трСния, Ρ‡Π΅ΠΌ старыС ΠΏΠ»Π΅Π½ΠΊΠΈ ΠΆΠΈΡ€Π½ΠΎΠ³ΠΎ Π°ΠΌΠΈΠ΄Π½ΠΎΠ³ΠΎ Ρ‚ΠΈΠΏΠ°, Π½ΠΎ ΠΏΡ€ΠΈ этом Π±Π΅Π·ΡƒΠΏΡ€Π΅Ρ‡Π½ΠΎ Ρ€Π°Π±ΠΎΡ‚Π°ΡŽΡ‚ Π½Π° этих ΠΌΠ°ΡˆΠΈΠ½Π°Ρ…. Π’Ρ€Π°Π΄ΠΈΡ†ΠΈΠΎΠ½Π½ΠΎΠ΅ ΠΌΡ‹ΡˆΠ»Π΅Π½ΠΈΠ΅ ΠΌΠΎΠΆΠ΅Ρ‚ ΡƒΡ‚Π²Π΅Ρ€ΠΆΠ΄Π°Ρ‚ΡŒ, Ρ‡Ρ‚ΠΎ Β«COF слишком высок» для этих Ρ„ΠΈΠ»ΡŒΠΌΠΎΠ², Π½ΠΎ это явно Π½Π΅ Ρ‚Π°ΠΊ; поэтому стандартный Π»Π°Π±ΠΎΡ€Π°Ρ‚ΠΎΡ€Π½Ρ‹ΠΉ тСст COF часто Π½Π΅ являСтся Ρ…ΠΎΡ€ΠΎΡˆΠΈΠΌ ΠΏΠΎΠΊΠ°Π·Π°Ρ‚Π΅Π»Π΅ΠΌ эффСктивности.

Π’ΠΠ˜ΠœΠΠΠ˜Π•: НС Π΄Π΅Π»Π°ΠΉΡ‚Π΅ Π²Ρ‹Π²ΠΎΠ΄ΠΎΠ² ΠΎ Ρ€Π°Π±ΠΎΡ‡ΠΈΡ… характСристиках Π½Π° основании Π½Π΅Π±ΠΎΠ»ΡŒΡˆΠΈΡ… Ρ€Π°Π·Π»ΠΈΡ‡ΠΈΠΉ (0,10 ΠΈΠ»ΠΈ мСньшС) Π² Π»Π°Π±ΠΎΡ€Π°Ρ‚ΠΎΡ€Π½Ρ‹Ρ… значСниях COF ΠΌΠ΅ΠΆΠ΄Ρƒ ΠΏΠ»Π΅Π½ΠΊΠ°ΠΌΠΈ Ρ€Π°Π·Π»ΠΈΡ‡Π½Ρ‹Ρ… ΠΏΠ»Π΅Π½ΠΎΡ‡Π½Ρ‹Ρ… ΠΏΡ€ΠΎΠ΄ΡƒΠΊΡ‚ΠΎΠ².ВмСсто этого слСдуСт ΡΡ€Π°Π²Π½ΠΈΡ‚ΡŒ Π΄Π²Π° Ρ€Π°Π·Π½Ρ‹Ρ… Ρ‚ΠΈΠΏΠ° ΠΏΠ»Π΅Π½ΠΎΠΊ, провСдя испытания Π² ΠΆΠ΅Π»Π°Π΅ΠΌΠΎΠΌ ΠΏΡ€ΠΈΠΌΠ΅Π½Π΅Π½ΠΈΠΈ.

Как измСняСтся COF Π² ΠΏΠ»Π΅Π½ΠΊΠ°Ρ…

НСмодифицированная ΠΏΠ»Π΅Π½ΠΊΠ° OPP ΠΌΠΎΠΆΠ΅Ρ‚ ΠΈΠΌΠ΅Ρ‚ΡŒ COF 0,70 ΠΈΠ»ΠΈ Π±ΠΎΠ»Π΅Π΅. Π‘ΠΎΠ»ΡŒΡˆΠΈΠ½ΡΡ‚Π²ΠΎ ΠΏΠΎΠ»ΠΈΠΌΠ΅Ρ€Π½Ρ‹Ρ… ΠΏΠ»Π΅Π½ΠΎΠΊ Π΄ΠΎΠ»ΠΆΠ½Ρ‹ ΠΈΠΌΠ΅Ρ‚ΡŒ ΡΠΏΠ΅Ρ†ΠΈΠ°Π»ΡŒΠ½Ρ‹ΠΉ состав для сниТСния COF ΠΈ ΠΈΠΌΠ΅Ρ‚ΡŒ свойства скольТСния, ΠΊΠΎΡ‚ΠΎΡ€Ρ‹Π΅ ΠΎΠ±Π΅ΡΠΏΠ΅Ρ‡ΠΈΠ²Π°ΡŽΡ‚ Ρ…ΠΎΡ€ΠΎΡˆΡƒΡŽ ΠΌΠ΅Ρ…Π°Π½ΠΈΡ‡Π΅ΡΠΊΡƒΡŽ ΠΎΠ±Ρ€Π°Π±ΠΎΡ‚ΠΊΡƒ, Ρ‚ΠΎ Π΅ΡΡ‚ΡŒ Ρ…ΠΎΡ€ΠΎΡˆΠΈΠ΅ характСристики ΡƒΠΏΠ°ΠΊΠΎΠ²ΠΎΡ‡Π½ΠΎΠ³ΠΎ оборудования.

Врадиционная тСхнология сниТСния COF Π·Π°ΠΊΠ»ΡŽΡ‡Π°Π΅Ρ‚ΡΡ Π² Π΄ΠΎΠ±Π°Π²Π»Π΅Π½ΠΈΠΈ ΠΆΠΈΡ€Π½ΠΎΠΉ Π°ΠΌΠΈΠ΄Π½ΠΎΠΉ Π΄ΠΎΠ±Π°Π²ΠΊΠΈ (ΠΎΠ±Ρ‹Ρ‡Π½ΠΎ Π½Π°Π·Ρ‹Π²Π°Π΅ΠΌΠΎΠΉ «скольТСниС») Π² смолу ΠΏΠ΅Ρ€Π΅Π΄ производством ΠΏΠ»Π΅Π½ΠΊΠΈ.Π‘ΠΎ Π²Ρ€Π΅ΠΌΠ΅Π½Π΅ΠΌ ΠΆΠΈΡ€Π½Ρ‹ΠΉ Π°ΠΌΠΈΠ΄ Π±ΡƒΠ΄Π΅Ρ‚ ΠΌΠΈΠ³Ρ€ΠΈΡ€ΠΎΠ²Π°Ρ‚ΡŒ Π½Π° ΠΏΠΎΠ²Π΅Ρ€Ρ…Π½ΠΎΡΡ‚ΡŒ ΠΏΠ»Π΅Π½ΠΊΠΈ (Β«ΠΏΠΎΡΡ‹ΠΏΠ°Ρ‚ΡŒΡΡΒ»), ΠΏΠΎΡ‚ΠΎΠΌΡƒ Ρ‡Ρ‚ΠΎ ΠΎΠ½ Π½Π΅ ΠΏΠΎΠ»Π½ΠΎΡΡ‚ΡŒΡŽ растворяСтся Π² ΠΏΠΎΠ»ΠΈΠΌΠ΅Ρ€Π΅. Π₯отя эта плСночная тСхнология Π΄ΠΎ сих ΠΏΠΎΡ€ ΡˆΠΈΡ€ΠΎΠΊΠΎ ΠΈΡΠΏΠΎΠ»ΡŒΠ·ΡƒΠ΅Ρ‚ΡΡ, ΠΎΠ½Π° ΠΈΠΌΠ΅Π΅Ρ‚ нСсколько ΠΏΡ€ΠΎΠ±Π»Π΅ΠΌ.

  1. ΠŸΡ€ΠΈ Π½Π°Π³Ρ€Π΅Π²Π°Π½ΠΈΠΈ ΠΆΠΈΡ€Π½Ρ‹Π΅ Π°ΠΌΠΈΠ΄Ρ‹ ΠΌΠΈΠ³Ρ€ΠΈΡ€ΡƒΡŽΡ‚ с повСрхности ΠΎΠ±Ρ€Π°Ρ‚Π½ΠΎ Π² ΠΎΡΠ½ΠΎΠ²Π½ΡƒΡŽ Ρ‡Π°ΡΡ‚ΡŒ ΠΏΠ»Π΅Π½ΠΊΠΈ, вызывая ΡƒΠ²Π΅Π»ΠΈΡ‡Π΅Π½ΠΈΠ΅ COF.
  2. Π—Π½Π°Ρ‡Π΅Π½ΠΈΠ΅ COF ΠΌΠΎΠΆΠ΅Ρ‚ сильно Π²Π°Ρ€ΡŒΠΈΡ€ΠΎΠ²Π°Ρ‚ΡŒΡΡ, ΠΊΠΎΡ‚ΠΎΡ€ΠΎΠ΅ Π±ΡƒΠ΄Π΅Ρ‚ ΠΌΠ΅Π½ΡΡ‚ΡŒΡΡ со Π²Ρ€Π΅ΠΌΠ΅Π½Π΅ΠΌ.
  3. Π­Ρ‚ΠΈ ΠΏΠ»Π΅Π½ΠΊΠΈ ΠΎΠ±Ρ‹Ρ‡Π½ΠΎ Π±ΠΎΠ»Π΅Π΅ ΠΌΡƒΡ‚Π½Ρ‹Π΅, Ρ‡Π΅ΠΌ ΠΏΠ»Π΅Π½ΠΊΠΈ, Π½Π΅ содСрТащиС ΠΆΠΈΡ€Π½Ρ‹Ρ… Π°ΠΌΠΈΠ΄ΠΎΠ².
  4. ΠŸΡ€ΠΈ наслоСнии Π½Π° ΠΏΠΎΠ»ΠΎΡ‚Π½ΠΎ Π³Π΅Ρ€ΠΌΠ΅Ρ‚ΠΈΠΊΠ° ΠΈ Π½Π°ΠΌΠΎΡ‚ΠΊΠ΅ Π² Ρ€ΡƒΠ»ΠΎΠ½ Тирная амидная Π΄ΠΎΠ±Π°Π²ΠΊΠ° ΠΌΠΎΠΆΠ΅Ρ‚ ΠΏΠ΅Ρ€Π΅Ρ…ΠΎΠ΄ΠΈΡ‚ΡŒ Π½Π° Π³Π΅Ρ€ΠΌΠ΅Ρ‚ΠΈΠ·ΠΈΡ€ΡƒΡŽΡ‰ΡƒΡŽ ΠΏΠΎΠ²Π΅Ρ€Ρ…Π½ΠΎΡΡ‚ΡŒ, Ρ‡Ρ‚ΠΎ Π² ΠΊΠΎΠ½Π΅Ρ‡Π½ΠΎΠΌ ΠΈΡ‚ΠΎΠ³Π΅ ΠΏΡ€ΠΈΠ²ΠΎΠ΄ΠΈΡ‚ ΠΊ ΡΡƒΠΆΠ΅Π½ΠΈΡŽ Π΄ΠΈΠ°ΠΏΠ°Π·ΠΎΠ½Π° Π³Π΅Ρ€ΠΌΠ΅Ρ‚ΠΈΠ·Π°Ρ†ΠΈΠΈ ΠΈ сниТСнию прочности Π³Π΅Ρ€ΠΌΠ΅Ρ‚ΠΈΠ·Π°Ρ†ΠΈΠΈ.
  5. Амид ΠΆΠΈΡ€Π½Ρ‹Ρ… кислот ΠΌΠΎΠΆΠ΅Ρ‚ ΠΎΡ‚ΠΊΠ»Π°Π΄Ρ‹Π²Π°Ρ‚ΡŒΡΡ ΠΈ Π½Π°ΠΊΠ°ΠΏΠ»ΠΈΠ²Π°Ρ‚ΡŒΡΡ Π½Π° Ρ€ΠΎΠ»ΠΈΠΊΠ°Ρ… ΠΈ повСрхностях ΡƒΠΏΠ°ΠΊΠΎΠ²ΠΎΡ‡Π½ΠΎΠΉ ΠΌΠ°ΡˆΠΈΠ½Ρ‹.
  6. Π”ΠΎΠ±Π°Π²ΠΊΠ° ΠΌΠΎΠΆΠ΅Ρ‚ ΡƒΡ…ΡƒΠ΄ΡˆΠΈΡ‚ΡŒ качСство ΠΏΠ΅Ρ‡Π°Ρ‚ΠΈ.

Π‘ΠΎΠ»ΡŒΡˆΠΈΠ½ΡΡ‚Π²ΠΎ ΠΏΠ»Π΅Π½ΠΎΠΊ ExxonMobil Π½Π΅ содСрТат систСм скольТСния ΠΌΠΈΠ³Ρ€ΠΈΡ€ΡƒΡŽΡ‰ΠΈΡ… Π°ΠΌΠΈΠ΄ΠΎΠ² ΠΆΠΈΡ€Π½Ρ‹Ρ… кислот. ВмСсто этого коэффициСнт трСния этих ΠΏΠ»Π΅Π½ΠΎΠΊ Π±Π΅Π· покрытия ΠΎΠΏΡ‚ΠΈΠΌΠΈΠ·ΠΈΡ€ΠΎΠ²Π°Π½ с ΠΏΠΎΠΌΠΎΡ‰ΡŒΡŽ Π·Π°ΠΏΠ°Ρ‚Π΅Π½Ρ‚ΠΎΠ²Π°Π½Π½Ρ‹Ρ… составов смол ΠΈ Π΄ΠΎΠ±Π°Π²ΠΎΠΊ, ΠΊΠΎΡ‚ΠΎΡ€Ρ‹Π΅ ΠΎΠ±Π΅ΡΠΏΠ΅Ρ‡ΠΈΠ²Π°ΡŽΡ‚ ΡΡ‚Π°Π±ΠΈΠ»ΡŒΠ½Ρ‹Π΅ характСристики Π½ΠΈΠ·ΠΊΠΎΠ³ΠΎ трСния.ΠžΠ±Ρ‹Ρ‡Π½ΠΎ Π½Π°Π·Ρ‹Π²Π°Π΅ΠΌΡ‹Π΅ Β«Π½Π΅ΠΌΠΈΠ³Ρ€ΠΈΡ€ΡƒΡŽΡ‰ΠΈΠΌΠΈΒ» систСмами скольТСния, эти ΠΌΠ΅Ρ‚ΠΎΠ΄Ρ‹ ΡƒΡΡ‚Ρ€Π°Π½ΡΡŽΡ‚ ΠΏΡ€ΠΎΠ±Π»Π΅ΠΌΡ‹, связанныС с использованиСм ΠΆΠΈΡ€Π½Ρ‹Ρ… Π°ΠΌΠΈΠ΄ΠΎΠ² для достиТСния Π±ΠΎΠ»Π΅Π΅ Π½ΠΈΠ·ΠΊΠΎΠ³ΠΎ COF.

ΠŸΠ Π˜ΠœΠ•Π§ΠΠΠ˜Π•: ΠŸΡ€Π΅Π΄Ρ‹Π΄ΡƒΡ‰Π΅Π΅ обсуТдСниС описываСт, ΠΊΠ°ΠΊ COF модифицируСтся Π² соэкструдированных ΠΈ ΡΠΊΠΎΠ»ΡŒΠ·ΡΡ‰ΠΈΡ… ΠΏΠ»Π΅Π½ΠΊΠ°Ρ… Π±Π΅Π· покрытия. ΠŸΠΎΠΌΠ½ΠΈΡ‚Π΅, Ρ‡Ρ‚ΠΎ повСрхности ExxonMobil с Π°ΠΊΡ€ΠΈΠ»ΠΎΠ²Ρ‹ΠΌ ΠΏΠΎΠΊΡ€Ρ‹Ρ‚ΠΈΠ΅ΠΌ ΠΈΠΌΠ΅ΡŽΡ‚ Π½ΠΈΠ·ΠΊΠΈΠΉ ΠΈ ΡΡ‚Π°Π±ΠΈΠ»ΡŒΠ½Ρ‹ΠΉ COF ΠΈ ΠΎΠ±Π΅ΡΠΏΠ΅Ρ‡ΠΈΠ²Π°ΡŽΡ‚ ΠΎΡ‚Π»ΠΈΡ‡Π½ΡƒΡŽ ΠΏΡ€ΠΎΠΈΠ·Π²ΠΎΠ΄ΠΈΡ‚Π΅Π»ΡŒΠ½ΠΎΡΡ‚ΡŒ ΠΌΠ°ΡˆΠΈΠ½Ρ‹.

ΠŸΡ€ΠΈΠ½Ρ†ΠΈΠΏΡ‹ испытаний

COF — это простоС ΡΠΎΠΎΡ‚Π½ΠΎΡˆΠ΅Π½ΠΈΠ΅, Ρ€Π°Π²Π½ΠΎΠ΅ силС, Π½Π΅ΠΎΠ±Ρ…ΠΎΠ΄ΠΈΠΌΠΎΠΉ для скольТСния ΠΎΠ΄Π½ΠΎΠΉ повСрхности ΠΏΠΎ Π΄Ρ€ΡƒΠ³ΠΎΠΉ, Π΄Π΅Π»Π΅Π½Π½ΠΎΠΉ Π½Π° силу, ΠΏΠ΅Ρ€ΠΏΠ΅Π½Π΄ΠΈΠΊΡƒΠ»ΡΡ€Π½ΡƒΡŽ ΠΊΠΎΠ½Ρ‚Π°ΠΊΡ‚ΠΈΡ€ΡƒΡŽΡ‰ΠΈΠΌ повСрхностям.

COF =
Π‘ΠΈΠ»Π°, Π²Ρ‹Π·Ρ‹Π²Π°ΡŽΡ‰Π°Ρ скольТСниС повСрхностСй ΠΏΠ»Π΅Π½ΠΊΠΈ (gf)
; Π±Π΅Π·Ρ€Π°Π·ΠΌΠ΅Ρ€Π½Ρ‹ΠΉ коэффициСнт
ВСс салазок (гс)

ExxonMobil ΠΈΡΠΏΠΎΠ»ΡŒΠ·ΡƒΠ΅Ρ‚ ΠΈΠΌΠ΅ΡŽΡ‰Π΅Π΅ΡΡ Π² ΠΏΡ€ΠΎΠ΄Π°ΠΆΠ΅ ΠΈΡΠΏΡ‹Ρ‚Π°Ρ‚Π΅Π»ΡŒΠ½ΠΎΠ΅ ΠΎΠ±ΠΎΡ€ΡƒΠ΄ΠΎΠ²Π°Π½ΠΈΠ΅ для измСрСния коэффициСнта трСния ΠΎΡ‚ ΠΏΠ»Π΅Π½ΠΊΠΈ ΠΊ ΠΏΠ»Π΅Π½ΠΊΠ΅ Π² условиях, ΠΎΠΏΡ€Π΅Π΄Π΅Π»Π΅Π½Π½Ρ‹Ρ… стандартом ASTM D 1894. Π’ Ρ‚Π°Π±Π»ΠΈΡ†Π΅ 8 приводится сводноС сравнСниС.

(Π΄ΡŽΠΉΠΌΡ‹)
ΠŸΡ€ΠΎΡ†Π΅Π΄ΡƒΡ€Π° испытания

Условия испытаний
Π˜Π·ΠΌΠ΅Ρ€Π΅Π½ΠΈΠ΅
Аппаратура
ΠšΠΎΠ½Ρ‚Π°ΠΊΡ‚Π½Ρ‹Π΅ повСрхности

Π‘Π°Π»Π°Π·ΠΊΠΈ
ВСс (Π³)
ΠšΠΎΠ½Ρ‚Π°ΠΊΡ‚Π½Ρ‹Π΅ Ρ€Π°Π·ΠΌΠ΅Ρ€Ρ‹ салазок
ΠœΠ΅Ρ‚ΠΎΠ΄ вытягивания
Π‘ΠΊΠΎΡ€ΠΎΡΡ‚ΡŒ вытягивания (дюйм / ΠΌΠΈΠ½)
ExxonMobil # 430
TMI # 32-06
ПлСнка Π½Π° ΠΏΠ»Π΅Π½ΠΊΡƒ
200
2.5 x 2,5
ΠŸΠΎΠ΄Π²ΠΈΠΆΠ½Ρ‹Π΅ салазки
6
ASTM D 1894
НС ΡƒΠΊΠ°Π·Π°Π½ΠΎ
НС ΡƒΠΊΠ°Π·Π°Π½ΠΎ
200
2,5 x 2,5
ΠŸΠΎΠ΄Π²ΠΈΠΆΠ½Ρ‹Π΅ салазки Π½Π° плоскости
6
Π’Π°Π±Π»ΠΈΡ†Π° 8: Π‘Ρ€Π°Π²Π½Π΅Π½ΠΈΠ΅ условий испытания COF ΠΌΠ΅ΠΆΠ΄Ρƒ ExxonMobil ΠΈ ΠΏΡ€ΠΎΡ†Π΅Π΄ΡƒΡ€Π°ΠΌΠΈ ASTM

ВСстСр TMI обСспСчиваСт Ρ†ΠΈΡ„Ρ€ΠΎΠ²ΠΎΠ΅ ΠΎΡ‚ΠΎΠ±Ρ€Π°ΠΆΠ΅Π½ΠΈΠ΅ статичСского ΠΈ кинСтичСского COF.БтатичСский COF — это Π±ΠΎΠ»Π΅Π΅ высокоС Π·Π½Π°Ρ‡Π΅Π½ΠΈΠ΅ ΠΈ прСдставляСт собой силу для Π½Π°Ρ‡Π°Π»Π° двиТСния, Π² Ρ‚ΠΎ врСмя ΠΊΠ°ΠΊ кинСтичСский COF — это число, ΠΎΠ±Ρ‹Ρ‡Π½ΠΎ ΠΎΡ‚ΠΎΠ±Ρ€Π°ΠΆΠ°Π΅ΠΌΠΎΠ΅ Π² тСхничСских Π΄Π°Π½Π½Ρ‹Ρ…, ΠΈ прСдставляСт собой срСднСС Π·Π½Π°Ρ‡Π΅Π½ΠΈΠ΅ послС Y 2 дюймов Ρ…ΠΎΠ΄Π°.

ЗначСния COF, ΠΏΠΎΠ»ΡƒΡ‡Π΅Π½Π½Ρ‹Π΅ Π² Ρ€Π°Π·Π½Ρ‹Ρ… лабораториях ΠΈΠ»ΠΈ Π½Π° Ρ€Π°Π·Π½Ρ‹Ρ… тСстСрах, ΠΌΠΎΠ³ΡƒΡ‚ сильно ΠΎΡ‚Π»ΠΈΡ‡Π°Ρ‚ΡŒΡΡ ΠΏΡ€ΠΈ ΠΈΠ·ΠΌΠ΅Ρ€Π΅Π½ΠΈΠΈ ΠΎΠ΄Π½ΠΎΠΉ ΠΈ Ρ‚ΠΎΠΉ ΠΆΠ΅ ΠΏΠ»Π΅Π½ΠΊΠΈ. НапримСр, ASTM ΠΏΡ€ΠΎΠ²Π΅Ρ€ΠΈΠ» Ρ‚ΠΎΡ‡Π½ΠΎΡΡ‚ΡŒ Π΄Π°Π½Π½Ρ‹Ρ… COF ΠΌΠ΅ΠΆΠ΄Ρƒ лабораториями ΠΈ ΠΎΠ±Π½Π°Ρ€ΡƒΠΆΠΈΠ» стандартныС отклонСния (Π² зависимости ΠΎΡ‚ Ρ‚ΠΈΠΏΠ° ΠΏΠ»Π΅Π½ΠΊΠΈ) ΠΌΠ΅ΠΆΠ΄Ρƒ 0,02 ΠΈ.12. Π‘Π»Π΅Π΄ΠΎΠ²Π°Ρ‚Π΅Π»ΡŒΠ½ΠΎ, Π½Π΅Ρ‚ смысла ΠΏΡ€Π΅Π΄ΠΏΠΎΠ»Π°Π³Π°Ρ‚ΡŒ, Ρ‡Ρ‚ΠΎ ΠΎΠ±ΡΠ·Π°Ρ‚Π΅Π»ΡŒΠ½ΠΎ сущСствуСт Ρ€Π΅Π°Π»ΡŒΠ½Π°Ρ Ρ€Π°Π·Π½ΠΈΡ†Π° ΠΌΠ΅ΠΆΠ΄Ρƒ, Π½Π°ΠΏΡ€ΠΈΠΌΠ΅Ρ€, 0,28, ΠΈΠ·ΠΌΠ΅Ρ€Π΅Π½Π½ΠΎΠΉ ΠΎΠ΄Π½ΠΎΠΉ ΠΊΠΎΠΌΠΏΠ°Π½ΠΈΠ΅ΠΉ, ΠΈ 0,34, ΠΈΠ·ΠΌΠ΅Ρ€Π΅Π½Π½ΠΎΠΉ Π΄Ρ€ΡƒΠ³ΠΎΠΉ.

Бвязанная тСрминология

Амид ΠΆΠΈΡ€Π½Ρ‹Ρ… кислот: Амид ΠΆΠΈΡ€Π½Ρ‹Ρ… кислот являСтся ΠΎΡ‡Π΅Π½ΡŒ распространСнной Π΄ΠΎΠ±Π°Π²ΠΊΠΎΠΉ, ΡΠ½ΠΈΠΆΠ°ΡŽΡ‰Π΅ΠΉ ΡΠΊΠΎΡ€ΠΎΡΡ‚ΡŒ ΠΌΠΈΠ³Ρ€Π°Ρ†ΠΈΠΈ пластмасс. Амиды ΠΆΠΈΡ€Π½Ρ‹Ρ… кислот ΠΏΡ€Π΅Π΄ΡΡ‚Π°Π²Π»ΡΡŽΡ‚ собой ΠΆΠΈΡ€Π½Ρ‹Π΅ органичСскиС соСдинСния, Ρ‚Π°ΠΊΠΈΠ΅ ΠΊΠ°ΠΊ стСарамид, эрукамид, Π±Π΅Π³Π΅Π½Π°ΠΌΠΈΠ΄ ΠΈ ΠΎΠ»Π΅Π°ΠΌΠΈΠ΄, ΠΊΠΎΡ‚ΠΎΡ€Ρ‹Π΅ ΠΏΡ€Π΅Π²Ρ€Π°Ρ‰Π°ΡŽΡ‚ΡΡ Π² смолу ΠΈ Π²Ρ‹Π΄Π΅Π»ΡΡŽΡ‚ΡΡ Π½Π° повСрхности послС образования ΠΏΠ»Π΅Π½ΠΊΠΈ.

ΠšΠΈΠ½Π΅Ρ‚ΠΈΡ‡Π΅ΡΠΊΠΈΠΉ COF: ΠšΠΈΠ½Π΅Ρ‚ΠΈΡ‡Π΅ΡΠΊΠΈΠΉ COF — это Π·Π½Π°Ρ‡Π΅Π½ΠΈΠ΅ коэффициСнта трСния, ΠΈΠ·ΠΌΠ΅Ρ€Π΅Π½Π½ΠΎΠ΅ послС Ρ‚ΠΎΠ³ΠΎ, ΠΊΠ°ΠΊ повСрхности находятся Π² Π΄Π²ΠΈΠΆΠ΅Π½ΠΈΠΈ с постоянной ΡΠΊΠΎΡ€ΠΎΡΡ‚ΡŒΡŽ.

ΠžΠ±Ρ€Π°Π±Π°Ρ‚Ρ‹Π²Π°Π΅ΠΌΠΎΡΡ‚ΡŒ: ΠžΠ±Ρ€Π°Π±Π°Ρ‚Ρ‹Π²Π°Π΅ΠΌΠΎΡΡ‚ΡŒ, Ρ‚Π°ΠΊΠΆΠ΅ извСстная ΠΊΠ°ΠΊ ΠΏΡ€ΠΎΠΈΠ·Π²ΠΎΠ΄ΠΈΡ‚Π΅Π»ΡŒΠ½ΠΎΡΡ‚ΡŒ ΠΌΠ°ΡˆΠΈΠ½Ρ‹, — это ΡΠΏΠΎΡΠΎΠ±Π½ΠΎΡΡ‚ΡŒ ΠΏΠ»Π΅Π½ΠΊΠΈ Ρ…ΠΎΡ€ΠΎΡˆΠΎ ΠΏΠ΅Ρ€Π΅ΠΌΠ΅Ρ‰Π°Ρ‚ΡŒΡΡ ΠΈ ΠΎΡ‚ΡΠ»Π΅ΠΆΠΈΠ²Π°Ρ‚ΡŒ Ρ‡Π΅Ρ€Π΅Π· ΡƒΠΏΠ°ΠΊΠΎΠ²ΠΎΡ‡Π½ΡƒΡŽ ΠΌΠ°ΡˆΠΈΠ½Ρƒ. Π Π°Π·Π½Ρ‹Π΅ ΠΌΠ°ΡˆΠΈΠ½Ρ‹ ΠΎΠ±Π»Π°Π΄Π°ΡŽΡ‚ Ρ€Π°Π·Π½Ρ‹ΠΌΠΈ свойствами ΠΏΠ»Π΅Π½ΠΊΠΈ, поэтому ΠΏΠ»Π΅Π½ΠΊΠ° ΠΌΠΎΠΆΠ΅Ρ‚ Π΄Π΅ΠΌΠΎΠ½ΡΡ‚Ρ€ΠΈΡ€ΠΎΠ²Π°Ρ‚ΡŒ Ρ…ΠΎΡ€ΠΎΡˆΡƒΡŽ ΠΎΠ±Ρ€Π°Π±Π°Ρ‚Ρ‹Π²Π°Π΅ΠΌΠΎΡΡ‚ΡŒ Π½Π° ΠΎΠ΄Π½ΠΎΠΌ ΠΎΠ±ΠΎΡ€ΡƒΠ΄ΠΎΠ²Π°Π½ΠΈΠΈ, Π½ΠΎ ΠΏΠ»ΠΎΡ…ΠΎ Ρ€Π°Π±ΠΎΡ‚Π°Ρ‚ΡŒ Π½Π° Π΄Ρ€ΡƒΠ³ΠΎΠΌ.

БистСма Π½Π΅ΠΌΠΈΠ³Ρ€ΠΈΡ€ΡƒΡŽΡ‰Π΅Π³ΠΎ скольТСния: Π­Ρ‚ΠΎΡ‚ Ρ‚Π΅Ρ€ΠΌΠΈΠ½ относится ΠΊ Π½ΠΎΠ²ΠΎΠΉ Ρ‚Π΅Ρ…Π½ΠΎΠ»ΠΎΠ³ΠΈΠΈ, которая сниТаСт коэффициСнт трСния ΠΏΠ»Π΅Π½ΠΊΠΈ Π±Π΅Π· ΠΏΡ€ΠΎΠ±Π»Π΅ΠΌ, связанных с ΠΆΠΈΡ€Π½Ρ‹ΠΌΠΈ Π°ΠΌΠΈΠ΄Π°ΠΌΠΈ.ExxonMobil являСтся Π»ΠΈΠ΄Π΅Ρ€ΠΎΠΌ Π² этой области, разрабатывая ΠΏΠ»Π΅Π½ΠΊΠΈ с Π½Π΅ΠΌΠΈΠ³Ρ€ΠΈΡ€ΡƒΡŽΡ‰ΠΈΠΌΠΈ Π΄ΠΎΠ±Π°Π²ΠΊΠ°ΠΌΠΈ ΠΈ ΠΌΠΎΠ΄ΠΈΡ„ΠΈΡ†ΠΈΡ€ΠΎΠ²Π°Π½Π½Ρ‹ΠΌΠΈ повСрхностями; Π­Ρ‚ΠΈ ΠΏΠ»Π΅Π½ΠΊΠΈ ΠΈΠΌΠ΅ΡŽΡ‚ ΡΡ‚Π°Π±ΠΈΠ»ΡŒΠ½Ρ‹ΠΉ коэффициСнт трСния ΠΈ ΠΎΡ‚Π»ΠΈΡ‡Π½Ρ‹Π΅ Ρ€Π°Π±ΠΎΡ‡ΠΈΠ΅ характСристики ΠΌΠ°ΡˆΠΈΠ½Ρ‹ для ΠΎΠ±Ρ€Π°Π±ΠΎΡ‚ΠΊΠΈ ΠΈ ΡƒΠΏΠ°ΠΊΠΎΠ²ΠΊΠΈ.

БкольТСниС: БкольТСниС ΠΏΡ€ΠΎΡ‚ΠΈΠ²ΠΎΠΏΠΎΠ»ΠΎΠΆΠ½ΠΎ Ρ‚Ρ€Π΅Π½ΠΈΡŽ. «ВысокоС скольТСниС» ΠΎΠ·Π½Π°Ρ‡Π°Π΅Ρ‚ Π½ΠΈΠ·ΠΊΠΈΠΉ коэффициСнт трСния, Π² Ρ‚ΠΎ врСмя ΠΊΠ°ΠΊ Π½ΠΈΠ·ΠΊΠΎΠ΅ скольТСниС ΠΎΠ·Π½Π°Ρ‡Π°Π΅Ρ‚ высокий коэффициСнт трСния. «ПлСнка скольТСния» прСдставляСт собой Π½Π΅Π·Π°ΠΏΠ΅Ρ‡Π°Ρ‚Ρ‹Π²Π°ΡŽΡ‰ΡƒΡŽΡΡ ΠΏΠ»Π΅Π½ΠΊΡƒ, Π² состав ΠΊΠΎΡ‚ΠΎΡ€ΠΎΠΉ ΡΠΏΠ΅Ρ†ΠΈΠ°Π»ΡŒΠ½ΠΎ входят Π΄ΠΎΠ±Π°Π²ΠΊΠΈ ΠΈΠ»ΠΈ модификация повСрхности для ΡƒΠΌΠ΅Π½ΡŒΡˆΠ΅Π½ΠΈΡ повСрхностного трСния. Β«Π”ΠΎΠ±Π°Π²ΠΊΠ° скольТСния» ΠΎΠ±Ρ‹Ρ‡Π½ΠΎ относится ΠΊ ΠΆΠΈΡ€Π½ΠΎΠΌΡƒ Π°ΠΌΠΈΠ΄Ρƒ.

БтатичСский COF: БтатичСский COF — это Π·Π½Π°Ρ‡Π΅Π½ΠΈΠ΅ коэффициСнта трСния, ΠΈΠ·ΠΌΠ΅Ρ€Π΅Π½Π½ΠΎΠ΅, ΠΊΠΎΠ³Π΄Π° Π΄Π²Π΅ повСрхности Ρ‚ΠΎΠ»ΡŒΠΊΠΎ Π½Π°Ρ‡ΠΈΠ½Π°ΡŽΡ‚ Π΄Π²ΠΈΠ³Π°Ρ‚ΡŒΡΡ Π΄Ρ€ΡƒΠ³ ΠΎΡ‚Π½ΠΎΡΠΈΡ‚Π΅Π»ΡŒΠ½ΠΎ Π΄Ρ€ΡƒΠ³Π°.

Π§Ρ‚ΠΎ Ρ‚Π°ΠΊΠΎΠ΅ COF (коэффициСнт трСния) ΠΈ ΠΏΠΎΡ‡Π΅ΠΌΡƒ ΠΎΠ½ Π²Π°ΠΆΠ΅Π½?

COF (коэффициСнт трСния) — это Π·Π½Π°Ρ‡Π΅Π½ΠΈΠ΅, ΠΊΠΎΡ‚ΠΎΡ€ΠΎΠ΅ прСдставляСт ΡΠΎΠΎΡ‚Π½ΠΎΡˆΠ΅Π½ΠΈΠ΅ ΠΌΠ΅ΠΆΠ΄Ρƒ силой трСния ΠΈ Π½ΠΎΡ€ΠΌΠ°Π»ΡŒΠ½ΠΎΠΉ силой ΠΌΠ΅ΠΆΠ΄Ρƒ двумя ΠΎΠ±ΡŠΠ΅ΠΊΡ‚Π°ΠΌΠΈ. Π₯отя это ΠΌΠΎΠΆΠ΅Ρ‚ ΠΏΠΎΠΊΠ°Π·Π°Ρ‚ΡŒΡΡ Π·Π°ΠΏΡƒΡ‚Π°Π½Π½Ρ‹ΠΌ, это ваТная Ρ†Π΅Π½Π½ΠΎΡΡ‚ΡŒ для ΠΊΠΎΠΌΠΏΠ°Π½ΠΈΠΉ, производящих мСшки для Π³Π°Π·ΠΎΠ½ΠΎΠ² ΠΈ сада, Π° Ρ‚Π°ΠΊΠΆΠ΅ полиэтилСновыС ΠΏΠ°ΠΊΠ΅Ρ‚Ρ‹ для ΠΌΡƒΠ»ΡŒΡ‡ΠΈ.COF Π²Π°ΠΆΠ΅Π½ ΠΊΠ°ΠΊ для ΠΏΠΎΡ‡Ρ‚ΠΎΠ²Ρ‹Ρ… ΠΎΡ‚ΠΏΡ€Π°Π²ΠΈΡ‚Π΅Π»Π΅ΠΉ, Ρ‚Π°ΠΊ ΠΈ для всСх Π³Π°Π·ΠΎΠ½Π½Ρ‹Ρ… ΠΈ садовых сумок, ΠΊΠΎΡ‚ΠΎΡ€Ρ‹Π΅ ΡƒΠΊΠ»Π°Π΄Ρ‹Π²Π°ΡŽΡ‚ΡΡ Π½Π° ΠΏΠΎΠ΄Π΄ΠΎΠ½Ρ‹ для транспортировки ΠΈ дСмонстрации.

Π’ повсСднСвной ΠΆΠΈΠ·Π½ΠΈ Π²ΠΎΠ·Π½ΠΈΠΊΠ°Π΅Ρ‚ Ρ‚Ρ€Π΅Π½ΠΈΠ΅. Π­Ρ‚Π° нСвидимая сила Π»ΠΈΠ±ΠΎ Ρ€Π°Π·Ρ€Π΅ΡˆΠ°Π΅Ρ‚, Π»ΠΈΠ±ΠΎ прСпятствуСт двиТСнию Π΄Π²ΡƒΡ… ΡΠΎΠΏΡ€ΠΈΠΊΠ°ΡΠ°ΡŽΡ‰ΠΈΡ…ΡΡ повСрхностСй. ΠšΠΎΠ½Ρ‚Ρ€ΠΎΠ»ΠΈΡ€ΡƒΡ COF, ΠΊΠΎΠΌΠΏΠ°Π½ΠΈΠΈ, ΠΊΠΎΡ‚ΠΎΡ€Ρ‹Π΅ производят ΠΈ ΠΏΡ€ΠΎΠ΄Π°ΡŽΡ‚ пластиковыС ΠΏΠ°ΠΊΠ΅Ρ‚Ρ‹ для ΠΌΡƒΠ»ΡŒΡ‡ΠΈ, ΠΏΠΎΡ‡Ρ‚ΠΎΠ²Ρ‹Ρ… ΠΎΡ‚ΠΏΡ€Π°Π²Π»Π΅Π½ΠΈΠΉ, Π³Π°Π·ΠΎΠ½ΠΎΠ² ΠΈ садов, ΠΌΠΎΠ³ΡƒΡ‚ ΠΎΠΏΡ‚ΠΈΠΌΠΈΠ·ΠΈΡ€ΠΎΠ²Π°Ρ‚ΡŒ ΠΏΡ€ΠΎΠΈΠ·Π²ΠΎΠ΄ΠΈΡ‚Π΅Π»ΡŒΠ½ΠΎΡΡ‚ΡŒ ΠΈ ΠΏΡ€Π΅Π΄ΠΎΡ‚Π²Ρ€Π°Ρ‚ΠΈΡ‚ΡŒ ΠΏΡ€ΠΎΠ±Π»Π΅ΠΌΡ‹, Ρ…Π°Ρ€Π°ΠΊΡ‚Π΅Ρ€Π½Ρ‹Π΅ для хранСния ΠΈ транспортировки.

Когда Π²ΠΎΠ·Π½ΠΈΠΊΠ°Π΅Ρ‚ Ρ‡Ρ€Π΅Π·ΠΌΠ΅Ρ€Π½ΠΎΠ΅ Ρ‚Ρ€Π΅Π½ΠΈΠ΅ Π½Π° Π³Π΅Ρ€ΠΌΠ΅Ρ‚ΠΈΠ·ΠΈΡ€ΡƒΡŽΡ‰Π΅ΠΉ сторонС ΠΏΠ»Π΅Π½ΠΊΠΈ ΠΏΡ€ΠΈ использовании Π³ΠΎΡ€ΠΈΠ·ΠΎΠ½Ρ‚Π°Π»ΡŒΠ½Ρ‹Ρ… систСм заполнСния ΠΈ уплотнСния, часто Π²ΠΎΠ·Π½ΠΈΠΊΠ°ΡŽΡ‚ ΠΏΡ€ΠΎΠ±Π»Π΅ΠΌΡ‹ с Π·Π°Π΅Π΄Π°Π½ΠΈΠ΅ΠΌ ΠΈΠ»ΠΈ Π²ΠΎΠ»ΠΎΡ‡Π΅Π½ΠΈΠ΅ΠΌ ΠΏΡ€ΠΈ ΠΏΡ€ΠΎΡ…ΠΎΠΆΠ΄Π΅Π½ΠΈΠΈ Ρ‡Π΅Ρ€Π΅Π· мСталличСскиС пластины.Π­Ρ‚ΠΎ ΠΏΠΎΠ²Ρ€Π΅ΠΆΠ΄Π°Π΅Ρ‚ Ρ‚ΠΎΠ²Π°Ρ€Ρ‹ ΠΈ замСдляСт производство, Ρ‡Ρ‚ΠΎ сказываСтся Π½Π° прибылях ΠΊΠΎΠΌΠΏΠ°Π½ΠΈΠΈ.

Для Π²Π΅Ρ€Ρ‚ΠΈΠΊΠ°Π»ΡŒΠ½Ρ‹Ρ… систСм заполнСния ΠΈ запСчатывания этот ΠΆΠ΅ сцСнарий ΠΏΡ€ΠΈΠ²ΠΎΠ΄ΠΈΡ‚ ΠΊ ΠΏΠ»ΠΎΡ…ΠΎΠΉ ΠΏΠΎΠ΄Π°Ρ‡Π΅ ΠΏΠ»Π΅Π½ΠΊΠΈ, ΠΏΠΎΡΠΊΠΎΠ»ΡŒΠΊΡƒ ΠΏΠ°ΠΊΠ΅Ρ‚Ρ‹ проходят Ρ‡Π΅Ρ€Π΅Π· мСталличСскиС Ρ„ΠΎΡ€ΠΌΠΎΠ²ΠΎΡ‡Π½Ρ‹Π΅ ΠΊΠΎΠ»ΡŒΡ†Π°. Из-Π·Π° этого Π²ΠΎΠ·Π½ΠΈΠΊΠ°Π΅Ρ‚ нСприятный Π²ΠΈΠ·Π³ ΠΈ ΠΏΠΎΠ»ΡƒΡ‡Π°ΡŽΡ‚ΡΡ ΡƒΠΏΠ°ΠΊΠΎΠ²ΠΊΠΈ нСстандартного Ρ€Π°Π·ΠΌΠ΅Ρ€Π°. НСзависимо ΠΎΡ‚ ΠΈΡΠΏΠΎΠ»ΡŒΠ·ΡƒΠ΅ΠΌΠΎΠΉ систСмы слишком большоС Ρ‚Ρ€Π΅Π½ΠΈΠ΅ ΠΏΡ€ΠΈΠ²ΠΎΠ΄ΠΈΡ‚ ΠΊ ΠΏΠ»ΠΎΡ…ΠΎΠΌΡƒ ΡƒΠΏΠ»ΠΎΡ‚Π½Π΅Π½ΠΈΡŽ. Для сравнСния, нСдостаточноС Ρ‚Ρ€Π΅Π½ΠΈΠ΅ заставляСт ΡƒΠΏΠ°ΠΊΠΎΠ²ΠΊΠΈ ΠΏΠ°Π΄Π°Ρ‚ΡŒ ΠΈΠ»ΠΈ ΡΠΎΡΠΊΠ°Π»ΡŒΠ·Ρ‹Π²Π°Ρ‚ΡŒ с ΠΊΠΎΠ½Π²Π΅ΠΉΠ΅Ρ€Π½ΠΎΠΉ Π»Π΅Π½Ρ‚Ρ‹, стопок ΠΈ ΠΏΠΎΠ΄Π΄ΠΎΠ½ΠΎΠ².

Π‘ΡƒΡ‚ΡŒ Π² Ρ‚ΠΎΠΌ, Ρ‡Ρ‚ΠΎ Ρ‡Ρ€Π΅Π·ΠΌΠ΅Ρ€Π½ΠΎΠ΅ Ρ‚Ρ€Π΅Π½ΠΈΠ΅ снаруТи замСдляСт ΠΏΡ€ΠΎΠ΄Π²ΠΈΠΆΠ΅Π½ΠΈΠ΅ ΡƒΠΏΠ°ΠΊΠΎΠ²ΠΎΠΊ ΠΏΠΎ Ρ€Π°Π·Π³Ρ€ΡƒΠ·ΠΎΡ‡Π½Ρ‹ΠΌ ΠΆΠ΅Π»ΠΎΠ±Π°ΠΌ, Π² Ρ‚ΠΎ врСмя ΠΊΠ°ΠΊ нСдостаточноС Ρ‚Ρ€Π΅Π½ΠΈΠ΅ Π²Ρ‹Π·Ρ‹Π²Π°Π΅Ρ‚ ΠΏΠ°Π΄Π΅Π½ΠΈΠ΅ ΠΈΠ»ΠΈ скольТСниС ΡƒΠΏΠ°ΠΊΠΎΠ²ΠΎΠΊ.Оба сцСнария ΠΏΠ»ΠΎΡ…ΠΈ ΠΏΡ€ΠΈ ΠΈΠ·Π³ΠΎΡ‚ΠΎΠ²Π»Π΅Π½ΠΈΠΈ полиэтилСновых ΠΏΠ°ΠΊΠ΅Ρ‚ΠΎΠ² для ΠΌΡƒΠ»ΡŒΡ‡ΠΈ.

Polypak Packaging Π·Π½Π°Π΅Ρ‚ Ρ‚ΠΎΡ‡Π½ΠΎΠ΅ количСство силы, Π½Π΅ΠΎΠ±Ρ…ΠΎΠ΄ΠΈΠΌΠΎΠ΅ для пСрСмСщСния пластиковых ΠΏΠ°ΠΊΠ΅Ρ‚ΠΎΠ² для ΠΌΡƒΠ»ΡŒΡ‡ΠΈ ΠΏΠΎ ΠΎΠ΄Π½ΠΎΠΉ повСрхности Π½Π° Π΄Ρ€ΡƒΠ³ΡƒΡŽ. Π’ свСрхпрочных ΠΌΠ΅ΡˆΠΊΠ°Ρ… ΠΊΠΎΠΌΠΏΠ°Π½ΠΈΠΈ Π΅ΡΡ‚ΡŒ новая ΠΏΠ»Π΅Π½ΠΊΠ° V-Zuh β„’ для повСрхности скольТСния, ΠΎΠ±Π΅ΡΠΏΠ΅Ρ‡ΠΈΠ²Π°ΡŽΡ‰Π°Ρ ΠΌΠ°ΠΊΡΠΈΠΌΠ°Π»ΡŒΠ½ΡƒΡŽ ΡƒΠ΄Π°Ρ€Π½ΡƒΡŽ Π²ΡΠ·ΠΊΠΎΡΡ‚ΡŒ ΠΈ Π³Π΅Ρ€ΠΌΠ΅Ρ‚ΠΈΡ‡Π½ΠΎΡΡ‚ΡŒ Π΄Π°ΠΆΠ΅ Π² нСблагоприятных условиях. ΠŸΠ°ΠΊΠ΅Ρ‚Ρ‹ Ρ‚Π°ΠΊΠΆΠ΅ сдСланы ΠΈΠ· устойчивых ΠΊ Π²Ρ‹Ρ†Π²Π΅Ρ‚Π°Π½ΠΈΡŽ Ρ‡Π΅Ρ€Π½ΠΈΠ» ΠΈ ΡΠΏΠ΅Ρ†ΠΈΠ°Π»ΡŒΠ½Ρ‹Ρ… ΠΈΠ½Π³Ρ€Π΅Π΄ΠΈΠ΅Π½Ρ‚ΠΎΠ², ΠΊΠΎΡ‚ΠΎΡ€Ρ‹Π΅ Π±Π»ΠΎΠΊΠΈΡ€ΡƒΡŽΡ‚ Ρ€Π΅Π·ΠΊΠΈΠ΅ ΡƒΠ»ΡŒΡ‚Ρ€Π°Ρ„ΠΈΠΎΠ»Π΅Ρ‚ΠΎΠ²Ρ‹Π΅ Π»ΡƒΡ‡ΠΈ. Для ΠΊΠ»ΠΈΠ΅Π½Ρ‚ΠΎΠ² это всС ΠΏΡ€ΠΈΡ‡ΠΈΠ½Ρ‹ Π²Ρ‹Π±Ρ€Π°Ρ‚ΡŒ ΠΏΡ€ΠΎΠ΄ΡƒΠΊΡ†ΠΈΡŽ Polypak Packaging.

Π§Ρ‚ΠΎ Ρ‚Π°ΠΊΠΎΠ΅ коэффициСнт трСния снСга? — MVOrganizing

Какой коэффициСнт трСния снСга?

Β«ΠšΠΎΡΡ„Ρ„ΠΈΡ†ΠΈΠ΅Π½Ρ‚ трСния для снСга ΠΈΠ»ΠΈ льда составляСт всСго 0,03, ΠΏΠΎΡ‚ΠΎΠΌΡƒ Ρ‡Ρ‚ΠΎ ΠΈΠ·-Π·Π° мСстного ΠΎΡ‡Π΅Π½ΡŒ высокого давлСния Ρ‚Π΅ΠΌΠΏΠ΅Ρ€Π°Ρ‚ΡƒΡ€Π° Ρ„Π°Π·ΠΎΠ²ΠΎΠ³ΠΎ прСвращСния Π²ΠΎΠ΄Π°-Π»Π΅Π΄ пониТаСтся, ΠΈ создаСтся слой Π²ΠΎΠ΄Ρ‹.

Как ΡΠΊΡΠΏΠ΅Ρ€ΠΈΠΌΠ΅Π½Ρ‚Π°Π»ΡŒΠ½ΠΎ ΠΎΠΏΡ€Π΅Π΄Π΅Π»ΠΈΡ‚ΡŒ коэффициСнт трСния?

ΠšΠΎΡΡ„Ρ„ΠΈΡ†ΠΈΠ΅Π½Ρ‚ трСния (fr) — это число, ΠΏΡ€Π΅Π΄ΡΡ‚Π°Π²Π»ΡΡŽΡ‰Π΅Π΅ собой ΠΎΡ‚Π½ΠΎΡˆΠ΅Π½ΠΈΠ΅ силы сопротивлСния трСния (Fr) ΠΊ Π½ΠΎΡ€ΠΌΠ°Π»ΡŒΠ½ΠΎΠΉ ΠΈΠ»ΠΈ пСрпСндикулярной силС (N), Ρ‚ΠΎΠ»ΠΊΠ°ΡŽΡ‰Π΅ΠΉ ΠΎΠ±ΡŠΠ΅ΠΊΡ‚Ρ‹ вмСстС.Он прСдставлСн ΡƒΡ€Π°Π²Π½Π΅Π½ΠΈΠ΅ΠΌ: fr = Fr / N.

Π§Ρ‚ΠΎ Ρ‚Π°ΠΊΠΎΠ΅ Mew K Π² Ρ„ΠΈΠ·ΠΈΠΊΠ΅?

ΠšΠΈΠ½Π΅Ρ‚ΠΈΡ‡Π΅ΡΠΊΠΎΠ΅ Ρ‚Ρ€Π΅Π½ΠΈΠ΅ — это сила, Π΄Π΅ΠΉΡΡ‚Π²ΡƒΡŽΡ‰Π°Ρ ΠΌΠ΅ΠΆΠ΄Ρƒ двиТущимися повСрхностями. ΠšΠΎΡΡ„Ρ„ΠΈΡ†ΠΈΠ΅Π½Ρ‚ кинСтичСского трСния обозначаСтся грСчСской Π±ΡƒΠΊΠ²ΠΎΠΉ «мю» (ΞΌ) с Π½ΠΈΠΆΠ½ΠΈΠΌ индСксом Β«kΒ». Π‘ΠΈΠ»Π° кинСтичСского трСния Π² ΞΌk Ρ€Π°Π· большС Π½ΠΎΡ€ΠΌΠ°Π»ΡŒΠ½ΠΎΠΉ силы, Π΄Π΅ΠΉΡΡ‚Π²ΡƒΡŽΡ‰Π΅ΠΉ Π½Π° ΠΎΠ±ΡŠΠ΅ΠΊΡ‚, ΠΈ выраТаСтся Π² Π΅Π΄ΠΈΠ½ΠΈΡ†Π°Ρ… Π½ΡŒΡŽΡ‚ΠΎΠ½ΠΎΠ² (Н).

Как ΠΏΠΎΠ»ΡƒΡ‡ΠΈΡ‚ΡŒ статичСскоС элСктричСство?

ΠšΠΎΡΡ„Ρ„ΠΈΡ†ΠΈΠ΅Π½Ρ‚ статичСского трСния ΠΌΠΎΠΆΠ½ΠΎ Π½Π°ΠΉΡ‚ΠΈ, пСрСписав Ρ„ΠΎΡ€ΠΌΡƒΠ»Ρƒ максимальной силы статичСского трСния: ΞΌs = 0.6020… ΠœΡ‹ Π½Π°Ρ…ΠΎΠ΄ΠΈΠΌ коэффициСнт статичСского трСния ΠΌΠ΅ΠΆΠ΄Ρƒ ΠΊΠΈΡ€ΠΏΠΈΡ‡ΠΎΠΌ ΠΈ Π΄Π΅Ρ€Π΅Π²ΠΎΠΌ Ρ€Π°Π²Π½Ρ‹ΠΌ 0,602.

Как Π½Π°ΠΉΡ‚ΠΈ FN?

ВСс ΠΎΠ±ΡŠΠ΅ΠΊΡ‚Π° Ρ€Π°Π²Π΅Π½ массС ΠΎΠ±ΡŠΠ΅ΠΊΡ‚Π°, ΡƒΠΌΠ½ΠΎΠΆΠ΅Π½Π½ΠΎΠΉ Π½Π° ускорСниС свободного падСния. Π£ΠΌΠ½ΠΎΠΆΡŒΡ‚Π΅ Π΄Π²Π° значСния вмСстС. Π§Ρ‚ΠΎΠ±Ρ‹ Π½Π°ΠΉΡ‚ΠΈ Π½ΠΎΡ€ΠΌΠ°Π»ΡŒΠ½ΡƒΡŽ силу, Π½ΡƒΠΆΠ½ΠΎ ΡƒΠΌΠ½ΠΎΠΆΠΈΡ‚ΡŒ вСс ΠΎΠ±ΡŠΠ΅ΠΊΡ‚Π° Π½Π° косинус ΡƒΠ³Π»Π° Π½Π°ΠΊΠ»ΠΎΠ½Π°.

ΠšΠΎΡΡ„Ρ„ΠΈΡ†ΠΈΠ΅Π½Ρ‚ трСния ΠΏΠΎΠ»ΠΎΠΆΠΈΡ‚Π΅Π»ΡŒΠ½Ρ‹ΠΉ?

Но Π²Π΅Π»ΠΈΡ‡ΠΈΠ½Π° трСния всСгда ΠΏΠΎΠ»ΠΎΠΆΠΈΡ‚Π΅Π»ΡŒΠ½Π°.Π­Ρ‚ΠΎ ΠΏΠΎΠ»ΠΎΠΆΠΈΡ‚Π΅Π»ΡŒΠ½ΠΎ ΠΏΠΎ ΠΎΠΏΡ€Π΅Π΄Π΅Π»Π΅Π½ΠΈΡŽ. Π’Π΅Π»ΠΈΡ‡ΠΈΠ½Π° любой силы (ΠΈΠ»ΠΈ любого Π²Π΅ΠΊΡ‚ΠΎΡ€Π°) ΠΏΠΎΠ»ΠΎΠΆΠΈΡ‚Π΅Π»ΡŒΠ½Π°.

ΠœΠΎΠ³ΡƒΡ‚ Π»ΠΈ коэффициСнты статичСского ΠΈ кинСтичСского трСния Π±Ρ‹Ρ‚ΡŒ мСньшС нуля?

МСньшС нуля ΠΎΠ·Π½Π°Ρ‡Π°Π»ΠΎ Π±Ρ‹, Ρ‡Ρ‚ΠΎ ΠΊ ΠΎΠ±ΡŠΠ΅ΠΊΡ‚Ρƒ добавляСтся энСргия ΠΏΠΎ ΠΌΠ΅Ρ€Π΅ Π΅Π³ΠΎ двиТСния ΠΏΠΎ повСрхности. Π˜Ρ‚Π°ΠΊ, ΠΎΡ‚Π²Π΅Ρ‚ Π½Π° этот вопрос — Β«Π½Π΅Ρ‚Β», Π·Π° ΠΈΡΠΊΠ»ΡŽΡ‡Π΅Π½ΠΈΠ΅ΠΌ случаСв, ΠΊΠΎΠ³Π΄Π° Π²Ρ‹ Ρ…ΠΎΡ‚ΠΈΡ‚Π΅ ΡƒΡ‡ΠΈΡ‚Ρ‹Π²Π°Ρ‚ΡŒ двиТущиСся повСрхности Π² коэффициСнтС трСния ΠΈΠ»ΠΈ ΠΊΠ°ΠΊΠΎΠΉ-Ρ‚ΠΎ странной Π²Π΅Ρ‰ΠΈ Π² этом Ρ€ΠΎΠ΄Π΅, ΠΊΠΎΠ³Π΄Π° Π²Ρ‹ пСрСопрСдСляСтС, Ρ‡Ρ‚ΠΎ ΠΎΠ·Π½Π°Ρ‡Π°Π΅Ρ‚ коэффициСнт трСния.

Как ΡƒΠ·Π½Π°Ρ‚ΡŒ, ΠΊΠΎΠ³Π΄Π° ΠΈΡΠΏΠΎΠ»ΡŒΠ·ΠΎΠ²Π°Ρ‚ΡŒ статичСскоС ΠΈΠ»ΠΈ кинСтичСскоС Ρ‚Ρ€Π΅Π½ΠΈΠ΅?

Π’ ΠΎΠ±Ρ‰Π΅ΠΌ, кинСтичСскоС Ρ‚Ρ€Π΅Π½ΠΈΠ΅ слСдуСт ΠΈΡΠΏΠΎΠ»ΡŒΠ·ΠΎΠ²Π°Ρ‚ΡŒ ΠΏΡ€ΠΈ скольТСнии ΠΌΠ΅ΠΆΠ΄Ρƒ интСрфСйсами ΠΈ статичСскоС Ρ‚Ρ€Π΅Π½ΠΈΠ΅ ΠΏΡ€ΠΈ отсутствии скольТСния.

ΠŸΠΎΡ‡Π΅ΠΌΡƒ Π²Π°ΠΆΠ΅Π½ коэффициСнт трСния?

COF (коэффициСнт трСния) — это Π·Π½Π°Ρ‡Π΅Π½ΠΈΠ΅, ΠΊΠΎΡ‚ΠΎΡ€ΠΎΠ΅ прСдставляСт ΡΠΎΠΎΡ‚Π½ΠΎΡˆΠ΅Π½ΠΈΠ΅ ΠΌΠ΅ΠΆΠ΄Ρƒ силой трСния ΠΈ Π½ΠΎΡ€ΠΌΠ°Π»ΡŒΠ½ΠΎΠΉ силой ΠΌΠ΅ΠΆΠ΄Ρƒ двумя ΠΎΠ±ΡŠΠ΅ΠΊΡ‚Π°ΠΌΠΈ. Π­Ρ‚Π° нСвидимая сила Π»ΠΈΠ±ΠΎ Ρ€Π°Π·Ρ€Π΅ΡˆΠ°Π΅Ρ‚, Π»ΠΈΠ±ΠΎ прСпятствуСт двиТСнию Π΄Π²ΡƒΡ… ΡΠΎΠΏΡ€ΠΈΠΊΠ°ΡΠ°ΡŽΡ‰ΠΈΡ…ΡΡ повСрхностСй. …

Какой ΠΌΠ°ΠΊΡΠΈΠΌΠ°Π»ΡŒΠ½Ρ‹ΠΉ коэффициСнт статичСского трСния?

Максимальная сила трСния покоя составляСт: (fs) max = ΞΌs N, Π³Π΄Π΅ ΞΌs — коэффициСнт трСния покоя. БтатичСскоС Ρ‚Ρ€Π΅Π½ΠΈΠ΅ Π½Π΅ΡƒΠ»ΠΎΠ²ΠΈΠΌΠΎ, ΠΏΠΎΡ‚ΠΎΠΌΡƒ Ρ‡Ρ‚ΠΎ сила статичСского трСния являСтся ΠΏΠ΅Ρ€Π΅ΠΌΠ΅Π½Π½ΠΎΠΉ ΠΈ зависит ΠΎΡ‚ Π²Π½Π΅ΡˆΠ½ΠΈΡ… сил, Π΄Π΅ΠΉΡΡ‚Π²ΡƒΡŽΡ‰ΠΈΡ… Π½Π° ΠΎΠ±ΡŠΠ΅ΠΊΡ‚.

Как ΠΈΡΠΏΠΎΠ»ΡŒΠ·ΠΎΠ²Π°Ρ‚ΡŒ коэффициСнт статичСского трСния?

Π˜Ρ‚Π°ΠΊ, коэффициСнт статичСского трСния Ρ€Π°Π²Π΅Π½ тангСнсу ΡƒΠ³Π»Π°, ΠΏΠΎΠ΄ ΠΊΠΎΡ‚ΠΎΡ€Ρ‹ΠΌ ΡΠΊΠΎΠ»ΡŒΠ·ΡΡ‚ ΠΏΡ€Π΅Π΄ΠΌΠ΅Ρ‚Ρ‹. Аналогичный ΠΌΠ΅Ρ‚ΠΎΠ΄ ΠΌΠΎΠΆΠ½ΠΎ ΠΈΡΠΏΠΎΠ»ΡŒΠ·ΠΎΠ²Π°Ρ‚ΡŒ для измСрСния ΞΌk. Для этого Π²Ρ‹ Ρ‚ΠΎΠ»ΠΊΠ°Π΅Ρ‚Π΅ Π²Π΅Ρ€Ρ…Π½ΠΈΠΉ ΠΎΠ±ΡŠΠ΅ΠΊΡ‚ ΠΏΠΎ ΠΌΠ΅Ρ€Π΅ увСличСния ΡƒΠ³Π»Π°. Когда Π²Π΅Ρ€Ρ…Π½ΠΈΠΉ ΠΎΠ±ΡŠΠ΅ΠΊΡ‚ двиТСтся с постоянной ΡΠΊΠΎΡ€ΠΎΡΡ‚ΡŒΡŽ, тангСнс этого ΡƒΠ³Π»Π° Ρ€Π°Π²Π΅Π½ ΞΌk.

Как Π½Π°ΠΉΡ‚ΠΈ коэффициСнт статичСского ΠΈ кинСтичСского трСния?

Π€ΠΎΡ€ΠΌΡƒΠ»Π°: Β΅ = f / N, Π³Π΄Π΅ Β΅ — коэффициСнт трСния, f — Π²Π΅Π»ΠΈΡ‡ΠΈΠ½Π° силы, ΠΏΡ€Π΅ΠΏΡΡ‚ΡΡ‚Π²ΡƒΡŽΡ‰Π΅ΠΉ двиТСнию, Π° N — Π½ΠΎΡ€ΠΌΠ°Π»ΡŒΠ½Π°Ρ сила.ΠΠΎΡ€ΠΌΠ°Π»ΡŒΠ½Π°Ρ сила — это сила, с ΠΊΠΎΡ‚ΠΎΡ€ΠΎΠΉ ΠΎΠ΄Π½Π° ΠΏΠΎΠ²Π΅Ρ€Ρ…Π½ΠΎΡΡ‚ΡŒ толкаСтся Π² Π΄Ρ€ΡƒΠ³ΡƒΡŽ.

Как Π½Π°ΠΉΡ‚ΠΈ коэффициСнт трСния?

ΠœΠ΅Π½ΡΠ΅Ρ‚ΡΡ Π»ΠΈ коэффициСнт трСния с ΡƒΠ³Π»ΠΎΠΌ?

Π’Π°ΠΊ Π²ΠΎΡ‚, коэффициСнт трСния являСтся свойством ΠΌΠ°Ρ‚Π΅Ρ€ΠΈΠ°Π»ΠΎΠ² ΠΈ Π½Π΅ мСняСтся с ΡƒΠ³Π»ΠΎΠΌ — Π½ΠΎ это Ρ‚ΠΎΡ‚ случай, ΠΊΠΎΠ³Π΄Π° сила трСния Π±ΡƒΠ΄Π΅Ρ‚ ΡƒΠΌΠ΅Π½ΡŒΡˆΠ°Ρ‚ΡŒΡΡ, ΠΏΠΎΡΠΊΠΎΠ»ΡŒΠΊΡƒ ΠΎΠ½Π° Ρ€Π°Π²Π½Π° Fk = ΞΌkFn.

Π§Ρ‚ΠΎ происходит с коэффициСнтом трСния ΠΏΡ€ΠΈ ΡƒΠ²Π΅Π»ΠΈΡ‡Π΅Π½ΠΈΠΈ ΡƒΠ³Π»Π°?

По ΠΌΠ΅Ρ€Π΅ увСличСния ΡƒΠ³Π»Π° Π½Π°ΠΊΠ»ΠΎΠ½Π° Π½ΠΎΡ€ΠΌΠ°Π»ΡŒΠ½Π°Ρ сила ΡƒΠΌΠ΅Π½ΡŒΡˆΠ°Π΅Ρ‚ΡΡ, Ρ‡Ρ‚ΠΎ ΡƒΠΌΠ΅Π½ΡŒΡˆΠ°Π΅Ρ‚ силу трСния.Наклон ΠΌΠΎΠΆΠ½ΠΎ ΡƒΠ²Π΅Π»ΠΈΡ‡ΠΈΠ²Π°Ρ‚ΡŒ Π΄ΠΎ Ρ‚Π΅Ρ… ΠΏΠΎΡ€, ΠΏΠΎΠΊΠ° ΠΎΠ±ΡŠΠ΅ΠΊΡ‚ Π½Π΅ Π½Π°Ρ‡Π½Π΅Ρ‚ ΡΠΊΠΎΠ»ΡŒΠ·ΠΈΡ‚ΡŒ. По ΠΌΠ΅Ρ€Π΅ увСличСния ΡƒΠ³Π»Π° Π½Π°ΠΊΠ»ΠΎΠ½Π° сила трСния Π±ΡƒΠ΄Π΅Ρ‚ ΡƒΠΌΠ΅Π½ΡŒΡˆΠ°Ρ‚ΡŒΡΡ ΠΈΠ·-Π·Π° измСнСния Π½ΠΎΡ€ΠΌΠ°Π»ΡŒΠ½ΠΎΠΉ силы.

Зависит Π»ΠΈ коэффициСнт трСния ΠΎΡ‚ массы?

Π‘ΠΈΠ»Π° трСния Π² этом случаС зависит ΠΎΡ‚ Π΄Π²ΡƒΡ… Ρ‚ΠΈΠΏΠΎΠ² Π²Π·Π°ΠΈΠΌΠΎΠ΄Π΅ΠΉΡΡ‚Π²ΡƒΡŽΡ‰ΠΈΡ… ΠΌΠ°Ρ‚Π΅Ρ€ΠΈΠ°Π»ΠΎΠ² (описываСмых коэффициСнтом ΞΌk) ΠΈ ΠΎΡ‚ Ρ‚ΠΎΠ³ΠΎ, насколько сильно эти Π΄Π²Π΅ повСрхности ΠΏΡ€ΠΈΠΆΠΈΠΌΠ°ΡŽΡ‚ΡΡ Π΄Ρ€ΡƒΠ³ ΠΊ Π΄Ρ€ΡƒΠ³Ρƒ (Π½ΠΎΡ€ΠΌΠ°Π»ΡŒΠ½Π°Ρ сила). Π’Π°ΠΊΠΆΠ΅ коэффициСнт трСния Π½Π΅ зависит ΠΎΡ‚ массы ΠΎΠ±ΡŠΠ΅ΠΊΡ‚Π°.

ΠœΠ΅Π½ΡΠ΅Ρ‚ΡΡ Π»ΠΈ коэффициСнт трСния с Π½Π°Π³Ρ€ΡƒΠ·ΠΊΠΎΠΉ?

Показано, Ρ‡Ρ‚ΠΎ коэффициСнт трСния измСняСтся ΠΎΡ‚ 0,215 Π΄ΠΎ 0,343, ΠΎΡ‚ 0,27 Π΄ΠΎ 0,352 ΠΈ ΠΎΡ‚ 0,352 Π΄ΠΎ 0,372 ΠΏΡ€ΠΈ ΠΈΠ·ΠΌΠ΅Π½Π΅Π½ΠΈΠΈ Π½ΠΎΡ€ΠΌΠ°Π»ΡŒΠ½ΠΎΠΉ Π½Π°Π³Ρ€ΡƒΠ·ΠΊΠΈ ΠΎΡ‚ 5 Π΄ΠΎ 10 Н для ΠΏΠ°Ρ€ алюминий-алюминий, алюминий-мСдь ΠΈ алюминий-Π»Π°Ρ‚ΡƒΠ½ΡŒ соотвСтствСнно. Π­Ρ‚ΠΈ Ρ€Π΅Π·ΡƒΠ»ΡŒΡ‚Π°Ρ‚Ρ‹ ΠΏΠΎΠΊΠ°Π·Ρ‹Π²Π°ΡŽΡ‚, Ρ‡Ρ‚ΠΎ коэффициСнт трСния увСличиваСтся с ΡƒΠ²Π΅Π»ΠΈΡ‡Π΅Π½ΠΈΠ΅ΠΌ Π½ΠΎΡ€ΠΌΠ°Π»ΡŒΠ½ΠΎΠΉ Π½Π°Π³Ρ€ΡƒΠ·ΠΊΠΈ.

КакоС самоС скользкоС вСщСство Π² ΠΌΠΈΡ€Π΅?

ПВЀЭ

Π§Ρ‚ΠΎ самоС скользкоС Π½Π° Π·Π΅ΠΌΠ»Π΅?

Π‘ΠΠœ

ΠŸΠΎΡ‡Π΅ΠΌΡƒ PTFE скользкий?

Π­Ρ‚Π° химичСская совмСстная Ρ€Π°Π±ΠΎΡ‚Π° ΡƒΠ³Π»Π΅Ρ€ΠΎΠ΄Π° ΠΈ Ρ„Ρ‚ΠΎΡ€Π° Π΄Π΅Π»Π°Π΅Ρ‚ Ρ‚Π΅Ρ„Π»ΠΎΠ½ Ρ‡Ρ€Π΅Π·Π²Ρ‹Ρ‡Π°ΠΉΠ½ΠΎ химичСски ΡΡ‚Π°Π±ΠΈΠ»ΡŒΠ½Ρ‹ΠΌ, ΠΈ ΠΈΠΌΠ΅Π½Π½ΠΎ эта химичСская ΡΡ‚Π°Π±ΠΈΠ»ΡŒΠ½ΠΎΡΡ‚ΡŒ Π΄Π΅Π»Π°Π΅Ρ‚ Ρ‚Π΅Ρ„Π»ΠΎΠ½ Ρ‚Π°ΠΊΠΈΠΌ скользким.ΠŸΠΎΡΡ‚ΠΎΡ€ΠΎΠ½Π½ΠΈΠ΅ вСщСства, Ρ‚Π°ΠΊΠΈΠ΅ ΠΊΠ°ΠΊ ΠΆΠ°Ρ€ΠΊΠΎΠ΅ яйцо, Π½Π΅ ΠΌΠΎΠ³ΡƒΡ‚ Π½Π°ΠΉΡ‚ΠΈ химичСской Ρ‚ΠΎΡ‡ΠΊΠΈ ΠΎΠΏΠΎΡ€Ρ‹ Π½Π° фторсодСрТащСй Π±Ρ€ΠΎΠ½Π΅, поэтому ΠΎΠ½ΠΈ просто ΡƒΡΠΊΠΎΠ»ΡŒΠ·Π½ΡƒΡ‚.

Π§Ρ‚ΠΎ самоС скользкоС ΠΈΠ· извСстных Ρ‡Π΅Π»ΠΎΠ²Π΅ΠΊΡƒ?

ISTR, Ρ‡Ρ‚ΠΎ ΠΌΠΎΠΊΡ€Ρ‹ΠΉ Π»Π΅Π΄ Π½Π° ΠΌΠΎΠΊΡ€ΠΎΠΌ Π»ΡŒΠ΄Ρƒ — самая скользкая Π²Π΅Ρ‰ΡŒ, извСстная Ρ‡Π΅Π»ΠΎΠ²Π΅ΠΊΡƒ.

ΠŸΠΎΠ²Ρ€Π΅ΠΆΠ΄Π°Π΅Ρ‚ Π»ΠΈ литиСвая смазка Ρ€Π΅Π·ΠΈΠ½Ρƒ?

Биликоновая смазка бСзопасна для Ρ€Π΅Π·ΠΈΠ½Ρ‹ ΠΈ Π΄Π΅ΠΉΡΡ‚Π²ΠΈΡ‚Π΅Π»ΡŒΠ½ΠΎ ΠΏΠΎΠΌΠΎΠ³Π°Π΅Ρ‚ ΡΠΎΡ…Ρ€Π°Π½ΠΈΡ‚ΡŒ Π΅Π΅ ΠΌΡΠ³ΠΊΠΎΡΡ‚ΡŒ. Π›ΡŽΠ±Π°Ρ другая смазка Π½Π° основС ΠΌΠΈΠ½Π΅Ρ€Π°Π»ΡŒΠ½ΠΎΠ³ΠΎ масла Ρ€Π°Π·Ρ€ΡƒΡˆΠ°Π΅Ρ‚ Π½Π°Ρ‚ΡƒΡ€Π°Π»ΡŒΠ½Ρ‹ΠΉ ΠΊΠ°ΡƒΡ‡ΡƒΠΊ. «БСлая литиСвая смазка» — это довольно ΠΎΠ±Ρ‰ΠΈΠΉ Ρ‚Π΅Ρ€ΠΌΠΈΠ½. Если Π²Ρ‹ Π½Π΅ Π·Π½Π°Π΅Ρ‚Π΅ Π΅Π΅ состав, я Π±Ρ‹ посовСтовал Π΄Π΅Ρ€ΠΆΠ°Ρ‚ΡŒ Π΅Π΅ подальшС ΠΎΡ‚ Π½Π°Ρ‚ΡƒΡ€Π°Π»ΡŒΠ½ΠΎΠ³ΠΎ ΠΊΠ°ΡƒΡ‡ΡƒΠΊΠ°.

Π‘ΠΈΠ»ΠΈΠΊΠΎΠ½ΠΎΠ²Ρ‹ΠΉ спрСй Π»ΡƒΡ‡ΡˆΠ΅ WD40?

WD40. Π‘ΠΈΠ»ΠΈΠΊΠΎΠ½ΠΎΠ²Ρ‹ΠΉ спрСй ΠΈΡΠΏΠΎΠ»ΡŒΠ·ΡƒΠ΅Ρ‚ΡΡ для смазывания ΠΌΠ½ΠΎΠ³ΠΈΡ… повСрхностСй, Ρ‚Π°ΠΊΠΈΡ… ΠΊΠ°ΠΊ ΠΌΠ΅Ρ‚Π°Π»Π», Ρ€Π΅Π·ΠΈΠ½Π°, пластмассы ΠΈ Π΄Π°ΠΆΠ΅ Π΄Π΅Ρ€Π΅Π²ΠΎ. По этой ΠΏΡ€ΠΈΡ‡ΠΈΠ½Π΅, Ссли Π²Ρ‹ смазываСтС ΠΏΠ»ΠΎΡ‚Π½Ρ‹ΠΉ участок, Π½Π΅ΠΎΠ±Ρ…ΠΎΠ΄ΠΈΠΌΠΎ сначала ΠΎΡ‡ΠΈΡΡ‚ΠΈΡ‚ΡŒ Π΅Π³ΠΎ ΠΎΠ±Π΅Π·ΠΆΠΈΡ€ΠΈΠ²Π°ΡŽΡ‰ΠΈΠΌ срСдством, Ρ‚Π°ΠΊΠΈΠΌ ΠΊΠ°ΠΊ WD40, Π° Π·Π°Ρ‚Π΅ΠΌ ΠΈΡΠΏΠΎΠ»ΡŒΠ·ΠΎΠ²Π°Ρ‚ΡŒ нСбольшоС количСство силиконового спрСя, Ρ‡Ρ‚ΠΎΠ±Ρ‹ ΡΠΌΠ°Π·Π°Ρ‚ΡŒ участок. …

ΠœΠΎΠ³Ρƒ Π»ΠΈ я ΠΈΡΠΏΠΎΠ»ΡŒΠ·ΠΎΠ²Π°Ρ‚ΡŒ WD40 Π½Π° Π²ΠΈΠ½ΠΈΠ»ΠΎΠ²Ρ‹Ρ… ΠΎΠΊΠ½Π°Ρ…?

НС ΠΈΡΠΏΠΎΠ»ΡŒΠ·ΡƒΠΉΡ‚Π΅ масло. ΠžΡ‡Π΅Π½ΡŒ распространСнноС Ρ€Π΅ΡˆΠ΅Π½ΠΈΠ΅ — ΠΈ ΠΎΡ‡Π΅Π½ΡŒ ΠΏΠ»ΠΎΡ…ΠΎΠ΅ — смазка ΠΎΠΊΠΎΠ½Π½Ρ‹Ρ… Π½Π°ΠΏΡ€Π°Π²Π»ΡΡŽΡ‰ΠΈΡ… ΠΏΡ€ΠΎΠ½ΠΈΠΊΠ°ΡŽΡ‰ΠΈΠΌ маслом, Π½Π°ΠΏΡ€ΠΈΠΌΠ΅Ρ€ WD-40.Масла ΠΏΡ€Π΅Π΄Π»ΠΎΠΆΠ°Ρ‚ Ρ‚ΠΎΠ»ΡŒΠΊΠΎ Π²Ρ€Π΅ΠΌΠ΅Π½Π½ΠΎΠ΅ Ρ€Π΅ΡˆΠ΅Π½ΠΈΠ΅ ΠΈ скоро станут ΠΌΠ°Π³Π½ΠΈΡ‚ΠΎΠΌ для ΠΏΡ‹Π»ΠΈ ΠΈ грязи, дСлая ваши ΠΎΠΊΠ½Π° Π±ΠΎΠ»Π΅Π΅ Π»ΠΈΠΏΠΊΠΈΠΌΠΈ, Ρ‡Π΅ΠΌ ΠΊΠΎΠ³Π΄Π°-Π»ΠΈΠ±ΠΎ. Никогда Π½Π΅ ΠΈΡΠΏΠΎΠ»ΡŒΠ·ΡƒΠΉΡ‚Π΅ Π½Π΅Ρ„Ρ‚Π΅ΠΏΡ€ΠΎΠ΄ΡƒΠΊΡ‚Ρ‹ Π½Π° дСрСвянных ΠΈΠ»ΠΈ Π²ΠΈΠ½ΠΈΠ»ΠΎΠ²Ρ‹Ρ… ΠΎΠΊΠ½Π°Ρ….

ΠšΠΎΡΡ„Ρ„ΠΈΡ†ΠΈΠ΅Π½Ρ‚ трСния, Π‘ΠΎΠΏΡ€ΠΎΡ‚ΠΈΠ²Π»Π΅Π½ΠΈΠ΅ ΠΊΠ°Ρ‡Π΅Π½ΠΈΡŽ, Π‘ΠΎΠΏΡ€ΠΎΡ‚ΠΈΠ²Π»Π΅Π½ΠΈΠ΅ Π²ΠΎΠ·Π΄ΡƒΡ…Π°, Аэродинамика

ΠšΠΎΡΡ„Ρ„ΠΈΡ†ΠΈΠ΅Π½Ρ‚ трСния Π² ΠΏΠΎΠ΄ΡˆΠΈΠΏΠ½ΠΈΠΊΠ°Ρ…

ΠšΠΎΡΡ„Ρ„ΠΈΡ†ΠΈΠ΅Π½Ρ‚ трСния [-]

Подшипник скольТСния гидродинамичСский

0.003 … 0,04

Подшипник скольТСния, спСчСнная Π±Ρ€ΠΎΠ½Π·Π°, с масляной смазкой

0,04 … 0,07

Подшипники скольТСния, твСрдая Π±Ρ€ΠΎΠ½Π·Π°, консистСнтная смазка

0,07 … 0,12

ΠŸΠΎΠ»ΠΈΠΌΠ΅Ρ€Π½Ρ‹ΠΉ подшипник скольТСния, ΠΏΠΎΠ»ΠΈΠ°ΠΌΠΈΠ΄, сухой

0,2 … 0,3

Подшипник ΠΏΠΎΠ»ΠΈΠΌΠ΅Ρ€Π½Ρ‹ΠΉ, ΠΊΠΎΠΌΠΏΠΎΠ·ΠΈΡ‚Π½Ρ‹ΠΉ, сухой

0.05 … 0,15

Π¨Π°Ρ€ΠΈΠΊΠΎΠ²Ρ‹Π΅ подшипники

0,001 … 0,0015

Подшипники Ρ€ΠΎΠ»ΠΈΠΊΠΎΠ²Ρ‹Π΅

0,0018

Π˜Π³ΠΎΠ»ΡŒΡ‡Π°Ρ‚Ρ‹Π΅ подшипники

0,0045

Π’ΠΎΠ·Π΄ΡƒΡˆΠ½Ρ‹Π΅ подшипники ΠΏΠΎΠ΄ Π΄Π°Π²Π»Π΅Π½ΠΈΠ΅ΠΌ

0,0

ГидростатичСскиС подшипники

0.001 … 0,002, исх. Вязк. Π‘Π΄Π²ΠΈΠ³

ΠšΠΎΡΡ„Ρ„ΠΈΡ†ΠΈΠ΅Π½Ρ‚ трСния Π² Ρ€Π΅Π·ΡŒΠ±ΠΎΠ²Ρ‹Ρ… соСдинСниях

со смазкой

со смазкой

сухой

сухой

в маслС

МоБ 2

Π¦ΠΈΠ½ΠΊΠΎΠ²Π°Π½ΠΈΠ΅

Π Π’Π‘

0.12 … 0,18 0,08 … 0,12 0,12 … 0,18 0,3 … 0,8
смазка для Π²ΠΈΠ½Ρ‚ΠΎΠ² / Ρ‚Ρ€Π΅Π½ΠΈΠ΅ >>

Π’Π°Π»ΡŒΡ†ΠΎΠ²ΠΊΠ° сопротивлСниС

C r [-]

Π‘Ρ‚Π°Π»ΡŒΠ½ΠΎΠ΅ колСсо Π½Π° Ρ€Π΅Π»ΡŒΡΠ΅

0.0002 … 0,0010

ΠΠ²Ρ‚ΠΎΠΌΠΎΠ±ΠΈΠ»ΡŒΠ½Π°Ρ шина Π½Π° Π΄ΠΎΡ€ΠΎΠ³Π΅

0,010 … 0,035

Π­Π½Π΅Ρ€Π³ΠΎΠ±Π΅Π·ΠΎΠΏΠ°ΡΠ½ΠΎΡΡ‚ΡŒ Π°Π²Ρ‚ΠΎΠΌΠΎΠ±ΠΈΠ»ΡŒΠ½Ρ‹Ρ… шин

0,006 … 0,009

Π’Ρ€ΡƒΠ±ΠΊΠ° 22 ΠΌΠΌ, 8 Π±Π°Ρ€

0,002

Гоночная шина 23 ΠΌΠΌ, 7 Π±Π°Ρ€

0,003

УнивСрсал 32 ΠΌΠΌ, 5 Π±Π°Ρ€

0.005

Π¨ΠΈΠ½Π° с Π·Π°Ρ‰ΠΈΡ‚ΠΎΠΉ ΠΎΡ‚ ΠΏΡ€ΠΎΡ‚Π΅Ρ‡Π΅ΠΊ 37 ΠΌΠΌ, 5 Π±Π°Ρ€ / 3 Π±Π°Ρ€Π° 0,007 / 0,01
ΠšΠ°Π»ΡŒΠΊΡƒΠ»ΡΡ‚ΠΎΡ€ сопротивлСния ΠΊΠ°Ρ‡Π΅Π½ΠΈΡŽ >>

Π’ΠΎΠ·Π΄ΡƒΡ… сопротивлСниС

C w [-]

A f [ΠΌ 2 ]

A f C w [ΠΌ 2 ]

Π‘ΠΎΠ²Ρ€Π΅ΠΌΠ΅Π½Π½Ρ‹ΠΉ Π°Π²Ρ‚ΠΎΠΌΠΎΠ±ΠΈΠ»ΡŒ

0.25 … 0,35

1,8

0,54

Π£Π½ΠΈΠ²Π΅Ρ€ΡΠ°Π»ΡŒΠ½Ρ‹ΠΉ Π°Π²Ρ‚ΠΎΠΌΠΎΠ±ΠΈΠ»ΡŒ

0,4 ​​… 0,5

3

1,35

Π“Ρ€ΡƒΠ·ΠΎΠ²ΠΈΠΊ

0,7

6

4,2

Π€ΠΎΡ€ΠΌΡƒΠ»Π° 1

0.75

1,6

1,20

Nuna Solar Car

0,07

0,86

0,06

Π‘ΠΏΠΎΡ€Ρ‚ΠΈΠ²Π½Ρ‹ΠΉ самолСт

0,12

5

0.60

ΠšΠ°Ρ€Ρ‚ΠΎΡ‡ΠΊΠ°

0.8

1

0,80

ВСлосипСд Grand pa

1,1

0,6

0,66

Π“ΠΎΠ½ΠΎΡ‡Π½Ρ‹ΠΉ вСлосипСд

0,9

0,35

0,31

Π“ΠΎΡ€ΠΈΠ·ΠΎΠ½Ρ‚Π°Π»ΡŒΠ½Ρ‹ΠΉ вСлосипСд

0.8

0,25

0,20

аэродинамичСский вСлосипСд

0,4 ​​… 0,15

0,25

0,07

Π¨Π°Ρ€ΠΈΠΊ

0,5

Ο€R 2

0,5Ο€R 2

ΠŸΠΎΠ»ΡƒΡΡ„Π΅Ρ€Π°

0.4

Ο€R 2

0,4Ο€R 2

ΠŸΠΎΠ»ΡƒΡΡ„Π΅Ρ€Π°

1,3

Ο€R 2

1,3Ο€R 2

Плоский ΠΊΡ€ΡƒΠ³ΠΎΠ²ΠΎΠΉ диск 1,1 Ο€R 2 1,1Ο€R 2
ΠšΠ°Π»ΡŒΠΊΡƒΠ»ΡΡ‚ΠΎΡ€ сопротивлСния Π²ΠΎΠ·Π΄ΡƒΡ…Π° >>

ΠŸΡ€Π΅ΠΎΠ±Ρ€Π°Π·ΠΎΠ²Π°Π½ΠΈΠ΅ скорости Π²Π΅Ρ‚Ρ€Π° ΠΏΠΎ шкалС Π‘ΠΎΡ„ΠΎΡ€Ρ‚Π° Π² ΡΠΊΠΎΡ€ΠΎΡΡ‚ΡŒ Π²Π΅Ρ‚Ρ€Π° Π² ΠΌ / с:

1: 1 м / с

2: 2 м / с

3: 4 м / с

4: 7 м / с

5:10 м / с

6:13 м / с

7:16 м / с

8:19 м / с

9: 23 м / с

10:27 м / с

11: 31м / с

12:> 36 м / с

ΠŸΡ€Π΅ΠΎΠ±Ρ€Π°Π·ΠΎΠ²Π°Π½ΠΈΠ΅ ΠΏΡ€ΠΎΡ†Π΅Π½Ρ‚ ΡƒΠΊΠ»ΠΎΠ½Π° ΠΊ ΡƒΠ³Π»Ρƒ Π½Π°ΠΊΠ»ΠΎΠ½Π°

Π£ΠΊΠ»ΠΎΠ½ 5% — это ΡƒΠΊΠ»ΠΎΠ½, ΠΊΠΎΡ‚ΠΎΡ€Ρ‹ΠΉ поднимаСтся Π½Π° 5 ΠΌ Π½Π° ΠΊΠ°ΠΆΠ΄Ρ‹Π΅ 100 ΠΌ Π³ΠΎΡ€ΠΈΠ·ΠΎΠ½Ρ‚Π°Π»ΡŒΠ½ΠΎΠ³ΠΎ расстояния, Ρ‚.Π΅.Π΅. ΡƒΠΊΠ»ΠΎΠ½ Π°Ρ‚Π°Π½Π° (5/100) = 2,86 градуса. 100% ΡƒΠΊΠ»ΠΎΠ½ составляСт 45 градусов.

Friction — The Physics Hypertextbook

ΠžΠ±ΡΡƒΠΆΠ΄Π΅Π½ΠΈΠ΅

Π‘ΠΈΠ»Π° ΠΌΠ΅ΠΆΠ΄Ρƒ ΠΊΠΎΠ½Ρ‚Π°ΠΊΡ‚ΠΈΡ€ΡƒΡŽΡ‰ΠΈΠΌΠΈ повСрхностями, которая сопротивляСтся ΠΈΡ… ΠΎΡ‚Π½ΠΎΡΠΈΡ‚Π΅Π»ΡŒΠ½ΠΎΠΌΡƒ ΠΊΠ°ΡΠ°Ρ‚Π΅Π»ΡŒΠ½ΠΎΠΌΡƒ двиТСнию (скольТСнию).

Π’ΠΈΠΏΡ‹: статичСский ΠΈ кинСтичСский

ΠšΠ»Π°ΡΡΠΈΡ‡Π΅ΡΠΊΠΎΠ΅ ΠΏΡ€ΠΈΠ±Π»ΠΈΠΆΠ΅Π½ΠΈΠ΅

  • нСзависимо ΠΎΡ‚
    • ΠΏΠ»ΠΎΡ‰Π°Π΄ΡŒ повСрхности
    • ΡΠΊΠΎΡ€ΠΎΡΡ‚ΡŒ (ΠΊΡ€ΠΎΠΌΠ΅ v = 0)
    • Ρ‚Π΅ΠΌΠΏΠ΅Ρ€Π°Ρ‚ΡƒΡ€Π°
  • зависит ΠΎΡ‚ Ρ…Π°Ρ€Π°ΠΊΡ‚Π΅Ρ€Π° ΠΊΠΎΠ½Ρ‚Π°ΠΊΡ‚ΠΈΡ€ΡƒΡŽΡ‰ΠΈΡ… повСрхностСй ΠΈ составляСт
  • прямо ΠΏΡ€ΠΎΠΏΠΎΡ€Ρ†ΠΈΠΎΠ½Π°Π»ΡŒΠ½Π° Π½ΠΎΡ€ΠΌΠ°Π»ΡŒΠ½ΠΎΠΉ силС.
    • Π˜Π½Ρ‚Π΅Ρ€Π΅ΡΠ½Π°Ρ цитата…
      Π“ΠΈΠΉΠΎΠΌ Амонтон (1663–1705) Ѐранция
      Π“ΠΈΠΉΠΎΠΌ Амонтон ΠΏΠ΅Ρ€Π²Ρ‹ΠΌ установил, Ρ‡Ρ‚ΠΎ сущСствуСт ΠΏΡ€ΠΎΠΏΠΎΡ€Ρ†ΠΈΠΎΠ½Π°Π»ΡŒΠ½Π°Ρ Π·Π°Π²ΠΈΡΠΈΠΌΠΎΡΡ‚ΡŒ ΠΌΠ΅ΠΆΠ΄Ρƒ силой трСния ΠΈ Π²Π·Π°ΠΈΠΌΠ½Ρ‹ΠΌ Π΄Π°Π²Π»Π΅Π½ΠΈΠ΅ΠΌ (ΠΈΠ»ΠΈ силой) ΠΌΠ΅ΠΆΠ΄Ρƒ ΠΊΠΎΠ½Ρ‚Π°ΠΊΡ‚ΠΈΡ€ΡƒΡŽΡ‰ΠΈΠΌΠΈ Ρ‚Π΅Π»Π°ΠΌΠΈ. ΠœΡ‹ осознаСм эту взаимосвязь, ΠΊΠΎΠ³Π΄Π° Π΄Π΅Π»ΠΈΠΌ силу трСния Π½Π° Π½ΠΎΡ€ΠΌΠ°Π»ΡŒΠ½ΡƒΡŽ силу ΠΈ опрСдСляСм частноС ΠΊΠ°ΠΊ «коэффициСнт трСния». Π‘Ρ‚Π°Ρ‚ΡŒΡ Амонтона «De la rΓ©sistance causΓ©e dans les machines» Π±Ρ‹Π»Π° ΠΎΠΏΡƒΠ±Π»ΠΈΠΊΠΎΠ²Π°Π½Π° Π² 1699 Π³ΠΎΠ΄Ρƒ Π² Memoires de l’AcadΓ©mie des Sciences .

f ≀ ΞΌ N

f с ≀ ΞΌ с N

f k = ΞΌ k N

микроскопичСскоС описаниС

Ρ€Π°Π·Π½ΠΎΠ΅

  • НСдавно Π±Ρ‹Π»ΠΎ ΠΏΠΎΠΊΠ°Π·Π°Π½ΠΎ, Ρ‡Ρ‚ΠΎ смазочныС свойства Π³Ρ€Π°Ρ„ΠΈΡ‚Π° ΠΈΡΡ‡Π΅Π·Π°ΡŽΡ‚ Π² свСрхвысоком Π²Π°ΠΊΡƒΡƒΠΌΠ΅, ΠΈ, ΡΠ»Π΅Π΄ΠΎΠ²Π°Ρ‚Π΅Π»ΡŒΠ½ΠΎ, ΠΌΠΎΠ»Π΅ΠΊΡƒΠ»Ρ‹ Π³Π°Π·ΠΎΠ², Ρ‚Π°ΠΊΠΈΡ… ΠΊΠ°ΠΊ кислород ΠΈ Π°Π·ΠΎΡ‚, скорСС всСго, Π΄Π΅ΠΉΡΡ‚Π²ΡƒΡŽΡ‚ ΠΊΠ°ΠΊ своСго Ρ€ΠΎΠ΄Π° молСкулярная смазка, помогая листам ΡΠΊΠΎΠ»ΡŒΠ·ΠΈΡ‚ΡŒ Π΄Ρ€ΡƒΠ³ ΠΌΠΈΠΌΠΎ Π΄Ρ€ΡƒΠ³Π°.
  • Π¨Π΅Ρ€ΠΎΡ…ΠΎΠ²Π°Ρ‚ΠΎΡΡ‚ΡŒ — второстСпСнный Ρ„Π°ΠΊΡ‚ΠΎΡ€, Π²Π»ΠΈΡΡŽΡ‰ΠΈΠΉ Π½Π° Ρ‚Ρ€Π΅Π½ΠΈΠ΅. Π’Ρ€Π΅Π½ΠΈΠ΅ ΠΌΠ΅ΠΆΠ΄Ρƒ Π³Π»Π°Π΄ΠΊΠΈΠΌΠΈ повСрхностями часто Π²Ρ‹ΡˆΠ΅. НасСкомыС ΠΌΠΎΠ³ΡƒΡ‚ Ρ…ΠΎΠ΄ΠΈΡ‚ΡŒ ΠΏΠΎ ΠΎΠΊΠ½Π°ΠΌ.
  • Если Ρ‚Ρ€Π΅Π½ΠΈΠ΅ Π½Π΅ зависит ΠΎΡ‚ ΡˆΠ΅Ρ€ΠΎΡ…ΠΎΠ²Π°Ρ‚ΠΎΡΡ‚ΠΈ повСрхности, ΠΏΠΎΡ‡Π΅ΠΌΡƒ Ρƒ шин Π΅ΡΡ‚ΡŒ ΠΏΡ€ΠΎΡ‚Π΅ΠΊΡ‚ΠΎΡ€? ΠŸΡ€ΠΎΡ‚Π΅ΠΊΡ‚ΠΎΡ€ ΡˆΠΈΠ½Ρ‹ ΠΏΡ€ΠΎΠ»ΠΈΠ²Π°Π΅Ρ‚ Π²ΠΎΠ΄Ρƒ.
  • Π’Π΅Ρ„Π»ΠΎΠ½ ΠΈΠΌΠ΅Π΅Ρ‚ Π½Π°ΡΡ‚ΠΎΠ»ΡŒΠΊΠΎ Π½ΠΈΠ·ΠΊΠΈΠΉ коэффициСнт трСния, Ρ‡Ρ‚ΠΎ часто отслаиваСтся ΠΎΡ‚ ΠΊΠ°ΡΡ‚Ρ€ΡŽΠ»ΡŒ ΠΈ сковородок. (Π˜ΡΠΏΠΎΠ»ΡŒΠ·ΡƒΠΉΡ‚Π΅ Π΄Π΅Ρ€Π΅Π²ΡΠ½Π½ΡƒΡŽ ΠΈΠ»ΠΈ ΠΏΠ»Π°ΡΡ‚ΠΈΠΊΠΎΠ²ΡƒΡŽ посуду.) Как Π·Π°ΡΡ‚Π°Π²ΠΈΡ‚ΡŒ Π΅Π΅ ΠΏΡ€ΠΈΠ»ΠΈΠΏΠ½ΡƒΡ‚ΡŒ? Аналогия с Π΄Ρ€Π΅Π΄Π°ΠΌΠΈ: Ρ‚Π΅Ρ„Π»ΠΎΠ½ — это ΠΏΠΎΠ»ΠΈΠΌΠ΅Ρ€, ΠΎΡ‚Π΄Π΅Π»ΡŒΠ½Ρ‹Π΅ пряди волос скользкиС, Π½ΠΎ пряди ΠΌΠΎΠ³ΡƒΡ‚ Π·Π°ΠΏΡƒΡ‚Π°Ρ‚ΡŒΡΡ Π΄ΠΎ Ρ‚Π°ΠΊΠΎΠΉ стСпСни, Ρ‡Ρ‚ΠΎ ΠΈΡ… Π½Π΅Π²ΠΎΠ·ΠΌΠΎΠΆΠ½ΠΎ Ρ€Π°Π·Π΄Π΅Π»ΠΈΡ‚ΡŒ.
  • Π£ людСй нСбольшиС волосы Π½Π° Ρ‚Π΅Π»Π΅. ΠŸΠΎΡ‡Π΅ΠΌΡƒ ΠΎΠΏΡ€Π΅Π΄Π΅Π»Π΅Π½Π½Ρ‹Π΅ участки всС Π΅Ρ‰Π΅ густо ΠΏΠΎΠΊΡ€Ρ‹Ρ‚Ρ‹ волосами? Π­Π²ΠΎΠ»ΡŽΡ†ΠΈΠΎΠ½Π½Ρ‹Π΅ прСимущСства. ΠžΠΏΠΈΡˆΠΈΡ‚Π΅ ΠΈΡ…!
  • ДинамичСскоС Ρ‚Ρ€Π΅Π½ΠΈΠ΅ сущСствуСт Π΄Π°ΠΆΠ΅ Π² галактичСском ΠΌΠ°ΡΡˆΡ‚Π°Π±Π΅. Π“Ρ€Π°Π²ΠΈΡ‚Π°Ρ†ΠΈΠΎΠ½Π½ΠΎΠ΅ притяТСниС проходящих ΠΏΠ»Π°Π½Π΅Ρ‚ Π²ΠΎ ΠΌΠ½ΠΎΠ³ΠΎΠΌ ΠΏΠΎΡ…ΠΎΠΆΠ΅ Π½Π° элСктростатичСскиС силы ΠΌΠ΅ΠΆΠ΄Ρƒ проходящими Π°Ρ‚ΠΎΠΌΠ°ΠΌΠΈ. ΠšΠΎΠ³Π΅Ρ€Π΅Π½Ρ‚Π½ΠΎΠ΅ Π΄Π²ΠΈΠΆΠ΅Π½ΠΈΠ΅ Π³Ρ€ΡƒΠΏΠΏ ΠΏΠ»Π°Π½Π΅Ρ‚ Π² ΠΊΠΎΠ½Π΅Ρ‡Π½ΠΎΠΌ ΠΈΡ‚ΠΎΠ³Π΅ ΠΏΠ΅Ρ€Π΅ΠΉΠ΄Π΅Ρ‚ Π² бСспорядочноС Π΄Π²ΠΈΠΆΠ΅Π½ΠΈΠ΅ ΠΎΡ‚Π΄Π΅Π»ΡŒΠ½Ρ‹Ρ… ΠΏΠ»Π°Π½Π΅Ρ‚.
ΠšΠΎΡΡ„Ρ„ΠΈΡ†ΠΈΠ΅Π½Ρ‚Ρ‹ трСния для Π²Ρ‹Π±Ρ€Π°Π½Π½Ρ‹Ρ… повСрхностСй Ρ€Π°Π·Π΄Π΅Π»Π° (Π² порядкС ΠΎΠ±Ρ‰Π΅Π³ΠΎ ΡƒΠΌΠ΅Π½ΡŒΡˆΠ΅Π½ΠΈΡ значСния)
ΠΌΠΊΠΌ с ΠΌΠΊΠΌ ΠΊ интСрфСйс
1.16 каучук– ΠΊΠ°ΡƒΡ‡ΡƒΠΊ
1,02 каучук– Π±Π΅Ρ‚ΠΎΠ½
0,84–0,98 0,72 Π°Π²Ρ‚ΠΎΠΌΠΎΠ±ΠΈΠ»ΡŒΠ½Π°Ρ ΡˆΠΈΠ½Π°β€“ Π°ΡΡ„Π°Π»ΡŒΡ‚
0,35 Π°Π²Ρ‚ΠΎΠΌΠΎΠ±ΠΈΠ»ΡŒΠ½Π°Ρ ΡˆΠΈΠ½Π°β€“ Ρ‚Ρ€Π°Π²Π°
0,78–1,00 скин– ΠΌΠ΅Ρ‚Π°Π»Π»ΠΎΠ²
0.9–1,0 стСкло– стСкло
0,9 овСц– ΡΡ‚Π°Π»ΡŒΠ½Π°Ρ сСтка
0,7 овСц– пластиковая Ρ€Π΅ΠΉΠΊΠ° (βŠ₯)
0,6 овСц– пластиковая Ρ€Π΅ΠΉΠΊΠ° (βˆ₯)
0,6 овСц– дСрСвянная Ρ€Π΅ΠΉΠΊΠ° (βŠ₯)
0.5 овСц– дСрСвянная Ρ€Π΅ΠΉΠΊΠ° (βˆ₯)
0,68 0,67 подкова– ΠΊΠ°ΡƒΡ‡ΡƒΠΊ
0,48 0,47 подкова– Π±Π΅Ρ‚ΠΎΠ½
0,44 0,42 подкова– Π°ΡΡ„Π°Π»ΡŒΡ‚
0,42 0,38 подкова– Π±ΡƒΠ»Ρ‹ΠΆΠ½ΠΈΠΊ
0.58 ΡΡ‚Π°Π»ΡŒβ€“ ΡΡ‚Π°Π»ΡŒ
0,40 ΠΊΠΎΠ»ΠΎΠ΄ΠΊΠΈ тормозныС– Ρ‡ΡƒΠ³ΡƒΠ½
0,30–0,70 ΡƒΡ‚ΡŽΠ³β€“ камСнь
0,62 дСрСво– Π±Π΅Ρ‚ΠΎΠ½
0,6 дСрСво– ΠΊΠΈΡ€ΠΏΠΈΡ‡
0,40 дСрСво– камСнь
0.2–0,6 дСрСво– ΠΌΠ΅Ρ‚Π°Π»Π»ΠΎΠ²
0,29 0,22 дСрСво– Π²ΠΎΠΉΠ»ΠΎΠΊ
0,28 0,17 дСрСво– Π΄Π΅Ρ€Π΅Π²ΠΎ
0,56 коТа– ΠΌΠ΅Ρ‚Π°Π»Π»ΠΎΠ²
0,27–0,38 коТа– Π΄Π΅Ρ€Π΅Π²ΠΎ
0.225 ΠΊΠΎΠΆΡƒΡ€Π° мандарина– Π»ΠΈΠ½ΠΎΠ»Π΅ΡƒΠΌ ΠΏΠΎΠ»
0,125 яблочная коТура– Π»ΠΈΠ½ΠΎΠ»Π΅ΡƒΠΌ ΠΏΠΎΠ»
0,066 банановая коТура– Π»ΠΈΠ½ΠΎΠ»Π΅ΡƒΠΌ ΠΏΠΎΠ»
0,3 снСг– Π½Π΅ΠΉΠ»ΠΎΠ½
0,04–0,4 0,04–0,4 снСг– Π³ΠΈΠΊΠΎΡ€ΠΈ, Π²ΠΎΡ‰Π΅Π½Ρ‹ΠΉ
0.1 графит– Π³Ρ€Π°Ρ„ΠΈΡ‚
0,1 графит– ΡΡ‚Π°Π»ΡŒ
0,05–0,08 тСфлон– ΡΡ‚Π°Π»ΡŒ
0,32–0,36 тСфлон– Ρ‚Π΅Ρ„Π»ΠΎΠ½ (189 см / с)
0,05–0,08 тСфлон– Ρ‚Π΅Ρ„Π»ΠΎΠ½ (1,1 см / с)
0.03 лСд– ΡΡ‚Π°Π»ΡŒ
0,05–0,5 0,02–0,09 лСд– Π»Π΅Π΄
0,0044–0,0057 хрящ– синовиальная ΠΆΠΈΠ΄ΠΊΠΎΡΡ‚ΡŒ
0,0013 сухоТилиС– ΠΎΠ±ΠΎΠ»ΠΎΡ‡ΠΊΠ°

ΠšΠΎΡΡ„Ρ„ΠΈΡ†ΠΈΠ΅Π½Ρ‚ трСния — ΠžΠΏΡ€Π΅Π΄Π΅Π»Π΅Π½ΠΈΠ΅ | Ρ„ΠΎΡ€ΠΌΡƒΠ»Ρ‹ | ΠŸΡ€ΠΈΠΌΠ΅Ρ€Ρ‹

ΠšΠΎΡΡ„Ρ„ΠΈΡ†ΠΈΠ΅Π½Ρ‚ трСния — это постоянная Π²Π΅Π»ΠΈΡ‡ΠΈΠ½Π°, ΠΎΠΏΡ€Π΅Π΄Π΅Π»ΡΡŽΡ‰Π°Ρ сопротивлСниС двиТСнию ΠΈΠ·-Π·Π° трСния.Π•Π³ΠΎ Ρ‚Π°ΠΊΠΆΠ΅ ΠΌΠΎΠΆΠ½ΠΎ ΠΎΠΏΡ€Π΅Π΄Π΅Π»ΠΈΡ‚ΡŒ ΠΊΠ°ΠΊ ΠΎΡ‚Π½ΠΎΡˆΠ΅Π½ΠΈΠ΅ силы трСния ΠΊ Π½ΠΎΡ€ΠΌΠ°Π»ΡŒΠ½ΠΎΠΉ силС. Π­Ρ‚ΠΎ Π±Π΅Π·Ρ€Π°Π·ΠΌΠ΅Ρ€Π½Ρ‹ΠΉ коэффициСнт, ΠΊΠΎΡ‚ΠΎΡ€Ρ‹ΠΉ позволяСт Π½Π°ΠΌ ΡΡ€Π°Π²Π½ΠΈΡ‚Π΅Π»ΡŒΠ½ΠΎ ΠΎΠΏΡ€Π΅Π΄Π΅Π»ΠΈΡ‚ΡŒ Ρ‚Ρ€Π΅Π½ΠΈΠ΅ ΠΌΠ΅ΠΆΠ΄Ρƒ ΠΏΠ°Ρ€ΠΎΠΉ ΠΌΠ°Ρ‚Π΅Ρ€ΠΈΠ°Π»ΠΎΠ². Π­Ρ‚ΠΎ скалярная Π²Π΅Π»ΠΈΡ‡ΠΈΠ½Π°.

Π—Π½Π°Ρ‡Π΅Π½ΠΈΠ΅ коэффициСнта трСния Ρ€Π°Π·Π½ΠΎΠ΅ для ΠΊΠ°ΠΆΠ΄ΠΎΠΉ ΠΏΠ°Ρ€Ρ‹ ΠΏΡ€Π΅Π΄ΠΌΠ΅Ρ‚ΠΎΠ². Π­Ρ‚ΠΎ ΠΎΠ·Π½Π°Ρ‡Π°Π΅Ρ‚, Ρ‡Ρ‚ΠΎ Ρ‚Ρ€Π΅Π½ΠΈΠ΅ зависит ΠΊΠ°ΠΊ ΠΎΡ‚ ΡΠΊΠΎΠ»ΡŒΠ·ΡΡ‰Π΅Π³ΠΎ ΠΎΠ±ΡŠΠ΅ΠΊΡ‚Π°, Ρ‚Π°ΠΊ ΠΈ ΠΎΡ‚ ΡΠΊΠΎΠ»ΡŒΠ·ΡΡ‰Π΅ΠΉ повСрхности. ΠŸΠΎΠ΄Ρ€ΠΎΠ±Π½Π΅Π΅ Ρ‡ΠΈΡ‚Π°ΠΉΡ‚Π΅ Π² нашСм постС ΠΎ Ρ„Π°ΠΊΡ‚ΠΎΡ€Π°Ρ…, Π²Π»ΠΈΡΡŽΡ‰ΠΈΡ… Π½Π° Ρ‚Ρ€Π΅Π½ΠΈΠ΅. Π—Π½Π°Ρ‡Π΅Π½ΠΈΠ΅ коэффициСнта трСния колСблСтся ΠΎΡ‚ нуля Π΄ΠΎ Π½Π΅ΠΌΠ½ΠΎΠ³ΠΎ большС Π΅Π΄ΠΈΠ½ΠΈΡ†Ρ‹.Π§Π΅ΠΌ Π²Ρ‹ΡˆΠ΅ ΡˆΠ΅Ρ€ΠΎΡ…ΠΎΠ²Π°Ρ‚ΠΎΡΡ‚ΡŒ, Ρ‚Π΅ΠΌ Π²Ρ‹ΡˆΠ΅ Π·Π½Π°Ρ‡Π΅Π½ΠΈΠ΅.

Π€ΠΎΡ€ΠΌΡƒΠ»Π° коэффициСнта трСния

ΞΌ = f / N, Π³Π΄Π΅ ΞΌ — коэффициСнт трСния, f — сила трСния, Π° N — Π½ΠΎΡ€ΠΌΠ°Π»ΡŒΠ½Π°Ρ сила, Π΄Π΅ΠΉΡΡ‚Π²ΡƒΡŽΡ‰Π°Ρ Π² Ρ‚ΠΎΡ‡ΠΊΠ΅ ΠΊΠΎΠ½Ρ‚Π°ΠΊΡ‚Π°.

ΠšΠ»Π°ΡΡΠΈΡ„ΠΈΠΊΠ°Ρ†ΠΈΡ коэффициСнта трСния

Π—Π½Π°Ρ‡Π΅Π½ΠΈΠ΅ коэффициСнта трСния Π±ΡƒΠ΄Π΅Ρ‚ Ρ€Π°Π·Π»ΠΈΡ‡Π½Ρ‹ΠΌ для Ρ€Π°Π·Π½Ρ‹Ρ… случаСв, ΠΊΠΎΡ‚ΠΎΡ€Ρ‹Π΅ ΠΌΠΎΠΆΠ½ΠΎ ΠΊΠ»Π°ΡΡΠΈΡ„ΠΈΡ†ΠΈΡ€ΠΎΠ²Π°Ρ‚ΡŒ ΡΠ»Π΅Π΄ΡƒΡŽΡ‰ΠΈΠΌ ΠΎΠ±Ρ€Π°Π·ΠΎΠΌ.

ΠšΠΎΡΡ„Ρ„ΠΈΡ†ΠΈΠ΅Π½Ρ‚ кинСтичСского трСния всСгда мСньшС ΠΏΡ€Π΅Π΄Π΅Π»ΡŒΠ½ΠΎΠ³ΠΎ трСния.

Π—Π½Π°Ρ‡Π΅Π½ΠΈΠ΅ коэффициСнта статичСского трСния рСгулируСтся автоматичСски.

ΠšΠΎΡΡ„Ρ„ΠΈΡ†ΠΈΠ΅Π½Ρ‚ трСния скольТСния всСгда большС, Ρ‡Π΅ΠΌ Π·Π½Π°Ρ‡Π΅Π½ΠΈΠ΅ коэффициСнта трСния качСния

Π“Ρ€Π°Ρ„ΠΈΠΊ трСния, ΠΏΠΎΠΊΠ°Π·Ρ‹Π²Π°ΡŽΡ‰ΠΈΠΉ области статичСского ΠΈ кинСтичСского трСния

Π§Ρ‚ΠΎ Ρ‚Π°ΠΊΠΎΠ΅ коэффициСнт ΠΏΡ€Π΅Π΄Π΅Π»ΡŒΠ½ΠΎΠ³ΠΎ трСния?

ΠŸΡ€Π΅Π΄Π΅Π»ΡŒΠ½Ρ‹ΠΉ коэффициСнт трСния — это максимальноС Π·Π½Π°Ρ‡Π΅Π½ΠΈΠ΅, ΠΊΠΎΡ‚ΠΎΡ€ΠΎΠ³ΠΎ ΠΌΠΎΠΆΠ΅Ρ‚ Π΄ΠΎΡΡ‚ΠΈΡ‡ΡŒ коэффициСнт, ΠΊΠΎΠ³Π΄Π° Ρ‚Π΅Π»ΠΎ находится Π² состоянии покоя. Π­Ρ‚ΠΎ ΠΌΠΎΠΆΠ½ΠΎ ΠΏΠΎΠ½ΡΡ‚ΡŒ, посмотрСв Π½Π° Π³Ρ€Π°Ρ„ΠΈΠΊ трСния.Π’ случаС трСния Π½Π° склонС Π΅Π³ΠΎ ΠΌΠΎΠΆΠ½ΠΎ Ρ€Π°ΡΡΡ‡ΠΈΡ‚Π°Ρ‚ΡŒ ΠΊΠ°ΠΊ тангСнс ΡƒΠ³Π»Π° трСния.

Π§Ρ‚ΠΎ Ρ‚Π°ΠΊΠΎΠ΅ Π·Π°ΠΊΠΎΠ½ трСния Амонтона?

Π—Π°ΠΊΠΎΠ½ трСния Амонтона гласит, Ρ‡Ρ‚ΠΎ сила трСния прямо ΠΏΡ€ΠΎΠΏΠΎΡ€Ρ†ΠΈΠΎΠ½Π°Π»ΡŒΠ½Π° Π½ΠΎΡ€ΠΌΠ°Π»ΡŒΠ½ΠΎΠΉ Π½Π°Π³Ρ€ΡƒΠ·ΠΊΠ΅. Π­Ρ‚ΠΎ ΠΎΠ·Π½Π°Ρ‡Π°Π΅Ρ‚, Ρ‡Ρ‚ΠΎ ΠΎΡ‚Π½ΠΎΡˆΠ΅Π½ΠΈΠ΅ силы трСния ΠΊ Π½ΠΎΡ€ΠΌΠ°Π»ΡŒΠ½ΠΎΠΉ силС постоянно. Π­Ρ‚ΠΎ классичСская вСрсия Π·Π°ΠΊΠΎΠ½Π° трСния, данная Π“ΠΈΠΉΠΎΠΌΠΎΠΌ Амонтоном Π² 1699 Π³ΠΎΠ΄Ρƒ.

Π’Ρ‚ΠΎΡ€ΠΎΠΉ Π·Π°ΠΊΠΎΠ½ Амонтона гласит, Ρ‡Ρ‚ΠΎ Ρ‚Ρ€Π΅Π½ΠΈΠ΅ Π½Π΅ зависит ΠΎΡ‚ ΠΏΠ»ΠΎΡ‰Π°Π΄ΠΈ ΠΊΠΎΠ½Ρ‚Π°ΠΊΡ‚Π°.УпомянутоС здСсь Ρ‚Ρ€Π΅Π½ΠΈΠ΅ Π½Π° самом Π΄Π΅Π»Π΅ являСтся сухим Ρ‚Ρ€Π΅Π½ΠΈΠ΅ΠΌ.

ΠŸΡ€ΠΈΠΌΠ΅Ρ€Ρ‹ коэффициСнта трСния

ΠŸΡ€ΠΈΠΌΠ΅Ρ€Ρ‹ высокого коэффициСнта трСния
  • БкольТСниС ΠΏΠΎ ΡˆΠ΅Ρ€ΠΎΡ…ΠΎΠ²Π°Ρ‚ΠΎΠΉ дСрСвянной повСрхности
  • Π Π΅Π·ΠΈΠ½Π° Π½Π° Π±ΡƒΠΌΠ°Π³Π΅
  • Π₯ΠΎΠΆΠ΄Π΅Π½ΠΈΠ΅ Π² ΠΎΠ±ΡƒΠ²ΠΈ с высоким сцСплСниСм ΠΏΠΎ Ρ‚Ρ€ΠΎΡ‚ΡƒΠ°Ρ€Ρƒ
  • наТдачная Π±ΡƒΠΌΠ°Π³Π° ΠΏΠΎ дСрСвянной повСрхности
  • Π—Π°Ρ‚ΠΎΡ‡ΠΊΠ° ΡƒΡ‚ΡŽΠ³Π° ΠΏΠΎ ΠΌΠ΅Π΄ΠΈ
Π—Π°Ρ‚ΠΎΡ‡ΠΊΠ° Π½ΠΎΠΆΠ° ΠΏΡƒΡ‚Π΅ΠΌ трСния ΠΎ Π΄Ρ€ΡƒΠ³ΡƒΡŽ ΠΌΠ΅Ρ‚Π°Π»Π»ΠΈΡ‡Π΅ΡΠΊΡƒΡŽ ΠΏΠΎΠ²Π΅Ρ€Ρ…Π½ΠΎΡΡ‚ΡŒ для создания трСния Π˜Π·ΠΎΠ±Ρ€Π°ΠΆΠ΅Π½ΠΈΠ΅ Ben Kerckx с сайта Pixabay

ΠŸΡ€ΠΈΠΌΠ΅Ρ€Ρ‹ Π½ΠΈΠ·ΠΊΠΎΠ³ΠΎ коэффициСнта трСния
  • Π₯одьба ΠΏΠΎ Π»ΡŒΠ΄Ρƒ
  • Π‘ΠΌΠ°Π·ΠΊΠ° Π½Π° стали
  • Масло Π½Π° дСрСвянных повСрхностях
  • Π₯одьба ΠΏΠΎ Π±ΠΎΠ»ΠΎΡ‚Π°ΠΌ

Π₯одьба ΠΏΠΎ Ρ‚ΠΎΠ½ΠΊΠΎΠΌΡƒ Π»ΡŒΠ΄Ρƒ — мСтафоричСскоС ΠΏΡ€Π΅Π΄ΡƒΠΏΡ€Π΅ΠΆΠ΄Π΅Π½ΠΈΠ΅, ΠΊΠΎΡ‚ΠΎΡ€ΠΎΠ΅ ΠΎΠ·Π½Π°Ρ‡Π°Π΅Ρ‚, Ρ‡Ρ‚ΠΎ Π²Ρ‹ Π½Π°Ρ…ΠΎΠ΄ΠΈΡ‚Π΅ΡΡŒ Π² опасности ΠΈΠ»ΠΈ рискуСтС Ρ‡Π΅ΠΌ-Ρ‚ΠΎ.ИмСнно ΠΈΠ·-Π·Π° Ρ‚Π°ΠΊΠΎΠ³ΠΎ Π½ΠΈΠ·ΠΊΠΎΠ³ΠΎ коэффициСнта трСния ΠΌΠ΅ΠΆΠ΄Ρƒ нашими Π½ΠΎΠ³Π°ΠΌΠΈ ΠΈ трСния ΠΌΡ‹ ΠΏΠΎΡΠΊΠΎΠ»ΡŒΠ·Π½ΡƒΠ»ΠΈΡΡŒ ΠΈ ΠΏΠ°Π΄Π°Π»ΠΈ.

ΠŸΡ€ΠΈΠΌΠ΅Ρ€ Π½ΠΈΠ·ΠΊΠΎΠ³ΠΎ коэффициСнта трСния: ΠΏΡ‚ΠΈΡ†Π° Π˜Ρ€ΠΎΠ΄ ΠΈΠ΄Π΅Ρ‚ ΠΏΠΎ Π»ΡŒΠ΄Ρƒ. Π˜Π·ΠΎΠ±Ρ€Π°ΠΆΠ΅Π½ΠΈΠ΅ Mabel Amber с сайта Pixabay

Π—Π΄Π΅ΡΡŒ Π²Ρ‹ ΠΌΠΎΠΆΠ΅Ρ‚Π΅ ΡƒΠ²ΠΈΠ΄Π΅Ρ‚ΡŒ значСния коэффициСнта статичСского ΠΈ кинСтичСского трСния для Π½Π°ΠΈΠ±ΠΎΠ»Π΅Π΅ часто ΠΈΡΠΏΠΎΠ»ΡŒΠ·ΡƒΠ΅ΠΌΡ‹Ρ… ΠΏΠ°Ρ€ ΠΌΠ°Ρ‚Π΅Ρ€ΠΈΠ°Π»ΠΎΠ².

Π’Ρ‹ ΠΌΠΎΠ³Π»ΠΈ Π·Π°ΠΌΠ΅Ρ‚ΠΈΡ‚ΡŒ, Ρ‡Ρ‚ΠΎ смазочныС ΠΌΠ°Ρ‚Π΅Ρ€ΠΈΠ°Π»Ρ‹ ΠΈ скользкиС ΠΌΠ°Ρ‚Π΅Ρ€ΠΈΠ°Π»Ρ‹ ΠΈΠΌΠ΅ΡŽΡ‚ коэффициСнт статичСского трСния всСго 0.05.

ΠšΠΎΠΌΠ±ΠΈΠ½Π°Ρ†ΠΈΡ Π³Ρ€ΡƒΠ±ΠΎΠ³ΠΎ ΠΈ Π³Π»Π°Π΄ΠΊΠΎΠ³ΠΎ ΠΌΠ°Ρ‚Π΅Ρ€ΠΈΠ°Π»Π° Π΄Π°Π΅Ρ‚ ΡƒΠΌΠ΅Ρ€Π΅Π½Π½ΠΎΠ΅ Π·Π½Π°Ρ‡Π΅Π½ΠΈΠ΅. ΠŸΡ€ΠΈΠΌΠ΅Ρ€Ρ‹ Π²ΠΊΠ»ΡŽΡ‡Π°ΡŽΡ‚ алюминий Π½Π° снСгу, стСкло Π½Π° Π½ΠΈΠΊΠ΅Π»Π΅ ΠΈ ΠΊΠΎΠΆΡƒ Π½Π° ΠΆΠ΅Π»Π΅Π·Π΅.

И Π±ΠΎΠ»Π΅Π΅ Π³Ρ€ΡƒΠ±Ρ‹Π΅ ΠΌΠ°Ρ‚Π΅Ρ€ΠΈΠ°Π»Ρ‹, Ρ‚Π°ΠΊΠΈΠ΅ ΠΊΠ°ΠΊ ΠΆΠ΅Π»Π΅Π·ΠΎ Π½Π° ΠΌΠ΅Π΄ΠΈ, ΠΈΠΌΠ΅ΡŽΡ‚ Π±ΠΎΠ»Π΅Π΅ высокиС значСния, Π½Π°ΠΏΡ€ΠΈΠΌΠ΅Ρ€ ~ 1. Π”Ρ€ΡƒΠ³ΠΈΠ΅ ΠΏΡ€ΠΈΠΌΠ΅Ρ€Ρ‹ Π²ΠΊΠ»ΡŽΡ‡Π°ΡŽΡ‚ Π»Π°Ρ‚ΡƒΠ½ΡŒ Π½Π° стали, ΠΌΠ΅Ρ‚Π°Π»Π» Π½Π° ΠΌΠ΅Ρ‚Π°Π»Π»Π΅, оргстСкло Π½Π° оргстСклС ΠΈ Ρ‚. Π”.

Π’Ρ‹ Ρ‚Π°ΠΊΠΆΠ΅ ΠΌΠΎΠ³Π»ΠΈ Π·Π°ΠΌΠ΅Ρ‚ΠΈΡ‚ΡŒ, Ρ‡Ρ‚ΠΎ состояниС повСрхности ΠΈ Ρ‚Π΅ΠΌΠΏΠ΅Ρ€Π°Ρ‚ΡƒΡ€Π° Ρ‚Π°ΠΊΠΆΠ΅ Π²Π»ΠΈΡΡŽΡ‚ Π½Π° коэффициСнт трСния. Условия повСрхности, Ρ‚Π°ΠΊΠΈΠ΅ ΠΊΠ°ΠΊ чистая ΠΈΠ»ΠΈ сухая, ΠΆΠΈΡ€ ΠΈΠ»ΠΈ смазка, ΠΌΠΎΠ³ΡƒΡ‚ Π·Π½Π°Ρ‡ΠΈΡ‚Π΅Π»ΡŒΠ½ΠΎ ΠΏΠΎΠ²Π»ΠΈΡΡ‚ΡŒ Π½Π° Π·Π½Π°Ρ‡Π΅Π½ΠΈΠ΅ коэффициСнта трСния.Π’Π΅ΠΌΠΏΠ΅Ρ€Π°Ρ‚ΡƒΡ€Π° влияСт Π½Π° Ρ‚Π°ΠΊΠΈΠ΅ ΠΌΠ°Ρ‚Π΅Ρ€ΠΈΠ°Π»Ρ‹, ΠΊΠ°ΠΊ Π²ΠΎΠ΄Π°, которая ΠΏΡ€ΠΈ Π½ΠΈΠ·ΠΊΠΈΡ… Ρ‚Π΅ΠΌΠΏΠ΅Ρ€Π°Ρ‚ΡƒΡ€Π°Ρ… мСняСт состояниС вСщСства.

Какой ΠΏΡ€ΠΈΠ±ΠΎΡ€ ΠΈΡΠΏΠΎΠ»ΡŒΠ·ΡƒΠ΅Ρ‚ΡΡ для измСрСния коэффициСнта трСния? Π’Ρ€ΠΈΠ±ΠΎΠΌΠ΅Ρ‚Ρ€Ρ‹

ΠΈΡΠΏΠΎΠ»ΡŒΠ·ΡƒΡŽΡ‚ΡΡ для измСрСния коэффициСнта трСния ΠΈ Π΄Ρ€ΡƒΠ³ΠΈΡ… связанных с Ρ‚Ρ€Π΅Π½ΠΈΠ΅ΠΌ свойств, Ρ‚Π°ΠΊΠΈΡ… ΠΊΠ°ΠΊ ΡΠΊΠΎΡ€ΠΎΡΡ‚ΡŒ износа ΠΈ Ρ‚. Π”. Π’ настоящСС врСмя доступны ΠΌΠΎΡ‚ΠΎΡ€ΠΈΠ·ΠΎΠ²Π°Π½Π½Ρ‹Π΅ Ρ‚Ρ€ΠΈΠ±ΠΎΠΌΠ΅Ρ‚Ρ€Ρ‹, ΠΊΠΎΡ‚ΠΎΡ€Ρ‹Π΅ ΠΌΠΎΠ³ΡƒΡ‚ ΠΈΠ·ΠΌΠ΅Ρ€ΡΡ‚ΡŒ динамичСскиС значСния трСния.

ΠšΠΎΡΡ„Ρ„ΠΈΡ†ΠΈΠ΅Π½Ρ‚ трСния Ρ‚Π°ΠΊΠΆΠ΅ ΠΌΠΎΠΆΠ½ΠΎ Ρ€Π°ΡΡΡ‡ΠΈΡ‚Π°Ρ‚ΡŒ, ΠΈΡΠΏΠΎΠ»ΡŒΠ·ΡƒΡ основныС классичСскиС Π·Π°ΠΊΠΎΠ½Ρ‹ Ρ„ΠΈΠ·ΠΈΠΊΠΈ.НСкоторыС ΠΈΠ· ΠΌΠ΅Ρ‚ΠΎΠ΄ΠΎΠ² пСрСчислСны Π½ΠΈΠΆΠ΅.

  • ΠŸΡ€ΡƒΠΆΠΈΠ½Π° баланса
  • Наклонная ΠΏΠ»ΠΎΡΠΊΠΎΡΡ‚ΡŒ
  • Π—Π°ΠΆΠΈΠΌ
  • ΠœΠ°ΡΡ‚Π½ΠΈΠΊ

Π’Ρ‹ ΠΌΠΎΠΆΠ΅Ρ‚Π΅ ΠΏΡ€ΠΎΡ‡ΠΈΡ‚Π°Ρ‚ΡŒ эту ΡΡ‚Π°Ρ‚ΡŒΡŽ с DOI 10.24874 / PES01.01.004, Ρ‡Ρ‚ΠΎΠ±Ρ‹ ΡƒΠ·Π½Π°Ρ‚ΡŒ большС ΠΎ Ρ‚Ρ€ΠΈΠ±ΠΎΠΌΠ΅Ρ‚Ρ€Π°Ρ….

ВлияСт Π»ΠΈ вСс Π½Π° коэффициСнт трСния?

ΠšΠΎΡΡ„Ρ„ΠΈΡ†ΠΈΠ΅Π½Ρ‚ трСния Π½Π΅ зависит ΠΎΡ‚ вСса ΠΎΠ±ΡŠΠ΅ΠΊΡ‚Π°. Π­Ρ‚ΠΎ зависит Ρ‚ΠΎΠ»ΡŒΠΊΠΎ ΠΎΡ‚ ΠΏΡ€ΠΈΡ€ΠΎΠ΄Ρ‹ ΠΌΠ°Ρ‚Π΅Ρ€ΠΈΠ°Π»ΠΎΠ². НСровности Π½Π° повСрхности, заряды ΠΌΠ°Ρ‚Π΅Ρ€ΠΈΠ°Π»ΠΎΠ² ΠΈ Π΄Ρ€ΡƒΠ³ΠΈΠ΅ ΠΏΠΎΠ΄ΠΎΠ±Π½Ρ‹Π΅ Ρ„Π°ΠΊΡ‚ΠΎΡ€Ρ‹ ΠΌΠΎΠ³ΡƒΡ‚ ΠΏΠΎΠ²Π»ΠΈΡΡ‚ΡŒ Π½Π° Π·Π½Π°Ρ‡Π΅Π½ΠΈΠ΅ коэффициСнта.

Π‘ΠΌ. Π’Π°ΠΊΠΆΠ΅:

ΠšΠΎΡΡ„Ρ„ΠΈΡ†ΠΈΠ΅Π½Ρ‚ трСния — ΠΎΠΏΡ€Π΅Π΄Π΅Π»Π΅Π½ΠΈΠ΅, Ρ„ΠΎΡ€ΠΌΡƒΠ»Π° ΠΈ ΠΎΡ‚Π²Π΅Ρ‚Ρ‹ Π½Π° часто Π·Π°Π΄Π°Π²Π°Π΅ΠΌΡ‹Π΅ вопросы

Для Π΄Π΅Ρ‚Π°Π»ΡŒΠ½ΠΎΠ³ΠΎ понимания ΠΊΠΎΠ½Ρ†Π΅ΠΏΡ†ΠΈΠΈ коэффициСнта трСния ΠΌΡ‹ Π΄ΠΎΠ»ΠΆΠ½Ρ‹ Ρ…ΠΎΡ€ΠΎΡˆΠΎ Ρ€Π°Π·Π±ΠΈΡ€Π°Ρ‚ΡŒΡΡ Π² понятиях трСния, силы трСния, Ρ‚ΠΈΠΏΠ° трСния ΠΈ Ρ‚. Π”. Π’Ρ€Π΅Π½ΠΈΠ΅ ΠΌΠΎΠΆΠ½ΠΎ ΠΎΠΏΡ€Π΅Π΄Π΅Π»ΠΈΡ‚ΡŒ ΠΊΠ°ΠΊ сопротивлСниС ΠΎΡ‚Π½ΠΎΡΠΈΡ‚Π΅Π»ΡŒΠ½ΠΎΠΌΡƒ двиТСнию ΠΌΠ΅ΠΆΠ΄Ρƒ двумя ΠΊΠΎΠ½Ρ‚Π°ΠΊΡ‚ΠΈΡ€ΡƒΡŽΡ‰ΠΈΠΌΠΈ повСрхностями, Ρ‚. Π΅. Π΄Π²Π΅ повСрхности ΠΌΠΎΠ³ΡƒΡ‚ ΠΊΠΎΠ½Ρ‚Π°ΠΊΡ‚ΠΈΡ€ΠΎΠ²Π°Ρ‚ΡŒ Π²ΠΎ врСмя скольТСния, качСния ΠΈΠ»ΠΈ Π² состоянии покоя. Π’Ρ€Π΅Π½ΠΈΠ΅ ΠΎΠ±Ρ‹Ρ‡Π½ΠΎ Π²ΠΎΠ·Π½ΠΈΠΊΠ°Π΅Ρ‚ ΠΈΠ·-Π·Π° ΡˆΠ΅Ρ€ΠΎΡ…ΠΎΠ²Π°Ρ‚Ρ‹Ρ… повСрхностСй.Π”Ρ€ΡƒΠ³ΠΈΠΌΠΈ словами, Ссли повСрхности Π½Π΅ ΠΈΠ΄Π΅Π°Π»ΡŒΠ½Ρ‹ для двиТСния, Ρ‚ΠΎ ΠΎΡ‚Π½ΠΎΡΠΈΡ‚Π΅Π»ΡŒΠ½ΠΎΠ΅ Π΄Π²ΠΈΠΆΠ΅Π½ΠΈΠ΅ ΠΌΠ΅ΠΆΠ΄Ρƒ двумя повСрхностями ΠΏΡ€ΠΈΠ²ΠΎΠ΄ΠΈΡ‚ ΠΊ Ρ‚Ρ€Π΅Π½ΠΈΡŽ.

Π’Π΅Π»ΠΈΡ‡ΠΈΠ½Π° трСния всСгда прямо ΠΏΡ€ΠΎΠΏΠΎΡ€Ρ†ΠΈΠΎΠ½Π°Π»ΡŒΠ½Π° ΡˆΠ΅Ρ€ΠΎΡ…ΠΎΠ²Π°Ρ‚ΠΎΡΡ‚ΠΈ повСрхности.

Π§Π΅ΠΌ Π³Ρ€ΡƒΠ±Π΅Π΅ ΠΏΠΎΠ²Π΅Ρ€Ρ…Π½ΠΎΡΡ‚ΡŒ, Ρ‚Π΅ΠΌ большС Ρ‚Ρ€Π΅Π½ΠΈΠ΅. Если ΠΏΠΎΠ²Π΅Ρ€Ρ…Π½ΠΎΡΡ‚ΡŒ Π±ΠΎΠ»Π΅Π΅ гладкая, Ρ‚Ρ€Π΅Π½ΠΈΠ΅ Π±ΡƒΠ΄Π΅Ρ‚ ΠΎΡ‚Π½ΠΎΡΠΈΡ‚Π΅Π»ΡŒΠ½ΠΎ мСньшС.

Π‘ΠΈΠ»Π° трСния ΠΈΠ»ΠΈ Π‘ΠΈΠ»Π° трСния:

Π‘ΠΈΠ»Π° трСния являСтся Π²Π΅ΠΊΡ‚ΠΎΡ€Π½ΠΎΠΉ Π²Π΅Π»ΠΈΡ‡ΠΈΠ½ΠΎΠΉ, поэтому ΠΎΠ½Π° ΠΈΠΌΠ΅Π΅Ρ‚ ΠΊΠ°ΠΊ Π²Π΅Π»ΠΈΡ‡ΠΈΠ½Ρƒ, Ρ‚Π°ΠΊ ΠΈ Π½Π°ΠΏΡ€Π°Π²Π»Π΅Π½ΠΈΠ΅.Π‘ΠΈΠ»Π° трСния — это сила Ρ€Π΅Π°ΠΊΡ†ΠΈΠΈ. Он сущСствуСт ΠΊΠ°ΠΊ рСакция Π΄Ρ€ΡƒΠ³ΠΎΠΉ силы, Π΄Π΅ΠΉΡΡ‚Π²ΡƒΡŽΡ‰Π΅ΠΉ Π½Π° Ρ‚Π΅Π»ΠΎ, ΠΌΡ‹ ΠΌΠΎΠΆΠ΅ΠΌ ΡΠΊΠ°Π·Π°Ρ‚ΡŒ, Ρ‡Ρ‚ΠΎ сила трСния Π½Π΅ ΠΌΠΎΠΆΠ΅Ρ‚ ΡΡƒΡ‰Π΅ΡΡ‚Π²ΠΎΠ²Π°Ρ‚ΡŒ Π½Π°ΠΏΡ€ΡΠΌΡƒΡŽ. ΠšΡ€ΠΎΠΌΠ΅ Ρ‚ΠΎΠ³ΠΎ, сила трСния — это сила сопротивлСния.

Π€ΠΎΡ€ΠΌΡƒΠ»Π° коэффициСнта трСния:

ΠšΠΎΡΡ„Ρ„ΠΈΡ†ΠΈΠ΅Π½Ρ‚ трСния — это ΠΌΠ΅Ρ€Π° стСпСни взаимодСйствия ΠΌΠ΅ΠΆΠ΄Ρƒ двумя повСрхностями, связанная с Ρ‚Ρ€Π΅Π½ΠΈΠ΅ΠΌ ΠΌΠ΅ΠΆΠ΄Ρƒ двумя повСрхностями. Π—Π½Π°Ρ‡Π΅Π½ΠΈΠ΅ коэффициСнта трСния ΠΏΠΎΠΌΠΎΠΆΠ΅Ρ‚ ΠΎΠΏΡ€Π΅Π΄Π΅Π»ΠΈΡ‚ΡŒ ΡˆΠ΅Ρ€ΠΎΡ…ΠΎΠ²Π°Ρ‚ΠΎΡΡ‚ΡŒ повСрхности.

Π‘ΠΈΠ»Π° трСния зависит ΠΎΡ‚ ΠΈΠ»ΠΈ сила трСния прямо ΠΏΡ€ΠΎΠΏΠΎΡ€Ρ†ΠΈΠΎΠ½Π°Π»ΡŒΠ½Π° Π½ΠΎΡ€ΠΌΠ°Π»ΡŒΠ½ΠΎΠΉ силС, Π΄Π΅ΠΉΡΡ‚Π²ΡƒΡŽΡ‰Π΅ΠΉ Π½Π° любой ΠΎΠ±ΡŠΠ΅ΠΊΡ‚ наблюдСния.ΠœΠ°Ρ‚Π΅ΠΌΠ°Ρ‚ΠΈΡ‡Π΅ΡΠΊΠΈ ΠΌΡ‹ ΠΌΠΎΠΆΠ΅ΠΌ Π·Π°ΠΏΠΈΡΠ°Ρ‚ΡŒ:

β‡’F \ [_ {r} \] Ξ± N …… (1)

Рассмотрим Π±Π»ΠΎΠΊ массы m, располоТСнный Π½Π° Π³ΠΎΡ€ΠΈΠ·ΠΎΠ½Ρ‚Π°Π»ΡŒΠ½ΠΎΠΉ ΡˆΠ΅Ρ€ΠΎΡ…ΠΎΠ²Π°Ρ‚ΠΎΠΉ повСрхности (это ΠΌΠΎΠΆΠ΅Ρ‚ Π±Ρ‹Ρ‚ΡŒ Π΄Π°ΠΆΠ΅ наклонная ΠΏΠΎΠ²Π΅Ρ€Ρ…Π½ΠΎΡΡ‚ΡŒ). ВСс ΠΎΠ±ΡŠΠ΅ΠΊΡ‚Π° дСйствуСт Π²Π½ΠΈΠ·, ΠΈ ΠΏΠΎΡΠΊΠΎΠ»ΡŒΠΊΡƒ Π±Π»ΠΎΠΊ находится Π² ΠΊΠΎΠ½Ρ‚Π°ΠΊΡ‚Π΅ с ΠΏΠΎΠ²Π΅Ρ€Ρ…Π½ΠΎΡΡ‚ΡŒΡŽ, Π½Π° Π½Π΅Π³ΠΎ Π±ΡƒΠ΄Π΅Ρ‚ Π΄Π΅ΠΉΡΡ‚Π²ΠΎΠ²Π°Ρ‚ΡŒ Π½ΠΎΡ€ΠΌΠ°Π»ΡŒΠ½Π°Ρ сила, Π΄Π΅ΠΉΡΡ‚Π²ΡƒΡŽΡ‰Π°Ρ Π² Π½Π°ΠΏΡ€Π°Π²Π»Π΅Π½ΠΈΠΈ, ΠΏΡ€ΠΎΡ‚ΠΈΠ²ΠΎΠΏΠΎΠ»ΠΎΠΆΠ½ΠΎΠΌ вСсу ΠΎΠ±ΡŠΠ΅ΠΊΡ‚Π°. ΠŸΡ€Π΅Π΄ΠΏΠΎΠ»ΠΎΠΆΠΈΠΌ, Ρ‡Ρ‚ΠΎ Π±Π»ΠΎΠΊ двиТСтся Π² ΠΏΡ€Π°Π²ΠΎΠΌ Π½Π°ΠΏΡ€Π°Π²Π»Π΅Π½ΠΈΠΈ, Ρ‚ΠΎΠ³Π΄Π° ΡΠΎΠΎΡ‚Π²Π΅Ρ‚ΡΡ‚Π²ΡƒΡŽΡ‰Π°Ρ сила трСния Π±ΡƒΠ΄Π΅Ρ‚ Π΄Π΅ΠΉΡΡ‚Π²ΠΎΠ²Π°Ρ‚ΡŒ Π²Π»Π΅Π²ΠΎ.

[Π˜Π·ΠΎΠ±Ρ€Π°ΠΆΠ΅Π½ΠΈΠ΅ Π±ΡƒΠ΄Π΅Ρ‚ Π·Π°Π³Ρ€ΡƒΠΆΠ΅Π½ΠΎ Π² блиТайшСС врСмя]

ΠœΡ‹ Π·Π½Π°Π΅ΠΌ, Ρ‡Ρ‚ΠΎ сила трСния прямо ΠΏΡ€ΠΎΠΏΠΎΡ€Ρ†ΠΈΠΎΠ½Π°Π»ΡŒΠ½Π° Π½ΠΎΡ€ΠΌΠ°Π»ΡŒΠ½ΠΎΠΉ силС, Π΄Π΅ΠΉΡΡ‚Π²ΡƒΡŽΡ‰Π΅ΠΉ Π½Π° Π½Π΅Π΅, ΠΈ ΠΏΡ€ΠΎΠΏΠΎΡ€Ρ†ΠΈΠΎΠ½Π°Π»ΡŒΠ½ΠΎΡΡ‚ΡŒ замСняСтся Π²Π²Π΅Π΄Π΅Π½ΠΈΠ΅ΠΌ константы ΠΏΡ€ΠΎΠΏΠΎΡ€Ρ†ΠΈΠΎΠ½Π°Π»ΡŒΠ½ΠΎΡΡ‚ΠΈ, извСстной ΠΊΠ°ΠΊ коэффициСнт трСния.

ΠœΠ°Ρ‚Π΅ΠΌΠ°Ρ‚ΠΈΡ‡Π΅ΡΠΊΠΈ запишСм,

β‡’F \ [_ {r} \] = ΞΌN …… .. (2)

Π“Π΄Π΅,

ΞΌ — коэффициСнт трСния

β‡’ ΞΌ = \ [\ frac {F_ {r}} {N} \] ……… (3)

Π£Ρ€Π°Π²Π½Π΅Π½ΠΈΠ΅ (3) извСстно ΠΊΠ°ΠΊ Ρ„ΠΎΡ€ΠΌΡƒΠ»Π° коэффициСнта трСния ΠΈΠ»ΠΈ ΡƒΡ€Π°Π²Π½Π΅Π½ΠΈΠ΅ коэффициСнта трСния.Из уравнСния (3) ясно, Ρ‡Ρ‚ΠΎ сила трСния прямо ΠΏΡ€ΠΎΠΏΠΎΡ€Ρ†ΠΈΠΎΠ½Π°Π»ΡŒΠ½Π° коэффициСнту трСния. Если коэффициСнт трСния большС, Ρ‚ΠΎ ΡΠΎΠΎΡ‚Π²Π΅Ρ‚ΡΡ‚Π²ΡƒΡŽΡ‰Π°Ρ сила трСния Ρ‚Π°ΠΊΠΆΠ΅ Π±ΡƒΠ΄Π΅Ρ‚ большС.

Π€ΠΎΡ€ΠΌΡƒΠ»Π° статичСского трСния:

БтатичСскоС Ρ‚Ρ€Π΅Π½ΠΈΠ΅ ΠΌΠΎΠΆΠ½ΠΎ ΠΎΠΏΡ€Π΅Π΄Π΅Π»ΠΈΡ‚ΡŒ ΠΊΠ°ΠΊ Ρ‚Π΅Π½Π΄Π΅Π½Ρ†ΠΈΡŽ ΠΎΡ‚Π½ΠΎΡΠΈΡ‚Π΅Π»ΡŒΠ½ΠΎΠ³ΠΎ двиТСния, ΠΊΠΎΠ³Π΄Π° ΠΎΠ±ΡŠΠ΅ΠΊΡ‚ находится Π² состоянии покоя. Если ΠΌΠ΅ΠΆΠ΄Ρƒ двумя повСрхностями сущСствуСт Π½ΠΎΡ€ΠΌΠ°Π»ΡŒΠ½Π°Ρ сила ΠΈ ΠΌΡ‹ скользим ΠΏΠΎ ΠΎΠ±ΡŠΠ΅ΠΊΡ‚Ρƒ, Ρ‚ΠΎ сила статичСского трСния дСйствуСт Π΅Ρ‰Π΅ Π΄ΠΎ скольТСния.

Π˜Ρ‚Π°ΠΊ, матСматичСская Ρ„ΠΎΡ€ΠΌΡƒΠ»Π° статичСского трСния ΠΈΠΌΠ΅Π΅Ρ‚ ΡΠ»Π΅Π΄ΡƒΡŽΡ‰ΠΈΠΉ Π²ΠΈΠ΄:

β‡’ F \ [_ {s} \] = ΞΌ \ [_ {s} \] N

Π“Π΄Π΅,

ΞΌ \ [_ {s} \] — ΠšΠΎΡΡ„Ρ„ΠΈΡ†ΠΈΠ΅Π½Ρ‚ статичСского трСния.

β‡’ΞΌ \ [_ {s} \] = \ [\ frac {F_ {s}} {N} \] …… .. (1)

Π£Ρ€Π°Π²Π½Π΅Π½ΠΈΠ΅ (1) извСстно ΠΊΠ°ΠΊ коэффициСнт статичСского трСния. .

Π€ΠΎΡ€ΠΌΡƒΠ»Π° кинСтичСского трСния:

ΠšΠΈΠ½Π΅Ρ‚ΠΈΡ‡Π΅ΡΠΊΠΎΠ΅ Ρ‚Ρ€Π΅Π½ΠΈΠ΅ — это сила, которая сопротивляСтся ΠΎΡ‚Π½ΠΎΡΠΈΡ‚Π΅Π»ΡŒΠ½ΠΎΠΌΡƒ двиТСнию повСрхностСй, ΠΊΠΎΠ³Π΄Π° ΠΎΠ½ΠΈ Π½Π°Ρ‡ΠΈΠ½Π°ΡŽΡ‚ Π΄Π²ΠΈΠ³Π°Ρ‚ΡŒΡΡ. Π’ΠΎΠ³Π΄Π° Ρ„ΠΎΡ€ΠΌΡƒΠ»Π° кинСтичСского трСния ΠΈΠΌΠ΅Π΅Ρ‚ Π²ΠΈΠ΄:

β‡’ F \ [_ {k} \] = ΞΌ \ [_ {k} \] N

Π“Π΄Π΅,

ΞΌ \ [_ {k} \] — коэффициСнт кинСтичСского трСния

β‡’ ΞΌ \ [_ {k} \] = \ [\ frac {F_ {k}} {N} \]

Π’Ρ‹ΡˆΠ΅ΠΏΡ€ΠΈΠ²Π΅Π΄Π΅Π½Π½ΠΎΠ΅ Π²Ρ‹Ρ€Π°ΠΆΠ΅Π½ΠΈΠ΅ извСстно ΠΊΠ°ΠΊ Ρ„ΠΎΡ€ΠΌΡƒΠ»Π° коэффициСнта кинСтичСского трСния.

Π—Π½Π°Π΅Ρ‚Π΅ Π»ΠΈ Π²Ρ‹?

ΠšΠ°ΠΆΠ΄Ρ‹ΠΉ ΠΌΠ°Ρ‚Π΅Ρ€ΠΈΠ°Π» ΠΈΠΌΠ΅Π΅Ρ‚ Ρ€Π°Π·Π½Ρ‹ΠΉ коэффициСнт трСния Π² зависимости ΠΎΡ‚ ΡˆΠ΅Ρ€ΠΎΡ…ΠΎΠ²Π°Ρ‚ΠΎΡΡ‚ΠΈ Π΅Π³ΠΎ повСрхности. НапримСр, Ссли Π²Ρ‹ ΠΏΡ€ΠΎΠ²Π΅Π΄Π΅Ρ‚Π΅ стСкло ΠΏΠΎ стСклу, Π²Ρ‹ смоТСтС Π»Π΅Π³ΠΊΠΎ ΡΠΊΠΎΠ»ΡŒΠ·ΠΈΡ‚ΡŒ Π±Π΅Π· Ρ€Ρ‹Π²ΠΊΠΎΠ² ΠΏΡ€ΠΈ Π΄Π²ΠΈΠΆΠ΅Π½ΠΈΠΈ. Π’ Ρ‚ΠΎ ΠΆΠ΅ врСмя, Ссли Π²Ρ‹ ΠΏΡ€ΠΎΠ²Π΅Π΄Π΅Ρ‚Π΅ стСклом ΠΏΠΎ Π΄ΠΎΡ€ΠΎΠ³Π΅ ΠΈΠ»ΠΈ ΠΊΠ°ΠΊΠΎΠΉ-Π»ΠΈΠ±ΠΎ Π½Π΅Π·Π°Π²Π΅Ρ€ΡˆΠ΅Π½Π½ΠΎΠΉ повСрхности, Π΄Π²ΠΈΠΆΠ΅Π½ΠΈΠ΅ Π½Π΅ Π±ΡƒΠ΄Π΅Ρ‚ ΠΏΠ»Π°Π²Π½Ρ‹ΠΌ ΠΈ Π²Ρ‹ ΡƒΠ²ΠΈΠ΄ΠΈΡ‚Π΅ ΠΈΠ·ΠΌΠ΅Π½Π΅Π½ΠΈΠ΅ силы.

Π”Π°Π²Π°ΠΉΡ‚Π΅ посмотрим Π½Π° коэффициСнт трСния Π½Π΅ΡΠΊΠΎΠ»ΡŒΠΊΠΈΡ… ΠΌΠ°Ρ‚Π΅Ρ€ΠΈΠ°Π»ΠΎΠ², пСрСчислСнных Π½ΠΈΠΆΠ΅:

900 s

ΠœΠ°Ρ‚Π΅Ρ€ΠΈΠ°Π»Ρ‹

ΠšΠΎΡΡ„Ρ„ΠΈΡ†ΠΈΠ΅Π½Ρ‚ кинСтичСского трСния 𝞡k

ΠšΠΎΡΡ„Ρ„ΠΈΡ†ΠΈΠ΅Π½Ρ‚ статичСского трСния

Π‘Ρ‚Π΅ΠΊΠ»ΠΎ Π½Π° стСклС

0.4

0.94

Π›Π΅Π΄ Π½Π° Π»ΡŒΠ΄Ρƒ

0,03

0,1

Алюминий Π½Π° стали

0,47

30 900 Π‘ΠΈΠ½ΠΎΠ²ΠΈΠ°Π»ΡŒΠ½Ρ‹Π΅ суставы Ρ‡Π΅Π»ΠΎΠ²Π΅ΠΊΠ°

0,003

0,01

Π’Π°ΠΊΠΆΠ΅ доступны ΠΊΠ°Π»ΡŒΠΊΡƒΠ»ΡΡ‚ΠΎΡ€Ρ‹ коэффициСнта трСния, ΠΊΠΎΡ‚ΠΎΡ€Ρ‹Π΅ ΠΎΠ±Π»Π΅Π³Ρ‡Π°Ρ‚ наши вычислСния ΠΏΡ€ΠΈ Π²Ρ‹ΠΏΠΎΠ»Π½Π΅Π½ΠΈΠΈ числСнных расчСтов.

alexxlab

Π”ΠΎΠ±Π°Π²ΠΈΡ‚ΡŒ ΠΊΠΎΠΌΠΌΠ΅Π½Ρ‚Π°Ρ€ΠΈΠΉ

Π’Π°Ρˆ адрСс email Π½Π΅ Π±ΡƒΠ΄Π΅Ρ‚ ΠΎΠΏΡƒΠ±Π»ΠΈΠΊΠΎΠ²Π°Π½. ΠžΠ±ΡΠ·Π°Ρ‚Π΅Π»ΡŒΠ½Ρ‹Π΅ поля ΠΏΠΎΠΌΠ΅Ρ‡Π΅Π½Ρ‹ *